You are on page 1of 233

C.S.

E-Pre 1993

1 of 12

GEOGRAPHY
Directions: The following 13 (thirteen) items
consist of two lists labelled List I and List II. You
are to match items under List I with those under
List II. Select the correct answers to the following
13 (thirteen) items using the codes given under the
respective lists.
1.
Match List I with List II.
List I
A. Simulating diffusion of innovation.
B. Economic rent and declining Soilfertility
C. Crop combination analysis technique.
D. Isolated state and concentric donation
of land use
List II
1. Ricardo, D.
2. Hagerstrand, T.
3. Von Thunen, J.H.
4. Weaver, J.C.
A B
C
D
a. 2 1
4
3
b. 1 4
3
2
c. 3 2
1
4
d. 4 3
2
1
2.
Match List I with List II
List I
A. Concentric Zone Theory
B. Fused Growth Theory
C. Sector Theory
D. Multiple Nuclei Theory
List II
1. Garrison
2. Hoyt
3. Harris and Ullman
4. Burgess
Codes: A
B
C
D
a.
4
2
3
1
b.
4
1
2
3
c.
1
2
3
4
d.
2
1
4
3
3.
Match List I with List II

4.

5.

List I (Land form)


A. Natural levee
B. Drnmlin
C. Polje
D. Seif
List II (Agent of erosion)
1. Wind
2. Glacier
3. Running water
4. Underground water
Codes: A
B
C
D
a.
1
2
3
4
b.
2
3
4
1
c.
3
2
4
1
d.
1
4
2
3
Match List I with List II
List I (Name of Projection)
A. Transverse Mercator
B. Lambert's
C. Conical
D. Poly conic
List II (Use for)
1. Mid-latitude areas
2. Topographical maps (Survey of India)
3. Countries of limited east-west extent
4. Countries of limited north-south extent
Codes: A
B
C
D
a.
4
1
2
3
b.
3
4
1
2
c.
2
3
1
4
d.
1
2
3
4
Match List I with List II
List I
A. Areal Differentiation
B. Behavioural approach
C. Locational Analysis
D. Diffusion of Innovations
List II
1. Sauer

6.

7.

8.

2. Haggett
3. Hagerstrand
4. Hartshorne
5. Gould
Codes; A
B
C
D
a.
4
3
1
2
b.
3
5
2
4
c.
4
5
2
3
d.
2
1
5
3
Match List with List II
List I
A. Atlas
B. Central Sahara
C. Chad
D. Ethiopia
List II
1. Coastal plain
2. Interior plain
3. Massif
4. Mountain
5. Plateua
Codes: A
B
C
D
a.
3
4
5
1
b.
4
3
2
5
c.
1
5
3
4
d.
5
2
4
3
Match List I with List II
List I
A. Cotton textile
B. Iron & Steel
C. Woollen textile
D. Chemicals
List II
1. Bedford, Moscow and Kobe
2. Buffalo, Krivoi Rog and Newcastle
3. Philadelphia, Dortmund and Kiev
4. Fall river, Osaka and Ivanovo
5. Tokyo, Milan and Lyons
Codes:A
B
C
D
a.
5
1
2
4
b.
3
4
5
2
c.
4
2
1
3
d.
1
3
4
5
Match List I with List II
List I (Tribes)

9.

10.

2 of 12
A. Khasis
B. Gonds
C. Garos
D. Kurubas
List II (States)
1. Meghalaya
2. Karnataka
3. Madhya Pradesh
4. Meghalaya
Codes:
A
B
C
D
a.
4
3
2
1
b.
2
3
4
1
c.
1
4
3
2
d.
4
1
2
3
Match List I with
List II
List I (Agricultural products in U.S.S.R.)
A. Cereals, hemp, potatoes, cattle and pig
rearing
B. Cereals, flax and dairy
C. Cattle rearing
D. Wheat and dairy
List II (Natural environment)
1. Arid and samiarid lands of Central
Asia
2. Coniferous forests
3. Northern Mixed and deciduous forests
4. Southern mixed and deciduous forests
5. Steppes
Codes:A
B
C
D
a.
3
4
2
1
b.
4
3
1
5
c.
2
3
4
5
d.
1
2
4
3
Match List I with List II
List I
A. Downs
B. Prairies
C. Pampas
D. Veld
List II
1. South Africa
2. Australia
3. North America
4. South America

11.

12.

13.

5. Asia
Codes: A
B
C
a.
1
2
3
b.
2
3
4
c.
4
3
2
d.
2
3
4
Match List I with List II
List I
A. Dodabetta
B. Nanda Devi
C. Amar Kantak
D. Gani Sikhar
List II
1. Nilgiri Hills
2. Himalaya Mountains
3. Aravallis Range
4. Maikal Range
Codes:
A
B
C
a.
2
1
3
b.
3
2
1
c.
1
2
4
d.
4
1
2
Match List I with List II
List I (Town)
A. Kalpakkam
B. Jharia
C. Mathura
D. Nepanagar
List II (Industry)
1. Newsprint
2. Oil Refining
3. Coal Mining
4. Nuclear Power Plant
Codes:A
B
C
a.
4
2
1
b.
4
3
2
c.
1
2
3
d.
3
4
1
Match List I with List II
List I
A. Bhil
B. Tharu
C. Gond
D. Toda

D
5
5
1
1

D
4
4
3
3

D
3
1
4
2

3 of 12
List II
1. Terai
2. Nilgiris
3. Bastar
4. Malwa
5. Chota Nagpur
Codes: A
B
C
D
a.
1
2
3
4
b.
5
4
3
2
c.
4
1
3
2
d.
5
4
2
1
Directions: The following 16 (Sixteen) items
consists of two statements, one labelled the
'Assertion (A)' and the other labelled the 'Reason
(R)'. You are to examine these two statements
carefully and decide if the Assertion (A) and the
Reason (R) are individually true and if so, whether
the Reason is a correct explanation of the
Assertion. Select your answers from the codes
given below and mark your answer sheets
accordingly.
14.
Assertion (A): Areas of dense population
in Asia lie between 10 and 40. North
latitudes.
Reason (R): These areas have monsoon
climate.
a. Both A and R are true and R is the
correct explanation of A
b. Both A and R are true but R is not a
correct explanation of A
c. A is true but R is false
d. A is false but R is true
15.
Assertion (A): The atmosphere is said to
be conditionally unstable when the lapse
rate is between the dry adiabatic lapse rate
and the wet adiabatic lapse rate.
Reason (R): The wet adiabatic lapse rate is
always greater than the dry adiabatic lapse
rate.
a. Both A and R are true and R is the
correct explanation of A
b. Both A and R are true but R is not a
correct explanation of A
c. A is true but R is false
d. A is false but R is true
16.
Assertion (A): The population of the world
has experienced an unprecedented growth
during in 20th century.

17.

18.

19.

20.

Reason (R): Death rate fell significantly


during the 20th century due to
development of health services.
a. Both A and R are true and R is the
correct explanation of A
b. Both A and R are true but R is not a
correct explanation of A
c. A is true but R is false
d. A is false but R is true
Assertion (A): Japan is one of the leading
industrial countries of the world.
Reason (R): It has a large mineral resource
base.
a. Both A and R are true and R is the
correct explanation of A
b. Both A and R are true but R is not a
correct explanation of A
c. A is true but R is false
d. A is false but R is true
Assertion (A): Plantation agriculture is
highly developed in South-east Asia.
Reason (R): South-east Asia remained
under the European colonial rule for a long
time.
a. Both A and R are true and R is the
correct explanation of A
b. Both A and R are true but R is not a
correct explanation of A
c. A is true but R is false
d. A is false but R is true
Assertion (A): Assam grows maximum tea
in India.
Reason (R): A lot of cheap tribal labour is
available.
a. Both A and R are true and R is the
correct explanation of A
b. Both A and R are true but R is not a
correct explanation of A
c. A is true but R is false
d. A is false but R is true
Assertion (A): Areas along the equator
record the highest temperature through out
the year.
Reason (R): On the equator, days and
nights are equal for the largest part of the
year.
a. Both A and R are true and R is the
correct explanation of A

21.

22.

23.

24.

4 of 12
b. Both A and R are true but R is not a
correct explanation of A
c. A is true but R is false
d. A is false but R is true
Assertion (A): Thunderstorm activity in
India is characteristic of the pre-and post
monsoon periods.
Reason (R): These are of the periods of
transition
between
the
seasonal
circulations.
a. Both A and R are true and R is the
correct explanation of A
b. Both A and R are true but R is not a
correct explanation of A
c. A is true but R is false
d. A is false but R is true
Assertion (A): Cotton cultivation in Egypt
is largely concentrated in the upper Nile
valley.
Reason (R): Cotton cultivation requires
fertile clayey soil, abundant sunshine,
frost-free weather and cheap labour.
a. Both A and R are true and R is the
correct explanation of A
b. Both A and R are true but R is not a
correct explanation of A
c. A is true but R is false
d. A is false but R is true
Assertion (A): The Great Dividing Range
of Australia is an example of a block
mountain.
Reason (R): It was uplifted by the Alpien
earth movements.
a. Both A and R are true and R is the
correct explanation of A
b. Both A and R are true but R is not a
correct explanation of A
c. A is true but R is false
d. A is false but R is true
Assertion (A): About 80% of Canadian
manufacturing is concentrated in South
Ontario and St. Lawrence lowland region.
Reason (R): There are abundant deposits
of coal in the South Ontario and St.
Lawrence lowland region.
a. Both A and R are true and R is the
correct explanation of A
b. Both A and R are true but R is not a
correct explanation of A

25.

26.

27.

28.

29.

c. A is true but R is false


d. A is false but R is true
Assertion (A): The Mediterranean regions
of the Northern Hemisphere specialize in
horticulture.
Reason (R): The climatic conditions in the
Mediterranean regions are favourable.
a. Both A and R are true and R is the
correct explanation of A
b. Both A and R are true but R is not a
correct explanation of A
c. A is true but R is false
d. A is false but R is true
Assertion (A): India has a tropical
monsoon climate.
Reason (R): India is located within tropical
latitudes.
a. Both A and R are true and R is the
correct explanation of A
b. Both A and R are true but R is not a
correct explanation of A
c. A is true but R is false
d. A is false but R is true
Assertion (A): The eastern coast of India is
affected by tropical cyclones more than the
western coast.
Reason (R): Tropical cyclores originate
only in the Bay of Bengal.
a. Both A and R are true and R is the
correct explanation of A
b. Both A and R are true but R is not a
correct explanation of A
c. A is true but R is false
d. A is false but R is true
Assertion (A): Equatorial region is called
'the region of debilitation'.
Reason (R): The Equatorial region has unit
formally unfavorable climatic conditions.
a. Both A and R are true and R is the
correct explanation of A
b. Both A and R are true but R is not a
correct explanation of A
c. A is true but R is false
d. A is false but R is true
Assertion (A): The Regur soils of the
Deccan Plateau are black in colour.
Reason (R): They contain a lot of humus.

30.

31.

32.

33.

34.

35.

36.

5 of 12
a. Both A and R are true and R is the
correct explanation of A
b. Both A and R are true but R is not a
correct explanation of A
c. A is true but R is false
d. A is false but R is true
The molten igneous material is called
a. Mantle
b. Magma
c. Sial
d. Sima
Which one of the following theories
postulates that orogenic-belts have roots?
a. Kober's Mountain-building theory
b. Pratt's theory of Isostasy
c. Holmes Convectional current theory
d. Airys theroy of Isostasy
Clear nights are cooler than cloudy nights
because of
a. condensation
b. radiation
c. insolation
d. conduction
Which among the following represents the
correct sequence in increasing height, of
different types of cloud formation?
a. Cirrus,
cumulonimbus,
stratus,
cumulus
b. Stratus, cumulus, cirrus cumulonimbus
c. Cumulonimbus, cumulus, stratus,
cirrus
d. Cirrus,
cumulus,
cumulonimbus,
stratus
Which one of the following is the
characteristic feature of kars topography?
a. Coves
b. Caves
c. Caverns
d. Cavities
The 'Moho' is a density discontinuity in the
a. earth's crust
b. upper atmosphere
c. lower atmosphere
d. greater depths of the oceans
Which one of the following ocean currents
is associated with the 'El Nino'
phenomenon?

37.

38.

39.

40.

41.

a. Humboldt
b. Benguela
c. Canaries
d. Kumshio
Drift of material along a shore from
opposite directions results in the building
of a pointed depositional feature known as
a. offshore bar
b. offshore horn
c. cuspate bar
d. tombolo
The plain formed due to coalescence of
series of alluvial fans in the piedmont zone
is known as
a. Pediment
b. Bajada
c. Pediplain
d. Hamada
By what name are the numerous
depressions found in a limestone region
called?
a. Caverns
b. Pot-holes
c. Plunge-pools
d. Sink-holes
Spatial interaction is reflected in the flow
of
1. People
2. goods
3. information
Select the correct answer from the codes
given below:
a. 1, 2 and 3
b. 1 and 2
c. 1 and 3
d. 2 and 3
Which of the following were the main
reasons for the great migrations from
Europe of America?
1. Over population in Europe
2. Economic prospects in Europe
3. Climatic similarity between Europe
and America
4. Adventurous spirit and navigational
skills of Europeans
Select the correct answer form the codes
given below

42.

43.

44.

45.

46.

6 of 12
a. 1 and 2
b. 2 and 3
c. 3 and 4
d. 1 and 4
Which one of the following tribes
practices pastoral nomadism?
a. Eskimo
b. Boro
c. Pygmy
d. Masai
Which of the following countries is not a
member of the SAARC?
a. Afghanistan
b. Bhutan
c. Nepal
d. Sir Lanka
Which of the following makes the
temperate forests easier of exploit than the
Tropical Rain forests?
1. Proximity of market
2. Softer wood
3. Taller trees
4. Greater homogeneity
Select the correct answer form the codes
given below
a. 1 and 3
b. 2 and 4
c. 2 and 3
d. 3 and 4
Pittsburg, as a centre of the iron and steel
industry has a great disadvantage in
respect of
a. availability of cooking coal
b. supply of iron ore
c. transportation
d. market
The North Pole lies within the Arctic
Ocean whereas the South Pole is located
within the Antarctica Continent
Which of the following hypotheses uses
this phenomenon as a supportive
argument?
a. Tetrahedron theory
b. Theory of Isostasy
c. Theory of Continental Drift
d. Convective Current Theory

47.

48.

49.

50.

51.

52.

53.

Lines joining places of same total


transportation costs in Weber's theory are
termed
a. isotims
b. isogonic lines
c. isodapanes
d. isochrones
If the highest point in locality is 1500
meters and the lowest is 1000 meters, 500
meters represents the
a. height above base-level
b. gradient
c. elevation
d. relative relief
The age and sex composition of the
population is best represented by
a. choropleths
b. isopleths
c. pie diagram
d. pyramid diagram
The proportion of the area of the surface
at various elevations above or depths
below a given datum is properly
represented by the
a. Clinographics curve
b. Altimetric frequency curve
c. Hypsometric curve
d. Lorenz curve
Topographical maps are distinguished
from atlas maps as they
a. represent only relief
b. are large-scale maps
c. are small-scale maps
d. are drawn on a different projection
The usage of the term 'ecosystem' for the
whole complex f organisms-both animals
and plants, and their habitat has been
proposed by
a. Cole
b. Tansley
c. Forbes
d. Sukachev
The gravity and potential models in
geography were the contribution of
a. Stewart
b. Clark and Evans
c. Haggett and Chorley

54.

55.

56.

57.

58.

59.

60.

7 of 12
d. Bunge
The rank-size rule is useful in studies of
a. Plant communities
b. climetic regions
c. settlement systems
d. soil types
From the viewpoint of laud forms, which
of the following pairs of continents are
mirror images of one another?
a. AfricaAustralia
b. AfricaSouth America
c. Asia Africa
d. Australia South America
Which of the following features is
common to the Deccan Plateau, the
Columbian Plateau and Iceland?
a. Composed of lava
b. Extremely rugged
c. Great height above sea level
d. Inland drainage
Which one of the following climatic types
identified by Koppen occurs in both China
and Japan?
a. Cwa
b. Cfa
c. Dfb
d. Dwb
The deep chernozem soil rich in humus is
found in
a. Amazon basin
b. Congo basin
c. Egypt
d. Ukraine
Terra rossa typically develops in terrains
composed of
a. limestone
b. basalt
c. granite
d. red sandstone
What natural vegetation can be expected in
subhumid
middle
latituded
where
evaportanspiration
almost
balances
precipitation?
a. Conifers
b. Prairies
c. Savanna
d. Selvas

61.

62.

63.

64.

65.

66.

Which of the following are reasons for the


location of the Australian Desert?
1. Presence of a cold ocean current along
the coast
2. Trade winds blowing over land for a
considerable stretch
3. Existence of a mountain chain
obstructing moisture bearing winds
4. Occurrences of sub-tropical high
pressure
Choose the correct answer from the codes
given below
a. 1 and 2
b. 2 and 3
c. 1, 2 and 3
d. 1, 2, 3 and 4 '
Which one of the following pairs is
correctly matched?
a. The Mediterranean Region :
Summer rain
b. The Equatorial Region :
Afternoon thundershowers
c. The Monsoon Region
:
Heavey rain throughout the Year
d. The Desert Region : Winter rain
The indigenous people living in the
Steppes of the Soviet Union arc the
a. Khirghiz
b. Taureg
c. Bushmen
d. Bhils
Which group contains the monsoon
regions among the following'?
a. U.S.A., Canada, Mexico, Ecuador
b. India, Bangladesh, Myanmar, Thailand
c. Sri Lanka, Japan, Australia, Antarctica
d. U.S.S.R., North China, Iran, Iraq
Which one among the following natural
regions occurs only in the Northern
Hemisphere?
a. Mediterranean
b. Steppe
c. Taiga
d. Tropical desert
The greatest number and widest variety of
plants and animals would be found in
a. tropical grasslands

8 of 12
b. tropical forests
c. temperate forests
d. temperate grasslands
67.
A much fragmented or clustered pattern of
population distribution is conspicuous in
parts of Asia because it is closely
associated with
a. soil and rainfall distribution
b. surface configuration and temperature
characteristics
c. routes of transport and trade centers
d. distribution of mineral and forest
resources
68.
Which of the following countries of
Europe had reached the last stage of the
demographic transition?
a. Portugal
b. France
c. Spain
d. Italy
69.
The non ecumeue areas of the world are
the
a. sparsely populated areas
b. densely populated areas
c. areas of high fertility
d. areas of high mortality
Directions: The following three questions are
related to the map given below. Observe the map
carefully and answer the questions.

70.

71.

The shaded area indicates the cultural


realm identified as
a. African
b. "Dry"
c. Oriental
d. Occidental
The arrow line shows the route of
migration of
a. Australasians
b. Polynesians
c. Southern Chinese
d. Vikings

72.

73.

74.

75.

76.

77.

78.

79.

The cross-hatched area is inhabited by


a. Bantu
b. Capoids
c. Zulus
d. Europeans
The major fishing grounds of the world are
located on the
a. continental shelves of the mid-latitudes
b. deep-sea plains of the North Pacific
c. Equatorial waters of the Atlantic
d. Equatorial waters of the Pacific
The leading sulphur producing country in
the world is
a. U.S.A.
b. U.S.S.R.
c. Japan
d. U.K.
The principal rubber exporting nations in
the world are
a. India, U.S.S.R., China, U.K.
b. Malaysia, Indonesia, Thailand, Nigeria
c. Banglandesh, Burma, Pakistan, Sri
Lanka
d. U.S.A, Canada, Mexico, Chile
'Ladang' is a type of cultivation
characterized by
a. dry farming
b. high degree of mechanization
c. mixed farming
d. shifting cultivation
What is Karaganda?
a. An animal
b. A montain
c. A coalfield
d. An ocean deep
The development of synthetic rubber
industry in U.S.A. is the outcome of
a. destruction of rubber trees by the early
settlers
b. clearing of rubber producing areas for
agricultural purposes
c. First World War
d. Second World War
'Port of Portugal" is a variety of
a. tobacco
b. fish

80.

81.

82.

83.

84.

85.

9 of 12
c. fruit
d. wine
The American cotton textile industry
moved from New England to the southern
states because
a. the latter have a more humid climate
b. New
England
diversified
its
manufacturing industries
c. the population growth in the south
provided a major market
d. the south offered cheaper labour, lower
taxes and more spacious sites
Which one of the following it the type of
vegetation characteristic of the area shown
in the map?
a. Rainforest
b. Forest with grassland
c. Desert and semi desert
d. Mediterranean
The Chindwin basin is situated in
a. Myanmar
b. Thailand
c. Malaysia
d. Indonesia
In which one of the following situations in
South-East Asia are the largest
concentrations of peasant population
found?
a. On low-land forest areas cleared for
plantation agriculture
b. On highland areas where the climate is
cooler and healthier
c. On low-land areas where grain farming
is carried on
d. In the mining areas
West Bank is the name given to the land
area west of river
a. Jordan
b. Mississippi
c. Nile
d. Tigris
The area marked X represents the Strait of

90.

91.

86.

87.

88.

89.

a. Dover
b. Babel Mandeb
c. Hoimuz
d. Magellan
Which one of the following South-West
Asian countries has the shortest coastline?
a. Iran
b. Oman
c. Syria
d. Jordan
The climate of North America is
influenced during winter by the
a. warm Gulf stream
b. cold Labrador current
c. cold polar airmasses
d. tropical airmasses
Which one of the following statements is
most plausible?
The Moscow industrial region is different
from the Ural, Kuzbas and Ukraine
industrial regions in that
a. it is based on local raw materials
b. It has an abundant supply of locally
available cheap power
c. it has vast market and ancillary
industries
d. it is specialised in agricultural
machinery
In the map of China the river X represents

92.

93.

94.

95.

a.
b.
c.
d.

Hwang Ho
Si Kiang
Sungari
Yangtse-Kiang

96.

10 of 12
The Loess plateau of North China is
covered with a thick mantle of loess
carried here by strong winter winds from
a. the Sahara Desert
b. the Gobi Desert
c. the Tarim basin
d. the Thar Desert
The Indus and Brahmputra rivers are
examples of
a. subsequent drainage
b. super-imposed drainage
c. consequent drainage
d. antecedent drainage
What is the correct descending order of the
following states area-wise ?
1. Andhra Pradesh
2. Bihar
3. Madhya Pradesh
4. UP.
Choose the correct answer from the codes
given below
a. 1, 2, 3, 4
b. 2, 3, 4, 1
c. 4,3,2,1
d. 3,4,1,2
Which one of the following states does not
form part of the Narmada basin?
a. Madhya Pradesh
b. Rajasthan
c. Gujarat
d. Maharashtra
Which of the following pairs is not
correctly matched?
a. Kosi
: Consequent river
b. Narmada : Rift valley river
c. Suttej
: Antecedent river
d. Ghaggbai : River with eodoreic
drainage
The alluvial soils of the Gangetk Valley
are described as
a. Intrazonal
b. Azonal
c. Zonal
d. None of the above
The following factors have caused water
logging in the black: lava soil of Deccan:

97.

98.

99.

100.

101.

1. Introduction of irrigation
2. Sugarcane cultivation
3. High rainfall
Select the correct answer from the codes
given below
a. 1, 2 and 3
b. 1 and 2
c. only 1
d. 1 and 3
The typical area of 'sal' forest in the Indian
Peninsular upland occurs
a. on the Western Chats
b. between the Tapti and the Narmada
c. to the North-east of the Godavari
d. on the Malwa Plateau
Which of the following are the rivers at the
delta mouths of which mangroves occur in
abundance?
1. Narmada
2. Subamarekha
3. Krishna
4. Ganga
Choose the correct answer from the codes
given below
a. 1 and 3
b. 3 and 4
c. 2 and 4
d. 1, 3 and 4
I.R. 20 and Ratna are two important High
Yielding Varieties of
a. wheat
b. bajra
c. paddy
d. jowar
In respect of which one of the following
crops the Green Revolution in India had
limited impact?
a. maize
b. wheat
c. rice
d. pulses
Which of the following forest species is
not characteristic of the deciduous forests?
a. Teak
b. Sal
c. Sandalwood

102.

103.

104.

105.

106.

107.

108.

11 of 12
d. Deodar
Tuticorin, an important fishing harbour of
India, is situated along the
a. Malabar Coast
b. Coromandel Coast
c. Konkan Coast
d. Gulf of Kutch Coast
The most important fisheries in India are
a. deep-sea
b. offshore
c. natural inland
d. cultural inland
The following states
produce coal.
Identify the correct order in which they are
arranged as per their importance
a. West Bengal, Bihar, Orissa, Madhya
Pradesh
b. Bihar, West Bengal, Orissa, Madhya
Pradesh
c. Orissa, Madhya Pradesh, West Bengal,
Bihar
d. Bihar, Orissa, Madhya Pradesh, West
Bengal
Which of the following has a potentiality
for harnessing of tidal energy in India?
a. Gulf of Cambay
b. Gulf of Mannar
c. Backwaters of Kerala
d. Chilka lake
At which place are diesel locomotives
manufactured?
a. Chittaranjan
b. Varanasi
c. Jamshedpur
d. Perambur
Although almost all the states of India
have paper mills halt" of the total paper
produced in the country comes from
a. Orissa
b. Maharashtra
c. West Bengal
d. Andhra Pradesh
The dominant type of irrigation in
Peninsular India is irrigation by
a. canal
b. tank
c. well

109.

110.

111.

112.

113.

d. sprinkler
Which of the following factors is more
important for the location of the steel mills
at Bokaro?
114.
a. Nearness to iron ore deposits
b. Nearness to coal deposits
c. Nearness to both coal and iron are
deposits
d. Nearness to the port city of Calcutta
Which of the following factors are
115.
responsible for the rapid growth of sugar
production in South India as compared to
North India?
1. Higher per acre field of sugarcane
2. Higher sucrose content of sugarcane
3. Lower labour cost
116.
4. Longer crushing period
Select the correct answer from the codes
given below
a. 1 and 2
b. 1,2 and 3
c. 1,3 and 4
117.
d. 1,2 and 4
The Rourkela Steel Plant was built on the
banks of the
a. Bhadra river
b. Brahmani river
c. Damodar river
118.
d. Bhima river
The most populous states of India
according to the latest census were the
following :
1. Bihar
2. Uttar Pradesh
119.
3. West Bengal
4. Maharashtra
What is their sequence arranged in the
descending order? Choose the correct
answer from the codes given below
a. 2, 1, 4, 3
b. 1, 2, 4, 3
120.
c. 2, 3, 4, 1
d. 1, 2, 3, 4
Which of the following cities was a
million city in 1981 census?
a. Dibrugarh
***********

12 of 12
b. Kanpur
c. Pun
d. Patna
The region
that has a typical dispersed
rural settlement pattern is the
a. Kerala Coastal plain
b. Western Ganga plain
c. Eastern Rajasthan
d. Telengana plateau
Which of the following industrial towns is
located on the Chota Nagpur Plateau ?
a. Bhilai
b. Ranchi
c. Asansol
d. Durgapur
Which one of the following major seaports
of India does not have a natural harbour?
a. Bombay
b. Cochin
c. Mannagao
d. Paradeep
The highest rainfall in the world is
received at
a. Cherrapunji
b. Mawsynram
c. Shillong
d. Calicut
Which one of the following is a satellite
town?
a. Faizabad
b. Murshidabad
c. Ghaziabad
d. Moradabad
Which one of the natural regions is known
as the 'Bread-Basket' of the world?
a. The steppe region
b. The Mediterranean region
c. The monsoon region
d. The equatorial region
The term "boat people" refers to the
migrants from
a. Sri Lanka to India
b. Taiwan to mainland China
c. Vietnam to Thailand
d. Vietnam to the U.S.A.

C.S.E-Pre 1994

1 of 11

GEOGRAPHY
1.

2.

3.

4.

5.

6.

The "cirque" is a French word which is


known in Scotland as
a. Corris
b. cors
c. cwm
d. corrie
Which one of the following groups is the
product of erosion?
a. Drumlins, moraines, aretes
b. U-shaped valleys, ventifacts, canyons
c. Eskers, fiords, outwash plain
d. Cirques, V- shaped valleys, levees
Soil that owes its colour to oxides of iron
is
a. regur
b. bangar
c. laterite
d. alkaline
The correct explanation of cyclogenesis is
a. occurrence
of
difference
in
temperature resulting in abrupt
lowering of pressure
b. occurrence
of
difference
in
temperature resulting in formation of
high pressure
c. parallel movement of warm and cold
fronts
d. interaction of land and sea breeze
Increase of carbon-dioxide in the
atmosphere due to industrial pollution will
lead to
a. increase in surface albedo
b. increase in surface temperature
c. decrease in surface temperature
d. increase in snow cover of the earth
Match List I with List II and select the
correct answer using the codes given
below the lists :
List I (Country/Region)
A. Chad
B. Guyana
C. South-West Australia
D. Sri Lanka
List II (climatic type)
1. Equatorial
2. Mediterranean
3. Monsoon
4. Tropical savanna

Codes:

7.

8.

9.

10.

A
B
C
D
a.
1
3
4
2
b.
1
5
4
3
c.
4
5
2
3
d.
5
1
2
3
Which one of the following is drought
resistant?
a. Xerophyte
b. Halophyte
c. Mesophyte
d. Hydrophyte
Which one of the following regions is
called the 'bread basket' of the world?
a. Tropical monsoonal
b. Mediterranean
c. Temperate grassland
d. Savanna grassland
Consider the temperature and rainfall data
below
Temp
Rainfall (in
T(C)
cms)
January

14.4

2.5

February

16.7

1.5

March

23.3

1.3

April

30.0

1.0

May

33.3

1.8

June

33.3

7.4

July

30.0

19.3

August

29.4

17.8

September

28.9

11.9

October

25.6

1.3

November

19.4

0.2

December

15.6

1.0

This would be typical of


a. an equatorial region
b. a monsoon region
c. a warm temperate region
d. a cool temperate region
The main occupations of the Lapps would
include
a. herding and fishing

11.

12.

13.

14.

15.

16.

b. hunting and food gathering


c. pastoral nomadism
d. shifting cultivation
A region best known for trapping of
animals is the
a. British type
b. Laurentian region
c. Taiga region
d. Polar region
Consider the following States :
1. Kerala
2. Mizoram
3. Goa
4. Tamil Nadu
As per the 91 census, the correct sequence
of the descending order of the effective
literary rates in these States is
a. 1, 2, 3, 4
b. 1, 4, 2, 3
c. 1, 3, 4, 2
d. 4, 1, 2, 3
In South-East Asia, the highest population
density is found in
a. Vietnam
b. Cambodia
c. Malaysia
d. Indonesia
Consider the following statements :
Developing countries in the past two
decades have registered increase in
population despite significant decline in
fertility rates because of declining infant
mortality better living conditions
1. increasing life-expectancy
2. declining infant mortality
3. failing death rates
4. better living conditions
Of these statements
a. 1, 3, and 4 are correct
b. 2, 3 and 4 are correct
c. 1, 2, and 3 are correct
d. 2, 3 and 4 are correct
Britishers constitute 90% of the population
of
a. Australia
b. New Zealand
c. Canada
d. South Africa
Griffith Taylor applied heconcep of Zones
and Strata for the
a. evolution of the races of mankind
b. evolution of cities and towns
c. spread of civilization
d. diffusion of settled agriculture

17.

18.

19.

20.

21.

22.

23.

2 of 11
In which one of the following groups of
countries are the Nordics found ?
a. Spain, France, Romania and Turkey
b. Sweden, Norway, England and
Germany
c. Sweden, Norway, Germany and Italy
d. Romania, Turkey, France and Portugal
The indigenous people living in Fiji, Papua
and New Guinea belong to
a. Mongolian group
b. Micronesian group
c. Polynesian group
d. Melanesian group
The 'Hottentots' are the
a. Pigmies of Congo (Zaire basin)
b. Negritoes of Shri Lanka having brown
colour
c. Yellow skinned people of South-West
Africa
d. Negritoes of Eastern Africa
The four major cultural realms according
to Jan Brock comprise
a. Anglo-American, Main Islamic, India
and E. Asian
b. Occidental, Main Islamic, India and
E.Asian
c. Latin American, Australian, New
Zealand, Main Islamic and E. Asian
d. Anglo-American,
Mediterranean
European, East Asian and Main
Islamic
Which one of the following features does
NOT apply to the polar culture realms?
a. Mostly the people depend on animals
b. Hunting and herding are the main ways
of obtaining livelihood
c. The Eskino are hunters of the land
animals
d. Mostly the people are Mongoloid
Intensive
subsistence
farming
is
characteristic of areas of
a. high density of population and high
technology
b. low density of population and high
technology
c. low density of population and low
technology
d. high density of population and low
technology
The ancient 'home region' of sugar beet
was the land between
a. Mediterranean Sea and Caspian Sea
b. Baltic Sea and North Sea
c. Caspian Sea and Aral Sea

24.

25.

26.

27.

28.

d. Mediterranean Sea and Adriatic Sea


Which one of the following is generally
considered to be the most important reason
for the decline of the Indus Valley
civilization?
a. Repeated droughts
b. Epidemics
c. Frequent floods
d. Internal feuds
Consider the following factors
1. Convergence of cool and warm ocean
currents
2. Shallow depth of ocean waters
3. Nearness of fishing ports
4. Abundance of drifting planktons
Those responsible for the location of the
great fishing ground would include
a. 1 alone
b. 1 and 2
c. 2, 3 and 4
d. 1, 2 and 4
The coniferous forests have been exploited
to a considerable extent because of the
a. presence of extensive solid stands of
one or two species of timber trees
b. presence of a vast network of perennial
streams
c. existence of large markets in close
proximity
d. availability of cheap labour
Match List I with List II and select the
correct answer using the cedes given
below lists :
List I (Material producer)
A. Tin
B. Silver
C. Mica
D. Copper
List II (World's largest)
1. Mexico
2. U.S.A.
3. Malaysia
4. India
Codes: A
B
C
D
a.
3
2
1
4
b.
3
1
4
2
c.
4
2
3
1
d.
4
1
2
3
Iron is obtained mostly from
a. hematite ores
b. magnetite ores
c. siderite ores
d. iron pyrites

29.

30.

31.

32.

33.

3 of 11
Match List I with List II and select the
correct answer using the codes given
below the lists :
List I (Project)
A. The Hoover Dam
B. Keokuk
C. Grand Coulee
D. Kachovka
List II (Rivers)
1. Columbia
2. Colorado
3. Mississippi
4. Dnieper
Codes: A
B
C
D
a.
2
3
4
1
b.
1
2
3
4
c.
3
2
4
1
d.
2
3
1
4
Which one of the following pairs is
correctly matched?
a. Automobile industry ................ Los
Angeles
b. Shipbuilding ................Lusaka
c. Aircraft industry ................ Seattle
d. Iron and Steel industry ................
Florence
The 'Horn of Africa' includes
a. Namibia, Botswana and Republic of
South Africa
b. Ethiopia,
Djibouti
and
Somali
Republic
c. Senegal, Guinea, Sierra Leone and
Liberia
d. Libya, Egypt and Sudan
The shift of the cotton textile industry
from New England to the Southern
Appalachians, was mainly due to
a. warmer climate
b. uniform working hours
c. abundant supply of raw cotton
d. low standard of living of the people
Match List I with List II and select the
correct answer from the codes given below
the lists :
List I : (Country)
A. Angola
B. Morocco
C. Namibia
D. Zaire
List II : (Capital)
1. Kinshasa
2. Luanda
3. Rabat
4. Windhoek

34.

35.

36.

37.

38.

39.

Codes: A
B
C
D
a.
1
3
4
2
b.
1
3
2
4
c.
2
1
3
4
d.
2
3
4
1
The geographical importance of SouthEast Asia is mainly due to
a. its religious diversity
b. its strategic location with reference to
trade routes
c. its production of tin and oil
d. its exposure to western influences
Which one of the following statements
regarding South-West Asia is NOT
correct?
a. United Arab Emirates is made up of
six Emirates
b. Yemen produces the best quality
coffee
c. Rice is cultivated in Iraq
d. Jordan grows cotton
The only important rail-road which
connects the Caspian Sea coast with the
Persian gulf area, passes through
a. Tabriz
b. Shiraz
c. Teheran
d. Esfahan
The Mesopotamia valley produces large
quantities of
a. wheat and orange
b. rice and date
c. maize and apple
d. barley and oat
The major coconut oil producing country
in South-East Asia is
a. Laos
b. Cambodia
c. Malaysia
d. Philippines
The lakes labelled I, II, II, IV and V in the
given map are respectively

40.

41.

42.

43.

44.

45.

46.
a. Ontario, Superior, Huron, Erie,
Michigan
b. Michigan, Ontario, Huron, Superior,
Erie
c. Superior, Michigan, Huron, Erie,
Ontario

4 of 11
d. Superior, Michigan, Erie, Huron,
Ontario
The heart of American Agriculture and the
most important agricultural region in the
world is the
a. corn belt
b. cotton belt
c. hay and dairy belt
d. truck-farming belt
The
Appalachian
Highlands
are
economically important mainly because of
a. ready source of timber and water
power
b. natural passes facilitating easy
movement
c. rich mineral wealth formig the basis
off lourishing industries
d. large number of both summer and
winter tourist centres
Which one of the following is the main
reason for the industrial backwardness of
Myanmar?
a. Easy livelihood from agriculture
b. Uneven surface of land
c. Limited local market
d. Lack of coal and iron ore
Moscow's engineering workshops and
textile factories are supplied with coal
from the
a. Valdai hills to the north
b. Tula coalfield to the south
c. Ural mountains
d. Caucasus mountains
Ukraine is an ideal industrial centre of
Eastern Europe because the region is rich
in
a. iron ore, coal and manganese
b. copper, zinc and oil
c. natural gas, iron ore and manganese
d. iron ore, limestone and copper
The erstwhile USSR became one of the
leading industrial power of the world
because of her
a. vast population
b. large number of skilled manpower
c. vast area
d. vast mineral and power resources
Which one of the following cities of China
leads the rest in both rade and population ?
a. Shanghai
b. Dairen
c. Tianjin
d. Canton

5 of 11
47.

48.

49.

50.

51.

52.

53.

54.

The enormous volume of dust swept


across the Steppe lands f Asia and
deposited in the North-Western part of
China is nown as
a. Reg
b. Loess
c. Terra Rossa
d. Kankar
Which one of the following mountain
ranges provides the most effective regional
division of China?
a. Nanling
b. Tienshan
c. Qulianshan
d. Tsinglingshan
Western Ghats are
a. mountains
b. plateaus
c. escarpment of the plateaus
d. hills
The Siwalik Hills have been made out of
the debris coming from the
a. Himalayas
b. Sutlej Valley
c. Ganga Valley
d. Southern Peninsula
Which one of the following physiographic
units has been created by both exogamic
and androgenic forces?
a. The Peninsular Plateau
b. The Thar Desert
c. The Indo-Gangetic Plain
d. The Himalayas
Copious rains in the northern coast of
Tamil Nadu in October and November are
due to
a. the south-east trade winds
b. easterly depression and the presence
of inter-tropical convergence
c. westerly depressions
d. occlusion
The immediate cause for the S.E. Trades to
cross the equator and blow as S. W.
monsoon winds over India is the
a. intense low pressure over Tibet
b. heated Ganga plain
c. intense low pressure area of Thar
(Rajasthan) desert
d. high temperature over Chotanagpur
plateau
The given map relates to climatic division
of India according to

55.

56.

57.

58.

59.

60.

a. Thornthwaite
b. Koppen
c. Stamp
d. Trewartha
Dark and hard are the characteristic
features of
a. teak
b. pine
c. ebony
d. sal
Soil erosion in India occurs in almost all
the littoral states but it is most serious
along the coast of
a. Kerala
b. Tamil Nadu
c. Orissa
d. Karnataka
Which one of the following factors is
responsible for excessive soil erosion in
Chotanagupr plateau?
a. Heavy rain throughout the year
b. Loose sandy soil
c. Deep ploughing by tractors
d. Large scale felling of trees
Which of the following measures are
effective for soil conservation in India?
1. Avoiding crop rotation
2. Afforestation
3. Encouraging the use of chemical
fertilizers
4. Limiting shifting cultivation
Select the correct answer from the codes
given below:
a. 1 and 2
b. 2 and 4
c. 3 and 4
d. 1, 2 and 3
The Ban Sagar project, under construction,
is on the river
a. Mahanadi
b. Indravati
c. Sone
d. Tapti
Which of the following natural
characteristics are associated with the dry
monsoon forests of India

61.

62.

63.

64.

1. Annual rainfall is below 50 cms


2. The trees have short roots
3. Thorny shrubs and grasslands grow
between the trees
4. Mango, mahua, sisam, keekar etc are.
the prominent trees.
Select the correct answer from the codes
given below:
a. 1 and 2
b. 1, 2 and 3
c. 2 and 4
d. 3 and 4
Consider the following statements about
Indian forestry:
1. About 40% of the Indian forests are in
the inaccessible mountainous regions
which impede the speedy transfer of
felled trees.
2. Although according to India's forest
policy, a high proportion of the land
area shall be covered by forest except
in some States, the percentage of forest
cover is precariously low.
3. Exploitation of forests is less time
consuming and less costly, as almost in
every forest a single type of tree
dominates.
4. The annual productivity of the forests
is 3, 5 cubic metres per hectare,
a. 1 and 2 are correct
b. 1, 2 and 3 are correct
c. 3 and 4 are correct
d. 1, 2, 3 and 4 are correct
The highest yield of rice per hectare in
India is in
a. West Bengal
b. Tamil Nadu
c. Punjab
d. Haryana
The extent of increase in the cumulative
irrigation potential in India during 1951-81
is
a. 1.5 times
b. 2.5 times
c. 3.0 times
d. 3.5 times
The substantial increase in agricultural
production in recent years in India has
been largely due to
a. uncultivated land shaving been brought
under the plough on a large scale
b. large scale employment of agricultural
labour

65.

66.

67.

68.

69.

6 of 11
c. increased use of high yielding varieties
of seeds and of fertilizers
d. better transport facilities
Consider the following
Andhra Pradesh is one of the leading
producers of rice because
1. it has fertile alluvial soil in the coastal
plain.
2. it receives about 125 cm of rainfall per
annum in the coastal tract.
3. it has a lot of tank irrigation in the
coastal plain
a. 1, 2 and 3 are correct
b. 1 and 2 are correct
c. 2 and 3 are correct
d. 1 and 3 are correct
Contribution of forestry to national income
is around
a. 2.0 percent
b. 3.0 percent
c. 1.0 percent
d. 4.0 percent
Consider the following States:
1. Jammu and Kashmir
2. Uttar Pradesh
3. Meghalaya
4. Arunachal Pradesh
The correct sequence of the descending
order in terms of the area of coverage of
coniferous forests in these States is
a. 1, 4, 2, 3
b. 1, 3, 2 4
c. 2, 3, 4, 1
d. 1, 2, 4, 3
The following States are involved in the
export offish and fish products:
1. Kerala
2. Maharashtra
3. Karnataka
4. Andhra Pradesh
The correct sequence of the descending
order in terms of the export offish and fish
products from these State is
a. 1, 2, 3, 4
b. 1, 4, 2, 3
c. 3, 2, 1, 4
d. 2, 1, 4, 3
Which of the following factors are
responsible for India's failure to fully
exploit the' inland fisheries during the last
five decades?
1. Silting and pollution! of the inland
water bodies

70.

71.

72.

73.

74.

75.

76.

2. Deforestation in the catchments area of


the river
3. Lack of marketing facilities.
Select the correct answer from the codes
given below:
a. 1, 2 and 3
b. 1 and 2
c. 1 and 3
d. 2 and 3
Diamond mines are located in
a. Uttar Pradesh
b. Karnataka
c. Madhya Pradesh
d. Gujarat
The super-thermal plant located near
Singareni Collieries is
a. Farakka
b. Singrauli
c. Korba
d. Ramagundan
The iron ore mined at Bailadila is mostly
a. hematite
b. siderite
c. limonite
d. magnetite
Asperse 1991 census, the correct
descending order of the given religious
groups among the population of India is
a. Sikhs, Buddhists, Christians, Jains
b. Jains, Sikhs, Buddhists, Christians
c. Buddhists, Jains, Sikhs, Christians
d. Christians, Sikhs, Buddhists, Jains
A centre of mica mining in India is located
at
a. Khetri,
b. Kodarma
c. Kalahandi
d. Guru Mahasani
Consider the following coal producing
states:
1. Bihar
2. Madhya Pradesh
3. West Bengal The correct descending
order in terms of the amount of coal
produced in these states is
a. 1, 2, 3
b. 3, 1, 2
c. 1, 3, 2
d. 3, 2, 1
Match List I with List II and select the
correct answer from the codes given below
the lists :
List I (Industrial Products)
A. Paper

77.

78.

79.

80.

7 of 11
B. Heavy Engineering
C. Locomotive
D. Steel production
List II (centers)
1. Vijaynagar
2. Varanasi
3. Hatia
4. Nepanagar
Codes: A
B
C
D
a. (a) 3
4
1
2
b. (b) 4
3
2
1
c. (c) 3
1
2
4
d. (d) 4
2
1
3
Hydroelectric project is located at
a. Pykara
b. Korba
c. Debari
d. Haldia
Following sectors are important consumers
of natural gas in India
1. Power
2. Industry
3. Fertilizers
4. Production of liquefied petroleum gas
The correct sequence of the descending
order in terms of the consumption of
natural gas by them, is
a. 1, 3, 2, 4
b. 1, 3, 4, 2
c. 2, 1, 3, 4
d. 2, 4, 3, 1
Match List I with List II and select the
correct answer from the codes given below
the lists :
List I: (Form of non-conventional energy)
A. Geothermal
B. Tidal
C. Wind
D. Solar
List II (Areas in India which are developed
or can be developed)
1. Lamba
2. Ahmedabad
3. Madras
4. Manikaran
Codes A
B
C
D
a.
1
2
3
4
b.
1
4
2
3
c.
4
1
3
2
d.
4
3
1
2
The rural settlements of Kerala are mostly
a. compact
b. dispersed
c. linear

d. radial
81.
The proportion of the number of villages
above population of 5000 to the total
number of villages statewide is the highest
in
a. Tamil Nadu
b. Uttar Pradesh
c. Bihar
d. Kerala
82.
Which of the following factors are
responsible for the present crisis in the jute
industry in India?
1. The decline in overseas market
2. Inadequate supply of raw jute
3. Stiff competition from synthetic
packing materials
Select the correct answer from the codes
given below
a. 1 and 3
b. 1, 2 and 3
c. 1 and 2
d. 2 and 3
83.
Lahaul and Spiti district in H.P. has the
highest
a. agricultural density of population
b. arithemetic density of population
c. physiological density of population
d. rural density of population
84.
Which one of the following is most
plausible
solution
for
planning
improvement of a metropolitan city in
India?
a. Constructing high rise building and
skyscrapers
b. Building up suburbs and satellite towns
around the city
c. Dispersing factories and industries to
the neighboring but independent
locations
d. Preventing immigration of people into
the metropolitan city
The next 17 items consist of two statements,
one labelled as 'assertion A' and the other
labelled as 'Reason R*. You are to examine
these two statements carefully and decide if
the Assertion A and the Reason R are
individually true and if so, whether the
Reason is a correct explanation of the
Assertion. Select your answer to these items
using the codes given below and mark your
answer sheet accordingly.

85.

86.

87.

88.

89.

8 of 11
Assertion (A): A Karst topography is
characterized by the formation of
stalactites and stalagmites.
Reason (R): Solution is a dominant
process in the development of landforms
in Karst region.
a. Both A and R are true and R is the
correct explanation of A
b. Both A and R are true but R is NOT a
correct explanation of A
c. A is true but R is false
d. A is false but R is true
Assertion (A): Physiological density is
preferable to arithmetic density as an index
of population density
Reason (R): Physiological density is based
on arable land while arithmetic density is
based on total area.
a. Both A and R are true and R is the
correct explanation of A
b. Both A and R are true but R is NOT a
correct explanation of A
c. A is true but R is false
d. A is false but R is true
Assertion (A): The late mature stage is the
last stage of city development according to
Taylor.
Reason (R): Taylor has suggested seven
stages in the development of an urban
centre.
a. Both A and R are true and R is the
correct explanation of A
b. Both A and R are true but R is NOT a
correct explanation of A
c. A is true but R is false
d. A is false but R is true
Assertion (A): On the west side of very
continent between latitude 20 and 30
intensely dry desert borders the ocean
Reason (R): The cold current flows near
the western coast of every continent
a. Both A and R are true and R is the
correct explanation of A
b. Both A and R are true but R is NOT a
correct explanation of A
c. A is true but R is false
d. A is false but R is true
Assertion (A): Loess is a find grained
yellow deposit.
Reason (R): It originated as wind blown
dust from desert surface.
a. Both A and R are true and R is the
correct explanation of A

90.

91.

92.

93.

94.

b. Both A and R are true but R is NOT a


correct explanation of A
c. A is true but R is false
d. A is false but R is true
Assertion (A): Saline soils are commonly
formed in poorly drained locations such as
low lying valley floors and basins.
Reason (R): In poorly drained locations,
the surface runoff (Water) containing
dissolved salts gets totally evaporated.
a. Both A and R are true and R is the
correct explanation of A
b. Both A and R are true but R is NOT a
correct explanation of A
c. A is true but R is false
d. A is false but R is true
Assertion (A): The equatorial forest in the
world is dense and impenetrable with
many types of plants.
Reason (R): The enormous growth of
vegetation in this region is due to
extremely warm and humid climate.
a. Both A and R are true and R is the
correct explanation of A
b. Both A and R are true but R is NOT a
correct explanation of A
c. A is true but R is false
d. A is false but R is true
Assertion (A): The people who depend
upon tropical crops suffer from
malnutrition
Reason (R): The tropical crops are rich in
carbohydrate
a. Both A and R are true and R is the
correct explanation of A
b. Both A and R are true but R is NOT a
correct explanation of A
c. A is true but R is false
d. A is false but R is true
Assertion (A): Brock has included South
African the occidental culture realm.
Reason (R): The British and the Dutch
have superimposed western culture on
South Africa.
a. Both A and R are true and R is the
correct explanation of A
b. Both A and R are true but R is NOT a
correct explanation of A
c. A is true but R is false
d. A is false but R is true
Assertion (A): Cotton is graded according
to staples.
Reason (R): The staples mean the degree
of whiteness of threads.

95.

96.

97.

98.

99.

9 of 11
a. Both A and R are true and R is the
correct explanation of A
b. Both A and R are true but R is NOT a
correct explanation of A
c. A is true but R is false
d. A is false but R is true
Assertion (A): Commercial fisheries have
not developed in the tropics.
Reason (R): The demand for marine food
from low income population is low in the
tropics
a. Both A and R are true and R is the
correct explanation of A
b. Both A and R are true but R is NOT a
correct explanation of A
c. A is true but R is false
d. A is false but R is true
Assertion (A): Paper industry is located
mainly along the southern edge of the
coniferous forest belt in North America
Reason (R): These areas have abundant
raw materials and water power.
a. Both A and R are true and R is the
correct explanation of A
b. Both A and R are true but R is NOT a
correct explanation of A
c. A is true but R is false
d. A is false but R is true
Assertion (A): Africa has immense natural
resources andyst it is industrially the most
backward continent.
Reason (R): The continent had been under
colonial rule for a long time.
a. Both A and R are true and R is the
correct explanation of A
b. Both A and R are true but R is NOT a
correct explanation of A
c. A is true but R is false
d. A is false but R is true
Assertion (A): The Rajasthan desert is
covered with shifting sand dunes.
Reason (R): For want of rain and surface
flowing of water, the work of wind is felt
every where.
a. Both A and R are true and R is the
correct explanation of A
b. Both A and R are true but R is NOT a
correct explanation of A
c. A is true but R is false
d. A is false but R is true
Assertion (A): The demand of potash
fertilizers in India is met by indigenous
production.

100.

101.

102.

103.

104.

Reason (R): India's requirements of potash


fertilizers have been very small.
a. Both A and R are true and R is the
correct explanation of A
b. Both A and R are true but R is NOT a
correct explanation of A
c. A is true but R is false
d. A is false but R is true
Assertion (A): India's cement producing
capacity is concentrated in Madhya
Pradesh,
Gujarat,
Tamil
Nadu,
Maharashtra and Karnataka.
Reason (R): Madhya Pradesh, Gujarat,
Tamil Nadu, Maharashtra and Karnataka
have bulk of the limestone resources of
India.
a. Both A and R are true and R is the
correct explanation of A
b. Both A and R are true but R is NOT a
correct explanation of A
c. A is true but R is false
d. A is false but R is true
Assertion (A): Sex ratio in Kerala,
southern most part of Tamil Nadu, north
eastern Madhya Pradesh and some parts of
UP, is above 1000,
Reason (R): The death rate among the
males is high in the said regions.
a. Both A and R are true and R is the
correct explanation of A
b. Both A and R are true but R is NOT a
correct explanation of A
c. A is true but R is false
d. A is false but R is true
The presence of incised meanders
indicates that the area has experienced
a. ecstatic changes
b. glaciations in the past
c. rejuvenation
d. faulting
Which one of the following sequences of
types of clouds would one meet, 500
meters above mean sea level?
a. Stratus,
Nimbus,
Cirro-stratus,
Cirrocumulus
b. Nimbus, Cirro-stratus, Cirrocumulus,
Stratus
c. Cirrocumulus, Stratus, Nimbus, Cirronimbus
d. Stratus,
Cirro-stratus,
Nimbus,
Cirrocumulus
Match List I with List II and select the
correct answer by using the codes given
below the lists.

105.

106.

107.

108.

109.

10 of 11
List I (Ocean Ridge)
A. Wyville thompson Ridge
B. East Jan Mayer Ridge
C. New Zealand Ridge
D. Chagos-St Paul Ridge
List II (Ocean)
1. Indian Ocean
2. Pacific Ocean
3. Arctic Ocean
4. Antarctic Ocean
Codes;
A
B
C
D
a.
1
2
3
5
b.
1
3
4
2
c.
4
5
2
3
d.
4
3
2
1
The area marked X in the sketch map
belongs to the tribe called

a. Bushman
b. Pygmy
c. Masai
d. Bedouin
The process of gradual spread over space
of people or ideas from the centre of origin
is known as
a. Distribution
b. Diffusion
c. Innovation
d. Communication
Which one of the following indicates the
principle of marketing in Central Place
Theory?
a. K3
b. K5
c. K7
d. K9
The most important producer of nickel in
the world is
a. Russia
b. Canada
c. Brazil
d. Venezuela
The area marked X in the given map
represents in important region for the
production of

110.

a. rubber
b. olives
c. oil plam
d. tobacco
Agricultural production in the area marked
as 'X' in the given Von-Thunes diagram
represents

116.

117.

118.

111.

112.

113.

114.

115.

a. grain farming fellow and pasture


b. grain farming without fellow
c. livestock farming
d. firewood and timber production
The antipodes of a place situated 25 S and
80 45' 45" W is
a. 25 N and 99 l4' 15" E
b. 2 7475"
c. 99 14' 15" E
d. 65 N and 29 74' 75" E
A telegram was sent from Greenwich at 12
noon. The time for transmission was 12
minutes. It reached a town at 6 P.M. The
longitude of the town is
a. 97 E
b. 87 E
c. 87 W
d. 97 W
The projection most suitable for showing
ocean routes is
a. Sinusoidal,
b. Polar Zenithal
c. Mollweide's
d. Mercator's
The central meridian indicated by a line
half the length of the equator represents
a. Mercator projection
b. Polyconic projection
c. Mollweide's projection
d. Bonne's projection
Which one of the following pair is
correctly matched?

119.

120.

11 of 11
a. Spatial diffusion Theory ..........
Hagerstrand
b. Landscape theory ................... Chorley
c. Trade Theory .............. ........ King
d. Transportation
Coast
Theory....
.............. . Smith
The traditional agricultural classification
as presented by D. Whittlesey was based
on four major dimensions of variationintensity, commerciality, mobility and
a. marketability
b. mechanisation
c. crop speciality
d. crop productivity
The concept of 'Heartland' was
propounded by
a. Hartshorne
b. Haushofer
c. Spykman
d. Mackinder
Match List 1 and List II and select the
answer using the codes given below the
lists :
List I
A. Jen Brunhes
B. O.H.K. Spate
C. G. Tylor
D. Friedrick Ratzel
List II
1. Determinism
2. Neo-detenninism
3. Possibilism
4. Prababilism
Codes:
A
B
C
D
a.
1
2
3
4
b.
1
3
2
4
c.
3
4
2
1
d.
3
4
1
2
The mantle rock dropped by the ice is
commonly referred to as
a. alluvium
b. glacial drift
c. iceberg
d. esker
In the given figure X represents

a.
b.
c.
d.

symmetrical fold
asymmetrical fold
isoclinal fold
recumbent fold

C.S.E-Pre 1995

1 of 12

GEOGRAPHY
1.

2.

3.

4.

5.

A majority of the populations of


Newfoundland are engaged in
a. lumbering
b. fishing
c. hunting
d. dairying
Consider the following statements :
Decline of world fish haul is due to
changing food habit over fishing
1. availably of cheep
2. changing food habit
3. indiscriminate fishing
4. overfishing
5. water pollution
a. 1, 2 and 3 are correct
b. 2, 3 and 4 are correct
c. 1, 4 and 5 are correct
d. 3, 4 and 5 are correct
The most important reason for dwining
forest resources today is
a. soil erosion
b. forest fire
c. floods
d. over felling
Aluminium, one of the metals more
abundant in the crust of the earth, came
into use of man only by the end of 19th
century when.
a. its use in different industries was fully
realised
b. three stages of production became
known
c. electrolytic reduction of alumina to
metallic aluminium was discovered
d. comparative hardness and tensile
strength was fully realised
Match List f with II and select the correct
answer.
List I
A. Basic inexhaustible
B. Conventional renewable source of
energy
C. Non-Conventional renewable source
of energy

6.

7.

8.

D. Non-Conventional nonrenewable
Source of energy
List II
1. Hydel Power
2. Tidal
3. Solar energy
4. Natural gas
Codes:
A
B
C
D
a.
3
2
1
4
b.
2
3
4
1
c.
2
1
3
4
d.
3
1
2
4
Match List I with List II and select the
correct answer.
List I: Mining Area
A. Donbas
B. Krivoi Rog
C. Globe-Miami
D. Lohardaga
List II: Mineral
1. Iron ore
2. Copper
3. Bauxite
4. Coal
Codes:
A
B
C
D
a.
4
2
1
3
b.
1
4
3
2
c.
4
1
2
3
d.
1
4
2
3
Which one of the following pairs is NOT
correctly matched ?
a. Lorraine . France
b. Ruhr

Westphalia
.
Germany
c. Pittshurg - Lake Erie
U.S.A.
d. Ise Wan Bay . Japan
Match List I with List II and select the
correct answer
List I: Industry
A. Iron and Steel
B. Automobile

9.

10.

11.

12.

13.

C. Ship building
D. Woollen Textile
List II: Place of Manufacture
1. Leeds
2. Kure
3. Atlanta
4. Birmingham
A
B
C
D
a.
4
3
1
2
b.
1
3
4
2
c.
1
2
3
4
d.
4
3
2
1
The greatest and the most productive
industrial region in Asia is ;
a. Southern Honshu
b. Bombay
c. Shanghai region
d. Singapore
The largest number of tribal groups live in
a. Uganda
b. Tanzania
c. South Africa
d. Nigeria
Ethiopia lies in the
a. Horn of Africa
b. Sahel of Africa
c. Cape of Africa
d. Central highlands of Africa
Consider the following statements :
The term Barbary States' denotes
1. the region comparising Morocco,
Tunisia and the northern part of
Algeria.
2. the original population of the region
belonging to the Semitic group known
as the Barbers
3. the irrigated crops which account for a
large part of the value of all
agricultural exports
a. 1, 2 and 3 are correct
b. 1 and 2 are correct
c. 2 and 3 are correct
d. 1 and 3 are correct
Karkatoa, the most famous volcano of
Indonesia, is located between
a. Sumatra and Java
b. Sumatra and Malaysia
c. Java and Borneo
d. Bali and Lombok

14.

15.

16.

17.

18.

19.

20.

2 of 12
Most part of Central Africa is forested
because
a. most of the countries are thinly
populated
b. of the rugged terrain
c. the sub-Saharan soil is most suitable
for forest growth
d. equator passes through the meddle of
the region
The geographic unity among the SouthEast Asia countries is maintained by
a. climate
b. culture
c. economy
d. topography
The major significance of South West Asia
lies in its
a. command over land routes
b. most picturesque desert landscape
c. enormous oil wealth
d. holy pilgrimage centres of Christians,
Muslims and Jews
Oil refineries-in South West Asia are
indicated
a. at the industrial centres
b. in the interior
c. in or near the oilfields
d. at posts of exports
he contrast in economic development
between Israel and the other South West
Asian countries is due to
a. desertic conditions of other countries,
b. mineral wealth of Israel
c. irrigational facilities available in Israel
d. application of high technology in Israel
The largest producer of fruits in the AngloAmerican Region is
a. New-England Region
b. Great Lakes Region
c. California Region
d. Appalachian Region
The
Appalachian
mountains
are
economically important because they
provide
a. a store-house of timber and fuel wood
b. natural passes, easily negotiable by
modern means of transport
c. mineral wealth forming the basis of
key industries
d. attracting tourists both in summer and
winter.

21.

22.

23.

24.

25.

26.

27.

Match List I with List II and select the


correct answer
List I: Producers
A. Baku
B. Kuzbas Basin
C. Urals
D. Eastern Siberia
List II: Products
1. Diamond
2. Iron Ore
3. Coal
4. Oil
Codes; A
B
C
D
a.
4
3
2
1
b.
2
3
4
1
c.
1
2
3
4
d.
4
3
1
2
Tea is grown in
a. Ukraine
b. Georgia
c. Kazakhstan
d. Uzbekistan
The largest industrial region of Russia is at
a. Irkutsk
b. Moscow-Gorki
c. Tomsk
d. Karaganda'
The provinces of Hunan and Chang-Jiang
of the Southern region in China, is known
as the
a. rich-bowl
b. region of sorrow
c. playground of China
d. industrial care
The loess uplands in China are largely
devoid of forests due to
a. agricultural expansion
b. arid climate
c. excessive exploitation of forests
d. over-grazing
The loess uplands in China are largely
devoid of forests dus to
a. agriculture expansion
b. arid climate
c. excessive exploitation of forests
d. over-grazing
Which one of the following pairs is NOT
correctly matched ?
a. Himalayas .. Tertiary Fold
Mountain

3 of 12
b. Deccan Trap..Volcanic Cone
eruption
c. Western GhatPalaeozoic fold
mountains
d. Aravalli .Pre-Cambrian relict
mountain
28.
he expression of earliest volcanic activity
in India is found in
a. Dalma hill of Bihar
b. North Western Deccan Plateau
c. Buldana District of Maharashtra
d. Kumaon Hill region
29.
The El Nino which influences the Indian
monsoon is
a. a cold ocean current that flows
northward along the coast f Chile
b. a warm ocean current that flows westwards along the coast of Ecuador and
Peru
c. a low pressure system over western
coast of Spain
d. none of the above
30.
Consider the following statements : The
Black Cotton soil of India
1. is mainly distributed over the river
valleys of the lava region of the
Deccan
2. has a great capacity of retaining
moisture
3. is extremely fertile at surface
4. occupies about 10% the total area of
the country
Of these statements
a. 1,2 and 3 are correct
b. 1,3 and 4 are correct
c. 1 and 2 are correct
d. 3 and 4 are correct
31.
The term 'Regur' refers to
a. Deltaic alluvial soils
b. Laterite soils
c. Black cotton soils
d. Red and yellow soils
32.
Which one of the following pairs is NOT
correctly matched?
a. laterite . Uttar Pradesh
b. Sandy Soil........ South Punjab
c. Red Soil... Tamil Nadu
d. Black Soil.... Madhya Pradesh
The following 2 questions pertain to the given
map:

33.

34.

35.

36.

37.

The type of forests found in the shaded


region of the map is

a. Thorn and scrub forests


b. Monsoon forests
c. Tropical rain forests
d. Deciduous forests
The forest found in the map marked 'A' is
a. Tropical dry deciduous forest
b. Tropical thorn forest
c. Tropical dry evergreen forest
d. Littoral and swamp
A person overlying India saw the natural
vegetation below in the sequential order of
tropical evergreen forest, savanna, dry
deciducous and deciduous. His flight was
from
a. Calcutta to Delhi
b. Bombay to Bhubaneshwar
c. Trivandrum to Delhi
d. Delhi to Madurai
Match List 1 with II and select the correct
answer.
List I
A. Sirhind canal
B. East Yamuna canal
C. Indira Gandhi canal
D. Wainganga canal
List II
1. U.P.
2. M.P.
3. Punab
4. Rajasthan
A
B
C
D
a.
1
2
3
4
b.
3
2
4
1
c.
3
1
4
2
d.
4
2
1
3
Which one of the following problems
NOT encountered in Indira Gandhi Canal
command area ?
a. Increase in soil salinity
b. Progressive waterlogging
c. Aeolian silting of the canal
d. Decline in water supply

38.

39.

40.

41.

42.

4 of 12
Which of the fallowing statements are
related to the favourable conditions for the
growth of tobacco?
1. It requires a moderately high
temperature
2. It cannot be grown 1000 m above sea
level
3. It requires a heavy average annual
rainfall
4. Red soil is suitable for its growing
Select the correct answer using the codes
given below
a. 3 and 4
b. 2 and 4
c. 1 and 4
d. 1 and 3
Doon vally is able to grow rice because
a. it has warm summers and snow melts
waters for irrigation
b. people in the valley are rice eaters
c. other crops cannot be grown
d. there is a huge export demand
The given map with shaded portion shows

a. Area affected by gully erosion


b. Distribution of sugarcane
c. Area of deciduous forest
d. Rice growing area
Match List I with List II and select the
correct answer:
List I: Name of the crops
A. Rice
B. Jowar
C. Jute
D. Coffee
List II: States Grown
1. Maharashtra
2. Orissa
3. Karnataka
4. West Bengal
A
B
C
D
a.
1
2
3
4
b.
2
3
4
1
c.
3
4
1
2
d.
2
1
4
3
The given map with shaded region shows:

5 of 12

43.

44.
45.

46.

47.

48.

a. Sugarcane growing areas


b. Cotton growing areas
c. Area having high density of population
d. Areas of high industrial development
"A mean monthly temperature of about
27C, relatively high humidity and a
rainfall of 175-200 cm during spring and
summer are the physical requirements
during the period of its vegetative growth".
Which one of the following crops is
referred to in this quotation?
a. Khair (acacia catechu)
b. Teak
c. Deodar
d. Rosewood
The proportion of forest to the total
national geographical area of India as
envisaged by National Forest Policy, is
a. 30.3 percent
b. 33.3 percent
c. 38.3 percent
d. 42.3 percent
Which one of the following sectors of the
economy is state-owned and its service
sector is a public enterprise?
a. Irrigation
b. Agriculture
c. Forests
d. Plantation
The State which records the highest
productivity of fish (per unit area) in India,
is
a. Kerala
b. Tamilnadu
c. Orissa
d. West Bengal
The shaded region in the give map relates
to

49.

50.

51.

52.

53.

a. Iron ore producing regions


b. Mineral bearing regions
c. Regions deficient in nonmetallic
minerals
d. Regions of mica mining
The main ore from which manganese is
extracted in India is
a. siderite
b. chalcopyrite
c. pyrolusite
d. covariate
Coal is produced in the states of
1. Bihar
2. West Bengal
3. Orissa
4. Andhra Pradesh
Select the correct sequence as per their
production from maximum to minimum.
a. 1, 2, 3, 4
b. 2, 1, 3, 4
c. 3, 4, 1, 2
d. 4, 3, 1, 2
Consider the Hydro-electric power
generation in the following states :
1. Maharashtra
2. Tamilnadu
3. Uttar Pradesh
4. Kerala
The correct sequence of installed hydroelectric power capacity in DECREASING
order will be
a. 1,2,4,3
b. 2, 1, 3,4
c. 3, 2, 1, 4
d. 3, 4, 2, 1
Nuclear power plant is associated with
a. Madras
b. Kota
c. Ganganagar
d. Mathura
The location of Iron & Steel mills in India
is determined in the order of availability
factor of

54.

55.

56.

57.

a. Iron ore, Coking coal, Limestone and


Managese
b. Coking coal, Iron ore, Limestone and
Managenese
c. Iron ore, Managenese, Limestone
and,Coking coal
d. Coking coal, Managanese, Iron ore and
Limestone
The biggest ship-building yeard of India is
a. Garden Reach Workshop, Calcutta.
b. Hindustan Shipyard, Vishakhapatnam
c. Mazagaon Dock. Bombay
d. Cochin Shipyard, Kochi
Modern industrial development of India is
largely due to
a. rationalisation of raw materials
b. availability of skilled manpower
c. liberalization of government policies
d. increase in market demand
In recent years the ratio between the tribal
and non-tribal population has been
adversely affected in Tripura. This is
because of
a. high death rate among the tribes due to
lack of medical and sanitation facilities
b. mass scale migration of tribes from
Tripura due to unemployment and
death of economic resources
c. immigration of large number of nontrial population from Bangladesh
d. high birth rate among the non-tribes in
comparison to the tribes
Occupation pattern of population in India
(in percentage of total workers)
Year
Primary Secondary Tertiary
1951
74.40
10.56
15.04
1961
76.44
10.96
12.60
1971
72.00
12.00
16.00
1981
69.00
13.00
18.00
From the given table it can be generalized
that
a. the occupation pattern in India has
remained more or less the same
b. the occupation pattern in India has
significantly changed
c. the dependence of workers on primary
sector has been reduced
d. the dependence of workers on tertiary
sector has been reduced

6 of 12
In India, while defining an urban area, the
most important consideration besides the
size of its population, is that it should have
a. half of its male working population in
non-primary sector
b. a police station, a telegraph office and
a telephone facility
c. a police station, a college and a
hospital
d. three-fourths of its male working
population in non-agricultural pursuits
59.
The most important cement producing
states in India are
1. Andhra Pradesh
2. Rajasthan
3. Madhya Pradesh
4. Tamil Nadu
Select the correct sequence in order of
DECREASING importance in the
production of cement,
a. 1, 2, 3, 4
b. 3, 1, 4, 2
c. 2, 3, 4, 1
d. 4, 2, 1,3
The next 11 items consist of two statements,
one labelled the 'Assertion (A)' and the other
labelled the 'Reason (R)', You are to examine
these two statements carefully and decide if the
Assertion (A) and the Reason (R) are
individually true and if so, whether the reason
is a correct explanation of the Assertion. Select
your answers from the codes given below.
60.
Assertion (A): In kerala the density of
human settlement decreases from south to
north along the coast.
Reason (R): The width of the coastal plain
decreases from south to north.
a. Both A and R are true and R is the
correct explanation of A
b. Both A and R are true but R is NOT a
correct explanation of A
c. A is true, but R is false
d. A is false, but R is true
61.
Assertion (A): The level of urbanization is
the hallmark of development.
Reason (R): Increasing urbanization means
phenomenal increase in the concentration
of human population in a limited space.
a. Both A and R are true and R is the
correct explanation of A
b. Both A and R are true but R is NOT a
correct explanation of A

58.

62.

63.

64.

65.

66.

c. A is true, but R is false


d. A is false, but R is true
Assertion (A): The one who led the
development of the new geography in
France was Paul Vidal de la Blache.
Reason (R): He found his way into
geography through his study of ancient
history and classical literature.
a. Both A and R are true and R is the
correct explanation of A
b. Both A and R are true but R is NOT a
correct explanation of A
c. A is true, but R is false
d. A is false, but R is true
Assertion (A): The population of North
America increased by more than six times
in the last one hundred years.
Reason (R): North America had
experienced
industrialization
and
immigration.
a. Both A and R are true and R is the
correct explanation of A
b. Both A and R are true but R is NOT a
correct explanation of A
c. A is true, but R is false
d. A is false, but R is true
Assertion (A): Population is heavily
concentrated in some low lands.
Reason (R): These lands could produce
large returns with minimum equipments.
a. Both A and R are true and R is the
correct explanation of A
b. Both A and R are true but R is NOT a
correct explanation of A
c. A is true, but R is false
d. A is false, but R is true
Assertion (A): Gulf Coast of North
America experiences cool winters
Reason (R): Gulf Cost of North America is
situated at a higher latitude.
a. Both A and R are true and R is the
correct explanation of A
b. Both A and R are true but R is NOT a
correct explanation of A
c. A is true, but R is false
d. A is false, but R is true
Assertion (A): Irrigation is an essential
input to revitalize Indian agriculture.
Reason (R): Two thirds of the farm lands
of India are located in semi arid regions.

67.

68.

69.

70.

71.

7 of 12
a. Both A and R are true and R is the
correct explanation of A
b. Both A and R are true but R is NOT a
correct explanation of A
c. A is true, but R is false
d. A is false, but R is true
Assertion (R): Fish export from India is
progressively increasing.
Reason (R): Internal consumption of fish
in India is decreasing.
a. Both A and R are true and R is the
correct explanation of A
b. Both A and R are true but R is NOT a
correct explanation of A
c. A is true, but R is false
d. A is false, but R is true
Assertion (A): Off-shore fishing in
Malabar coast is more developed than that
in Coriander coast.
Reason (R): Malabar coast has better
facilities to export sea fish to African
countries.
a. Both A and R are true and R is the
correct explanation of A
b. Both A and R are true but R is NOT a
correct explanation of A
c. A is true, but R is false
d. A is false, but R is true
Assertion (A): Rihand power plant is
located where rainfall is limited and
seasonal.
Reason (R): Hydel Power plants are found
on the hills with heavy rainfall.
a. Both A and R are true and R is the
correct explanation of A
b. Both A and R are true but R is NOT a
correct explanation of A
c. A is true, but R is false
d. A is false, but R is true
Assertion (A): Aluminium industry is
located near the source of power.
Reason (R): Industries are usually located
close to the source of raw materials.
a. Both A and R are true and R is the
correct explanation of A
b. Both A and R are true but R is NOT a
correct explanation of A
c. A is true, but R is false
d. A is false, but R is true
Match List I with List II select the correct
answer.

72.

73.

74.

75.

List I: Composition
A. Quartz and felds par
B. Quartz, feldspar and mica
C. Compacted volcanic ash
D. Feldspar, mica, pyroxenes and olivine
List II Rock type
1. Basalt
2. Tuff
3. Granite
4. Sandstone
A
B
C
D
a.
4
1
2
3
b.
1
2
3
4
c.
3
2
1
4
d.
4
3
2
1
Mohorovicic discontinuity separates
a. core and mantle
b. crust and mantle
c. inner core .and outer core
d. sima and nifs
The profile of equilibrium refrs to the
a. Profile of a glaciated valley
b. transverse profile of a graded stream
c. longitudinal profile of a graded stream
d. profile along the shoreline
Relative humidity refers to
a. absolute amount of water vapour in the
atmosphere
b. the amount of hygroscopic nuclei in
the atmosphere
c. saturated vapour pressure
d. ratio of actual amount of water vapour
in the air to the amount it could hold at
that temperature.
The area marked X in the given diagram
representingioncen-tric zone theory of
urban land use is called :

77.

78.

79.

80.

76.

a. Central Business District


b. Low Class Residential
c. Medium class Residential
d. High Class Residential
An empirical method used to describe the
city-size distribution of many countries
and regions, is called.
a. urban hierarchy
b. rank-size rule

8 of 12
c. central place model
d. urban location model
Match List I with List II and select the
correct answer
List I: Authors
A. P. Gould
B. P. Haggett
C. EC. Semple
D. E. Huntington
List II: Books
1. Mental Maps
2. Influences of Geographic Environment
3. Mainsprings of Civilization
4. Lavational Analysis in Human
Geography
A
B
C
D
a. 1 4
2
3
b. 1 4
3
2
c. 4 3
2
1
d. 2 3
1
4
Chris taller postulated his central place
theory which does NOTrefer to
a. K = 7
b. K = 4
c. K = 3
d. K = 1
Which one of the following statements is
correct?
a. All the abiotic resources are nonrenewable and all the biotic resources
are renewable
b. All the abiotic resources are nonrenewable but some of the biotic
resources are non-renewable.
c. Some of the abiotic resources are
renewable but the biotic resources are
renewable.
d. Some of the abiotic resources are
renewable and some of the biotic
resources are non-renewable.
Which one of the following statements is
correct?
a. While most soft woods are deciduous,
most hardwoods are evergreen.
b. The trees of equatorial evergreen
forests are mostly of softwood type.
c. While most softwoods occur in the
tropical areas, the hardwoods are
characteristically of temperate region.

81.

82.

83.

84.

85.

d. While most hardwoods are either


deciduous or evergreen, softwood is
usually coniferous in characteristics.
Match List I with List II and select the
correct answer
List I: Canal
A. Suez Canal
B. Panama Canal
C. Soo and Wetland Canal
D. Mittelland Canal
List II: Areas Joining
1. Great Lakesh and St.Lawrence.
2. Mediterranean and Red Sea
3. Pacific and Atlantic
4. North Sea and North European Plan:
A
B
C
D
a.
1
2
3
4
b.
1
3
2
4
c.
2
3
1
4
d.
2
1
4
3
Rhumb line is a
a. line on the earth's surface which cuts
all meridians at the same angle
b. technical name for the international
date line
c. line connecting the Greenwich
meridian with the international date
line
d. line which gives correct distance on
Mercator's projection
For drawing the map of the world Ptolemy
apparently used a modified form of
a. conical projection
b. azimuthally equidistant projection
c. Mercator's projection
d. cylindrical equal area projection
Which one of the following is NOT
common between cylindrical equal area
and Mollweide's map projection?
a. Parallels are closer towards the poles
b. Parallels are straight lines
c. Meridians are curves
d. Area is shown correctly
If the map drawn to a scale of 1 cm to 30
km is to be enlarged three times, then
representative fraction of the enlarged map
will be
a. 1 :1,000,000
b. 1 :3,000,000
c. 1 :6,000,000
d. 1: 9,000,000

86.

87.

88.

89.

90.

91.

92.

9 of 12
Which one of the following forms the
subject matter of Radical Geography?
a. Socio-economic inequalities of man
b. Geo-politics in different areas.
c. Disparities in industrial development.
d. Agricultural problems
Which one of the following statements
applies to the Arab Geographers ?
a. They developed various concepts of
geography
b. Their main contribution to Geography
was in the field of measurement based
on astronomical observations.
c. They laid the foundation of modern
geography
d. They did not place importance to
writing travel narratives.
Which one of the following paris is NOT
correctly matched ?
a. Jafferson . Primate city
b. Monte Carlo .. Simulation model
c. Penck... Treppean concept
d. Strahler...Waxing slope
Which one of the following sets indicates
the aeolian process?
a. Corosion, transportation, deposition
b. Nivation, down-cutting, solifluction
c. Corrosion, attrition, deflation
d. Ablation, deposition, oxidation
The areas marked 'X on the given map are
the plateaus of the type known as

a. Coastal
b. Continental
c. Intermundane
d. Piedmont
The rivers of Africa are as large as those in
North or South America, but they do not
attract much transport because
a. they pass through dense forest
b. the people there are not commercial
minded
c. their courses are broken by cataracts
and waterfalls
d. they traverse through resource poor
area
Consider the following peaks :

93.

94.

95.

96.

1. Dhaulagiri
2. Kanchenjunga
3. Nanda-Devi
4. Kamet Peak
The correct sequence of the peaks from
West to East will be
a. 3, 1, 4, 2
b. 3, 4, 1, 2
c. 1, 3, 2, 4
d. 4,3, 1,2
Match List I with List II and select the
correct answer.
List I
A. S.W. Monsoon
B. Westerlies
C. S.E. Monsoon
D. Doldrums
List II
1. Pacific coast of Canada
2. India
3. Indonesia
4. China
A
B
C
D
a.
2
1
4
3
b.
1
2
4
3
c.
2
3
1
4
d.
4
2
1
3
Mean monthly temperature means
a. the sum of daily temperature in a
month
b. The sum of daily temperature in a year
divided by 12
c. the sum of daily mean temperature in a
month divided by the number of days
in that month
d. the sum of daily noon temperature
divided by the number of days in that
month
The cyclones of temperate latitudes differ
from those of the tropics in a number of
ways. Which one of the following is the
most prominent of them all?
a. Their size is far greater
b. They are caused by frontal systems
whereas the tropical cyclones are not
c. These occur more frequently
d. The precipitation is more evenly
distributed in a temperate cyclone
Which one of the following pairs is
correctly matched according to Koppen's
scheme?

97.

98.

99.

100.

101.

10 of 12
a. Cs Summer rain
b. Af rain throughout the year
c. Et winter rain
d. Df summer snow
Match List I with List II and select the
correct answer
List I Soil Type
A. Chernozems
B. Laterite
C. Podzol
D. Alluvia!
List II: Region
1. Eastern Romania
2. Northern plains of India
3. Central Africa
4. Eastern Russia
A
B
C
D
a.
1
3
4
2
b.
4
3
1
2
c.
1
4
3
2
d.
4
3
2
1
Guano is a unique fertilizer product of
a. Atacama
b. Grand Banks
c. Paraguay
d. Coastal areas and islands off Peru
The major amount of wheat, in Ukraine is
produced in area of
a. Acidic soil
b. Podzol soil
c. Black Earth (soil)
d. Red soil
The vegetation which survives on aerial
roots is
a. mangrove
b. selvas
c. catinga,
d. taiga
Which one of the following statements,
NOT true of coniferous forests ?
a. Spruce is the important tree of this
jungle which is used in preparing pulp
and artificial silk
b. They are evergreen forests of soft
wood trees with short and pointed
leaves
c. Such jungles are also called 'boreal'
d. Tree trunks of this region are quite
thick and are covered with thick barks

102.

103.

104.

105.

106.

107.

so as to protect them from the dryness


of the summer season.
Match List I with List II and select the
correct answer
List I: Altitudinal location of Mt.
Kilimajaro
A. Higher slopes
B. Middle slopes
C. lower slopes
D. Foothill zone
List II : Vegetation Type
1. Savannah
2. Forest
3. Dry grassland
4. Alpine grasses
A
B
C
D
a.
2
1
4
3
b.
3
4
2
1
c.
4
2
1
3
d.
4
1
3
2
A natural region has the similarity of
a. climate-and natural vegetation
b. climate and occupation
c. soil and drainage
d. economic base and natural vegetation
The natural region produced by the
creation of climatic homogeneity by the
sea is
a. Humid subtropics
b. Marine west coast
c. Monsoon tropics
d. Short summer humid continental.
Fishing, hunting and food gathering
activities are carried out by
a. Semangs of Malaysia
b. Masais of East Africa
c. Bedouins of Arabia
d. Kirghiz of Central Asia
The region with grasslands, extensive
rolling plains, dull uniformity and
monotony of scene, no trees, extreme cold
and dryness in winter, can be described as
having,
a. Steppe type of climate
b. Tundra type of climate
c. China type of climate
d. Warm Temperate Eastern Margin type
of climate
Approximately one-half of the world
population lives in

108.

109.

110.

111.

112.

113.

11 of 12
a. East Asia and Europe
b. Europe and Eastern-north America
c. South and South-east Asia
d. East and South Asia
The correct sequence in the descending
order with respect to present total
population of the given countries is
a. Columbia, Brazil, U.S.A., Mexico
b. Mexico, U.S.A., Columbia, Brazil
c. U.S.A., Brazil, Mexico, Columbia
d. Brazil, U.S.A., Mexico, Ccolumbia
The following Gulf countries have Indian
Communities:
1. Saudi Arabia
2. Oman
3. Kuwait
4. Iraq
The correct sequence in DESCENDING
order of the size of Indian community, as
per 1990 figures, will be
a. Saudi Arabia, Oman, Kuwait, Iraq
b. Oman, Kuwait, Iraq, Squdi Arabia
c. Iraq, Saudi Arabia, Kuwait, Oman
d. Kuwait, Iraq, Oman, Saudi Arabia
South-West Asia is inhabited chiefly by
a. Caucasians
b. Mongoloids
c. Negroes
d. Red Indians
Consider the following statements :
Caucasoid race is associated with
1. long, wavy sot hair
2. narrow to medium broad face
3. very broad, flat broad to very broad
nose
Of these statements
a. 1,2 and 3 are correct
b. 1, and 2 are correct
c. 2 and 3 are correct
d. 1 and 3 are correct
The International migrations, of population
in Eurasia in the late forties after the
Second World War were caused by
a. natural calamities such as floods and
famines
b. favourable immigration laws
c. religious fanaticism
d. stabilized international situation
Match List I with List II and select the
correct answer

114.

115.

List I: Climate
A. Arctic
B. Humid Temperature
C. Steppes
D. Tropical Rainforest
List II: House Type
1. Brick and stone house
2. House on hill slope
3. Hut on tree
4. Igloo
5. Mud house
A
B
C
D
a.
1
5
4
3
b.
4
1
5
3
c.
2
4
3
1
d.
4
2
1
5
Match List I with List II and select the
correct answer:
List I: Languages
A. Flemish
B. Basque
C. Ainu
D. Swahili
List II: Country
1. North Japan
2. Kanya
3. North Eastern Spain
4. Belgium
A
B
C
D
a.
4
2
1
3
b.
2
3
4
1
c.
1
4
3
2
d.
4
3
1
2
Match List I with List II and select the
correct answer:
List I: Scholars
A. Weber
B. Von Thunen
C. Christaller
D. Malthus
List: Theories
1. Population Theory
2. Theory of industrial location
3. Agricultural location
4. Central Place Theory
A
B
C
D
a.
1
3
2
4
b.
2
4
3
1
c.
1
4
2
3

116.

117.

118.

119.

120.

12 of 12
d.
2
3
4
1
The Japanese are fish and rice eaters as
they
a. are Asians
b. are Buddhists
c. cultivate rice and catch fish
d. import fish and rice
The largest religious system in the world
in both geographic extent and number of
adherents is
a. Christianity
b. Islam
c. Buddhism
d. Hinduism
Match List I with List II and select the
correct answer:
List I (Growing Condition)
A. 20-26 above 150
B. 10-20 50-70
C. 25
75-100
D. 21-27 100-150
List II: Name of crops (Temp 0CV
(Rainfall (cm)
1. Cotton
2. Rice
3. Rubber
4. Sugarcane
5. Wheat
A
B
C
D
a.
3
1
5
4
4
5
3
2
b.
c.
2
3
1
5
d.
2
5
1
4
The correct DESCENDING order of the
given continents in terms of per capita
availability of cultivable land is
a. Africa, Asia, South America, Australia
b. Australia, Africa, South America, Asia
c. Asia, Australia, Africa, South America
d. South America, Asia, Australia, Africa
World's leading producer of tobacco is
a. China
b. U.S.A.
c. India
d. Russia

C.S.E-Pre 1996

1 of 12

GEOGRAPHY
1.

2.

3.

4.

5.

6.

7.

The yield of cotton per hectare is the


highest in
a. C.I.S.
b. U.S.A.
c. Egypt
d. India
The most densely populated African
country is
a. Sudan
b. Egypt
c. Libya
d. Uganda
"Unit area" approach was first adopted
in/by
a. Land use survey in Britain
b. Land utilisation in China
c. Tennessee Valley Authority
d. Damodar Valley Corporation
Coconut triangle is located in
a. India
b. Sri Lanka
c. Bangladesh
d. Maldives
The largest area under wasteland in India
is in
a. Uttar Pradesh
b. Madhya Pradesh
c. Rajasthan
d. Gujarat
Consider the following statements
associated with the bajra cultivation in
India :
1. Bajra occupies about 11 % of the total
area under foodgrains.
2. Rajasthan is the leading producer of
bajra in India.
3. India exports a large quantity of bajra.
Of these statements
a. 1 and 2 are correct
b. 1,2 and 3 are correct
c. 2 and 3 are correct
d. 1 and 3 are correct
The nearest town Mecca in South West
Asia is
a. Medina
b. Jeddah

8.

9.

10.

11.

12.

c. Jerusalem
d. Ankara
About two-third of Canada's population is
concentrated in the St. Lawrence Basin
and Ontario peninsula on account of
a. their closeness to the U.S. border
b. history of settlement
c. concentration of economic activities
and facility of transport
d. their easy accessibility to western
Europe
The most urbanised region in AngloAmerica is
a. Mexico Gulf Coastal Region
b. North East Atlantic Coastal Region
c. Pacific Coastal Region
d. The Great Lakes Region
The largest coal resources are found at
a. Irkutsk region
b. Omsk-Tomsk region
c. Kuznetsk Basin
d. Tashkent Basin
The State which leads the others in marine
ashine is
a. Kerala
b. Tamil Nadu
c. West Bengal
d. Andhra Pradesh
Match List I with List II and select the
correct answer
List I (Mining area)
A. Kudremukh
B. Mosabani
C. Birmitrapur
D. Lohardaga
List II (Mineral)
1. Dolomite
2. Bauxite
3. Iron ore
4. Copper
A
B
C
D
a.
4
3
1
2
b.
3
4
1
2
c.
3
2
1
4
d.
3
4
2
1

13.

14.

15.

16.

The most plausible explanation of the


location of the Thardesert in Western India
is
a. the obstruction caused by the Aravallis
to the rainbearing wind that proceeds
to the Ganga Valley.
b. the evaporation of moisture by heat
c. the absence of mountains to the north
of Rajasthan to cause orographic
rainfall in it.
d. that the moisture carried by the southwest monsoon is driven away by the
dry upper air current.
Consider the following statements
associated with the Peccan Traps :
1. From the Cretaceous to the Eocene,
intense volcanic activity took place in
peninsular India.
2. The volcanic lava spread out in
horizontal
sheets,
filling
the
irregularities of the pre-existing
topography.
3. The trap country is characterized by
conical hills and extensive flat
surfaces.
Of these statements
a. 1 and 2 are correct
b. 1, 2 and 3 are correct
c. 2 and 3 are correct
d. 1 and 3 are correct
Match List I with List II and select the
correct answer
List I (Waterfalls)
A. Jog
B. Bheraghat
C. Sivasamudram
D. Hundru
List II (Rivers)
1. Narmada
2. Cauvery
3. Subarnarekha
4. Sharavati
A
B
C
D
a.
1
4
3
2
b.
1
4
2
3
c.
4
1
3
2
d.
4
1
2
3
Under Thornthwaite's classification of
climate almost the entire state of Orissa
comes under
a. dry subhumid
b. moist subhumid

17.

18.

19.

20.

21.

22.

23.

2 of 12
c. semi-arid
d. humid
Koeppen's Amw type of climate prevails
over
a. the interior peninsula of India
b. the Coromandal coast of India
c. Western coast of India, south of Goa
d. Eastern coast of India, south of
Calcutta
The Zonal Soil type of peninsular India
belongs to
a. Red Soils
b. Yellow Soils
c. Black Soils
d. Older Alluvium
The major forest product in India is
a. Timper
b. Fuel Wood
c. Cane products
d. Resins
Consider the following statements :
The S.W. Monsoon originates in India
primarily due to
1. low pressure in the Punjab plain.
2. high pressure in areas south of 8 N
latitude.
3. equatorial low being filled up by
descending currents.
4. the Himalayas.
Of these statements:
a. 1 and 4 are correct
b. 1 and 2 are correct
c. 1,2 and 3 are correct
d. 3 and 4 are correct
The correct sequence in DESCENDING
order of the given soils with respect to
areal coverage of India is
a. Alluvial, black, red, laterite
b. Alluvial, red, black, laterite
c. Alluvial, red, laterite, black
d. Red, alluvial, black, laterite
'Sal' trees are concentrated in
a. Tamil Nadu
b. Assam
c. Bihar
d. Madhya Pradesh
Which one of the following agricultural
products in the largest earner of foreign
exchange for India?
a. Sugarcane
b. Jute

c. Tea
d. Tobacco
24.
Match List I with List II and select the
correct answer
List I (Crop)
A. Gram
B. Wheat
C. Tobacco
D. Jowar
List II (State leading in production)
1. Gujarat
2. Maharashtra
3. Madhya Pradesh
4. Uttar Pradesh
A
B
C
D
a.
4
3
2
1
b.
3
4
1
2
c.
1
2
3
4
d.
3
4
2
1
25.
Per capital availability of forest land is the
highest in
a. Madhya Pradesh
b. Assam
c. Jammu and Kashmir
d. Sikkim
26.
Consider the following objectives:
1. Flood control
2. Irrigation
3. Soil conservation Power generation
4. Industrial Development
The primary objectives of Damodar Valley
Project include
a. 2, 3 and 4
b. 1, 2 and 4
c. 2, 4 and 5
d. 1, 4 and 5
27.
Which one of the following mountain
chains has two dissimilar types of
vegetation on its two slopes?
a. Aravallis
b. Vindhyas
c. Eastern Ghats
d. Western Ghats
28.
Which one of the following dams is NOT
meant for irrigation?
a. Bhavani Sagar
b. Sivasamudram
c. Krishnaraja Sagar
d. Bhakra-Nangal
29.
The correct sequence in DESCENDING
order of the given states in coal mining is

30.

31.

32.

33.

34.

3 of 12
a. West Bengal, Bihar, Madhya Pradesh
b. Bihar, Madhya Pradesh, West Bengal
c. Bihar, West Bengal, Madhya Pradesh
d. Madhya Pradesh, Bihar, West Bengal
Which one of the following statements is
NOT correct?
a. Maximum number of cotton yarn and
textile mills is lo cated in Gujarat.
b. Carpet industry is chiefly in the state of
Uttar Pradesh.
c. Cotton textile industry provides jobs to
the maximum number of people
d. The first cotton mill was set up at Fort
Gloster in Calcutta
Consider the following measures:
1. Creation of mass awareness on the
need for conservation of petroleum
2. Promotion of measures to curb
wasteful practices
3. Reducing the number of vehicles and
engines using petrol
4. Improving the oil use efficiency of
equipments devices and vehicles
5. Research and development for
improving oil use efficiency
The measures adopted by the Petroleum
Conservation Research Association of
India to conserve petroleum, include
a. 1, 2 and 3
b. 2, 3, 4 and 5
c. 1, 3, 4 and 5
d. 1, 2, 4 and 5
Copper is produced in
a. Rajasthan and Bihar
b. Uttar Pradesh and Rajasthan
c. Bihar, Uttar Pradesh and Rajasthan
d. Orissa, Rajasthan, Bihar and Uttar
Pradesh
The clustered pattern of a village depends
upon
a. distance between villages
b. placing of tenements
c. population of a village
d. location of settlements
Which of the following pairs are correctly
matched?
1. Okha .. Salt
2. Bastar.. Mica
3. Jodhpur.. Gypsum
4. Nasik .. Chromite
a. 2, 3 and 4
b. 1,3 and 4

35.

36.

37.

38.

c. 1, 2 and 4
d. 1,2 and 3
The greater potential for the generation of
tidal power in India is available in the
a. Malabar coast
b. Konkan coast
c. Gujarat coast
d. Coromandal coast
The following are the important events in
coal mining in India :
1. The first production of coal at
Raniganj
2. The establishment of Coal India
Limited
3. Nationalisation of coal mines
4. Establishment of Neyveli Lignite
Corporation
Their correct chronological sequencers
a. 1, 2, 3, 4
b. 2, 1, 3, 4
c. 1, 2, 4, 3
d. 2, 1, 4, 3
Match List I with List II and select the
correct answer
List I
A. Ranchi
B. Perambur
C. Narora
D. Bhopal
List II
1. Intergral coach
2. Heavy electricals
3. Heavy engg.
4. Atomic Power.
A
B
C
D
a.
2
4
1
3
b.
1
4
2
3
c.
3
1
4
2
d.
3
2
1
4
Consider the following states :
1. Maharashtra
2. Gujarat
3. West Bengal
4. Tamil Nadu
As per the 1991 census, the correct
sequence in DESCENDING order of these
states in terms of the number of workers
employed in industries is
a. 1, 3, 2, 4
b. 1, 2, 4, 3
c. 3, 1, 2, 4

4 of 12
d. 3, 2, 1, 4
39.
Consider the following states :
1. Kerala
2. Goa
3. Mizoram
4. Nagaland
As per 1991 census, the correct sequence
in DESCENDING order of female literacy
rate in these states is
a. 5,2,4,3
b. 2, 1,3, 4
c. 1,3,2,4
d. 1,2,3,4
40.
In terms of value, India is the largest
exporter of
a. textiles
b. gems and jewellery
c. engineering goods
d. agricultural products
41.
In terms of absolute numbers, which one
of the following pairs of states has a higher
level of Scheduled Castes population than
the rest?
a. Tamil Nadu and Rajasthan
b. Uttar Pradesh and West Bengal
c. Uttar Pradesh and Bihar
d. Rajasthan and Uttar Pradesh
42.
Consider the following states :
1. Gujarat
2. Karnataka
3. Maharashtra
4. Tamil Nadu
As per 1991 census, the correct sequence
in DESCENDING order of the level of
urbanization of these states is
a. 1, 3, 4, 2
b. 3, 4, 2, 1
c. 3, 1, 4, 2
d. 4, 3, 1, 2
The following nineteen items consist of two
statements, one labelled the Assertion A' and
the other labelled the 'Reason R'. You are to
examine these two statements carefully and
decide if the Assertion A and the Reason R and
individually true and if so, whether the reason
is a correct explanation of the Assertion. Select
your answer to these items using the codes
given below.
43.

Assertion (A): Plutonic rocks are intrusive


rocks of deep seated origin.

44.

45.

46.

47.

Reason (R): Plutonic rocks get cooled


slowly at great depth with large crystals.
a. Both A and R are true and R is the
correct explanation of A
b. Both A and R are true but R is not a
correct explanation of A
c. A is true but R is false
d. A is false but R is true
Assertion (A): Straight slope is a type
recognized by W. Penck.
Reason (R): Straight slope develops due to
increasing intensity of erosion.
a. Both A and R are true and R is the
correct explanation of A
b. Both A and R are true but R is not a
correct explanation of A
c. A is true but R is false
d. A is false but R is true
Assertion (A): The desert soils of India are
poor in mineral nutrients and hence
infertile.
Reason (R): The dearth of rainfall and
humus does not allow good soil formation.
a. Both A and R are true and R is the
correct explanation of A
b. Both A and R are true but R is not a
correct explanation of A
c. A is true but R is false
d. A is false but R is true
Assertion (A): Mediterranean civilization
of Greece and Rome did not develop in
similar climatic conditions as in Australia,
South Africa, Chile or California.
Reason (R): Similar physical environments
do not always produce the same response.
a. Both A and R are true and R is the
correct explanation of A
b. Both A and R are true but R is not a
correct explanation of A
c. A is true but R is false
d. A is false but R is true
Assertion (A): The production of rubber is
decreasing in the Amazon basin.
Reason (R): Equatorial climate is
favourable for rubber plantation.
a. Both A and R are true and R is the
correct explanation of A
b. Both A and R are true but R is not a
correct explanation of A
c. A is true but R is false
d. A is false but R is true

48.

49.

50.

51.

52.

5 of 12
Assertion (A): If a surveyor is to select for
surveying, only one instrument out of
Dumpy level and Theologize, he will
select the Theologize
Reason (R): Theologize can perform both
horizontal and vertical measurements.
a. Both A and R are true and R is the
correct explanation of A
b. Both A and R are true but R is not a
correct explanation of A
c. A is true but R is false
d. A is false but R is true
Assertion (A): The equatorial regions of
the world provide favourable environment
for plants, but not for man.
Reason (R): The average annual range of
temperatures in the region is very small
(about 3 C)
a. Both A and R are true and R is the
correct explanation of A
b. Both A and R are true but R is not a
correct explanation of A
c. A is true but R is false
d. A is false but R is true
Assertion (A): River Hwang Ho often
overflows its banks and inundates
innumerable villages causing widespread
devastation.
Reason (R): It is fed by melting glaciers in
its upper course and heavy monsoon rain
in the highlands.
a. Both A and R are true and R is the
correct explanation of A
b. Both A and R are true but R is not a
correct explanation of A
c. A is true but R is false
d. A is false but R is true
Assertion (A): Tea is grown on the hill
slopes of South East Asia.
Reason (R): The slopes are easily terraced
for tea plantation.
a. Both A and R are true and R is the
correct explanation of A
b. Both A and R are true but R is not a
correct explanation of A
c. A is true but R is false
d. A is false but R is true
Assertion (A): The whole of the present
vegetation of North-west Europe is of
recent origin as compared to much older
vegetations of Zaire and Amazon basin.

53.

54.

55.

56.

a. Reason (R): North West Europe was


subjected to ice age glaciations while
the Zaire and Amazon basins were not.
Both A and R are true and R is the
correct explanation of A
b. Both A and R are true but R is not a
correct explanation of A
c. A is true but R is false
d. A is false but R is true
Assertion (A): Indian foreign trade has
been experiencing heavy growth rate
during the past two years.
Reason (R): Indian currency has been
devalued against the world's major
currencies.
a. Both A and R are true and R is the
correct explanation of A
b. Both A and R are true but R is not a
correct explanation of A
c. A is true but R is false
d. A is false but R is true
Assertion (A): The study of sex
composition of population assumes added
significance for national planners.
Reason (R): The two sexes play partly
complementry roles in the economy and
society.
a. Both A and R are true and R is the
correct explanation of A
b. Both A and R are true but R is not a
correct explanation of A
c. A is true but R is false
d. A is false but R is true
Assertion (A): Forest cover in India needs
to be increased from 22% to 33% level of
the total land.
Reason (R): This will provide enough fuel
wood to the rural poor.
a. Both A and R are true and R is the
correct explanation of A
b. Both A and R are true but R is not a
correct explanation of A
c. A is true but R is false
d. A is false but R is true
Assertion (A): In India the larger cities are
growing faster.
Reason (R): Inspite of the emphasis on
their development, the small and medium
towns have failed to come up to expected
norms.
a. Both A and R are true and R is the
correct explanation of A

57.

58.

59.

60.

61.

6 of 12
b. Both A and R are true but R is not a
correct explanation of A
c. A is true but R is false
d. A is false but R is true
Assertion (A): Chemical industries have
reached a high level of development in the
Hoogly-Calcutta belt.
Reason (R): Cheap industrial labour force
is readily available in this belt.
a. Both A and R are true and R is the
correct explanation of A
b. Both A and R are true but R is not a
correct explanation of A
c. A is true but R is false
d. A is false but R is true
Assertion (A): Friedrich Ratzel has been
the greatest single contributor to the
development of the geography of man.
Reason (R): His contributions are highly
esteemed by
a. Both A and R are true and R is the
correct explanation of A
b. Both A and R are true but R is not a
correct explanation of A
c. A is true but R is false
d. A is false but R is true
Assertion (A): In India, fishing occupies a
very important place in its economy.
Reason (R): The production offish in India
has shown appreciable increase after
Independence.
a. Both A and R are true and R is the
correct explanation of A
b. Both A and R are true but R is not a
correct explanation of A
c. A is true but R is false
d. A is false but R is true
Assertion (A): India is witnessing a rapid
growth of population.
Reason (R): The rate of decrease in
fertility has tended to lag behind that of
mortality.
a. Both A and R are true and R is the
correct explanation of A
b. Both A and R are true but R is not a
correct explanation of A
c. A is true but R is false
d. A is false but R is true
Assertion (A): In India, per hectare
production of sugarcane is the highest in
the world.

62.

63.

64.

65.

66.

Reason (R): India has about one-third of


the world's sugarcane area.
a. Both A and R are true and R is the
correct explanation of A
b. Both A and R are true but R is not a
correct explanation of A
c. A is true but R is false
d. A is false but R is true
The concept of Ice Age was first put
forward by
a. Louis Agassiz
b. Jean de Charpentier
c. James Geike
d. De Geer
'Water table' refers to
a. the lower limit of the zone of
saturation
b. the upper limit of the zone of
saturation
c. seepage of water into fissures lying
underground
d. the contact zone of the permeable and
impermeable rocks
Population explosion is associated with
a. high birth rate and low death rate
b. high birth rate and high death rate
c. low birth rate and high death rate
d. low birth rate and low death rate
Which one of the following is NOT a soil
forming process
a. Weathering
b. Translocation
c. Gleying
d. Hydration
Match List I with List II and select the
correct answer
List I
A. Nubian Desert
B. Sinai Desert
C. Rub-al Khali
D. Dusht-e-Let
List II
1. Egypt
2. Sudan
3. Iran
4. Saudi Arabia
A
B
C
D
a.
1
2
3
4
b.
2
1
4
3
c.
2
4
1
3
d.
3
1
4
2

67.

68.

69.

70.

71.

7 of 12
Which of the following are examples of
pen plains? Plains of
1. Central Russia
2. Paris Basin
3. Eastern England
4. Upper Mississippi Basin
Select the correct answer
a. 2 and 3
b. 1,2 and 4
c. 1,3 and 4
d. 1,2, 3 and 4
The greatest concentration of China's coal
deposits is in
a. Hopei
b. Shantung
c. Shanshi and Shensi
d. Szechwan
The shaded protion in the given map is

a. Veld region
b. Lake Rudolf
c. Shott plateau
d. Chad basin
Fiords are very typical of the
a. Alaskan coast
b. Spanish coast
c. Norwegian coast
d. Peruvian coast
Match List I with List II and select the
correct answer
List I (Era)
A. Palaeozoic
B. Mesozoic
C. Pre-Cambrian
D. Cenozoic
List II (Period / Epoch)
1. Triassic
2. Archaean
3. Oligocene
4. Devonian
5. Neocene
A
B
C
D
a.
1
4
2
5
b.
4
1
2
5
c.
1
5
3
2
d.
4
1
3
2

72.

73.

74.

75.

76.

Match List I with List II and select the


correct answer
List I (Landforms)
A. Stalactite
B. Solanchak
C. Shott
D. Sill
List II (Associated features)
1. Saline soil
2. Limestone
3. Volcanic rock
4. Salt lake
A
B
C
D
a.
1
4
2
3
b.
2
3
4
1
c.
2
1
4
3
d.
4
1
2
3
While explaining spatial interaction,
Edward Ullmann introduced the concept of
a. transport network
b. functional classification
c. transferability
d. social physics
The agricultural density of population is
represented by
a. Total population
Total area
b. Total population
Total cultivated area
c. Total
population
engaged
in
agriculture
Total cultivated area
d. Total cultivators
Net sown area
Match List I with List II and select the
correct answer
List I
A. Bushman
B. Pygmy
C. Eskimo
D. Gond
A
B
C
D
a.
3
4
2
1
b.
3
4
1
2
c.
4
3
1
2
d.
4
3
2
1
The correct sequence of the types of
farming which a surveyor will come across
from Omaha to Chicago will be
a. livestock, dairy, grain
b. livestock, grain, dairy

77.

78.

79.

80.

8 of 12
c. grain, mixed fanning, dairy
d. grain, livestock, dairy
Which of the following scholars are
related to the geostatic viewpoint in human
geography?
1. Humboldt
2. Ritter
3. Ratzel
4. Semple
Codes;
a. 3 and 4
b. 2 and 4
c. 1, 2 and 3
d. 1, 2, 3 and 4
Weber's model of industrial location has
been usefully applied to interpret the
changing location of the British
a. iron and steel industry
b. cotton textile industry
c. ship-building industry
d. heavy engineering industry
Match List and List II and select the
correct answer
List I (Minerals)
A. Bauxite
B. Gold
C. Copper
D. Tin
List II (Major Producing Countries)
1. Brazil
2. U.S.A.
3. Malaysia
4. Australia
5. South Africa
A
B
C
D
a.
1
4
3
2
b.
4
5
2
3
c.
4
5
3
1
d.
1
5
2
3
Transportation cost - distance relationship
is shown in the following figures. If a
person wants to travel from A to B, then
which one of the following relationships
will be beneficial to him?
a.

b.

85.
c.

81.

82.

83.

84.

d.

86.

Match List I with List II and select the


correct answer
List I (Concepts)
A. Areal differentiation
B. Chorology
C. Human Welfare
D. Spatial organization
List II (Contributors)
1. Hettner
2. Hartshorne
3. D.M. Smith
4. David Harvey
A
B
C
D
a.
2
1
3
4
b.
1
3
4
2
c.
2
1
4
3
d.
4
2
1
3
'Kitab-ul-Hind' was written by
a. Al-Masudi
b. AI-Biruni
c. Al-Idrisi
d. Ibn-Batuta
Which one of the following is NOT a
conventional map projection ?
a. Interrupted Sinusoidal Projection
b. Mollweide's Projection
c. Sinusoidal Projection
d. Gail's Stereographic Projection
Consider the following situations :
1. Very low birth rate and low death rate
2. Birth rate greater than death rate
3. Very high birth and death rates
4. Low birth and death rates
Demographic transition in a country
follows the sequences
a. 3, 2, 4, 1

87.

88.

89.

90.

9 of 12
b. 1, 3, 2, 4
c. 1, 4, 2, 3
d. 4, 3, 2, 1
The first treatise on cartography was
written by
a. Aristotle
b. Eratosthenes
c. Mercato
d. Ptolemy
The heliocentric concept of the universe
was first formulated by
a. Ptolemy
b. Newton
c. Copernicus
d. Cassini
The most appropriate diagram for showing
the actual distribution of rainfall over a
substantial long period is the
a. wheel diagram
b. dispersion diagram
c. scatter diagram
d. bar diagram
The model developed by Hagerstrand in
relation to spatial diffusion is
a. Loschian model
b. Monte Carlo model
c. Lowvy-type model
d. Multiple-nuclei model
Match List I with List II and select the
correct answer
List I
A. A. H. Robinson
B. C. K. Wentworth
C. E. Rasiz and J. Henry
D. G. H. Smith
A
B
C
D
a.
4
2
3
1
b.
2
3
4
1
c.
4
3
2
1
d.
2
1
4
3
The following forms of relationship exist
between man and his environment
according to the development of
geographical 'thought:
1. Environment determinism
2. Possibilism
3. Nee-determinism
The correct sequence of the development
of these forms of relationships is
a. 2, 1, 3

91.

92.

b. 3, 1, 2
c. 1, 3, 2
d. 1, 2, 3
Consider the following statements
regarding conical projection with two
standard parallels :
1. The parallels are arcs of concentric
circles and are equispaced.
2. The meridians are straights lines
radiating from the common centre as
radii of concentric curves.
3. The scale is true along two standard
parallels only.
4. The pole is represented by a point.
a. 1, 2 and 3 are correct
b. 2 and 4 are correct
c. 1 and 2 are correct
d. 1, 2, 3 and 4 are correct
Match List I with List II and select the
correct answer
List I (Climatic type)
A. Savannah
B. Humid subtropical
C. West cost marine
D. Middle latitude desert

94.

95.

96.

List II (Characteristics)

93.

1. Rainfall below 15 cm and thorny scrub


vegetation
2. No prolonged dry season and rainforest
vegetation
3. Annual
rainfall 75
cm,
no
pronounced
dry
season
forest
vegetation
4. Rain scanty, long dry season, bush and
short grass vegetation
5. Annual temperature rang below 20C
and mixed hardwood to coniferous
vegetation
A
B
C
D
a.
4
3
5
1
b.
1
5
2
4
c.
5
2
1
4
d.
4
5
2
1
Gondwanaland does NOT include
a. India
b. Vietnam
c. China
d. Australia

97.

98.

99.

100.

10 of 12
On the basis of the given map, the best
place for locating a big iron and steel plant
would be the site marked as

a. A
b. B
c. C
d. D
Which one of the following stands for the
symbol pH?
a. Phosphorus and hydrogen content in
soil
b. Percentage of hydrogen in soil
c. Proportion of potash and hydrogen in
soil
d. Concentration of hydrogen ions in the
soil
The main cause of Kalahari being a desert
is
a. the cold currents near the coast
b. its location on the west coast in the
tropics
c. its location in the tropics
d. that its people are primitive
Augusta and Atlanta, the two important
cotton textile centres are located in
a. South Carolina
b. Massachusets
c. North Carolina
d. Georgia
Salinity in ocean water decreases when
a. evaporation is more
b. wind velocity is high
c. rainfall is heavy
d. humidity is high,
The country well-known for its small scale
precision high cost industrial goods is
a. Poland
b. Switzerland
c. France
d. Germany
Which of the following pairs are correctly
matched ?
1. Cwa . hot summer, dry
winter
2. Cwb . hot, dry summer

101.

102.

103.

104.

105.

106.

3. Csb .warm, dry summer


4. Cfb .warm summer
Select the correct answer
a. 1, 2, 3 and 4
b. 1, 3 and 4
c. 1 and 4
d. 2 and 3
Which of the following conditions
characterize the equate rial climates of the
world?
1. Cold winter
2. Hot summer
3. Afternoon thunder shower
4. Small diurnal range of temperature
Select the correct answer
a. 1 and 3
b. 2 and 4
c. 1 and 2
d. 3 and 4
The hot dust-laden wind blowing from
Sahara desert to Mediterranean region is
called the
a. chinook
b. forhn
c. mistral
d. sirocco
Which one of the following pairs are
correctly matched?
a. Latosols . Scrubs
b. Chernozem . Savannas
c. Sierozem . Selvas
d. Podsol . Conifers
In the rough outline map given below,

The shaded portion marked X represents a


region of
a. tropical grassland climate
b. sub-tropical semi-arid climate
c. sub-tropical grassland climate
d. humid sub-tropical climate
Which one of the following is a zonal soil?
a. Sierozem
b. Podsol
c. Lithosol
d. Solonchak
Match List I with List II and select the
correct answer

107.

108.

109.

110.

11 of 12
List I (Humidity Provinces)
A. Wet
B. Humid
C. Semi- arid
D. Arid
List II (Characteristics Natural Vegetation)
1. Forest
2. Rainforest
3. Scrub
4. Steppe
A
B
C
D
a.
2
3
4
3
b.
1
2
4
3
c.
2
4
1
3
d.
4
3
2
1
Which of the following pairs are correctly
matched ?
1. Sal .
Tropical deciduous
2. Pine .Coniferous
3. Teak .Evergreen
4. Cactus .Desert
Select the correct answer
a. 1 and 3
b. 2, 3 and 4
c. 1, 2 and 4
d. 1, 2, 3 and 4
Which of the following features
characterise Gran chaco type of vegetation
in Central South America?
1. Thorny bushes and dwarf shrubs
2. Thick impenetrable stands
3. Cacti
4. Tall grasses.
Select the correct answer
a. 1, 2, 3 and 4
b. 1,2 and 3
c. 2 and 4
d. 1 and 3
In which one of the following natural
regions, is subsistence fanning, the main
occupation?
a. Mediterranean type
b. Steppe type
c. China type
d. Monsoon type
A major part of fresh water fish is caught
in
a. eastern and southern Asia
b. western Africa
c. South America
d. western Europe

111.

112.

113.

114.

115.

116.

The term 'occidental' relates to the culture


of
a. South East Asia
b. North Western Europe
c. Middle East
d. Africa
The highest growth rate of population is
found in
a. India
b. Malaysia
c. Bangladesh
d. Sri Lanka
In developing countries, the population
pressure increases in some parts due to
immigration. Such parts are characterized
by
a. low cost of living
b. more suitable climate
c. large employment opportunities
d. accessibility
Africa (with much higher potential hydroelectric power than any other continent)
has developed only 0.5 percent of its
potential and contributes only a little more
than one percent of the hydro-electric
power of the world. This is because
a. the demand for power is small
b. the volume of water in all the rivers is
small
c. there is no steady flow of water in the
rivers
d. the topography does not permit its
transmission by wires
Masai belongs to
a. Gobi desert
b. Atacama desert
c. Tropical Africa
d. Sahara desert
Match List I with List II and select the
correct answer
List I (Petroleum producing countries)
A. C.I.S
B. China
C. Borneo (Indonesia)
D. Myanmar
List II (Centers of productions)
1. Yenangyaung
2. Baku
3. Kansu
4. Balikapapan
A
B
C
D
a.
2
3
4
1
b.
3
2
1
4
c.
3
2
4
1

117.

118.

119.

120.

12 of 12
d.
2
3
1
4
The largest permanent migration of the
Indians outside the country in the last
century was associated with the
a. demand for the professionals
b. petro-dollar revolution
c. sugarcane plantations
d. rubber and tea plantations
US. Which of the following characteristics
symbolize the oriental cultural realm?
1. Industrialization
2. Joint family bond
3. Subsistence agriculture
4. Urbanization
Select the correct answer
a. 1,2 and 4
b. 2 and 3
c. 1 and 3
d. 2, 3 and 4
Match List I with List II and select the
correct answer
List I (Countries)
A. Australia
B. Brazil
C. USA
D. Malaysia
List II (Crops)
1. Natural rubber
2. Coffee
3. Eucalyptus
4. Tobacco
A
B
C
D
a.
3
1
2
4
b.
1
3
4
2
c.
3
2
4
1
d.
1
3
2
4
Which of the following statements
regarding cotton cultivation in China are
correct?
1. A growing season of about 200 days
free from frost
2. Intermittent rainfall during harvesting
period benefits higher yields
3. Rainfall of 500 to 1500 mm during
growing season
4. Mechanized farming and heavy
irrigation is a common feature in the
east
Select the correct answer
a. 3 and 4
b. 2, 3 and 4
c. 1 and 2
d. 1 and 3

C.S.E-Pre 1997

1 of 14

GEOGRAPHY
1.

2.

3.

4.

The largest portion of India's wasteland


occurs in
a. upland without scrub
b. water-logged and marshy land
c. degraded pastures and grazing land
d. wastelands
created
by shifting
cultivation
Which of the following pairs are correctly
matched?
Industrial Regions Major Industrial
Centers
1. Ukraine
Krivoy Rog
2. Kuzbas

Novosibirsk

3. Central-Asian

Karaganda

4. Volga

Moscow

a. 1 and 3
b. 1 and 2
c. 2 and 3
d. 1, 2 and 3
The largest industrial region of China is
a. Mukden
b. Beijing
c. Shanghai
d. South Manchuria
Match List I with List II and select the
correct answer:
List I (Projects)
A. Sardar Sarovar
B. Dulhasti
C. Kadam
D. Gerusoppa
List II (Location)
1. Andhra Pradesh
2. Karnataka
3. Gujarat
4. Jammu and Kashmir
A
B
C
D
a.
3
4
1
2
b.
3
1
2
4
c.
1
4
3
2
d.
4
2
1
3

The percentage of irrigated land in India is


a. 45
b. 65
c. 35
d. 25
6.
Consider the following statements :
In India the problem of soil erosion is
associated with
1. excessive rainfall
2. deforestation
3. excessive cultivation
4. overgrazing
Of these statements
a. 1 and 2 are correct
b. 1, 2 and 3 are correct
c. 2, 3 and 4 are correct
d. 2 and 4 are correct
7.
The shaded portion in the given rough
outline map of India represents

5.

8.

9.

a. areas affected by drought


b. areas under sugarcane cultivation
c. densely urbanized areas
d. areas under jute cultivation
Karewas are terraces of glacial origin
found in
a. Teesta valley
b. Ravi valley
c. Jhelum valley
d. Alakananda valley
Consider the following statements
The type of natural vegetation in India
varies due to variation in soil types
altitude.
1. the amount of rainfall.
2. soil types
3. mean annual temperature
4. altitude
Of these statements

10.

11.

12.

13.

a. 1 and 2 are correct


b. 1, 2 and 3 are correct
c. 2, 3 and 4 are correct
d. 1, 3 and 4 are correct
Match List I with List II and select the
correct answer:
List I (Name of forest)
A. Manas
B. Betla
C. Gorumara
D. Mudumalai
List II (Location)
1. West Bengal
2. Assam
3. Bihar
4. Tamil Nadu
A
B
C
D
a.
2
3
1
4
b.
1
2
3
4
c.
4
3
1
2
d.
2
1
4
3
India has the highest productive potential
of fisheries in her
a. inland water bodies
b. shallow continental shelf
c. deep sea areas
d. brackish water lagoons
Consider the share of forest areas in the
following states :
1. Punjab
2. Gujarat
3. Haryana
4. Rajasthan
The correct sequence of the descending
order of forested areas in these states is
a. 3, 4, 2, 1
b. 4, 3, 2, 1
c. 3, 4, 1, 2
d. 4, 3, 1, 2
Which one of the following types of
forests grows in the shaded portion of the
given rough outline map of India?

14.

15.

16.

17.

2 of 14
a. Mixed deciduous
b. Tropical Savannah
c. Semi-arid scrublands
d. Mangrove forests
The present forest area of India, according
to satellite data is
a. increasing
b. decreasing
c. static
d. open forest area is decreasing but
closed forest area is increasing
Match List I with List II and select the
correct answer:
List I (Dam)
A. Nagarjunasagar
B. Matatilla
C. Maithon
D. Hirakud
List II (River)
1. Mahanadi
2. Barakar
3. Krishna
4. Betwa
A
B
C
D
a.
3
1
2
4
b.
1
4
3
2
c.
3
4
2
1
d.
4
2
3
1
Which one of the following is associated
with Indian Agriculture?
a. I.C.A.R.
b. I.C.C.R.
c. I.C.M.R.
d. I.C.W.A
Match List I with List II and select the
correct answer:
List I
A. Ferozabad
B. Mirzapur
C. Jaipur
D. Trivandrum
List II
1. Pottery
2. Glass Bangles
3. Wood carving
4. Gems Industry
A
B
C
D

18.

19.

20.

21.

22.

a.
2
1
4
3
b.
4
2
1
3
c.
3
1
2
4
d.
2
3
4
1
The refining capacity of crude oil is
highest in the
a. Haldia oil refinery
b. Mathura oil refinery
c. Gujarat oil refinery
d. Vishakhapatnam oil refinery
Which one of the following statements is
correct?
a. Diamonds occur in the sediments of
Cuddapah Series
b. Natural gas occurs in the Gondwana
beds
c. Mica occurs in abundance at Kodarma
d. Dharwars are famous for petroleum
The portions marked in the given rough
outline map of India indicate-the
distribution of A

a. coal-fields
b. hydel power plants
c. oil-fields
d. iron-ore deposits
The group of States which has forest
coverage of more than 75% of the total
geographical area is
a. Assam, Meghalaya, Nagaland
b. Assam, Arunachal Pradesh, Nagaland
c. Arunachal
Pradesh,
Manipur,
Nagaland
d. Arunachal Pradesh, Madhya Pradesh,
Nagaland
Which one of the following is not a
causative factor with respect to poor
coverage of forest area in Jammu and
Kashmir?
a. Low amount of rainfall
b. Large area under cultivation
c. Steep barren slopes
d. Snow covered peaks

23.

24.

3 of 14
The coal mines of Corrigenda are situated
in
a. Kazakhstan
b. Azerbaijan
c. Uzbekistan
d. Georgia
Match List I with List II and select the
correct answer
List I (Industries)
A. Lac
B. Mineral oil
C. Heavy Engineering
D. Railway wagon
List II (place)
1. Jhalda
2. Ranchi
3. Kadi
4. Pimpri
5. Perambur
A
B
C
D

a.
1
3
2
5
b.
4
3
1
2
c.
2
4
3
5
d.
1
4
2
3
Directions: The following fourteen items consist
of two statements, one labelled the 'Assertion A'
and the other labelled the 'Reason R'. You are to
examine these two statements carefully and decide
if the Assertion A and the Reason R are
individually true and if so, whether the Reason is
a correct explanation of the Assertion. Select your
answer to these items using the codes given below
and mark your answer sheet accordingly.
25.
Assertion (A): On the Interrupted
Sinusoidal Projection, shapes of continents
are much improved as compared to those
of the Sinusoidal Projection.
Reason (R): The Interrupted Sinusoidal
Projection employs several straight line
meridians, one for each continent.
a. Both A and R are true and R is
the correct explanation of A
b. Both A and R are true but R is not the
correct explanation of A
c. A is true but R is false
d. A is false but R is true
26.
Assertion (A) : There are regional
disparities in the spatial organizations of
economy

27.

28.

29.

30.

Reason (R): Interactions among inherent


factors in economic activities affect the
texture of spatial organization.
a. Both A and R are true and R is
the correct explanation of A
b. Both A and R are true but R is not the
correct explanation of A
c. A is true but R is false
d. A is false but R is true
Assertion (A): A negligible proportion of
the people of the hot desert regions of the
world are engaged in industrial pursuits.
Reason, (R): Those regions are very poorly
endowed with mineral and/or energy
resources.
a. Both A and R are true and R is
the correct explanation of A
b. Both A and R are true but R is not the
correct explanation of A
c. A is true but R is false
d. A is false but R is true
Assertion (A): When British rule ended in
India, many Anglo-Indians migrated to
Australia.
Reason (R): Some migrations are caused
by religious attraction.
a. Both A and R are true and R is
the correct explanation of A
b. Both A and R are true but R is not the
correct explanation of A
c. A is true but R is false
d. A is false but R is true
Assertion (A): Oceans are always shown
in blue in coloured maps.
Reason (R): It indicates the natural colour
of oceans.
a. Both A and R are true and R is
the correct explanation of A
b. Both A and R are true but R is not the
correct explanation of A
c. A is true but R is false
d. A is false but R is true
Assertion (A): More cyclones hit the
eastern coast of India than the western
coast.
Reason (R): The eastern coast of India lies
in the path of the North-East trade wind.
a. Both A and R are true and R is
the correct explanation of A

31.

32.

33.

34.

4 of 14
b. Both A and R are true but R is not the
correct explanation of A
c. A is true but R is false
d. A is false but R is true
Assertion (A): Though Britain and
Labrador have the same latitudinal
location, the climate of Britain is
temperate while that of Labrador is cold.
Reason (R): Warm ocean currents skirt
Britain while cold currents flow along the
Labrador coast.
a. Both A and R are true and R is
the correct explanation of A
b. Both A and R are true but R is not the
correct explanation of A
c. A is true but R is false
d. A is false but R is true
Assertion (A): In Africa, coastal lowlands
are few and small in extent.
Reason (R): Africa is a plateau continent
as nearly all of the continent lies above
300 meters elevation.
a. Both A and R are true and R is
the correct explanation of A
b. Both A and R are true but R is not the
correct explanation of A
c. A is true but R is false
d. A is false but R is true
Assertion (A): Kenya's birth rate rose
despite advances in education and child
survival.
Reason (R): Kenyans place high cultural
values on high fertility.
a. Both A and R are true and R is
the correct explanation of A
b. Both A and R are true but R is not the
correct explanation of A
c. A is true but R is false
d. A is false but R is true
Assertion (A): Assam has the largest area
under canal irrigation in India.
Reason (R): The Assam valley is flat and
the Brahmaputra carries enormous volume
of water.
a. Both A and R are true and R is
the correct explanation of A
b. Both A and R are true but R is not the
correct explanation of A
c. A is true but R is false
d. A is false but R is true

35.

36.

37.

38.

39.

Assertion (A): The boundary marking the


termination of tree growth is known as the
timber line.
Reason (R): Its location is determined by
the amount of moisture, exposure,
evaporation and depth of snow.
a. Both A and R are true and R is
the correct explanation of A
b. Both A and R are true but R is not the
correct explanation of A
c. A is true but R is false
d. A is false but R is true
Assertion (A): Indian agriculture is no
longer the gamble of .monsoons.
Reason (R): There is recent expansion of
dry farming in India.
a. Both A and R are true and R is
the correct explanation of A
b. Both A and R are true but R is not the
correct explanation of A
c. A is true but R is false
d. A is false but R is true
Assertion (A) : The western disturbance
causes winter rain in North-West India.
Reason (R): The South-West monsoon
starts retreating from North-West India
during winter.
a. Both A and R are true and R is
the correct explanation of A
b. Both A and R are true but R is not the
correct explanation of A
c. A is true but R is false
d. A is false but R is true
Assertion (A): Production of natural gas in
India has increased in recent years.
Reason (R): India has a high reserve of
natural gas.
a. Both A and R are true and R is
the correct explanation of A
b. Both A and R are true but R is not the
correct explanation of A
c. A is true but R is false
d. A is false but R is true
The classical model of industrial location
theory in which least cost approach is of
primary consideration is recognized as the
industrial location theory of
a. Weber
b. Smith
c. hard

40.

41.

42.

5 of 14
d. Fetter
In the sketch of a tropical cyclone shown
in the given figure, the area marked 'X' is
characterized by

a. rain
b. clear sky
c. clouds
d. dirty sky
Match List I with List II and select the
correct answer:
List I
A. Evaporation
B. Condensation
C. Precipitation
D. Isolation
List II
1. Release of moisture
2. Cold air moves into warm air
3. Transformation of water vapur into
water
4. Gain of heat
5. Incoming solar radiation
A
B
C
D
a.
5
3
1
2
b.
4
1
5
3
c.
2
4
3
5
d.
4
3
1
5
Match List I with List II and select the
correct answer:
List I (Instrument)
A. Planimeter
B. Pantograh
C. Parallax bar
D. Clinometer
List II (Purpose)
1. Measure angles and slopes on ground
2. Calculation of areas on maps
3. Enlargement and reduction of maps
4. Measures the elevation from the
photographs
A
B
C
D
a.
4
2
3
1
b.
2
1
3
4
c.
2
3
4
1

6 of 14
43.

44.

45.

46.

d.
1
3
4
2
On a map drawn on Gnomonic projection,
the great circle course between two points
is shown by
a. a sine curve
b. a straight line
c. a parabola
d. an arc of a circle
Match List I with List II and select the
correct answer:
List I (Characteristic)
A. Superimposed drainage
B. Antecedent drainage
C. Jet Streams
D. Inland drainage
List II (Area)
1. Peninsular India
2. Himalayan region
3. Desert area
4. Glacial region
5. Upper atmospheric zone
A
B
C
D
a.
1
2
5
3
b.
1
2
3
4
c.
4
5
2
3
d.
3
1
5
2
Match List I with List II and select the
correct answer:
List I (Authors)
A. A. Haggett P.
B. B. Berry B.J.L
C. C. Reilly W.J.
List II (Location theories)
1. Concept of threshold and Range
2. Law of gravitation
3. Location allocation models
A
B
C
a.
1
3
2
b.
3
2
1
c.
3
1
2
d.
1
2
3
The given diagram indicating the curve of
W. Penck's cycle of erosion, shows which
of the following features?

47.

48.

49.

1. Only upliftment in stage A


2. Rate of upliftment in stage A is lower
than that of B.
3. Upliftment is more than erosion in
stage B.
4. Rate of erosion in stage E is higher
than that of D.
Select the correct answer using the codes
given below:
a. 1, 2 and 3
b. 3 and 4
c. 2 and 4
d. 1, 2 and 4
According to Central Place theory, which
one of the following statements most aptly
describes the distribution area of a
commodity around a central place?
a. The area which supports the threshold
population for the commodity
b. The area in which the central place
supplies more of the commodity than
does any other central place
c. The area bounded by the line joining
the points of maximum distance upto
which the commodity is supplied by
the central place
d. The area bounded by the tine joining
the points of maximum distance from
which people come to the central place
to buy the commodity
The plantation farming regions of the
world differ from subsistence agricultural
regions because they
a. have organized agriculture
b. are carried on in entirely different
environmental conditions
c. do not occupy large portions of the
earths surface
d. provided livelihood for people
distributed over wide
Match List I with List II and select the
correct answer:
List I (Name of exponents)
A. Malthus
B. Burgess

50.

51.

52.

53.

C. Stamp
List I (Principles enunciated)
1. Concentric circles
2. Use and misuse of land
3. Network analysis
4. Growth of population and growth of
food supply
A
B
C
a.
1
2
3
b.
4
1
3
c.
4
3
2
d.
4
1
2
Consider the following countries
1. Bangladesh
2. Brazil
3. Indonesia
4. Japan
The correct sequence of these countries in
descending order of their population is
a. 1,2,3,4
b. 2, 1,4,3
c. 3,2,4, 1
d. 4,3,2, 1
The concept of Stop and Go
Determinism was put forward by
a. Griffith Taylor
b. Jean Brunhes
c. J. E. Spencer
d. Vidal de Blache
The basic distinction between urban and
rural settlements s
the
a. population size
b. population density
c. function
d. location
Match List I with List II and select the
correct answer:
List I (Commodity)
A. Mineral oil production
B. M production
C. Oil refining capacity
D. Tin ore production
List II (Country)
1. India
2. Russia
3. Malaysia
4. U.S.A.
5. Saudi Arabia

54.

55.

56.

57.

7 of 14
A
B
C
D
a.
2
1
4
3
b.
4
1
5
3
c.
5
2
4
1
d.
2
3
1
4
Consider the following takes in the St
Lawrence Great Lakes Waterway
1. Huron
2. Ontario
3. Erie
4. Superior
5. Michigan
The correct sequence of these lakes from
St. Lawrence onwards in the westward
direction is
a. 2, 3, 5, 1, 4
b. 2, 3, 1, 5, 4
c. 3, 2, 1, 5, 4
d. 3, 2, 5, 1, 4
Consider the following statements:
A climograph is drawn with the help of the
monthly data of 1. rainfall
2. relative humidity
3. wet bulb temperature
Of these statements a. 1, 2 and 3 are correct
b. 1 and 2 are correct
c. 2 and 3 are correct
d. 1 and 3 are correct
Consider the following ports:
1. Gibraltar
2. Colombo
3. Singapore
4. Aden
The correct sequence of the location of
ports along the Mediterranean Sea-Red
Sea ocean route from London to
Yokohama is
a. 1, 2, 3, 4
b. 1, 4, 2, 3
c. 4, 1, 3, 2
d. 3, 1, 4, 2
The position in philosophy that defines
meaning and knowledge in terms of their
function and experience, with reference to
adjustment and the resolution of
problematic situations is termed as
a. pragmatism

58.

59.

60.

61.

62.

b. idealism
c. realism
d. nominalism
The best cartographic method for
representing the distribution of rural
population in a region of greatty varying
size of villages is
a. Choropleth method
b. Isopleth method
c. Multiple Dot method
d. Proportional Squares method.
Who control the Rimland rules Eurasia,
who rules Eurasia rules the world. The
above statement was made by
a. Mackinder
b. Spykman
c. Alfred Mahan
d. Whittles
Match List I with List Hand select the
correct answer:
List I (Types of topo-sheets)
A. Million
B. One inch
C. Quarter inch
D. Half inch
List II (Standard contour intervals in feet)
1. 50
2. 500
3. 100
4. 250
A
B
C
D
a.
2
4
3
1
b.
4
1
2
3
c.
2
1
4
3
d.
1
3
4
2
In Christallers Central P theory for
organization of settlements, K = 4 denotes
a. Transport principle
b. Market principle
c. Administrative principle
d. Economic principles
Match List I with List II and select the
correct answer
List I (Characteristic)
A. Asthenosphere
B. Inversion
C. Lithosphere
D. Mantle

63.

64.

65.

8 of 14
List II (Area in which its exists)
1. Troposphere (Lower part)
2. The oceanic part of the earth
3. Mantle of the earth (upper part)
4. Solid earth
5. Mantle of thee earth (lower part)
A
B
C
D
a.
2
4
3
5
b.
3
1
4
5
c.
3
2
1
4
d.
5
1
4
3
Which of the following are correctly
matched?
1. E.C. Semple .(Influence of
Geographic Environment)
2. C.O. Sauer
.
(The
Morphology of Land scapa)
3. R. E. Dickinson . (The
Nature of Geography)
4. E.Huntington ...
(Civilization and Climate)
Select the correct answer using the codes
given below:
a. 1 and 3
b. 2 and 3
c. 1, 2 and
d. 1 and 4
Match List I with List I and select the
correct answer:
List I
A. Ozone Hole
B. Greenhouse effect
C. Global warming
D. Albedo
List II
1. Difference in the outgoing and
incoming long wave radiation of the
atmosphere
2. Acceleration of ozone de-structure
3. Increase in the carbon dioxide level
4. Ratio of reflected radiation to radiation
received
A
B
C
D
a.
2
1
3
4
b.
4
1
3
2
c.
3
2
1
4
d.
2
3
4
1
Which of the following deposition
landforms are produced by glaciers?

66.

67.

68.

1. Roches moutonnces
2. Outwash plains
3. Eskers
4. U-shaped valleys
Select the correct answer using the codes
given below:
a. 1 and 2
b. 2 and 3
c. 3 and 4
d. 1 and 4
Match List I with List II and select the
correct answers:
List I (landform type)
A. Mariana trench
B. Sunda trench
C. Puerto Rica trench
D. Carlsberg ridge
List II (location)
1. Indian Ocean
2. West-Pacific Ocean
3. Arabian Sea
4. Atlantic Ocean
A
B
C
D
a.
2
1
4
3
b.
2
3
1
4
c.
4
1
2
3
d.
3
1
4
2
Match list I with List II and select the
correct answer:
List I (Continents)
A. Asia
B. Africa
C. North America
D. South America
List II (Deserts)
1. Atacama
2. Mojave
3. Kalahari
4. Gobi
A
B
C
D
a.
3
1
2
4
b.
4
3
1
2
c.
4
3
2
1
d.
2
4
3
1
Playas are depressions formed due to
a. deflating action of winds
b. glacial erosion
c. river erosion

69.

70.

71.

72.

9 of 14
d. marine erosion
Which of the following towns are not
associated with silk weaving industry r
India?
1. Murshidabad
2. Ludhiana
3. Vadodara
4. Ambala
Select the correct answer using the codes
given below:
a. 2, 3 and 4
b. 1, 2 and 4
c. 1, 2 and 3
d. 1, 3 and 4
Match List I with List II and select the
correct answer:
List I (Location of from and Steel Plants)
A. Bhilai
B. Rourkela
C. Durgapur
D. Burnpur
List II (Countries / Companies associated
with)
1. Indian Iron and Steel Co
2. Russia
3. Germany
4. Great Britain
A
B
C
D
a.
3
1
4
2
b.
4
3
2
1
c.
2
3
4
1
d.
2
4
1
3
Consider the fo1low places:
1. Ghazithad
2. Siliguri
3. Pune
4. Rourkela
Place(s) better suited for the location of
cement industry would include
a. l and 4
b. 3 and 4
c. 3 alone
d. 4 alone
Consider the following states
1. Uttar Pradesh
2. Bihar
3. Kerala
4. West Bengal

73.

74.

75.

76.

The correct sequence of the ascending


order of population density, as per 1991
census, of these state is
a. 1,2,3,4
b. 1,3,2,4
c. 2, 1, 3, 4
d. 2, 1,4, 3
Which of the following statements
regarding literate soils of
1. Laterites are generally red hi colour.
2. Laterites are rich in nitrogen and
potash.
3. These are welt developed in Rajasthan
and UP.
4. Tapioca and cashew nuts grow welt on
this soil.
Select the correct answer using the codes
given below:
a. 1, 2 and 3
b. 1 and 4
c. 2, 3 and 4
d. 2 and 3
The proportion of Scheduled Tribe
population to the total population is
highest in
a. Manipur
b. Meghalaya
c. Madhya Pradesh
d. Nagaland
The growth of population in India was
negative in which one of the following
Censual Decades
a. 1901 to 1911
b. 1911 to 1921
c. 1971 to 1981
d. None of the above
Match List I with List II and select the
correct answer:
List I (Types of minerals)
A. Non-metallic
B. Metallic
C. Refractory
D. Fertilizer
List II (Name of the minerals)
1. Kyanite
2. Gypsum
3. Asbestos
4. Wolfram
A
B
C
D

77.

78.

79.

80.

10 of 14
a.
3
2
4
1
b.
2
4
1
3
c.
3
4
1
2
d.
4
1
3
2
In the process of industrial development of
India, the investment per enterprise in the
public sector was the highest during the
a. First Five Year Plan
b. Third Five Year Plan
c. Fifth Five Year Plan
d. Seventh Five Year Plan
Match List I with List II and select the
correct answer:
List I (Soil Class)
A. Podsols
B. Terra Rossa
C. Chernozerns
D. Serozems
List II (Climatic types)
1. Temperate grasslands
2. Hot desert
3. Cool temperate
4. Mediterranean
A
B
C
D
a.
2
3
1
4
b.
3
4
1
2
c.
3
4
2
1
d.
4
1
3
2
The spatial variations in the wind velocity
in a weather map can be best explained by
a reference to the distribution of
a. barometric gradient
b. cyclones and depressions
c. fronts and air masses
d. highs and lows
Match List I with List II and select the
correct answer:
List I (Vegetation type)
A. Catinga
B. Downs
C. Maquis
D. Veldt
List II (Regions)
1. Algeria
2. Australia
3. Eastern Brazil
4. South Africa
A
B
C
D

81.

82.

83.

84.

a.
3
1
2
4
b.
2
4
1
3
c.
3
2
1
4
d.
4
2
3
1
Marxist
philosophy
of
population
dynamics states that
a. population growth is not checked by
the society
b. economic and social benefits can only
be enhanced by an increase in the
labour force c. nature will reduce surplus population
by war, diseases, famine and vice
d. man tends to reproduce prolifically
Match List I with List II and select the
correct answer:
List I (Plains)
A. Chile valley
B. Guinea plain
C. Russian platform
D. Western Europe
List II (Mode of occurrence)
1. Erosion plain
2. Glacial depositional plain
3. Piedmont plain
4. Structural plain
A
B
C
D
a.
3
1
4
2
b.
3
2
1
4
c.
2
3
4
1
d.
4
1
3
2
Consider the following statements
The problem of overpopulation in most of
Asia ii due to
1. large birth rate and large death rate.
2. inadequate
agro-Industrial
development.
3. immobility of the people.
4. the continent being the original home
of mankind.
Of these statements
a. 1 and 4 are correct
b. 2 and 3 are correct
c. 2 alone is correct
d. 1, 2, 3 and 4 are correct
Which of the following pairs are not
correctly matched?
1. Indonesians Australoid
2. North Africans Caucasold

85.

86.

87.

11 of 14
3. West Europeans Mongoloid
4. South Americans Negroid
a. 1 and 2
b. 1,3 and 4
c. 2 and 3
d. 2 and 4
Match List I with List U and select the
correct answer:
List I (Climatic region)
A. Tundra
B. Monsoon
C. Mediterranean sub tropical
D. Equatorial
List II (Location and area)
1. Continental western margins between
300 N and 400 S
2. Very high latitudes
3. Large areas of the
4. Continental interiors of tow to
moderate elevation along the equator
A
B
C
D
a.
2
3
1
4
b.
3
1
2
4
c.
4
3
1
2
d.
2
4
3
1
The most widely accepted scheme of
cultural realms of the world were devised
by
a. Spencer and Thomas
b. Broek and Webb
c. Haggett and Chorley
d. Dicken and Pitts
Match List I with List II and select the
correct answer:
List I (Races)
A. Semang
B. Hottentots
C. Polynesian
D. Nordic
List II (Place of habitation)
1. Northern plains of India
2. Malaysia
3. Kalahari
4. Hawaii
A
B
C
D
a.
4
2
3
1
b.
1
3
2
4
c.
2
3
4
1

88.

89.

90.

91.

d.
2
1
4
3
Match List I with List II and select the
correct answer:
List I
A. Masai
B. Berbers
C. Red Indians
D. Kirghiz
List II
1. North America
2. East Africa
3. Asia
4. N.W. Africa
A
B
C
D
a.
3
2
1
4
b.
2
3
4
1
c.
2
4
1
3
d.
1
4
2
3
Fohn and Chinook winds are experienced
in
a. the valleys of the northern Alps in
Switzerland and on the eastern slopes
of Rockies in North America
b. the eastern S of southern Alps in
Switzerland and on the we slopes of
Andes in South America
c. Po valley of France and in
Newfoundland
d. Netherlands and New Zealand
The main producers of coffee in Africa
include
a. Algeria, Libya and Egypt
b. Somaija and Mozambique
c. Angola Ivory Coast and Uganda
d. French West Africa Botswana and
Libya
Match List I with List II and select the
correct answer:
List I (Rice)
A. Highest acreage
B. Highest production
C. Highest yield
D. Highest exports
List II (Countries)
1. U.S.A.
2. Japan
3. India
4. China
A
B
C
D

92.

93.

94.

95.

96.

97.

12 of 14
a.
3
2
1
4
b.
3
4
2
1
c.
4
1
2
3
d.
2
4
3
1
The most suitable measure for soil
conservation is
a. afforestation
b. irrigation
c. crop rotation
d. contour farming
Synthetic rubber is likely to replace natural
rubber for domestic and industrial
purposes in future because
a. natural rubber is grown in the tropical
countries but is utilized in the rich and
industrialized temperate countries
b. natural rubber can hardly meet the
growing demand of various inductees
c. the raw materials for synthetic rubber
are available in various parts of the
world
d. the demand for rubber in the world is
gradually on the increase
The most salient contribution to the study
of Spatial diffusion processes was made
by
a. Hagerstrand
b. Haggett
c. Hartshorne
d. Hirschmann
Which one of the following areas in
eastern U.S still contains extensive stands
of forest?
a. Areas away from market centers
b. Areas with poor sandy soils and
mountains
c. Areas with gentle slope and good soils
d. Areas with heavy rainfall and high
temperature
The unified elemental complex in which
all the features of the physical, biotic and
societal environments are functionally
associated with mans occupancy of the
earth has been called
a. functional region
b. compage
c. ecosystem
d. natural region
The Mongol culture hearth was originally
situated in

98.

99.

100.

101.

102.

103.

a. Southern China
b. Northern Manchuria
c. Eastern China
d. Mongolia
Fishing in inland waters is most important
in
a. Russia
b. Anglo-America
c. Monsoon Asia
d. Central Africa
Literate soils are formed due to
a. removal of silicates, lime and organic
matter
b. decayed organic matter
c. disintegration of volcanic matter
d. deposition of limestone
Match List I with List II and select the
coned answer
List I
A. Iron and Steel
B. Automobile
C. Shipbuilding
D. Aircraft
List II
1. Dortmund
2. Seattle
3. Detroit
4. Portsmouth
A
B
C
D
a.
2
1
4
3
b.
2
3
1
4
c.
1
3
4
2
d.
1
4
3
2
Match List! with List II and select the
correct answer:
List I (Energy resources)
A. Coal production
B. Petroleum production
C. Petroleum deposits
D. Potential water power
A
B
C
D
a.
4
1
2
3
b.
4
2
3
1
c.
1
4
2
3
d.
3
1
4
2
Main hunting ground for blue whales is
located in
a. Arctic region
b. Antarctic region
c. West Pacific Ocean
d. South Atlantic Ocean
Africa
is
rich
in
hydro-electric
potentialities but lagging in the production

104.

105.

106.

107.

108.

109.

110.

13 of 14
of hydro-electricity. This is due to the lack
of
a. good sites for power plants
b. healthy climate for work
c. human efforts for development
d. raw materials for industry
The daily and annual range of temperature
is maximum in the
a. Savanna grassland
b. Temperate grassland
c. Hot desert
d. Equatorial region
The continent that has occupied the largest
area in the equatorial belt is
a. South America
b. Africa
c. Australia
d. Asia
Which one of the following has influenced
most, the location of industries in the
world?
a. Water
b. Petroleum
c. Coal
d. Natural gas
The largest producer of paper in the world
is
a. Canada
b. Japan
c. Sweden
d. USA
Maximum human population belongs to
the
a. Australoid
b. Negroid
c. Mongoloid
d. Caucasian
Infant mortality and birth rate tend to vary
together, which suggests that
a. decrease in one Leads to decrease in
the other
b. decrease in one leads to increase in the
other
c. increase in one leads to decrease in the
other
d. increase in one leads to increase in the
other
Consider the following South-East Asian
countries which produce 90% of the
worlds rubber
1. Indonesia
2. Malaysia
3. Sri Lanka
4. Vietnam

111.

112.

113.

114.

115.

116.

The coned sequence of these rubber


producers n decreasing order importance is
a. 1, 2, 3, 4
b. 2, 1, 3, 4
c. 1, 2, 4, 3
d. 2, 1, 4, 3
Horticulture is most widespread in the
a. Mediterranean regions
b. Subtropical regions
c. Temperate regions
d. Equatorial regions
Nitrate deposits are found in the northern
part of
a. Ecuador
b. Chile
c. Brazil
d. Colombia
The largest oil-field of Iraq is
a. Al-Faw
b. Halt Sulaiman
c. Kirkuk
d. Khor-al-Amaya
The largest urban agglomeration in China
is
a. Shanghai
b. Canton
c. Beijing
d. Wu-Han
The extensive coffee plantations in Brazil
are called
a. Fazendas
b. Trasil
c. Corrals
d. Estancias
Match List I with List If and se the correct
answer:
List I
A. Angola
B. Ghana
C. Tunisia
D. Lebanon
List II
1. Cacao
2. Citrus
3. Olive
4. Coffee
A
B
C
D
a.
4
3
2
1
b.
1
3
4
2
c.
4
1
3
2

117.

118.

119.

120.

14 of 14
d.
2
1
3
4
Rich coal deposits of China are found in
a. Shansi and Shensi
b. Shantung
c. Human
d. Kwangtung and Kwangsi
Match List I with List II and select the
correct answer:
List I (Classification of towns)
A. Confluence town
B. Port town
C. Religious town
D. Health resort
List II (Examples of towns)
1. Panchmarhi
2. Tuticorin
3. Samath
4. Allahabad
A
B
C
D
a.
1
2
3
4
b.
3
4
2
1
c.
4
2
3
1
d.
4
3
1
2
Match List I with List II and select the
correct answer:
List I (Countries)
A. Maynmar
B. Thailand
C. Kampuchea
D. Vietnam
List II (Name of important rivers)
1. Maenam Chao Phraya
2. Mekong
3. Irrawaddy
4. Red River
A
B
C
D
a.
1
4
2
3
b.
3
1
2
4
c.
3
2
1
4
d.
4
1
3
2
The important iron ore mining region
situate in Siberian Russia is
a. Krivoy Rog
b. Kursk
c. Magnitogorsk
d. Kuznetsk

C.S.E-Pre 1998

1 of 14

GEOGRAPHY
1.

2.

3.

4.

5.

6.

Which one of the following power plants


is not a thermal plant?
a. Faridabad
b. Dhuvaram
c. Idukki
d. Loktak
Match List I- with List II and select the
correct answer :
List I (Place of manufacturing)
A. Haridwar
B. Rishikesh
C. Korba
D. Bhatinda
List II (Industry)
1. Pharmaceuticals
2. Fertilizer
3. Heavy electrical
4. Aluminium
A
B
C
D
a.
3
1
2
4
b.
4
3
1
2
c.
3
1
4
2
d.
1
3
2
4
What proportion of urban population of
India is clustered in Class I cities?
a. One-third
b. Two-thirds
c. Three-fourths
d. Two-fifths
The correct sequence of number of urban
places with respect to their size-class in
India according to 1991 Census was
a. III, IV, V, VI
b. IV, III, VI, V
c. IV, III, V, VI
d. III.,V, IV, VI
The correct chronological sequence of
periods of Pleistocene Ice Age is
a. RissGunzWurmMindel
b. GunzMindelRissWurm
c. MindelRissGunzWurm
d. RissWurmGunzMindel
Match List I with List II and select the
correct answer :
List I (Concept)
A. Cultural Hearth

7.

8.

9.

B. Conurbation
C. Prime City
D. Structuralism
List II (Names)
1. Mark Jefferson
2. Patrick Geddes
3. Carl Sauer
4. C. Lovi Strong
A
B
C
D
a.
1
2
3
4
b.
1
3
4
2
c.
3
2
1
4
d.
3
1
4
2
The correct sequence of Von Thunes
optional land use rings is a. cereal fanning, woodlands, marketgardening, grazing
b. market-gardening, woodlands, cereal
farming, grazing
c. market-gardening, cereal farming,,
grazing, woodlands
d. Grazing, market-gardening, cereal
farming, woodlands
A survey which is specifically concerned
with landed property is known as
a. Cadastral Survey
b. Geodelic Survey
c. Thematic Survey
d. Triangulation Survey
Match List I with List II and select-the
correct answer:
List I (Old Names)
A. Azrak
B. Ister
C. Paulus Meotus
D. Raha
List II New Names)
1. Azov
2. Blue Nile
3. Danube
4. Volga
A
B
C
D
a.
2
4
1
3
b.
1
4
2
3
c.
2
3
1
4
d.
1
3
4
2

10.

11.

12.

13.

Consider the following statements:


Low soil fertility in Rainy Tropics can be
caused by
1. high temperature and heavy rainfall.
2. eluviations and leaching.
3. the existence of an extremely active
bacterial life on the forest floor.
Of these statements
a. 1,2 and 3 are correct
b. 1 and 2 are correct
c. 2 and 3 are correct
d. 1 and 3 are correct
Which of the following leaf modifications
occur in the desert areas to inhibit water
loss?
1. Hard and waxy leaves
2. Tiny leaves or no leaves
3. Drip-tip leaves
4. Thorn instead of leaves
Select the correct answer using the codes
given below:
a. 1, 2 and 3
b. 2 and 3
c. 1, 2 and 4
d. 1 and 4
A region with summer temperature of 20
C, winter temperature of15 C, annual
rainfall of about 50 cm, mainly in spring
and early summer and coniferous
vegetation would lie in the
a. cool temperature marginal type
environment
b. prairie type environment
c. cool temperature central or Siberian
type environment
d. Manchurian type environment
Consider the following statements :
1. The distribution of population in the
southern continents is peripheral.
2. Among the continents. Europe has the
highest density of population.
3. The most densely populated region of
the USA is its Californian coast.
4. Equatorial regions generally have low
density of population due to dense
forest.
Of these statements
a. 1, 3 and 4 arc correct
b. 1,2 and 4 are correct
c. 2, 3 and 4 are correct
d. 1 and 2 are correct

14.

15.

16.

17.

18.

2 of 14
In the given outline map of the world, the
shaded areas indicate the cultivation of

a. barley
b. oats
c. maize
d. wheat
The most important type of forest in terms
of economic value and easy exploitation is
a. temperature deciduous forests
b. coniferous forests
c. tropical monsoon forests
d. equatorial forest
Which one of the following countries has
the highest percentage of developed water
power to the available potential?
a. Canada
b. France
c. Japan
d. Norway
Match List I with List II and select the
correct answer
List I (Location)
A. Osaka
B. Sheffield
C. Seattle
D. Magnitogorsk
List II (Industry)
1. Aircraft
2. Cuttery
3. Steel
4. Cotton textiles
A
B
C
D
a.
2
3
1
4
b.
4
2
1
3
c.
3
1
2
4
d.
4
1
2
3
The largest producer of cloves in the world
is
a. Comoros
b. Mauritius
c. Seychelles
d. Zanzibar

3 of 14
19.

20.

21.

22.

23.

Which one of the following sets of


commodities is imported by India from
South-West Asian countries?
a. Raw wool and carpets
b. Dates and Olives
c. Precious stones and pearls
d. Perfumes and coffee
In the given rough outline map of U.S.A.
the cities labelled 1, 2, 3 and 4 stand
respectively for

a. Seattle. San Diego, Denver and


Phoenix
b. Seattle. San Diego, Phoenix and
Denver
c. San Diego, Seattle., Phoenix and
Denver
d. San Diego. Seattle, Denver and
Phoenix
The Ural mountains were formed in
a. Hercynian period
b. Permean period
c. Tertiary period
d. Archaean period
The leading iron and steel producing
region of China is located in
a. Szechwan
b. Northern China
c. Yangtse Valley
d. Yunan plateau
Identify the islands marked 1,2,3 and 4 in
(he given map and select the correct
answer using the codes given below :

24.

25.

26.

A. Car Nicobar
B. Little Andaman
C. Great Nicobar

D. Little Nicobar
A
B
C
D
a.
3
4
1
2
b.
4
3
2
1
c.
4
3
1
2
d.
3
4
2
1
The dotted line in the map shows

a. average annual temperature divide


b. annual isohyets of 200-400cm
c. all-India water divide
d. contour of 300 meters
Match types of solid distribution shown in
the rough outline map of India with the
region labelled 1, 2, 3 and 4 and select the
correct answer using the codes given
below :
Soil Types
A. Red and yellow
B. Grey and brown
C. Mixed red and black
D. Red loamy

A
B
C
D
a.
1
2
3
4
b.
2
1
4
3
c.
1
2
4
3
d.
2
1
3
4
Consider the following principal group
crops of India ;
1. Pulses
2. Fibre crops
3. Oil seeds
4. Plantation crops
The correct sequence in descending order
principal groups of crops with respect to
the share in the total cropped area is
a. 1, 3, 2, 4
b. 1, 3, 4, 2

27.

28.

29.

30.

31.

c. 3, 1, 2, 4
d. 3, 1, 4, 2
Which one of the following pairs is not
correctly matched?
a. Bhittan Kanika: Olive Ridley turtles
b. Dalma Hill : Wild Elephants
c. Kaziranga : One-horned Rhinos
d. Dachigam : Asiatic Lions
Match List 1 with List II and select the
correct answer :
List I (Minerals)
A. Gondwana Coal
B. Lignite
C. Mineral Oil
D. Bauxite
List II (Places of occurrences)
1. Neyeli
2. Lohardaga
3. Talcher
4. Kolol
A
B
C
D
a.
1
3
4
2
b.
1
3
2
4
c.
3
1
2
4
d.
3
1
4
2
In terms of total energy generation in India
the correct sequence in descending order
of the concerned zones is
a. Southern, Western, Eastern, Northern
b. Western, Southern, Northern, Eastern
c. Western, Northern, Southern, Eastern
d. Southern, Eastern, Western, Northern
Match List I with List 11 and select the
correct answer :
List I (Industries)
A. Steel
B. Newsprint
C. Heavy Water
D. Shipbuilding
List II (Local ion)
1. Thai
2. Salem
3. Nepanagar
4. Mazgaon
A
B
C
D
a.
3
2
4
1
b.
3
2
1
4
c.
2
3
4
1
d.
2
3
1
4
Match List I with List II and select the
correct answer:

32.

33.

34.

35.

4 of 14
List I (State)
A. Assam
B. Gujarat
C. Karnataka
D. Maharashtra
List II (Population in millions)
1. 78.9
2. 44.9
3. 22.4
4. 41.3
A
B
C
D
a.
3
4
1
2
b.
3
4
2
1
c.
4
3
1
2
d.
4
3
2
1
Which one of the following pairs is
correctly matched?
a. A.N. Strahler
: Pediplain
b. W.N. Davis
: Peneplain
c. L.C. King : Panplain
d. C.H. Crickmay : Slope equilibrium
The approach in the study of geography
which claims that geography should be
concerned essentially with the formulation
of scientific laws is known as the
a. Idiographic Approach
b. Nomothetic Approach
c. Positivistic Approach
d. Radical Approach
Consider the following statements :
Japan is one of the leading industrial
countries in the world because it has
1. developed hydel power.
2. large deposits of metallic mineral.
3. high technological capability.
4. insular location.
Of these statements
a. 1, 2, 3 and 4 are correct
b. 1, 2 and 3 are correct
c. 1 and 3 are correct
d. 2 and 4 are correct
Match List I with List II and select the
correct answer:
List I
A. Isopleths maps are best suited for
B. Do not maps are useful for
C. Scatter groups are essential for
D. Star diagrams are suitable for
List II
1. Showing natural grouping
2. Showing distribution of population

36.

37.

38.

3. Indicating wind direction


4. Depicting changes which are relatively
gradual
A
B
C
D
a.
4
2
3
1
b.
2
4
3
1
c.
4
2
1
3
d.
2
4
1
3
Match List I with List II and select the
correct answer :
List I (Old name)
A. Albion
B. Euxine
C. Mare inlemum
D. Taprobana
List II (New name)
1. Btacksea
2. England
3. Sri Lanka
4. Mediterranean Sea
A
B
C
D
a.
1
4
3
2
b.
2
4
1
3
c.
2
1
4
3
d.
1
2
3
4
Consider the following statements :
1. Rice is predominantly cultivated in the
tropical areas.
2. USA is one of the exporters of rice.
3. Thailand is an important importer
office.
4. In Italy, rice is cultivated in the Povalley.
Of these statements
a. 1,2,3 and 4 are correct
b. 1,2 and 4 are correct
c. 3 and 3 are correct
d. 2 and 4 are correct
Consider the following statements
regarding equatorial forests :
1. They consist mostly of hardwood trees.
2. In very' dense areas, under growth is
absent.
3. There is segregation of the same
species of trees.
4. Lumbering is well developed in these
forests.
Of these statements
a. 1, 2 and 4 are correct
b. 2, 3 and 4 are correct
c. 1 and 2 are correct

39.

40.

41.

42.

43.

44.

5 of 14
d. 1 and 3 are correct
Which one of the following mining areas
is significant for a different mineral from
that of the other three areas?
a. Kuznetsk
b. Fushun
c. Karaganda
d. Kirkuk
Match List I with List II and select the
correct answer :
List I (Rivers)
A. Niger
B. Kasai
C. Orange
D. Nile
List II (Countries)
1. Egypt
2. S. Africa
3. Zaire
4. Mali
A
B
C
D
a.
4
3
1
2
b.
3
4
2
1
c.
3
4
1
2
d.
4
3
2
1
In Iran during the ancient period, the
system of utilizing the under ground water
table was known as
a. Wadi
b. Karez
c. Aquifer stratum
d. Kanats (Qanats)
The weir and canal connects
a. Lake Ontario and Erie
b. Lake Huron and Superior
c. Lake Michigan and Ontario
d. Lake Superior and Erie
The Aral Sea is located between
a. Kazakhstan and Turkmenia
b. Uzbekistan and Turkmenia
c. Kazakhstan and Uzbekistan
d. Uzbekistan and Tajikistan
The Amarkantak Hill is the source of two
rivers flowing in two different directions
(west and east). They are
a. Narmada and Tapi
b. Narmada and Mahanadi
c. Tapi and Belwa
d. Tapi and Son

45.

46.

47.

48.

The shaded area in the given rough outline


map represents which one of the following
types of climatic regions (after Koppen)?

a. Amw
b. BS hw
c. BS
d. Cwg
Bandland topography is characteristic of
the
a. Gulf of Kutch
b. Sundarban Delta
c. Konkan coast
d. Chambal valley
Match List I with List II and select the
correct answer
List I (Crop)
A. Tobacco
B. Bajra
C. Potato
D. Groundnut
List II (Stale leading in its production)
1. Rajasthan
2. Uttar Pradesh
3. Gujarat
4. Andhra Pradesh
A
B
C
D
a.
3
1
2
4
b.
1
2
3
4
c.
4
1
3
2
d.
4
1
2
3
Which of the following pairs is/are
correctly matched?
1. Korba coal mine : Orissa
2. Khetri copper mine: Rajaslhan
3. Kodarma mica mines: Madhya
Pradesh
Select the correct answer using the codes
given below:
a. 1, 2 and 3
b. 2 and 3
c. 2 alone
d. 1 and 3

49.

50.

51.

52.

6 of 14
Match List I with List II and select the
correct answer :
List I (Female literacy rate)
A. 78.60
B. 72.89
C. 66.99
D. 65.46
List II (State/Union Territories)
1. Delhi
2. Andaman and Nicobar Islands
3. Mizoram
4. Lakshadeep
A
B
C
D
a.
4
3
1
2
b.
3
4
2
1
c.
3
4
1
2
d.
4
3
2
1
Which one of the following sets of types
actions occurs in and topography?
a. Deflation, Attrition, Corrosion
b. Attrition, Corrosion, Abrasion
c. Deflation, Abrasion
d. Deflation, Attrition
Which of the following pairs are correctly
matched?
1. Karst topography: Blind valley
2. Humid topography: Stone lattice
3. Aeolian topography: Ventifacts
4. Periglacial topography: Pingo
Select the correct answer using the codes
given below :
a. 1, 2 and 3
b. 1, 3 and 4
c. 1, 2 and 4
d. 2, 3 and 4
Match List I with List II and select the
correct answer :
List I
A. Shifting cultivation
B. Pastoralism
C. Hunters and food gatherers
D. Hunters
List II
1. Pygmies
2. Eskimos
3. Kirghiz
4. Rengmas
A
B
C
D
a.
1
2
3
4
b.
1
3
2
4
c.
4
1
3
2

53.

54.

55.

56.

57.

d.
4
3
1
2
Some types of permanent adjustments of a
theologize during survey are given below :
1. Adjustment of the line of collimation.
2. Adjustment of the vertical index frame.
3. Adjustment of the plate levels.
4. Adjustment of the horizontal axis.
Select the correct answer using the codes
given below :
a. 1,3,4,2
b. 3,1,2,4
c. 1,3,2,4
d. 3,1,4,2
The approach in Geography which states
that 'actions are motivated by cognitive
process' is the essence of
a. Behaviouralism
b. Humanism
c. Positivism
d. Radicalism
Match List I with List II and select the
correct answer:
List I (Location)
A. Kiruna
B. Kalgoorlie
C. Karaganda
D. Kirkuk
List II (Mineral)
1. Coal
2. Petroleum
3. Iron ore
4. Gold
A
B
C
D
a.
2
4
1
3
b.
3
4
1
2
c.
3
4
2
1
d.
1
3
4
2
The Katanga copper and gold mines are
located in
a. The Republic of South Africa
b. The Republic of Congo
c. Zimbabwe
d. Zambia
Match List I (cities in USA) with List II
(factors for which they are known) and
select the correct answer:
List I
A. Chicago
B. St. Louis
C. Detroit
D. Kansas

58.

59.

60.

7 of 14
List II
1. Greatest centre of automobile industry
2. Biggest railways junction
3. Important river port
4. Famous livestock market
A
B
C
D
a.
3
2
4
1
b.
3
2
1
4
c.
2
3
4
1
d.
2
3
1
4
The given diagram represents:

1. Young
2. Old
3. Oldest
a. Monoclinal fold
b. Overfold
c. Recumbent fold
d. Symmetrical fold
The shaded area in the rough outline map
of India shows:

a. summer rainfall of 80-120 cm


b. summer humidity of 80-90 percent
c. January pressure ranging between
1016-1018 mb
d. winter rainfall of 5-10 cm
Match List I with List 11 and select the
correct answer :
List I (Mineral area)
A. Gurumahisani
B. Talcher
C. Jaduguda
D. Zawar
List II (Mineral)
1. Zinc
2. Uranium
3. Iron ore
4. Coal
A
B
C
D
a.
4
3
1
2

61.

62.

63.

64.

65.

b.
2
4
3
1
c.
3
1
2
4
d.
3
4
2
1
Following
relief
zones
are
the
characteristics of ocean basins
1. Continental slope
2. Continenal shell
3. Deep sea plain
4. Oceanic trench
The correct sequence of their seaward
appearance from the coasts is
a. 2,1,4,3
b. 1,2,4,3
c. 2,1,3,4
d. 1,2,3,4
When point symbols are inserted over nonquantitative shaded maps showing spatial
distribution, the map is
a. Chorochromatic
b. Choroschematic
c. Choropleth
d. Chorographic compage
The normal lapse rate of temperature in the
troposphere is 1 C. for every
a. 146m
b. 156m
c. 166m
d. 176m
The theory that events can only be
explained as stages in a movement towards
a pre-ordained end is known as
a. Functionalism
b. Humanism
c. Phenomenology
d. Teleology
Which of the following pairs are correctly
matched?
1. Winter rain in North West India :
Western disturbances
2. Summer rain in Malabar coast
:
Retreating monsoon
3. Summer rain in Bengal basin : Nor
westerly
4. Winter rain in Tamil Nadu coast :
N.E. Monsoon
Select the correct answer using the codes
given below :
a. 1, 2, 3 and 4
b. 1, 3 and 4
c. 3 and 4
d. 1 and 2

66.

8 of 14
Which one of the following is the correct
sequence in terms of descending values of
Albedo?
a. Forest, snow, water
b. Snow, forest, water
c. Water, snow, forest
d. Forest, water, snow

Directions:- The following 24 (twenty four) items


consist of two statements, one labelled as
Assertion A and the other labelled as 'Reason R'.
You are to examine these two statements carefully
and decide if the Assertion A and the Reason R
are individually true and if so, whether the Reason
is a correct explanation of the Assertion. Select
the answers to these items using the codes given
below and mark your answer sheet accordingly.
67.

68.

69.

Assertion (A): Corals ire not found near


the shores where rivers meet the sea.
Reason (R): Corals do not thrive in regions
where waters contain a of sill.
a. Both A and R are true and R is the
correct explanation of A
b. Both A and R are the but R is NOT the
correct explanation of A
c. A is true but R is false
d. A is false but R is true
Assertion (A): The temperature recorded at
the surface of enclosed seas of high
latitudes is lower than the temperature
recorded at relatively greater depth.
Reason (R): The surface of the ocean
directly receives insulation and heat is
transmitted to the lower part of the ocean
through the process of conduction.
a. Both A and R are true and R is the
correct explanation of A
b. Both A and R are the but R is NOT the
correct explanation of A
c. A is true but R is false
d. A is false but R is true
Assertion (A): The Tropical Cyclones are
thermal in origin.
Reason (R): The Tropical Cyclones
develop due to Carioles force.
a. Both A and R are true and R is the
correct explanation of A
b. Both A and R are the but R is NOT the
correct explanation of A
c. A is true but R is false
d. A is false but R is true

70.

71.

72.

73.

74.

Assertion (A): Human Geography is the


synthetic study of the relationship between
human societies and the Earths surface,
Reason (R): It includes all those parts of
Geography which are not solely concerned
either with the physical environment or
Cartography.
a. Both A and R are true and R is the
correct explanation of A
b. Both A and R are the but R is NOT the
correct explanation of A
c. A is true but R is false
d. A is false but R is true
Assertion (A): The Homo sapiens are
generally
divided
into
Caucasoid.
Mongoloid. Negroid and Australoid races.
Reason (R): Such a classification leads to
racism.
a. Both A and R are true and R is the
correct explanation of A
b. Both A and R are the but R is NOT the
correct explanation of A
c. A is true but R is false
d. A is false but R is true
Assertion (A): Freight rates decrease with
increasing distance in the case of water
transport.
Reason (R): Terminal changes are
reduced.
a. Both A and R are true and R is the
correct explanation of A
b. Both A and R are the but R is NOT the
correct explanation of A
c. A is true but R is false
d. A is false but R is true
Assertion (A): Humanistic Geography
gives the central and active role to human
awareness and human agency.
Reason (R): Humanistic approach to
developed as a reaction of positivism.
a. Both A and R are true and R is the
correct explanation of A
b. Both A and R are the but R is NOT the
correct explanation of A
c. A is true but R is false
d. A is false but R is true
Assertion (A): The origin of Himalayas is
due to the collision of Indian Subcontinent
with European landmass.
Reason (R): The convergence of
crystallites is often associated with
mountain building.

75.

76.

77.

78.

9 of 14
a. Both A and R are true and R is the
correct explanation of A
b. Both A and R are the but R is NOT the
correct explanation of A
c. A is true but R is false
d. A is false but R is true
Assertion (A): Considering the worlds
wind systems, the waterlines normally
below between 400C to 600 latitudes.
Reason (R): The flow of air from sub
tropical high pressure cells to the
temperature zone of pressure leads to the
formation of mid latitudinal plane wind
system.
a. Both A and R are true and R is the
correct explanation of A
b. Both A and R are the but R is NOT the
correct explanation of A
c. A is true but R is false
d. A is false but R is true
Assertion (A): The first major group of
migrants from the United Kingdom to
North eastern USA was to Pilgrim
Fathers.
Reason (R):
Roman Catholics were
allowed to practice the religion fully in the
UK at that time.
a. Both A and R are true and R is the
correct explanation of A
b. Both A and R are the but R is NOT the
correct explanation of A
c. A is true but R is false
d. A is false but R is true
Assertion
(A):
In
Kazakhstan,
transhumance is practiced.
Reason (R): There is a seasonal shifting of
snowline in the mountains.
a. Both A and R are true and R is the
correct explanation of A
b. Both A and R are the but R is NOT the
correct explanation of A
c. A is true but R is false
d. A is false but R is true
Assertion (A): The northern limits of
cotton belt in the USA follow 200 days
frost-free line.
Reason (R): Frost helps the plant to get its
required moisture.
a. Both A and R are true and R is the
correct explanation of A
b. Both A and R are the but R is NOT the
correct explanation of A
c. A is true but R is false

79.

80.

81.

82.

83.

d. A is false but R is true


Assertion (A): Japan is one of the leading
countries of the world in fish catch.
Reason (R): Around Japan, the continental
shelf is very wide.
a. Both A and R are true and R is the
correct explanation of A
b. Both A and R are the but R is NOT the
correct explanation of A
c. A is true but R is false
d. A is false but R is true
Assertion (A): Chicago-Gary district is one
of the important steel producing regions of
the USA.
Reason (R): Location of Chicago at the
head of Lake Michigan has helped its steel
industry.
a. Both A and R are true and R is the
correct explanation of A
b. Both A and R are the but R is NOT the
correct explanation of A
c. A is true but R is false
d. A is false but R is true
Assertion (A): Rhine-Ruhr valley became
the most important industrial region of
Europe because of the presence of large
coal-fields.
Reason (R): The industries such as
locomotives, machinery, cutlery and
armaments are concentrated in the RhineRuhr valley.
a. Both A and R are true and R is the
correct explanation of A
b. Both A and R are the but R is NOT the
correct explanation of A
c. A is true but R is false
d. A is false but R is true
Assertion (A): Japan is the largest
producer of heavy engineering products.
Reason (R): Most of the ship building
yards are located near the ports.
a. Both A and R are true and R is the
correct explanation of A
b. Both A and R are the but R is NOT the
correct explanation of A
c. A is true but R is false
d. A is false but R is true
Assertion (A): In most of the tropical
mainland of Southeast Asia, rainfall is low
in central areas.
Reason (R): The north-south mountain
ranges cause rain shadow effect in the
interior areas.

84.

85.

86.

87.

88.

red.

10 of 14
a. Both A and R are true and R is the
correct explanation of A
b. Both A and R are the but R is NOT the
correct explanation of A
c. A is true but R is false
d. A is false but R is true
Assertion (A): In spite of a fairly long
coastline China has been not able to
develop coastal fishery to a large extent.
Reason (R): Most of the China's
population lives away from the coast.
a. Both A and R are true and R is the
correct explanation of A
b. Both A and R are the but R is NOT the
correct explanation of A
c. A is true but R is false
d. A is false but R is true
Assertion (A): Mauusynram is the wettest
place in India.
Reason (R): It revives heavy rainfall
throughout the year.
a. Both A and R are true and R is the
correct explanation of A
b. Both A and R are the but R is NOT the
correct explanation of A
c. A is true but R is false
d. A is false but R is true
Assertion (A): The Himalayan meadows
are suitable for transshumance.
Reason (R): In these areas grasses grow in
abundance during summers.
a. Both A and R are true and R is the
correct explanation of A
b. Both A and R are the but R is NOT the
correct explanation of A
c. A is true but R is false
d. A is false but R is true
Assertion (A): Indira Gandhi Canal
irrigates large areas of western Rajasthan.
Reason (R): It is led by the Ravi river in
Rajasthan.
a. Both A and R are true and R is the
correct explanation of A
b. Both A and R are the but R is NOT the
correct explanation of A
c. A is true but R is false
d. A is false but R is true
Assertion (A): The hydel power plants in
Maharashtra are operated by the rivers
coming down from the Western ghats.
Reason (R): Most of the rivers are rain-

11 of 14

89.

90.

91.

92.

a. Both A and R are true and R is the


correct explanation of A
b. Both A and R are the but R is NOT the
correct explanation of A
c. A is true but R is false
d. A is false but R is true
Assertion (A): The once prosperous
Hooghly-side Industrial Complex is now
showing signs of decay.
Reason (R): The decline is mainly because
of the silting of Hooghly channel.
a. Both A and R are true and R is the
correct explanation of A
b. Both A and R are the but R is NOT the
correct explanation of A
c. A is true but R is false
d. A is false but R is true
Assertion (A): Volcanic soils are usually
fertile.
Reason (R): The heat of eruption causes
fertility on such soils.
a. Both A and R are true and R is the
correct explanation of A
b. Both A and R are the but R is NOT the
correct explanation of A
c. A is true but R is false
d. A is false but R is true
Which of the following factors supports
the hypothesis of the permanency of ocean
basins and continents?
1. The cross-section of the ocean basin,
comprising the broad abyssal trough
flanked by rim-like continental shelves
2. Occurrence of shallow-water sands and
muds limited to the continental shell
and slope, while that of pelagic oozes
being limited to the abyssal plain.
3. Absence of normal sedimentary rocks
on the oceanic islands
Select the correct answer using the codes
given below ;
a. 2 and 3
b. 1 and 3
c. 1 and 2
d. 1, 2 and 3
The birth rate (1) and death rate (2) are
marked on the graph given below
illustrating demographic transition. What
does the third continuous line (3) on the
graph show

93.

94.

95.

a. Fertility rate
b. Growth rate
c. Life expectancy
d. Total population
Match List I with List II and select the
correct answer:
List I
A. Hematite
B. Copper
C. Uranium
D. Manganese
List II
1. Production of electricity
2. Manufacture of steel
3. Manufactures of electrical appliances
4. Manufacture of alloy paints and
batteries
A
B
C
D
a.
3
1
2
4
b.
2
3
4
1
c.
2
3
1
4
d.
3
4
1
2
Which one of the following is the most
suitable projection for showing TransSiberian Railway ?
a. Cylindrical Equal Area Projection
b. Mercator's Projection
c. Conical with one Standard Parallel
d. Polyconic Projection
Match List 1 with List II and select the
correct answer:
List I
A. Lebensraum
B. Pays
C. Orbit Terrarum
D. Terra Incognita
List II
1. la Blache
2. Solinus
3. Ptolemy
4. Ratzel
A
B
C
D
a.
2
3
1
4
b.
1
4
2
3
c.
4
1
3
2

96.

97.

98.

99.

100.

d.
4
1
2
3
Which one of the following is a true
example of a Median Mass?
a. Hungarian Plain
b. Indo-Gangetic Plain
c. Hwangho Plain
d. Danube Plain
The world-wide jet stream that occurs in
winter above the troposphere resulting
from a very steep stratospheric thermal
gradient is the
a. Arctic jet stream
b. Sub-polar jet stream
c. Polar-night jet stream
d. Sub-tropical jet stream
Consider the following statements :
1. Mathogany is a native of the
Equatorial forests.
2. Cactus plants arc natives of areas of
heavy precipitation.
3. Fir trees are found widely in the
Amzon basin.
4. Tea is a plant of cool-dry climates.
Of these statements
a. 1 alone is correct
b. 1, 2 and 3 are correct
c. 1, 3 and 4 are correct
d. 2 and 4 are correct
The Amazon basin remains a backward
region because of its
a. lack of natural resources
b. inaccessibility
c. primitive tribal ways of life
d. climatic hazards
Consider the following statements :
1. As per the optimum theory of
population per capital income with
increase with population growth in
under populated areas.
2. According to Malthus, population
doubles once in every 20 years.
3. Celibacy,
self-control and
late
marriages are the preventive checks
postulated by Malthus.
4. According to Malthus wars, earthquake
and floods are some of the positive
checks on population growth.
Of these statements
a. 1, 3 and 4 are correct
b. 1, 2 and 4 are correct
c. 2, 3 and 4 are correct
d. 1, 2 and 3 are correct

101.

102.

103.

104.

105.

106.

12 of 14
Xanthoderons is the term of skin applied to
a. Caucasoid Race
b. Negroid Race
c. Mongoloid Race
d. Australoid Race
Which one of the following pairs is
correctly matched?
a. Bushmen :
Amazon basin
b. Badouin :
Arabia
c. Eski mo :
Greenland
d. Semand :
Malaysia
In the world, viniculture, horticulture,
sericulture are well developed in the
a. monsoon regions
b. tropical highlands
c. mediterranean regions
d. marine west coast areas
Consider the following statements :
1. Most of the fishing grounds occur in
areas where the continental shelf is
wide
2. Fishing activity is well developed in
warm tropical waters
3. Mixing of warm and cold currents
bring plant nutrients for fish
4. Inland fisheries are more significant
than other types in India.
Select the correct answer using the codes
given below:
a. 1 and 4 are correct
b. 1 and 3 are correct
c. 2, 3 and 4 are correct
d. 1,2 and 3 are correct
On global perspective, the pattern of
energy consumption (in descending order
of proportion) is
a. Coal Oil- Natural Gas - Hydel
power
b. Oil-Coal - Hydel power - Natural Gas
c. Oil- Coal - Natural Gas -Hydel power
d. Coal Oil - Hydel power - Natural
Gas
Consider the following statements :
1. the advantages of art early start.
2. availability of raw material.
3. availability of skilled labour.
4. large local market.
Of these statements
a. 1,2 and 3 are correct
b. 1,2 and 4 are correct
c. 1 and 3 are correct
d. 2, 3 and 4 are correct

13 of 14
107.

108.

109.

110.

In the given map A' refers to

a. Alexandria
b. Port Said
c. Suez
d. Cairo
Match List I with List II and select the
correct answer:
List I
A. N. Vietnam
B. S. Vietnam
C. China
D. Myanmar
List II
1. Si-Kiang
2. Salween
3. Red River
4. Mekong river
A
B
C
D

111.

a.
3
1
4
2
b.
2
1
3
4
c.
3
4
1
2
d.
4
2
1
3
Which of the following statements are
correct regarding S.W. Asia?
1. This region produces almost 35%
crude oil of the world
2. The crude oil is mostly refined at
Abadan and Kirkuk
3. It is mostly exported to East Asia
4. Pipelines are the main mode of
transport
Select the correct answer using the codes
given below:
a. 1, 2 and 4
b. 1, 2 and 3
c. 1 and 2
d. 2, 3 and 4
The shaded landform region in the given
Anglo-American map is

112.

113.

114.

a. Rocky mountains
b. Interior highlands
c. Great plains
d. Interior plains
The railway stations marked 1,2, 3, and the
given map in a sequential order from to
east are

a. Omsk, Irkutsk, Tomsk, Krasnoyarsh


b. Omsk, Tomsk, Irkutsk, Krasnoyarsh
c. Tomsk, Omsk, Krasnoyarsh, Irkutsk
d. Omsk, Tomsk, Krasnoyarsh, Irkutsk
The shaded region in the map of China
represents

a. Tarim basin
b. Tsaidam basin
c. Red basin
d. Dzungarian basin
Shillong Plateau is also known as
'Meghalaya' meaning 'abode of clouds'. It
was so named by
a. O. H. K. Spate
b. S. P. Chalterjee
c. D. N. Wadia
d. R. L. Singh
The climatic graph given below indicates
rainfall and temperature conditions of a
Average monthly rainfall

14 of 14

115.

116.

117.

1. __ Average Mean Temp.


2. Average Mean Temp.
a. Chandigarh
b. Delhi
c. Jaipur
d. Lucknow
Match List I with List II and select the
correct answer:
List I (Soil Types)
A. Katerute
B. Saline
C. Black
D. Red loam
List II (Places of occurrence)
1. Rahr plain
2. Maharashtra plain
3. Rajmahal hills
4. Kutch region
A
B
C
D
a.
4
3
2
1
b.
4
3
1
2
c.
3
4
2
1
d.
3
4
1
2
In terms of different sources of irrigation
(viz. canals, wells, tanks and others),
which one of the following states strikes a
balance between all?
a. Uttar Pradesh
b. Andhra Pradesh
c. Tamil Nadu
d. Karnataka
Consider the map given below:
In the above map the regions market 1, 2,
3 and 4 are respectively known for their
cultivation of

118.

119.

120.

a. Sugarcane, Ragi, Cotton and Gram


b. Sugarcane, Gram, Cotton and Ragi
c. Gram, Sugarcane, Cotton and Ragi
d. Cotton, Ragi, Gram and Sugarcane
Reserved Forest is a forest
a. totally under government control
b. meant maily for tribal people
c. earmarked for commercial exploitation
with restricted grazing
d. without any economic activity
Unlike other parts of the Indian Coast,
fishing industry has not developed along
the Saurashtra cost because
a. there are few indentations suitable for
fishing
b. of overwhelming dependence on
agriculture and animal husbandry
c. the sea water is relatively more saline
d. of industrial development leading to
widespread pollution of coastal area
The correct sequence in decreasing order
in terms of total production of the given
non-ferrous metals in Indian is
a. Lead, Zinc, Copper, Aluminium
b. Zinc, Lead, Aluminium, Copper
c. Aluminium, Zinc, Copper, Lead
d. Aluminium, Copper, Zinc, Lead

C.S.E-Pre 1999

1 of 15

GEOGRAPHY
1.

2.

3.

Match List I with list Hand select the


correct answer:
List I (Geological Era)
A. Mesczoic
B. Protcrozoic
C. Cainozoic
D. Palaezoic
List II (Geological/Period)
1. Tertiary
2. Triassic
3. Pre
4. Permian
A
B
C
D
a.
3
2
1
4
b.
2
3
1
4
c.
2
3
4
1
d.
3
2
4
1
Conceder the following statements:
1. An approach to Human Geograj that
emphasizes questions of inequality is
known as Welfare Geography.
2. An approach to Human Geography that
gives central) and active role to human
awareness is known as Behavioural
Geography
3. Radical Geography developed as a
reaction to Spatial Analys
Of these statements:
a. 1 alone is correct
b. 1 and 2 are correct
c. 1 and 3 are correct
d. 2 and 3 are correct
Match List! with List I! and select the
correct answer:
List I (Rivets)
A. Sac Fransisco
B. Mississippi
C. Niger
D. Suqehanna
List II (Shapes of their deltas)
1. Aicuate
2. Digitate
3. Estuarine

4.

5.

6.

4. Cuspate
A
B
C
D
a.
4
2
1
3
b.
4
2
3
1
c.
2
4
3
1
d.
2
4
1
3
En given map, the Himalayan peaks
Makalu, Annapuma, Kanchenjunga and
Dhaulagiri have been labdled respectively
as

a. 2, 3, 1 and 4
b. 3, 2, 4 and 1
c. 3, 2, 1 and 4
d. 2, 3, 4and 1
Which one of the following patterns of
drainage does the given diagram
represent?

a. Annular
b. Dendritic
c. Radial
d. Trellis
Who among the following is regarded as
the founder of Exceptionalism in
Geography?
a. Blaut
b. Haggett
c. Harvey
d. Hartshorne

7.

8.

9.

10.

11.

12.

Which of the following pails are correctly


matched?
1. Tienshan RDU mountain
2. Vosges
Structural dome
3. Ferghana Deep-seated
4. Mama Loa Volcanic cone
a. 1, 2, 3 and 4
b. 1, 3 and 4
c. 2 and 4
d. 1, 2and3
Which one of the following is at the lowest
height above the Mean Sea Level?
a. Delhi
b. Jodhpur
c. Rota
d. Nagpur
The doctrine of unoformitarianism, that is,
The present is the key to the past was
advocated by
a. Dutton; CE.
b. Gilbgn, G.R.
c. Hutton, J.
d. Playfair, J.
Consider the following Arab Geographers
1. lbn Hauqal
2. Al-Idrisi
3. AlBeruni
4. 1bn-Bath
The correct chorological order of these
Geographers is
a. 1, 3, 4, 2
b. 3, 1, 4, 2
c. 3, 1, 2, 4
d. 1, 3, 2, 4
Which one of following sets of lakes is to
tectonic origin?
a. Baikal, Tanganyika and Rudolf
b. Chilka, Pulicat and Dal
c. Ladoga, Onega and Superior
d. Chad Biwa and Sambar
The shaded areas in the given map
represent the

13.

14.

15.

16.

a. Dharwarian System
b. Gondwana System

2 of 15
c. Tertiary System
d. Vindhyan System
Which of the following statements are
true?
1. Moderate earthquakes are caused along
constructive plate boundaries,
2. Low magnitude earthquakes are caused
along conservative plate boundaries.
3. Disastrous earthquakes are caused
along destructive plate boundaries.
4. Severe earthquakes are caused along
the conservative plate boundaries with
the creation of transform faults.
Select the correct answer using the codes
given below:
a. 1 and 2
b. 1 and 3
c. 1, 3 and 4
d. 2, 3 and 4
Match List I with List II and select the
correct answer :
List I
A. Pragmatism
B. Humanism
C. Quantitative Revolution
D. Positivism
List II
1. Augtste Comte
2. Peirce
3. Burton
4. Tuan
A
B
C
D
a.
4
2
3
1
b.
4
2
1
3
c.
2
4
1
3
d.
2
4
3
1
Which one of the following pairs is
Correct matched?
a. Cs:
Mediterranean
b. Am:
N.W. Europe
Tropical monsoon
c. DW:
d. ET:
Siberia
In the given map of Deccan region of
India, rivers Mnijira, Godavari, Bhima and
Penganga have been labelled

17.

18.

19.

a. 2, 3, 4 and 1
b. 2, 3, 1 and 4
c. 3, 2, 1 and 4
d. 3, 2 4 and 1
Match List I with List II and select the
correct answer:
List I (Theories/Hypotheses)
A. Geosynclinals Theory
B. Tetrahedral hypothesis
C. Stationary Wave Theory
D. Subsidence Theory
List II (Related aspects)
1. Origin of tides
2. Origin of coral reefs and atolls
3. Mountain building
4. Origin of continents and ocean basins
A
B
C
D
a.
3
4
2
1
b.
4
3
2
1
c.
3
4
1
2
d.
4
3
1
2
Match List I with List H and select the
correct answer:
List I (Old names)
A. Sarmaticus
B. Erytharaen
C. Hibernia
D. Imaus
List II (New names)
1. Altai
2. Ireland
3. Arabian Sea
4. Baltic Sea
A
B
C
D
a.
1
3
2
4
b.
1
2
3
4
c.
4
3
2
1
d.
4
2
3
1
Match List I with List II and select the
correct answer:

20.

21.

22.

23.

3 of 15
List I (Type of Climate)
A. Equatonal
B. Monsoon
C. Mediterranean
D. Hot desert
List II (Country)
1. Namibia
2. Sri Lanka
3. Cyprus
4. Jawa
A
B
C
D
5.
1
2
3
4
6.
4
3
2
1
7.
4
2
3
1
8.
1
3
2
4
Which one of following rivers does NOT
represent the antecedent drainage?
a. Chenab
b. Sutlej
c. Ravi
d. Subansiri
The main differ between paternoster lake
and tarn is that paternoster lake is
a. formed by glacial action whereas tarn
is not
b. associated with cirque whereas tarn is
associated with glacial stairways
c. associated with glacial stairways
whereas tarn is associated with cirque
d. a lake whereas tarn s not
Who among the following developed the
technique shown in the figure for the
measurement of the length of Equator?

a. Anaximander
b. Eratoshenes
c. Hipparchus
d. Ptolemy
Soils, which have undergone gleying and
are associated with marshes, swamps or
poorly drained upland are called
a. calcimorphous soils

24.

25.

26.

27.

28.

b. hydromorphic soils
c. halomorphic soils
d. sierozems
Which one of the following places is
located at the confluence of Aleksandra
and Bhagirathi?
a. Vishnuprayag
b. Devaprayag
c. Rudraprayag
d. Kamaprayag
Match List I with List II and select the
correct answer:
List (Agent)
A. River
B. Wind
C. Glacier
D. Underground water
List II (Landform)
1. Blind valley
2. Roche Moutonnee
3. Natural levee
4. Zeugen
A
B
C
D
a.
1
2
4
3
b.
3
4
2
1
c.
3
2
4
1
d.
1
4
2
3
Which of the following is the correct
sequence of landform developed under
fluvial cycle of erosion?
a. Gorge, floodplain, piedmont plain,
delta
b. Gorge, piedmont plain, flood plain,
delta
c. Gorge, piedmont plain, delta, flood
plain
d. Piedmont plain, Gorge, flood plain.
delta
Allimial cone is predominant in
a. coastal belt
b. piedmont zone
c. delta region
d. pediment region
Which one of the following states has the
longest coast line?
a. Maharashtra
b. Tamil Nadu
c. Andhra Pradesh

29.

30.

31.

32.

33.

4 of 15
d. Karnataka
Chemical weathering is most predominant
in
a. arid region
b. semi-arid region
c. humid tropical region
d. Mediterranean region
Which one of the following is the
aboriginal tribe of Malaysia?
a. Ainu
b. Fula
c. Sakai
d. Onge
Social forestry aims at
a. adopting
scientific
silvicultural
practices for healthy growth of plants
b. motivating villagers in the generation
and management of forests
c. bringing wastelands under massive
afforestation programme
d. adopting measures for the maximum
utilisation of forest products
Which of the following statements are
correct?
1. The Peninsular India has not
undergone marine submergence since
Archaean times.
2. The Aravalli mountains were one of
the oldest geosyn-clines of the world.
3. Dharwar system is rich in minerals.
4. The Peninsular block of India is
regarded as an earth quake-free zone.
Select the correct answer using the codes
given below4:
a. 1, 2, 3 and 4
b. 2 and 4
c. l and 2
d. 1, 2 and 3
Which of the following pairs are correctly
matched?
1. Granites :
Batholiths
2. Marble :
Hogback ridges
3. Quartzite : Ridge crests
4. Hard rocks:
Monadnocks
Select the correct answer using the codes
given below:
a. 1, 2, 3 and 4
b. 1, 3 and 4
c. 2 and 4

34.

35.

36.

d. 1, 2 and 3
Human Geography is 'Human Ecology'.
This definition was given by
a. Barrow
b. Huntington
c. Ratzel
d. Vidal de la Blache
Which one of the following factors caused
the recent depletion of rain forests in
Indonesia?
a. Growth of forest based industries
b. Encroachment by urban areas
c. Forest fires in peat-bogs in marshy
areas
d. Lack of proper management of forests
The climate graph (showing mean monthly
distribution of temperature and rainfall) in
the figure depicts conditions ob tainting at

39.

40.

37.

38.

a. Bangalore
b. Nagpur
c. Chennai
d. Jodhpur
During the passage of a tropical cyclone at
sea coast, the following atmospheric
conditions are observed :
1. Torrential rainfall with gusty winds.
2. Barometric fall with dark clouds.
3. Total calm and clear sides.
The correct sequence of the occurrence of
these conditions is
a. 2, 1, 3
b. 1, 2, 3
c. 3, 2, 1
d. 2, 3, 1
Match List I with List I and select the
correct answer:
List I
A. Jarawa
B. Todas

41.

42.

5 of 15
C. Angami
D. Saura
List II
1. Shifting cultivation
2. Terraced cultivation
3. Pastroal
4. Food gathering
A
B
C
D
a.
3
4
1
2
b.
4
3
1
2
c.
4
3
2
1
d.
3
4
2
1
Non-commercial pastoral nomadism is a
way of life in the
a. velds of South Africa
b. Steppes of Mangolia
c. Fairies of Canada
d. Australian downs
In India, the tropical moist deciduous
forests form the In India, the tropical moist
deciduous forest form the natural cover in
nearly, all the places where the annual
rainfall is between
a. 201 cm and 250 cm
b. 251 cm and 300 cm
c. 70 cm and 100 cm
d. 101 cm and 200 cm
Match List I (Isoclines) with List II
(Features) and select
List I
A. Isotrpes
B. Isopachs
C. Isopytes
D. Isohypes
List II
1. Rainfall
2. Cloud base
3. Potential temperature
4. Cumulonimbus
A
B
C
D
a.
3
4
1
2
b.
4
3
1
2
c.
4
3
2
1
d.
3
4
2
1
Consider the following statements :
According to the Malthusian model
1. Population
tends
to
increase
arithmetically.

43.

2. Positive and preventive checks prevent


excessive growth rate of population.
3. the basic of preventive checks is 'moral
restraint
4. Postponement of marriage is to be
emphasized.
Of these statements
a. 1 and 2 are correct
b. 1, 2 and 3 are correct
c. 2, 3 and 4 are correct
d. 1, 3 and 4 are correct
The shaded regions in the given world map
represent

47.

48.

44.

45.

46.

a. temperate grasslands
b. temperate deciduous forests
c. tropical grasslands
d. mixed deciduous forests
Which one of the following species has the
largest areal coverage in India?
a. Chir
b. Deodar
c. Sal
d. Teak
In the case of saturated air, the column of
air tends to be in stable equilibrium when
the prevailing lapse rate
a. is equal to the wet-adiabatic lapse rate
b. is more than the wet-adiabatic lapse
rate
c. is less than the wet-adiabatic lapse rate
d. does not vary with the wet-adiabatic
lapse rate
According to Census of India 1991, which
one of the following pairs of states had a
lower sex-ratio than the other three pairs?
a. Punjab and Haryana
b. Haryana and Madhya Pradesh
c. Andhra Pradesh and Karnataka
d. Punjab and Rajasthan

49.

6 of 15
Budapest, Bucharest, Vienna and Belgrade
are four cities located on or near vier
Danube. In the given map, these four cities
have been labelled respectively as

a. 2, 4, 1 and 3
b. 2, 4, 3 and 1
c. 4, 2, 1 and 3
d. 4, 2, 3 and 1
Match the areas labelled as 1, 2, 3 and 4 in
the given rough map of India with the
following characteristics

A. Tropical dry deciduous forest


B. Mixed red and black soils
C. Hallikar cattle breed
D. Cultivated area > 60% of total
Select the correct answer using the codes
given below:
A
B
C
D
a.
3
1
4
2
b.
1
3
4
2
c.
1
3
2
4
d.
3
1
2
4
Which one of the following is the correct
sequence of cloud forms in ascending
order of their heights from the ground?
a. Cirrus, cumulus, stratus

50.

51.

52.

53.

b. Cumulus, stratus, cirrus


c. Stratus, cirrus, cumulus
d. Stratus, cumulus, cirrus
Consider the following statements about
the Central Place
1. It provides an account of the size and
distribution of settlements.
2. It assumes that shop-owners and
consumers are rational persons.
3. It is based on octagonal network.
4. It is based on three orders settlements.
Of these statements:
a. 1, 2 and 3 are correct
b. 2, 3 and 4 are correct
c. 1, 2 and 4 are correct
d. 1, 3 and 4 are correct
Which of the following pairs are correctly
matched?
1. Primary sex ratio : Sex at the time of
conception
2. Secondary sex ratio: Sex at the time of
birth
3. Tertiary sex ratio : Sex at the time of
enumeration
4. Imbalanced sex ratio: Sex of workers
Select the correct answer using the codes
given below:
a. 1, 2, 3 and 4
b. 1, 3 and 4
c. 2 and 4
d. 1, 2 and 3
Consider the following states :
1. Goa
2. Andhra Pradesh
3. Orissa
4. West Bengal
The correct sequence of these states in
ascending order in terms of mangrove
vegetation coverage is :
a. 2, 1, 3, 4
b. 1, 2, 3, 4
c. 2, 1, 4, 3
d. 1, 2, 4, 3
A cold local wind draining occasionally
down the Rhone valley is known as
a. Chinook
b. Mistral
c. Bora

54.

55.

56.

57.

7 of 15
d. Blizzard
Match List I with List II and select the
correct answer :
List I (Tribe)
A. Hottentot
B. Inuit
C. Berber
D. Yakut
List II (Region/Country)
1. Siberia
2. Libya
3. Canada
4. South Africa
A
B
C
D
a.
4
2
3
1
b.
4
3
2
1
c.
1
2
3
4
d.
1
3
2
4
Which one of the following represent the
correct sequence of descending order of
population size of the four given mega
cities of the world?
a. Tokyo Mexico city New York
Mumbai
b. Mexico city Tokyo Mumbai
New York
c. Mexico city Tokyo New York Mumbai
d. Tokyo Mexico city Mumbai
New York
The correct sequence of the given states in
descending order with respect to
production of Bajra is :
a. Rajasthan, Gujarat, Maharashtra, Uttar
Pradesh
b. Rajasthan, Gujarat, Uttar Pradesh,
Maharashtra
c. Gujarat, Rajasthan, Maharashtra, Uttar
Pradesh
d. Gujarat, Rajasthan, Uttar Pradesh,
Maharashtra
Which of the following statements
regarding Fjords are correct?
1. Most of the Fjords were formed due to
a rise in sea level after the melting of
Pleistocene ice-sheets,
2. It is an emergent coast which
originally was a sub merged glaciated
valley.

58.

3. It is a long, narrow inlet of the sea


bounded by steep mountain slopes
extending to considerable depths.
4. It is a glaciated valley that has been
inundated by the sea and forms a deep
steep-sided coastal inlet.
Select the correct answer using the codes
given below:
a. 1, 2, 3 and 4
b. 1, 3 and 4
c. 2 and 4
d. 1, 2 and 3
A rough map of Southern portion of
Africa is shown in the figure. In this map,
Botswana, Mozambique, Zambia and
Zimbabwe have been labelled respectively

61.

62.

59.

60.

a. 4,1 2, and 3
b. 4, 1, 3 and 2
c. 1, 4, 3 and 2
d. 1, 4, 2 and 3
Which one of the following pairs is
correctly matched?
a. Masai :
Patagonia
b. Kirghiz :
Kazakastan
c. Melanesians :
Malaysia
d. Pygmies :
Tanzania
Match List I with List II and select the
correct answer:
List I (Mango verities)
A. Neelam
B. Alphonso
C. Himasgar
D. Deseri
List II (State related with its production)
1. Uttar Pradesh
2. Andhra Pradesh
3. Maharasthra
4. West Bengal
A
B
C
D

63.

64.

8 of 15
a.
3
2
1
4
b.
2
3
4
1
c.
3
2
4
1
d.
2
3
1
4
Which one of the following statements
regarding El Nino is NOT true?
a. It is an extension of the Equatorial
current towards the western coast of
South America
b. It is an occasional warm current
leading to an increase of about 10C in
surface water temperature
c. It develops as a temporary replacement
of the usual cold Peruvian current
d. It causes an increase in planktons
which thrive in the cold Peruvian
current
Which of the following pairs of countries
and their capitals are correctly matched?
:
Amman
1. Jordan
2. Lebanon :
Beirut
3. Syria
:
Damuseus
4. Yemen
:
Aden
Select the correct answer using the codes
given below:
a. 1, 2, 3 and 4
b. 1, 3 and 4
c. 2 and 4
d. 1, 2 and 3
Mormonism is practiced widely in
a. Western USA
b. Eastern Africa
c. Northern China
d. South-East Asia
Match List I with List II and select the
correct answer:
List I (crop)
A. Sugarcane
B. Coconut
C. Coffee
D. Groundnut
List II (state leading in production)
1. Karnataka
2. Maharashtra
3. Andhra Pradesh
4. Kerala
A
B
C
D
a.
2
4
3
1

65.

66.

b.
4
2
3
1
c.
2
4
1
3
d.
4
2
1
3
Match List I with List II and select the
correct answer:
List I
A. Ca
B. Kiruna
C. Masabi
D. Pehaxa
List II
1. Aus
2. Brazil
3. Sweden
4. U.S.A.
A
B
C
D
a.
2
4
3
1
b.
1
3
4
2
c.
2
3
4
1
d.
1
4
3
2
The shaded are in the given map is a part
of

69.

70.

71.

67.

68.

a. Indonesia
b. Malaysia
c. Philippines
d. None of the above
Which one of the following regions is
culturally closer to India but ethnically
related to China?
a. Central Asia
b. Polynesia
c. West Asia
d. South-East Asia
Match List I with List II and select the
correct answer :
List I (Mineral/Ore)
A. Lignite
B. Oil

72.

9 of 15
C. Hematite
D. Lead and Zinc
List II (Occurrence)
1. Kalol
2. Noamundi
3. Neyveli
4. Zawar
A
B
C
D
a.
1
3
4
2
b.
1
3
2
4
c.
3
1
2
4
d.
3
1
4
2
Which one of the following commercially
important groups of trees belongs to cool
temperate hardwood species?
a. Maple, mahogany and oak
b. Mathogany, ebony and maple
c. Oak, poplar and maple
d. Ebony, oak and poplar
Lake Van-Golu is situated in
a. Iran
b. Iraq
c. Jordan
d. Turkey
Which of the following pairs of countries
and the local names for shifting cultivation
are correctly matched?
1. 1. Sri Lanka:
Chena
2. 2. Zaire :
Masole
3. 3. Indonesia :
Human
Select the correct answer using the codes
given below:
a. 1 and 3
b. 1, 2 and 3
c. 2 and 3
d. 1 and 2
Consider the following states :
1. Kamataka
2. Orissa
3. Madhya Pradesh
4. Bihar
The correct sequence of these states in
ascending order in terms of production of
iron ore is:
a. 1, 3, 2, 4
b. 1, 3, 4, 2
c. 3, 1, 2, 4
d. 3, 1, 4, 2

73.

In the map given below some of the world


shipping routes are labelled as 1, 2, 3 and
4. The correct sequence in decreasing
order of volume of traffic in these routes
is:

77.

78.

74.

75.

76.

a. 1, 4, 3, 2
b. 4, 1, 3, 2
c. 4, 1, 2, 3
d. 1, 4, 2, 3
Which one of the following cities does
NOT lie on the shore of Lake Michigan?
a. Chicago
b. Gary
c. Milwaukee
d. Duluth
Which of the following pairs of crop and
gene centers are correctly matched?
1. Tomato :
South America
2. Chickpeas :
North Africa
3. Coffee
:
Ethiopia
4. Wheat
:
Europe
Select the correct answer using the codes
given below:
a. 1, 2, 3 and 4
b. 1, 3 and 4
c. 2 and 4
d. 1, 2 and 3
Which of the following statements are
correct?
1. Lignite and low grade bituminous coal
were formed during the Quaternary
period.
2. Dolomite is used as refractory material
and flux in iron and steel industry.
3. Chhotanagpur plateau is the most
extensive, rich and diversified mineral
belt.
Select the correct answer using the codes
given below:
a. 1, 2 and 3
b. 1 and 2

79.

80.

81.

10 of 15
c. 2 and 3
d. 1 and 3
Which one of the following models is
concerned with the principle that "a central
city would generally be surrounded by
concentric zones"?
a. Agricultural regions
b. Agricultural location
c. Concentric zones
d. Isolated State
The shaded area in the given map
represents the distribution of

a. wheat and sugarcane


b. wheat and tobacco
c. maize a id tobacco
d. maize an sugarcane
Which one of the following agricultural
patterns is NOT practiced in Western
Europe?
a. Dairy farming
b. Mixed farming
c. Market gardening
d. Intensive subsistence farming
Talcher and Ennore ate noted fur
a. atomic power plants
b. hydel power plants
c. geo-thermal power plants
d. thermal power plants
Which of the following pairs are correctly
matched?
1. Nomadic herding : Afro-Asian arid
lands
2. Livestock ranching :
Amazon basin
3. Mixed farming:
Eastern U.S.A.
Select the correct answer using the codes
given below:
a. 1, 2 and 3
b. 1 and 3
c. 1 and 2

82.

83.

84.

85.

86.

87.

88.

d. 2 and 3
Which one of the following pairs of rivers
has caused maximum flood havoc in China
in 1998?
a. Yangtze Kiang and Peh Kiang
b. Hwang Ho and Si Kiang
c. Si Kiang and Yangtze Kiang
d. Yangtze Kiang and Hwang Ho
Who among the following has divided the
world into major agricultural regions?
a. Stamp
b. Whittlesey
c. Baker
d. Weaver
Which of the following pairs of thermal
power stations and their locations are
correctly matched?
1. Satpura :
Maharashtra
2. Neyvelli :
Tamil Nadu
3. Kottagudem :
Andhra Pradesh
4. Kota :
Rajasthan
Select the correct answer using the codes
given below:
a. 2, 3 and 4
b. 1 and 4
c. 1, 2 and 3
d. 1, 2, 3 and 4
A map with a scale of 1 cm to 30 km is
enlarged nine times in size. The R.F of the
new map will be
a. 1:1,000.000
b. 1:9,000.000
c. 1 : 100.000
d. 1 : 10,000,000
In China, cotton is grown mostly in the
a. North-east
b. South-east
c. North-west
d. South-central region
Which one of the following pairs is
correctly matched?
a. Taiga :
Rosewood
b. Tropical deciduous :
Maple
c. Mediterranean :
Olive
d. Selua :
Mahogamy
Which one of the following regions of
India has the potentiality of harnessing
tidal power?

89.

90.

91.

92.

11 of 15
a. Gulf of Khambhat
b. Gulf of Mannar
c. Palk Strait
d. Kerala Coast
Two places situated on the same isogonic
line in a world map will have the same
a. ground pollution
b. clound cover
c. seismic activity
d. magnetic declination
In the given map, Ashkabad, Bishkek,
Dushanbe and Tashkent, capitals of some
of the Central Asian Republics, have been
labelled respectively as

a. 4, 1, 2 and 3
b. 1, 4, 2 and 3
c. 4, 1, 3 and 2
d. 1, 4, 3 and 2
The most important pelagic fish found in
the North Sea adjacent to the U.K. is
a. cod
b. halibut
c. herring
d. mackerel
Match List I with List II and select the
correct answer :
List I (Place)
A. Anand
B. Tirunelveli
C. Ramagundam
D. Varanasi
List II (Industry)
1. Cotton Textile
2. Fertilizer
3. Locomotives
4. Dairy
A
B
C
D
a.
3
1
2
4

12 of 15

93.

94.

95.

96.

97.

98.

b.
3
2
1
4
c.
4
2
1
3
d.
4
1
2
3
The ratio of the 'map length' of the equator
and the meridian is ; in the
a. Mercator's Projection
b. Sinusoidal Projection
c. Cylindrical Equal-Area Projection
d. Simple Conical Projection with one
standard parallel
Which of the following pairs are correctly
matched?
1. Ekibartuz :
Coal
2. Kursk
:
Copper.
3. Tyumen :
Oil and natural gas
4. Zaporozhye:
Aluminium
Select the correct answer using the codes
given below:
a. 1, 2, 3 and 4
b. 1, 3 and 4
c. 2 and 4
d. 1, 2 and 3
Le Hare bank is a fishing bank in
a. North Pacific coast
b. South Pacific coast
c. North Atlantic coast
d. South Atlantic coast
Which one of the following states has the
highest proportion of Christians?
a. Goa
b. Kerala
c. Mizoram
d. Nagaland
Which one of the following map
projections look like a photograph of the
earth grid taken from a considerable
distance/height?
a. Gnomonic
b. Orthomorphic
c. Stereographic
d. Orthographic
In the given map of Europe, Liechtenstein,
Andorra, Luxembourg and Monaco, have
been labelled respectively as

99.

100.

101.

a. 4, 2, 3 and 1
b. 2, 4, 3 and 1
c. 2, 4, 1 and 3
d. 4, 2, 1 and 3
The per-capita hydel power output is the
highest in
a. Japan
b. Norway
c. Sweden
d. U.S.A.
Which of the following pairs arc correctly
matched as per 1991 Census?
1. Highest population density :
West
Bengal
2. Highest population growth rate : Uttar
Pradesh
3. Lowest female literacy :
Rajasthan
4. Highest literacy rate :
Kerala
Select the correct answer using the codes
given below:
a. 1, 2, 3 and 4
b. 1, 3 and 4
c. 2 and 4
d. 1, 2 and 3
In the given map of Antarctic Continent,
Dakshin Gangotri has been labelled as

a. 1

102.

103.

104.

b. 2
c. 3
d. 4
Match list I with list II and select the
correct answer:
List I (Dams & Waterfalls)
A. Hoover Dam
B. Grand Cadge Dam
C. Niagara Falls
D. St. Anthony Falls
List II (Rivers)
1. Mississippi
2. Sh Lawrence
3. Colombia
4. Colorado
A
B
C
D
a.
3
4
1
2
b.
4
3
1
2
c.
4
3
2
1
d.
3
4
2
1
Which one of the following pairs is NOT
correctly matched?
a. Tharu
:
Uttar Pradesh
b. Gaddi
:
Himachal Pradesh
c. Konyak :
Kerala
d. Toda
:
Tamil Nadu
Places marked as 1, 2, 3 and 4 in the given
map are noted for the following industries.

A. Iron and Steel


B. Dairy Products
C. Textile
D. Wine
Match the industry with the notation on the
map and select the correct answer using
the codes given below:

13 of 15
A
B
C
D
a.
4
3
1
2
b.
4
3
2
1
c.
3
4
2
1
d.
3
4
1
2
105. Which of the following cause(s) can be
attributed to the decline of population in
India between 1911 and 1921?
1. Severa natural hazards
2. Preventive checks
3. International migration
4. Epidemics
Select the correct answer using the codes
given below:
a. 1 and 2
b. 4 alone
c. 2 alone
d. 1 and 3
106. Since 1950, the trend of urbanization of
West Africa has led to
a. migration of people from the coast to
inland
b. migration of people from the upland to
lowland
c. concentration of people in the
Savannah belt
d. growth of shanty towns around the
cities
107. Which one of the following states has the
highest percentage of small population size
(<200) villages?
a. Jammu& Kashmir
b. Sikkim
c. Himachal Pradesh
d. Manipur
Directions: - The following 13 (thirteen) items
consist of two statements, one labelled as
'Assertion A' and the other labelled as 'Reason
R'. You are to examine these two statements
carefully and decide if the Assertion A and the
Reason R are individually true and if so,
whether the Reason is a correct explanation of
the Assertion. Select your answers to these
items using the codes given below and mark
your answer sheet accordingly.
108. Assertion (A): Average ocean salinity
generally decreases from equator towards

109.

110.

111.

the pole but tropical zone (20"-30") is


characterized by maximum salinity.
Reason (R): The tropical zone owes its
high salinity to high temperature, high
evaporation and significantly low rainfall.
a. Both A and R are true and R is the
correct explanation of A
b. Both A and Rare true but R is NOT the
correct explanation of A
c. A is true but R is false
d. A is fake but R is true
Assertion (A):
Pediments are the bed
rock portions of piedmont slopes which
extend downward to neighbouring basin
floors.
Reason {R}: It is commonly believed that
the dissected pediments are formed by
erosion of original flat pediments under
the second cycle of erosion.
a. Both A and R are true and R is the
correct explanation of A
b. Both A and Rare true but R is NOT the
correct explanation of A
c. A is true but R is false
d. A is fake but R is true
Assertion (A): Laterite soils are reddish in
colour and poor in humus and plant
nutrients.
Reason (R): In humid tropical climate,
excessive leaching takes place under
alternate arid and humid conditions.
a. Both A and R are true and R is the
correct explanation of A
b. Both A and Rare true but R is NOT the
correct explanation of A
c. A is true but R is false
d. A is fake but R is true
Assertion (A): Seas near the equator
receive? Rainfall throughout the year.
Reason (R): High temperature and high
humidity near the equator causes
convectional rain in most afternoons.
a. Both A and R are true and R is the
correct explanation of A
b. Both A and Rare true but R is NOT the
correct explanation of A
c. A is true but R is false
d. A is fake but R is true

112.

113.

114.

115.

116.

14 of 15
Assertion (A): Equatorial region is an area
of sluggish development.
Reason (R): Extreme climatic conditions
prevail here.
a. Both A and R are true and R is the
correct explanation of A
b. Both A and Rare true but R is NOT the
correct explanation of A
c. A is true but R is false
d. A is fake but R is true
Assertion (A): Chinese migrated in large
numbers to South-East Asia during the
Colonial period.
Reason (R): New economic opportunities
provided by the Colonial powers acted as
incentives.
a. Both A and R are true and R is the
correct explanation of A
b. Both A and Rare true but R is NOT the
correct explanation of A
c. A is true but R is false
d. A is fake but R is true
Assertion (A): The equatorial region of
Africa is sparsely populated.
Reason (R): It has hot and humid climate
and thick forests infested with insects and
Tse-Tse flies.
a. Both A and R are true and R is the
correct explanation of A
b. Both A and Rare true but R is NOT the
correct explanation of A
c. A is true but R is false
d. A is fake but R is true
Assertion (A):
South-eastern region of
South America is famous for sheep
rearing.
Reason (R): It receives moderate amount
of average annual rainfall.
a. Both A and R are true and R is the
correct explanation of A
b. Both A and Rare true but R is NOT the
correct explanation of A
c. A is true but R is false
d. A is fake but R is true
Assertion (A): The dichotomy of
ideographic and homothetic was started by
David Harvey.

117.

118.

Reason (R): Idiographic and homothetic


approaches are concerned with the
particular
and
the
general
laws
respectively.
a. Both A and R are true and R is the
correct explanation of A
b. Both A and Rare true but R is NOT the
correct explanation of A
c. A is true but R is false
d. A is fake but R is true
Assertion (A): The differences in the
geomorphic features of different areas of
the earth's surface are known as aerial
differentiation.
Reason (R): This definition of geography
is based on interrelation, differential
character and aerial expression of different
phenomena
a. Both A and R are true and R is the
correct explanation of A
b. Both A and Rare true but R is NOT the
correct explanation of A
c. A is true but R is false
d. A is fake but R is true
Assertion (A):
No significant progress
was made in the development of
geography during the Dark Ages.
Reason (R): Intellectual discourse during
that period was guided by theocracy.

119.

120.

15 of 15
a. Both A and R are true and R is the
correct explanation of A
b. Both A and Rare true but R is NOT the
correct explanation of A
c. A is true but R is false
d. A is fake but R is true
Assertion (A): Ahmedabad is a major
centre of sick cotton textile industry.
Reason (R): Cotton required by the
industry is grown on large scale in the
black soil around it.
a. Both A and R are true and R is the
correct explanation of A
b. Both A and Rare true but R is NOT the
correct explanation of A
c. A is true but R is false
d. A is fake but R is true
Assertion (A): Tank irrigation is mostly
practiced in Peninsular India including
Maharashtra and Gujarat regions.
Reason (R): Many streams in Deccan
Plateau region become torrential during
rainy season but dry up when the rains
cease.
a. Both A and R are true and R is the
correct explanation of A
b. Both A and Rare true but R is NOT the
correct explanation of A
c. A is true but R is false
d. A is fake but R is true

C.S.E-Pre 2000

1 of 15

GEOGRAPHY
1.

2.

3.

Match the rivers labelled as A, B, C and D


in the given map with the following
names:
1. Rhone
2. Loire
3. Seine
4. Elbe
Select the correct answer using the codes
given below:
A
B
C
D
a.
3
4
2
1
b.
2
3
4
1
c.
4
1
2
3
d.
4
1
2
3
Match List I with List II and select the
correct answer:
List I
A. Ivanovo
B. Magnitogorsk
C. Moscow
D. St. Petersburg
List II
1. Cotton textile industry
2. Engineering industry
3. Iron and steel industry
4. Ship-building industry
A
B
C
D
a.
4
3
2
1
b.
1
3
2
4
c.
1
2
3
4
d.
3
2
1
4
Consider the following statements
regarding Japan:
1. Japan's
long
latitudinal
sweep
combined with its insularity, give it a
beneficial climate

4.

5.

6.

2. Japan has no real dry season


3. West coast of Japan is warmer than the
east coast
4. Hokkaido has a greater agricultural
potential
5. Which of the above statements are
correct?
a. 1, 3 and 4
b. 1, 2 and 4
c. 1 and 2
d. 1 and 3
The correct sequence (in descending order)
of exports of the given commodities from
Southeast Asia region is
a. palm oil, sugar, rubber, copra
b. rubber, palm oil, sugar, copra
c. rubber, copra, palm oil, sugar
d. palm oil, rubber, copra, sugar
Match List I with List II and select the
correct answer:
List I (Country)
A. Iran
B. Israel
C. Kuwait
D. Lebanon
List II (Higher Rank in)
1. Adult literacy
2. Density of population
3. Agricultural population
4. Population size
5. Urban population
A
B
C
D
a.
1
4
2
3
b.
4
1
2
5
c.
3
4
5
2
d.
4
1
5
2
Consider the following statements
regarding Southern
1. Almost all goods that enter and leave
Zimbabwe and Botswana pass through
South Africa
2. Zimbabwe's land-locked position is a
hindrance to its development
3. South Africa is the leading producer of
platinum in the world

2 of 15
2. The concept of Terra-Incognita was
propounded by Eratosthenes
3. The book Outline of Geography was
written by Ptolemy
4. Ptolemy used astronomical principles
to prepare the world map
Which of the above statements are correct?
a. 1, 2 and 3
b. 1, 3 and 4
c. 2, 3 and 4
d. 1, 2 and 4

4. Diamond and coal are the major


minerals of Botswana
Which of the above statements are correct?
a. 1, 2, 3 and 4
b. 1 and 2
c. 1, 3 and 4
d. 3 and 4
7.

11.

8.

In the given map showing the three major


natural vegetations of Brazil, the areas
labelled as
a. 1 and 2 are equatorial rain forest and
monsoonal forest respectively
b. 2 and 3 are equatorial rain forest and
monsoonal forest respectively
c. 3 is monsoonal forest and 2 is tropical
grassland
d. 2 and 1 are tropical grassland and
monsoonal forest respectively
Which one of the following pairs of lands
is separated from each other by the Ten
Degree Channel"?
a. Andaman and Nicobar
b. Nicobar and Sumatra
c. Maldives and Lakshadweep
d. Sumatra and Java 9.

12.

9.

10.

The given outline map of the world was


prepared by
a. Hecataeus
b. Herodotus
c. Ptolemy
d. Srrabo
Consider the following statements:
1. The Romans' main contribution was in
the field of historical and mathematical
geography

13.

According to ancient Indian geographers,


1, 2, 3 and 4 marked on the modem map,
show respectively
a. Kraunca, Kusa, Salmali and Plaska
Dwipas
b. Plaska, Kusa, Kraunca and Salmali
Dwipas
c. Plaska, Kusa, Salmali and Kraunca
Dwipas
d. Salmali, Plaska, Kusa and Kraunca
Dwipas
Match List I with List II and select the
correct answer:
List I (Philosophical Approach)
A. Behaviouralism
B. Humanism
C. Quantitative Revolution
D. Radicalism
List II (Scholars)
1. R. J. Peel
2. I- Burton
3. W. Kirk
4. Y. E Tuan
A
B
C
D
a.
4
1
3
2
b.
3
2
4
1
c.
4
3
1
2
d.
3
4
2
1
The book The Geography of Puranas was
written by

14.

15.

16.

17.

18.

a. R. N. Dubey
b. S. M. Ali
c. T. Rizvi
d. S. P Chatterjee
Consider the following statements :
1. Al-Masudi translated Ratanjali's book
into Arabic
2. Ibn-Batuta was appointed as the
ambassador of the Sultanate of Delhi
to China
3. Baitul-Hikma (Academy of Science) at
Baghdad was founded by Caliph AlMamoon
4. The Arabs borrowed the 'Geocentric
concept' from the Greeks
Which of the above statements are correct?
a. 1 and 2
b. 1 and 3
c. 2 and 4
d. 1, 2, 3 and 4
The statement, 'History differs from
geography in the consideration of time and
space. The former is the report of
phenomena that follow one another and
has reference to time. The later is the
report of phenomena besides each other in
space' is attributed to
a. Alexander von Humboldt
b. Carl Ritter
c. Immanuel Kant
d. Bemhard Varen (Varenius)
The dichotomy of general geography and
special geography was advocated by
a. Alexander von Humboldt
b. Alfred Hettner
c. Elisee Reclus
d. B. Varenius
Which one of the following approaches to
geography stressed on a holistic view of
economy, society and polity?
a. Behaviouralism
b. Humanism
c. Positivisms
d. Radicalism

19.

20.

21.

22.

3 of 15
The model of environmental perception
and behaviour shown in the given block
diagram was propounded by:
a. Boulding
b. Down
c. Kirk
d. Porteous
Consider the following statements :
The main objectives of Quantitative
Revolution in geography was to
1. make geography an earth science
2. make geography the geometry of space
3. develop precise regional laws
4. develop universal laws in geography
Which of the above statements are correct?
a. 1, 2 and 3
b. 1, 3 and 4
c. 2, 3 and 4
d. 1, 2 and 4
Match List I with List II and select the
correct answer:
List I (Geographer)
A. Hettner
B. Demangeon
C. Mackinder
D. Berg
List II (Country)
1. France
2. Britain
3. Germany
4. Russia
A
B
C
D
a.
3
2
1
4
b.
2
3
4
1
c.
2
4
3
1
d.
3
1
2
4
While explaining the relation between
nature and man, Blache cited the instance
of the tradition of a regional concept of
France known as
a. Compage
b. Formal region
c. Functional region
d. Pays
The term 'Anthropogeography' coined by
Ratzel and was used by him for the
a. geography of man in terms of
individuals and races
b. geography of man and his works in
relation to earth's surface
c. organic theory of society and state

23.

d. study of harmonious reciprocal relation


of man and nature
India's Eastern Ghats and Western Ghats
meet at the
a. Cardamom hills
b. Annamalai hills
c. Nilgiri hills
d. Palani hills

27.

28.

24.

25.

The given diagram shows the river regime


of
a. Cauvery
b. Godavari
c. Ganga
d. Sutlej
Which one of the following characteristics
is NOT relevant to Ganga river?
a. It is a braided river with numerous
channels
b. It has multiple inter-twined sand-bars
c. It has extensive gully erosion
d. It deposits enormous sediments
annually into the Bay of Bengal

26.

The dotted areas marked on the given map


represent
a. humid sub-tropical climatic region
b. tropical wet and dry climatic region
c. semi-arid climatic region
d. sub-tropical dry climatic region

29.

4 of 15
During winter, north-western part of India
gets rainfall from
a. thunderstorms
b. retreating monsoon
c. western disturbances
d. tropical cyclones
Match the monthly distribution of
temperature indicated as 1, 2, 3 and 4 in
the given diagram with the following
towns :
A. Shillong
B. Delhi
C. Jodhpur
D. Tiruvananthapuram

Select the correct answer using the codes


given below
A
B
C
D
a.
2
4
3
1
b.
2
4
1
3
c.
4
2
3
1
d.
4
2
1
3
Match the four types of soil areas marked
as 1, 2, 3 and 4 in the given map with the
following soil types:
A. Red sandy soil
B. Mixed red and black soil
C. Red loamy soil
D. Red and yellow soil

Select the correct answer using the codes


given below
A
B
C
D
a.
3
1
4
2
b.
3
1
2
4

5 of 15

30.

c.
1
3
2
4
d.
1
3
4
2
The site of which one of the following
hydel power stations is marked as X in the
given map?

a. Koyna on Koyna river


b. Saravathi on Saravathi river
c. Srisailam on Krishna river
d. Nagarjuna Sagar on Krishna river
31.
Major share of the total production of
marine fish is landed along the
a. Kerala and Maharashtra coast
b. Maharashtra and Kamataka coast
c. Kamataka and West Bengal coast
d. West Bengal and Maharashtra coast
32.
The following table gives changing pattern
of forest cover in hectares for the area X
from 1990-91 to 1994-95:
Year
Reserved
Unreserved
Forest
Forest
1990-91
569.26
902.73
1991-92
406.26
885.00
1992-93
552.95
1124.02
1993-94
581.50
1143.95
1994-95
562.00
1202.20
In general, an increase in forest cover
when compared to that in the previous year
has been notices in the period
a. 1991-92 and 1992-93
b. 1993-94 and 1994-95
c. 1991-92 and 1994-95
d. 1992-93 and 1993-94 33.
33.
Which one of the following problems is
NOT encountered in Indira Gandhi Canal
command area?
a. Increase in soil salinity
b. Progressive water logging
c. Aeolian silting of the canal
d. Decline of water supply
34.

35.

36.

37.

38.

In the given map, the petrochemical


concentration, known as the 'Golden
corridor industrial area is located at the
place marked
a. 1
b. 2
c. 3
d. 4
Location of sugar industry in India is
shifting from north to south because of
a. cheap labour
b. expanding regional market
c. cheap and abundant supply of power
d. high yield and high sugar content in
sugarcane
Which one of the following statements
about KOYALI is correct? It is a/an/the
a. hydroelectric plant in Uttar Pradesh
b. historic-cultural tourist centre in
Rajasthan
c. oil refinery located in Gujarat
d. site of steel plant under construction in
MP
Match List I with List II and select the
correct answer:
List I
A. Agricultural machinery
B. Cement
C. Paper
D. Heavy electricals
List II
1. Saharanpur, Dalmianagar
2. Hardwar, Bhopal
3. Ernakulam, Faridabad
4. Dwarka, Chaibasa
A
B
C
D
a.
3
4
1
2
b.
3
4
2
1
c.
4
3
1
2
d.
4
3
2
1
Consider the following statements :
1. Textiles

39.

40.

41.

42.

43.

2. Agricultural and allied products


3. Chemical and related products
4. Gem and jewellery
The correct sequence in descending order
of these items in terms of value of exports
from India as per 1997-98 figures is
a. 1, 4, 3, 2
b. 1, 4, 2, 3
c. 4, 1, 2, 3
d. 4, 1, 3, 2
The
Palghat
Gap
serves
inland
communications from
a. Madurai to Tiruvananthapuram
b. Chennai to Kochi
c. Pune to Mumbai
d. Bangalore to Mangalore
Which one of the following ports in
Gujarat is famous for ship breaking and
ship repairing?
a. Porbandar
b. Patan
c. Pipavav
d. Mandvi
The correct sequence in descending order
of the given States in terms of density of
population (1991 Census) is
a. Punjab, Orissa, Gujarat, Assam
b. Assam, Gujarat, Punjab, Orissa
c. Assam, Orissa, Gujarat, Punjab
d. Punjab, Assam, Gujarat, Orissa
Match List I with List II and select the
correct answer:
List I
A. Jammu & Kashmir
B. Kerala
C. Maharashtra
D. Uttar Pradesh
List II
1. Highest in population size
2. Highest rate of population growth
3. Highest ratio of females/males
4. High proportion of urban population
5. Largest proportion of rural population
A
B
C
D
a.
3
2
4
1
b.
5
4
2
3
c.
5
3
4
1
d.
4
3
1
2
Which one of the following States had the
lowest level of urbanization as per 1991
Census?

44.

45.

46.

47.

48.

6 of 15
a. Himachal Pradesh
b. Manipur
c. Goa
d. Haryana
Consider the following pairs of places and
types of natural hazard faced by them:
1. Latur
:
Riverine flooding
2. Nashri
:
Landslide
3. Kandla
:
Cyclonic storm
Which of the above pairs are correctly
matched?
a. 1 and 2
b. 2 and 3
c. 1 and 3
d. 1, 2 and 3
Consider the following statements :
Majuli. the largest river island in the world
is environmentally degraded due to
1. floods
2. erosion
3. Increasing population pressure
4. Development of transport activities
Which of the above statements are correct?
a. 1 and 4
b. 1 and 3
c. 1, 2 and 3
d. 2, 3 and 4
Consider the following pairs of projects
are related environmental problem:
1. River valley project: Water body
induced seism city
2. Mining Project: Ground water and
surface water pollution
3. Thermal power project: Degradation of
land
Which of the above pairs (s) is/are
correctly matched?
a. 1, 2 and 3
b. 2 and 3
c. 1 and 2
d. 1 alone
The endangered Ridley turtles have the
world's largest aggregation at
a. Gahirmatha
b. Sagarmatha
c. Lakshadweep islands
d. Andaman and Nicobar group of islands
'Rally for Valley' programme in India was
organized to highlight the problem of
a. environmental degradation
b. biodiversity

49.

50.

51.

c. resettlement of displaced people


d. loss of agricultural land
Match List I with List II and select the
correct answer:
List I (Wildlife Sanctuaries)
A. Namdapha
B. Periyar
C. Bandipur
D. Lamjao
List II (States)
1. Kamataka
2. Arunachal Pradesh
3. Manipur
4. Kerala
A
B
C
D
a.
2
4
1
3
b.
4
2
3
1
c.
4
2
1
3
d.
2
4
3
1
Match List I with List II and select the
correct answer:
List I
A. Polar Gnomonic Zenithal Projection

53.

B. Polyconic Projection

54.

C. Simple Conical Projection with one


Standard Parallel
D. Mercator's Projection
List II
1. Parallels are acres of non concentric
circles
2. Distance between parallels increase
towards poles, greatly
3. Distance between parallels increases
towards equator
4. The pole is represented by an are arc of
a circle
A
B
C
D
a.
3
2
1
4
b.
3
1
4
3
c.
1
2
3
4
d.
2
1
4
3
A line diagram of a vertical air
photograph is shown in the figure:

52.

7 of 15
The scale of this photography is
a. 1 : 1,000
b. 1 : 10,000
c. 1 : 100,000
d. 1 : 1,000,00
Spatial data is stored in the computer by
using:
a. digitizer and scanner
b. plotter, digitizer and keyboard
c. keyboard and plotter
d. scanner, digitizer and keyboard
Match List I with List II and select the
correct answer:
List I (Index No. of Indian Toposheets)
A.
B.
C.
D.

65
64 M
64 M/NE
64 M/10
A
B
C
D
a.
3
2
4
1
b.
2
3
1
4
c.
3
2
1
4
d.
2
3
4
1
The scale of the one-inch Indian top sheet
covered into metric system is now system
is now given as
a. 1 : 5000
b. 1: 25000
c. 1: 5000
d. 1:250000

55.

The three profiles shown in diagram


and Z represent respectively
a. projected,
composite
superimposed profiles
b. superimposed,
projected
composite profiles
c. composite,
superimposed
projected profiles

X, Y
and
and
and

56.

d. superimposed,
composite
and
projected profiles
Match List I (Diagrams) with List II
(Uses) and select the correct answer:
List I
A. Hypsometic curve
B. Hythergraphy
C. Isochrones
D. Pyramids
List II
1. Population age-sex ratio diagram
2. Spatial travel time diagram
3. Elevation/depth area ratio diagram
4. Climatic diagram
A
B
C
D
a.
1
2
3
4
b.
3
4
1
2
c.
3
4
2
1
d.
4
3
1
2

59.

57.
60.

A comparison of Polar zenithal projections


drawn on same scale is shown in the
figure. Here X and Y are respectively
a. gnomonic and orthographic projections
b. orthographic and gnomonic projections
c. gnomonic and zenithal stereographic
projections
d. orthographic
and
zenithal
stereographic projections
Directions: - The following 13 (thirteen) items
consist of two statements, one labelled as
'Assertion A' and the other labelled as 'Reason R'.
You are to examine two statements carefully and
decide if the Assertion A and the Reason R
individually true and if so, whether the Reason is
a correct explanation of the ion. Select your
answers to these items using the codes given
below and our answer sheet accordingly.
58.

Assertion (A): The most important single


agent of physical weathering is the

61.

62.

8 of 15
freeze/thaw action of water in open spaces
in rock.
Reason (R): Repetition is the key to
understanding the force of frost shattering.
a. Both A and R are true and R is the
correct explanation of A
b. Both A and R are true but R is NOT
the correct explanation of A
c. A is true but R is false
d. A is false but R is true
Assertion (A): Fjords are the features
produced by glacial erosion.
Reason (R): Glaciers descending from
coastal mountains may reach the sea and
continue their erosion on the seafloor.
a. Both A and R are true and R is the
correct explanation of A
b. Both A and R are true but R is NOT
the correct explanation of A
c. A is true but R is false
d. A is false but R is true
Assertion (A): Hot deserts in the trade
wind belts occur invariably on the western
side of the continents.
Reason (R): In the tropical belt where the
trade winds blow, there are practically no
highlands on the western side to obstruct
the moisture-laden winds for causing
rainfall.
a. Both A and R are true and R is the
correct explanation of A
b. Both A and R are true but R is NOT
the correct explanation of A
c. A is true but R is false
d. A is false but R is true
Assertion (A): The moist adiabatic lapse
rate is lower than the dry adiabatic lapse
rate.
Reason (R): When condensation begins,
the latent heat of condensation is released
in the parcel of air.
a. Both A and R are true and R is the
correct explanation of A
b. Both A and R are true but R is NOT
the correct explanation of A
c. A is true but R is false
d. A is false but R is true
Assertion (A): Baltic Sea remains open for
international trade throughout the year.
Reason (R): Baltic Sea lies in the warm
temperate zone.

63.

64.

65.

66.

a. Both A and R are true and R is the


correct explanation of A
b. Both A and R are true but R is NOT
the correct explanation of A
c. A is true but R is false
d. A is false but R is true
Assertion (A): The Darwinian theory of
natural selection sometimes referred to as
'the survival of the fittest explains the
origin of any species as a normal process
of descent from parent forms.
Reason (R): The contemporary distribution
pattern of many organisms is often the
result of natural migration or dispersal
from original centre(s) of development.
a. Both A and R are true and R is the
correct explanation of A
b. Both A and R are true but R is NOT
the correct explanation of A
c. A is true but R is false
d. A is false but R is true
Assertion (A): The pH of acid rain lies
between 9 and 12.
Reason (R): Acid rain occurs due to heavy
air pollution.
a. Both A and R are true and R is the
correct explanation of A
b. Both A and R are true but R is NOT
the correct explanation of A
c. A is true but R is false
d. A is false but R is true
Assertion (A): According to determinists,
people living in similar environments
manifest similar social, economic and/or
historical development.
Reason (R): Every aspect of man's life
mirrors the influence of the physical
environment of his habitat.
a. Both A and R are true and R is the
correct explanation of A
b. Both A and R are true but R is NOT
the correct explanation of A
c. A is true but R is false
d. A is false but R is true
Assertion (A): In areas of intensive
subsistence agriculture, animal husbandry
is a meager sideline of crop farming.
Reason (R): Throughout the region, the
farmer is handicapped in raising cattle due
to lack of scientific methods of breeding.
a. Both A and R are true and R is the
correct explanation of A

67.

68.

69.

70.

71.

9 of 15
b. Both A and R are true but R is NOT
the correct explanation of A
c. A is true but R is false
d. A is false but R is true
Assertion (A): New England region in the
USA continues to be an important area for
cotton textiles.
Reason (R): Market and geographical
inertia are the main reasons for cotton
textiles here.
a. Both A and R are true and R is the
correct explanation of A
b. Both A and R are true but R is NOT
the correct explanation of A
c. A is true but R is false
d. A is false but R is true
Assertion (A): In
Western Europe,
rainfall decreases from the coastal areas
towards the interior.
Reason (R): Most of Western Europe lies
in the westernizes belt.
a. Both A and R are true and R is the
correct explanation of A
b. Both A and R are true but R is NOT
the correct explanation of A
c. A is true but R is false
d. A is false but R is true
Assertion (A): Most of the population
in Australia lives in urban centers.
Reason (R): Nearly two-thirds of Australia
receives less than 50 cm of rain making it
difficult to pursue agriculture as a major
occupation.
a. Both A and R are true and R is the
correct explanation of A
b. Both A and R are true but R is NOT
the correct explanation of A
c. A is true but R is false
d. A is false but R is true
Assertion (A): Hwang Ho shifts its
course often.
Reason (R): Much of the area over which
Hwang Ho flows is Loess land.
a. Both A and R are true and R is the
correct explanation of A
b. Both A and R are true but R is NOT
the correct explanation of A
c. A is true but R is false
d. A is false but R is true
Match List I with List II and select the
correct answer:
List I (Scholars)

72.

73.

74.

75.

A. Jeans and Jeffreys


B. Chamberiin and Moulton
C. Kant and Laplace
D. Hoyle and Littleton
List II (Hypotheses Regarding Origin of
the Earth)
1. Nebular Hypothesis
2. Tidal Hypothesis
3. Supernova Hypothesis
4. Collision Hypothesis
5. Planetesimal hypothesis
A
B
C
D
a.
4
1
3
2
b.
2
5
1
3
c.
5
2
1
3
d.
2
4
5
1
The correct sequence (from the oldest to
the youngest) of different periods of
Tertiary epoch is
a. Pliocene, Miocene, Eocene, Oligocene
b. Eocene, Oligocene, Pliocene, Miocene
c. Pliocene, Eocene, Miocene, Oligocene
d. Eocene, Oligocene, Miocene, Pliocene
Mono-discontinuity is found between the
a. crust and mantle
b. upper mantle and lower mantle
c. mantle and core
d. inner core and outer core
The development of synclinal ridges and
anticlinal valleys in a folded region is
named as
a. parallel belt of valleys and ridges
b. abrasion platform
c. topography of inversion of relief
d. ridge and valley landscape
The following features produced by
volcanic activities are shown in the
diagram :

76.

77.

78.

79.

1.
2.
3.
4.

Batholith
Dyke
Crater
Sill

10 of 15
Match the names of the feature with the
labels A, B, C and D in the diagram and
select the correct answer:
A
B
C
D
a.
4
2
3
1
b.
1
2
3
4
c.
1
3
2
4
d.
4
2
1
3
Consider the following statements
regarding earthquakes;
1. In earthquakes, shocks are generated at
a point known as the epicenter
2. The point of origin of shock may be
within the crust or mantle
3. The more earthquake-prone areas are
the margin of tectonic plates
Which of the above statements are correct?
a. 1,2 and 3
b. 1 and 2
c. 1 and 3
d. 2 and 3
The extensive flood plain with level to
gentle slopes (2 to 5) and very low
channel gradient makes the river flow so
sluggish that it becomes a tangled network
of
interconnected
diverging
and
converging shallow channels. Such a flood
plain is said to be
a. Braided
b. Graded
c. Entrenched
d. Meandered
The depositional feature, formed due to the
coalescence of several alluvial cones is an
arid region is called
a. Pediment
b. Bolson
c. Deflation basin
d. Bajada
Consider the following statements :
1. Ozone is found mostly in the
Stratosphere
2. Ozone layer lies 55-75 km above the
surface of the earth
3. Ozone absorbs ultraviolet radiation
from the Sun
4. Ozone layer has no significance for life
on the earth
Which of the above statements are correct?
a. 1 and 3
b. 2 and 4
c. 2 and 3

d. 1 and 4
80.

83.

84.

81.

82.

In the given map, the highest annual range


of temperature is recorded in the area
labelled as
a. 3
b. 1
c. 4
d. 2
Thunderstorms originate from
a. nimbostratus clouds
b. altocumulus clouds
c. status clouds
d. cumulonimbus clouds
Which one of the following precipitation
conditions is a NOT characteristic of
frontal rainfall type?
a. Precipitation falls during the passage
of a depression in zones facing the
passage
b. It occurs in the middle and high
latitudes
c. It is distinguished from the cyclonic
rain because of the prolonged drizzling
rain which falls from the warm front of
the depression
d. Cloud burst with long duration of rain
is associated with the squally passage
of the cold front
The following table contains temperature
and rainfall data of a station in the
southern hemisphere:
Month
Temp C
Precipitation
(in mm)
January
23
8
February
23
13
March
22
18
April
19
40
May
16
125
June
14
175
July
13
165

85.

86.

87.

11 of 15
August
13
145
September
14
84
October
16
54
November
18
20
December
22
15
The data is indicative of
a. sub-tropical dry summer
b. tropical wet dry summer
c. sub-tropical humid climate
d. marine west coast climate
Match List I with List II and select the
correct answer:
List I (Ocean Trench)
A. Aleutian
B. Pueto Rico
C. Ryukyu
D. Tonga
List II (Region)
1. East of Philippines
2. Fiji Islands
3. S.W. Alaska
4. West Indies
A
B
C
D
a.
4
3
1
2
b.
3
4
2
1
c.
3
4
1
2
d.
4
3
2
1
Consider the following conditions :
1. Anticyclone conditions with static air
and high temperature
2. Cyclonic conditions with unstable air
3. High
precipitation
with
low
evaporation
4. Low
precipitation
with
high
evaporation
High salinity of ocean waters is associated
with
a. 1 and 4
b. 2 and 3
c. 1 and 3
d. 2 and 4
Radiolarian oozes occurs in depths of
a. 600 to 1200 fathoms
b. 1500 to 2000 fathoms
c. 2000 to 5000 fathoms
d. 5000 to 6000 fathoms
Consider the following factors :
1. Rotation of the earth
2. Air pressure and wind
3. Ocean water density
4. Revolution of the earth

88.

89.

90.

91.

92.

Which of these factors modify ocean


currents?
a. 1 and 2
b. 1, 2 and 3
c. 1 and 4
d. 2, 3 and 4
Tsunamis are produced by
a. shrinking of the earth's crust
b. cyclones
c. submarine earthquakes
d. tides
Match List I with List II and select the
correct answer:
List I (Soil Type)
A. Entisol
B. Vertisol
C. Spodosol
D. Oxisol
List II (Characteristics)
1. Clay-rich soils
2. Highly weathered tropical and subtropical soils
3. Young soils with no horizons
4. Acid soils with leached upper horizon
A
B
C
D
a.
2
1
3
4
b.
3
1
4
2
c.
3
4
1
2
d.
4
1
2
3
Gleization is associated with
a. areas which have experienced recent
glacial deposition and have deranged
drainage pattern
b. areas where decomposition proceeds
slowly
c. areas where unrecompensed or
partially decomposed organic matter
accumulates in a swampy environment
d. highly acidic and oxygen-poor soils
The state of inactivity adopted by desert
animals during the hottest and driest
season is known as
a. basal metabolism
b. dormancy
c. aestivation
d. hibernation
Which one of the following sets of biomes
represents the right sequence in terms of
increasing water deficiency?
a. Tropical scrub', tropical savanna,
desert

93.

94.

95.

96.

12 of 15
b. Tropical
savanna,
midlatitude
grassland, desert
c. Mediterranean woodland, midlatitude
grassland, midlatitude deciduous forest
d. Desert, tropical savanna, tropical scrub
Water
bodies
which
have
low
concentration of nutrients are in
a. Littoral zones
b. Oligotrophic areas
c. Neritic province
d. Benthos zone
Consider the
following statements
regarding human geography :
1. It is the synthetic relationship between
human societies and earth surface
2. It describes alt those parts of
geography which are not solely
concerned either with physical
environment or with cartography
3. It is currently dominated by numerous
approaches
Which of the above statements are correct?
a. 1 and 2
b. 1 and 3
c. 2 and 3
d. 1, 2 and 3
Match List I with List II and select the
correct answer:
List I List II
A. Generesdevie
B. Lands haft
C. Lebensraum
D. Sequent occupancy
1. S. Passarge
2. Whittles
3. Vidal de La Blache
4. R Ratzel
A
B
C
D
a.
2
1
4
3
b.
3
4
1
2
c.
2
4
1
3
d.
3
1
4
2
Consider the following statements:
Demographic transition is a general model
1. describing the evolution of levels of
fertility and mortality
2. which has been developed with
particular reference to developed
countries
3. which has been accepted as the
universal model of population growth

4. which suggests four highly stylized


phases in the process
Which of the above statements are correct?
a. 1, 2 and 3
b. 2, 3 and 4
c. 1,2 and 4
d. 1, 3 and 4

100.

97.
101.

102.

98.

99.

The given diagram shows a


a. Fulani settlement
b. Masai settlement
c. Kirghiz settlement
d. Bushmen settlement
Match List I with List II and select the
correct answer:
List I (Country)
A. Canada
B. China
C. Thailand
D. USA
List II (Indigenous People)
1. Crow
2. Highur
3. Innu
4. Lisu
A
B
C
D
a.
3
2
1
4
b.
3
2
4
1
c.
1
3
2
4
d.
1
4
2
3
Consider the following statements :
1. The population of Europe is older in
age than that of any other continent
2. Population of many of the European
countries is showing a negative
growth
3. About 25 percent of the European
population is in the senile age group
4. Only 10 percent of the European
population is in the juvenile age group
Which of the above statements are correct?
a. 1 and 2
b. 1 and 3

103.

104.

105.

106.

107.

13 of 15
c. 2, 3 and 4
d. 1, 2, 3 and 4
The correct sequence in the descending
order of the given countries in terms of life
expectancy at birth is
a. China, India, Sri Lanka, Pakistan
b. China, Sri Lanka, Pakistan, India
c. Sri Lanka, India, China, Pakistan
d. Sri Lanka, China, Pakistan, India
The doubling period of population is the
lowest in
a. Bangladesh
b. Egypt
c. Iran
d. Pakistan
The first scholar to formulate the laws of
migration was
a. Kosinski, L. A.
b. Lee, E.
c. Ravenstein, E. G.
d. Zelinsky, W.
'Human activates are conditioned and
sometimes deter mined by biophysical
environmental is related to the doctrine of
a. determinism
b. neo-determinism
c. possibilism
d. humanism
Which one of the following geographical
conditions makes hunting and fishing the
principal means of livelihood in the
Tundra region?
a. Sparse population
b. Infertile dissected terrain
c. Scarcity of vegetation
d. Harsh climate
Transhumance' refers to
a. migration of animal in ranches
b. human migration in search of job
c. seasonal movement of people and their
herds from valley to mountain and vice
versa
d. migration of nomads
'Estancias1 are
a. vast cattle farms in Argentina
b. smaller estates in Brazil
c. plantation farms in Malaysia
d. commercial grain farms in Western
USA
Bamboo and pine trees are generally found
in which one of the following types of
climates?

108.

109.

110.

111.

a. Taiga
b. Tropical monsoon
c. Mediterranean
d. Savanna
Nagaland mountains are becoming
increasingly barren mountains, mainly due
to
a. insurgency
b. shifting cultivation
c. urbanization
d. rapid population growth
Match List I with List II and select the
correct answer:
List I (Crops)
A. Banana
B. Cocoa
C. Coffee
D. Tea
List II (Producer)
1. Colombia
2. Ghana
3. Jamaica
4. Kenya
A
B
C
D
a.
2
3
1
4
b.
3
2
1
4
c.
3
2
4
1
d.
2
3
4
1
Match List I with List II and select the
correct answer:
List I (Minerals)
A. Coal
B. Copper
C. Iron ore
D. Petroleum
List II (Areas)
1. Bisbee
2. Kirkuk
3. Donetz
4. Mesabi
A
B
C
D
a.
3
1
4
2
b.
3
4
1
2
c.
4
1
2
3
d.
2
4
1
3
Consider the following statements :
1. Iron and steel industries developed
around lake Michigan in the USA due
to the availability of good quality coal
2. Pittsburg-Youngstown area is the
leading producers of steel in the USA

112.

113.

114.

14 of 15
3. The Lorrains area in France is
significant for steel production
4. The South Manchurian region accounts
for nearly 60% of China's pig iron
production
Which of above statements are correct?
a. 1, 3 and 4
b. 1, 2 and 4
c. 2, 3 and 4
d. 2 and 3
Consider the following reasons :
1. Cane sugar is preferred to beet sugar
2. Beet sugar is insufficient to meet local
demand
3. There is low demand and surplus
production of cane sugar in producing
countries
4. Cane sugar producing countries were
colonies of importing countries
5. Production cost of cane sugar is low
Which of the above reasons account for
the fact that cane sugar has a major share
in the international sugar trade?
a. 1, 2 and 3
b. 2, 3 and 4
c. 1, 4 and 5
d. 2, 3 and 5
Consider the following statements
1. Low-income countries of Asia have
lower level of urbanization but had
some of the largest cities in the world
2. Most of Southeast Asian countries
have primate pattern of urbanization
3. In the addition of world urban
population after 1950 the share of
developed countries is higher than that
of the developing countries
Which of the above statements are correct?
a. 1 and 2
b. 2 and 3
c. 1 and 3
d. 1, 2 and 3

115.

116.

In the given Multiple Nuclei Model of


Harris-Ullman, the shaded area marked A
represents
a. heavy manufacturing area
b. residential suburb
c. outlying business district
d. industrial suburb
Match List I with List II and select the
correct answer:
List I (City)
A. Jammu
B. Kakinada
C. Faizabad
D. Udaipur
List II (Chrematistics)
1. Sea-front location
2. River-front location
3. Lake-front location
4. Hill-front location
A
B
C
D
a.
2
1
4
3
b.
4
1
2
3
c.
3
4
1
2
d.
3
2
4
1
The term 'Megalopolis for the description
of urban pattern of north-eastern seaboard
of the USA, was first introduced by
a. C. D. Harris
b. Jean Guttmann
c. Brian Berry
d. David Harvey

117.

118.

The shaded area in the given world map


represents the natural region characterized
by
a. warm summer continental climate
b. cool summer continental climate
c. southern Tundra climate
d. marine west coast climate
Between which one of the following pairs
of ports does Panama Canal shorten the
distance to the maximum?
a. Liverpool and Shanghai
b. New York and Honolulu

119.

15 of 15
c. Liverpool and Sydney
d. New York and San Francisco
Match List I (Cities) with List II (Rivers
on which the cities are located) and select
the correct answer:
List I
A. New York
B. Bonn
C. Quebec
D. Venice
List II
1. Daunbe
2. Po
3. St. Lawrence
4. Hudson
5. Rhine
A
B
C
D
a.
4
2
1
3
b.
4
5
3
2
c.
1
3
5
2
d.
3
1
4
5

120.

Match the sites marked as A, B, C and D


in the given map with the following cities:
1. Gary
2. Detroit
3. Kingston
4. Marquette
Select the correct answer using the codes
given below:
A
B
C
D
a.
4
1
3
2
b.
3
1
2
4
c.
3
2
1
4
d.
4
1
2
3

C.S.E-Pre 2001

1 of 15

GEOGRAPHY
1.

2.

3.

Which one of the following statements is


NOT correct?
a. High
weathered
tropical
and
subtropical soils alt called Oxisols.
b. Grassland soils rich in organic matter
and high in bases are called Mollisols
c. Soils in waterlogged areas with high
organic matter are called Histosols
d. Young soils with no horizons are
called Incept sols
The transition zone between two
Ecosystems is called
a. biome
b. biotope
c. ecotone
d. sere
Which one of the following statements is
hot correct from the diagram of the given
food web?

5.

6.

Planets
Grasshopper
Mouse

Praying
Mantis

OWL

Marten

Shrew

7.
a.
b.
c.
d.

4.

Mouse is a secondary consumer


Shrew is a tertiary consumer
Grasshopper is a primary consumer
Owl is both a secondary and
quaternary consumer
Which one of the following statements is
not correct?
a. The energy levels within food chain
are trophic level
b. In deep oceans, primary production is
almost nil
c. The primary consumers are called
autotrophs
d. The
decomposers
are
called
saprotrophs

8.

The first research project programme


which focussed on the El Nino-Southern
Oscillation and its predictability called
a. Climate Variability and Predictability
(CUVAR)
b. Global Energy and Water Cycle
Experiment (GEWEX)
c. Global Change and Terrestrial
Ecosystem (GCTE)
d. Tropical
Ocean
and
Global
Atmosphere (TOGA)
Consider the following statements
1. Human Geography covets alt those
aspects of Geography which are not
directly concerned with Physical
Geography.
2. Human Geography is the study of
interrelationship
between
human
beings and their environment.
3. Human Geography deals with the
description and explanation of human
phenomena around the variable earth
surface.
4. Human Geography does not cover
technical mailers of Geography.
Which of the above statements are correct?
a. 1 and 2
b. 2 and 3
c. 3 and 4
d. 1, 2, 3 and 4
Match List I with List II and select the
correct answer
List I (Tribe) List II (Country)
A. Chukchi 1. Indonesia
B. Dayak
2. Mexico
C. Lacandon 3. Russia
D. Lese
4. Zaire
A
B
C
D
a.
3
1
4
2
b.
1
3
2
4
c.
3
1
2
4
d.
1
3
4
2
The dotted area in the given diagram
shows

13.

9.

10.

11.

12.

a. Growth rate of population


b. Infant mortality rate of population
c. Natural growth rate of population
d. Net reproductive rate of population
The Mobility Transition Model of
Migration was propounded by
a. Clark W.A.V
b. Lee, E
c. Revenstein, E.G.
d. Zelinsky, W.
Consider the following statements:
1. Population of European countries is
stagnating.
2. European countries fertility rates are
below the replacement level.
3. The fertility rated in all the countries of
Europe are below 1.5 per cent
4. Above 12 per cent of population of
Europe is in the senile age group.
Which of the above statements are correct?
a. 1, 2 and 3
b. 2, 3 and 4
c. 1, 2 and 4
d. 1, 2, 3 and 4
The hunting and gathering economy can
support only
a. 1 person per sq. km
b. 3 persons per sq. km
c. 5 persons per q. km
d. 7 person per sq. km
Consider the following statements
regarding lapse rate
1. An unsaturated alt mass cools at dry
adiabatic lapse rate when rises in the
atmosphere.
2. The dry adiabatic, lapse rate amounts
to 1 C per 100 meters.
3. The wet adiabatic lapse rate may vary
between 04C and 09C per 100
meters.
4. Dry adiabatic lapse rate is higher
because of the release of latent heat of
condensation.

14.

15.

16.

17.

2 of 15
Which of the above statements are
correct?
a. 1, 2 and 4
b. 1, 3 and 4
c. 2, 3 and 4
d. 1, 2 and 3
Which one of the following statements
explains that mining is an increasing
cost economic activity?
a. Mine workers enhance their wages
b. Best and most easily available m
deposits are mined first
c. Mining techniques becomes expensive
d. Mining royalties increase
For his model of industrial location, Weber
makes use of
a. Isogons
b. Sophenes
c. Isodapanes
d. Isotachs
A person moving from Chile to Brazil
along the coast cross
a. Bass Strait
b. Cook Strait
c. Magellan Strait
d. Torres Strait
The settlements in the given region
represent

a. Cauvery delta
b. Lower Brahmaputra Valley
c. Mahanadi delta
d. Upper Ganga plain
Match List I with List Ii and select the
correct answer
List I (Term / Model)
A. Conurbation
B. Functional

classification

C. Hierarchy of settlements
D. Umland
List II (Author)

of

towns

1.
2.
3.
4.

18.

19.

20.

21.

Van Cleef
Christaller
Geddes
Harris
A
B
C
D
a.
3
4
1
2
b.
3
4
2
1
c.
4
3
2
1
d.
4
3
1
2
In the Newling model on urban population
density distribution, the term Density
Crater is applied to
a. Central Business District
b. High income residential area
c. Suburb
d. Zone in Transition
The Colby hypothesis offers a dynamic
explanation of
a. urban development
b. landuse development
c. sustainable development
d. rural development
Match List I with List II and select the
correct answer
List I (Characteristics)
Cloudiness and heavy precipitation
unfavorable for livestock farming
A. Westerlies throughout the year and
mild winter
B. Famous for game and parkland type of
vegetation
C. Dry summers, wet winters and Orchard
farming
List II (Regions)
1. Equatorial region
2. Mediterraneam region
3. Savanna region
4. West European region
A
B
C
D
a.
1
4
3
2
b.
4
1
3
2
c.
1
4
2
3
d.
4
1
2
3
Which one of the following natural regions
is not correctly matched with its economic
base?
a. Canadian Prairies : Spring wheat
cultivation
b. Argentine Pampas :
Beef
cattle
ranching
c. Australian Downs : Nomadic herding

22.

23.

24.

3 of 15
d. South African Veld: Maize growing
Consider the following similarities
between Canada and Australia
1. Close connections with the U.S.A. and
UK.
2. Environmental variety
3. Exporters of raw materials and food
4. Population concentration in some
peripheral pockets
Which of the above similarities are
correct?
a. 2, 3 and 4
b. 1, 2 and 3
c. 1, 3 and 4
d. 1, 2 and 4
Match List I with List II and select the
correct answer
List I (Industries)
A. Automobile
B. Cotton Textile
C. Iron and Steel
D. Ship building
List II (Industrial Centers)
1. Baltimore
2. Dallas
3. New York
4. Providence
5. Wheeling
A
B
C
D
a.
4
2
5
1
b.
2
4
5
1
c.
4
2
1
3
d.
2
4
1
3
Match List I with List II and select the
correct answer:
List I (Country)
A. Belgium
B. Germany
C. Netherlands
D. Sweden
List-II (Highest Ranking in Europe)
1. population size
2. population density
3. percent of urban population
4. percent of agricultural population
5. percent of population in age group
beyond 65 years
A
B
C
D
a.
3
1
5
4
b.
1
3
2
5
c.
3
1
2
5

4 of 15
25.

26.

27.

28.

d.
1
3
5
4
Which one o the following statements is
not correct about Netherlands ?
a. It is the most densely populated
country in Western Europe
b. It is an efficient dairy producer in the
European Community
c. It depends heavily on the imports of
livestock feed
d. It is the largest exporter of butter in the
world
The shift of aluminium industry from the
Dneiper region to Siberia was made
possible because of
a. exploration of new bauxite sources
b. development of hydroelectricity
c. new favourable market location
d. development of efficient transportation
Consider the following industrial regions
of Japan
A. Kinki region
B. Kitakyushu region
C. Kanto region
D. Nagoya region
Identity location of the respective regions
on the map

A
B
C
D
a.
3
1
5
6
b.
2
4
5
3
c.
3
4
2
5
d.
2
1
3
6
Consider the following regions of major
economic activities in Australia
A. Commercial grain farming
B. Intensive commercial fanning
C. Hunting -gathering
D. Ranching
Identify the location of the respective
regions on the map

29.

30.

31.

32.

A
B
C
D
a.
4
3
1
2
b.
3
4
5
1
c.
3
4
1
2
d.
4
3
5
1
In which one of the following areas of
South-East Asia is there the largest
concentration of peasant population
a. In areas of shifting cultivation
b. On island areas where grain farming is
carried on
c. On highland areas where the climate is
cooler and healthier
d. On cleared lowland forest area
The peninsular Malaysia has greater
densities of population on its western
coastland because of
1. Greater possibilities of agriculture on
the west.
2. The east coast has swampy plains and
gives way to more mountainous
terrain.
3. The western lowlands proved ideal for
colonial plantation development.
4. Fishing is yew important along with
agriculture on the west.
Select the correct answer using the codes
given below
a. 1 and 2
b. 1, 2 and 3
c. 1, 3 and 4
d. 1 and 4
The contrast in economic development
between Israel and the other South-West
Asian countries is due to
a. deserted conditions of other countries
b. mineral wealth of Israel
c. irrigation facilities available in Israel
d. high-technology revolution in Israel
Match List I with List II and select the
correct answer
List I (Country)
A. Iran

33.

34.

35.

36.

B. Iraq
C. Israel
D. Saudi Arabia
A
B
C
D
a.
2
1
3
4
b.
2
1
4
3
c.
1
2
3
4
d.
1
2
4
3
Which one of the 1ollow pairs is not
correctly matched?
a. Szechwan Basin :
Petroleum
b. Hainan region
:
Iron ore
c. Shansi region
:
Coal
d. Shensi region
:
Tin
Which one of the dotted lines labelled as 1,
2, 3 and 4 on the map of China acts as a
divide between tropical and temperature
crops?

a. 1
b. 2
c. 3
d. 4
The correct sequence of the natural regions
of Brazil from north to south is
a. Amazon Basin, Mato Grosso Plateau,
Brazilian High lands, Campos
b. Amazon Basin, Brazilian Highlands,
Campos, Mato Grosso Plateau
c. Brazilian Highlands, Mato Grosso
Plateau, Amazon Basin, Campos
d. Mato Grosso Plateau, Campus.
Brazilian Highlands, Amazon Basin
Consider he following c cities of the
Southern African countries
A. Harare
B. Lilonew
C. Maseru
D. Windhoek

37.

38.

39.

5 of 15
Identify the location of the respective
capital cities on the using the codes given
below:
A
B
C
D
1.
4
5
2
3
2.
5
4
2
3
3.
4
5
3
1
4.
5
4
3
1
Consider the following crop regions of Sri
Lanka
A. Coconut region
B. Corn and Millet region
C. Rubber region
D. Tea region

Identify the location of the respective


regions on the map:
A
B
C
D
a.
5
4
2
1
b.
4
5
3
2
c.
5
4
3
2
d.
4
5
2
1
Which one of the following is located in
Russia?
a. Donetsk
b. Krivoirong
c. Zhitomir
d. Pechora
Match List I with List II and select the
correct answer:
List-I (States as on Dec. 2000)
A. Madhya Pradesh
B. Maharashtra
C. Andhra Pradesh
D. Rajasthan
1. 1
2. 2
3. 3
4. 4
A
B
C
D
a.
2
3
4
1
b.
3
2
4
1
c.
2
3
1
4
d.
3
2
1
4

6 of 15
40.

41.

42.

43.

44.

45.

The correct sequence in the ascending


order of the given cities in terms of
altitude above mean sea-level is
a. Marmugao, Mumbai, Kolkata, Chennai
b. Mumbai, Kokata, Chennai, Marmugao
c. Chennai, Marmugao, Mumbai, Kolkata
d. Kolkata, Mumbai, Chennai, Marmuago
Kathiawar Peninsula is an example of
a. emerged shoreline
b. submerged shoreline
c. ria shorteline
d. dalmatian shorteline
Match List I and List II and select the
correct answer:
List-I (Rivers)
A. Krishna
B. Brahmaputra
C. Godavari
D. Yamuna
List-II (Their Tributaries)
1. Chambal
2. Indravati
3. Tista
4. Bhima
A
B
C
D
a.
4
3
2
1
b.
3
4
1
2
c.
4
3
1
2
d.
3
4
2
1
The upper course of Damodar river
occupies as:
a. rif valley
b. synclinal valley
c. eroded valley
d. depositional valley
In the given diagram, spatial-spread of a
river basin of India has been shown.
Identify the river:

a. Cauvery
b. Brahmini
c. Godavari
d. Mahanadi
The given diagram shows the temperature
regime (0C) of

46.

47.

48.

a. Koklata
b. Cherrapunji
c. Kochi
d. Bhopal
The following table contains temperature
and rainfall data of a city:
Months
Characteristics
Temperature Rainfall
in mm
00C
January
16
23
February
19
15
March
25
15
April
31
5
May
35
15
June
34
127
July
30
320
August
29
254
September
29
213
October
26
58
November
20
8
December
16
8
The data is associated with
a. Ahmedabad
b. Allahabad
c. Nagpur
d. New Delhi
Large area in India is subjected to
considerable rainfall variability. Rainfall
variability is likely to be the greates in area
of:
a. very high rainfall
b. high rainfall
c. medium rainfall
d. very low rainfall
In the given map, the shaded area
represents a region of important forest
resource of our country Name of the forest
is

53.

49.

50.

51.

52.

a. Tropical wet evergreen


b. Tropical wet deciduous
c. Sub-tropical wet forest
d. Tropical dry deciduous
Which one of the following ores is
abundant in the region Balaghat-BhandaraNagpur?
a. Iron
b. Manganese
c. Mica
d. Bauxite
Match List I with List I and select the
correct answer:
List I (Minerals)
A. Lignite
B. Asbestos
C. Titanium
D. Chromite
List II (Area)
1. Bikaner area
2. Chhotangapur region
3. Southern Malbar Coast
4. Ratnagiri region
A
B
C
D
a.
1
2
3
4
b.
1
2
4
3
c.
2
1
3
4
d.
2
1
4
3
The correct sequence (in descending order)
of the given industries in terms of
consumption of coal in India is:
a. iron and steel, cement, textile, thermal
power
b. thermal power, textile, iron and steel,
cement
c. iron and steel, thermal power, cement,
textile
d. thermal power, iron and steel, cement,
textile
The power plant at Manikaran based on
geothermal energy is the state of
a. Arunachal Pradesh
b. Himachal Pradesh
c. Jammu and Kashmir
d. Uttranchal

54.

55.

56.

57.

7 of 15
The areal spread of fishing ground of the
continental shelf along the peninsular coast
of India is over
a. 2 lakh km2
b. 3 lakh km2
c. 4 lakh km2
d. 5 lakh km2
In Eastern Himalaya, the subtropical board
leaved forests are commonly found
between the altitudes ranging from
a. 500 to 1000 metres
b. 750 to 1000 metre
c. 1000 to 2000 metres
d. 2000 to 2500 metres
The transhumance is still an important
component of the socio-economic life of
the people in region at:
a. Andaman and Nicobar islands
b. Meghalaya
c. Jammu and Kashmir
d. Himacha pradesh
Match List I with List II and select the
correct answer
List I (Agricultural produce)
A. Saffron
B. Jowar
C. Costor Seed
D. Sesamum
List I (Producing State)
1. Maharashtra
2. Jammu and Kashmir
3. Rajasthan
4. Andhra Pradesh
A
B
C
D
a.
2
1
4
3
b.
2
1
3
4
c.
1
2
4
3
d.
1
2
3
4
Match the four types of crop areas marked
as l, 2, 3 and 4 in the given map with the
following crops

A.
B.
C.
D.

Tur
Maize
Potato
Sugarcane

58.

59.

60.

61.

A
B
C
D
a.
4
2
3
1
b.
2
4
1
3
c.
4
2
1
3
d.
2
4
3
1
Match List I with List II and se the correct
answer
List I (Area/Region / Source)
A. Longest navigable waterway
B. Bhanger soils
C. Rock-formations for coal deposits in
India
D. Dandkaranya region
List II (Geographical characteristics)
1. Gondwana area
2. Indravati region
3. Higher tracts of plains
4. Uttar Prad
5. Bihar
A
B
C
D
a.
4
3
2
5
b.
3
4
1
2
c.
3
4
2
5
d.
4
3
1
2
There is now a tendency to set up more
sugar factories in South India because of
1. Longer crushing season
2. Higher productivity of sugarcane
3. Scarcity of other cash crops
4. Efficient transport system
Which of the above answers are correct?
a. 1, 2 and 3
b. 1, 3 and 4
c. 1, 2 and 4
d. 2, 3 and 4
The correct sequence of four railway
stations ( , 2, 3; 4) shown in the given map
between Delhi to Mumbai is

a. Jhansi, Itarsi, Jalgaon, Manmad


b. Itarsi, Manrnad, Jhansi, Jalgaon
c. Itarsi, Jhansi, Manmad, Jalgaon
d. Jhansi, Itarsi, Manmad, Jalgaon
The first high tect port of India is
a. Quilon

62.

63.

8 of 15
b. Nagapathinam
c. Paradeep
d. Nava Sheva
Ganga is navigable between
a. Hardwar and Kanpur
b. Mahabad and Varanasi
c. Patna and Calcutta
d. Allahabad and Haldia
Match List I with List II and select the
correct answer
List I (Female characteristics)
A. Life expectancy of birth (years)
B. Adult literacy rate (%)
C. Pregnant women with anaemia (%)
D. Women-held seats in Parliament (%)

64.

List I (Years/% etc.)


1. 88
2. 8.9
3. 43.5
4. 63.3
A
B
C
D
a.
3
4
1
2
b.
4
3
1
2
c.
3
4
2
1
d.
4
3
2
1
Match the four areas marked as 1, 2, 3 and
4 in the given map showing distribution of
the following population / irrigation
characteristic

A. Well irrigated areas


B. Percentage literacy rate of 45 to 50 in
1991
C. Population density distribution area of
200 400 persons per sq. km.
D. Population growth rate of 25 30%
during 198191.
A
B
C
D
a.
3
4
2
1
b.
4
3
2
1

65.

66.

67.

68.

69.

c.
4
3
1
2
d.
3
4
1
2
Match List I with List II and select the
correct answer
List I (National Parks and Wild life
sanctuaries)
A. Nandan Kanan
B. Kaziranga
C. Bandhavgarh
D. Melghat
List II (States)
1. Maharashtra
2. Madhya Pradesh
3. Orissa
4. Assam
A
B
C
D
a.
4
3
2
1
b.
4
3
1
2
c.
3
4
1
2
d.
3
4
2
1
Kanha National Park belongs to which one
of
the
following
bio-geographical
provinces?
a. Tropical Humid Forests
b. Warm Semi Desert
c. Tropical Dry Forests
d. Tropical Sub-humid Forests
Which one of the following scholars was
the first to divide the world landmass into
three continents Europe, Asia and Libya
(Africa)?
a. Anaximander
b. Hecataeus
c. Herodotus
d. Eratosthenes
The given map was prepared by the

a. Arabs
b. Greeks
c. Phoenicians
d. Romans
Match List I with List II and select the
correct answer
List I (Old name)
A. Asikni
B. Parushi

70.

71.

72.

73.

74.

75.

9 of 15
C. Sutudri
D. Vitasta
List II (New name)
1. Chenab
2. Jhelum
3. Ravi
4. Sutlej
A
B
C
D
a.
1
3
2
4
b.
3
1
4
2
c.
3
1
2
4
d.
1
3
4
2
An early statement of Geography as
Choroogy was provided by
a. Kant
b. Humboldt
c. Ritter
d. Varenius
Ratzels work was based on the concept
a. There is a dichotomy between physical
and cultural aspects of geography
b. Geography is a scientific discipline
c. Physical
environment
controlled
human activities
d. Geography is necessarily descriptive
Huntington is noted for describing
a. man as a product of the earths surface
b. the evolution of landforms
c. the effects of climate on human life
d. the historical study of the cultural
landscape
The kind of relationship that exists
between the elements given system is

a. complex
b. feedback
c. parallel
d. simple
The behavioural environment is meant by
a. reality as it exists in nature
b. the decisions based on reasoned
thought
c. mans behaviour as a function of the
environment
d. reality as is perceived by individual
Match List I with List I and select the
coned answer

List I (Book)
A. The Changing Nature of Geography
B. Conceptual Revolution in Geography
C. Geography and Geographers
D. Themes in Geographic Thought

76.

77.

List II (Author)
1. Davis, W.K.
2. Harvey, M.E.
3. Johnston, R.J.
4. Minshull, P.
A
B
C
D
a.
1
4
3
2
b.
1
4
2
3
c.
4
1
2
3
d.
4
1
3
2
Consider the following statements:
1. Quantitative Revolution means the use
of statistical and mathematical
techniques
in
understanding
geographical system.
2. Quantitative methods made geography
as a regional science.
3. Quantitative methods range from
passive
observation
to
active
intervention.
4. Quantitative
methods
influenced
geography
mainly through the
Location Theory.
Which of the above statements are correct
a. l and 2
b. 1, 2 and 3
c. 1, 2 and 4
d. 1, 2, 3 and 4
Match List I with List I and se the correct
answer
List I (Statements)
A. The earth itself must be asked for its
laws
B. The unity of geography is in its
methods
C. Man is the product of the earths
surface
D. Man everywhere becomes most
essentially associated with terrestrial
life
List II (Authors)
1. Humboldt

78.

79.

80.

81.

10 of 15
2. Ritter
3. Hettner
4. Ellen C. Sempl
A
B
C
D
a.
3
2
1
4
b.
2
3
4
1
c.
2
3
1
4
d.
3
2
4
1
Which one of the following concepts is
associated with Vidal de La Blache?
a. Detetminism
b. Possibilism
c. Environmentalism
d. Probabilism
Consider the following statements
1. Radicalism in geography came in the
form of a reaction to geography as a
spatial science,
2. The origin of radical approach can be
traced to civil rights, poverty and
inequality.
3. The founders of radicalism in
geography had a strong Marxist base.
4. Some of the radical geographers
termed radicalism as the politicaleconomy perspective.
Which of the above statements are correct?
a. 1, 2 and 3
b. 1, 2 and 4
c. 1, 3 and 4
d. 1, 2, 3 and 4
An approach of reasoning from particular
to general is known as a. Positivist
b. Normative
c. Inductive
d. Deductive
Consider the figures X and V

Which one of the following statements is


correct?
a. Figure X is small scale and the
distance between A and B is 40 metres
b. Figure Y is large scale and the distance
between C and D is 4 km
c. Figure X is large scale and Figure Y is
small scale. The distances between A

82.

83.

84.

85.

86.

and B and C and D are 40 meters and


20 km respectively
d. Figure X is large scale and Figure Y is
small scale. The ground distance
between A and B and C and D are
equal
The form lines are
a. the interpolated contours
b. the extrapolation of contours
c. significant contours
d. generalized contours
Which one of the following pairs is not
correctly matched?
a. Rotameter :
used for measuring
distances on maps
b. Planimeter :
used for measuring
areas on maps
c. Pantograph:
used for reduction
of maps
d. Ediograph :
used for measuring
volumes
Match List I with List II and select the
correct answer
List I (Slopes) List I (Contours)

A
B
C
D
a.
1
2
3
4
b.
1
2
4
3
c.
2
1
4
3
d.
2
1
3
4
The Sten-de-Geers diagram is used to
represent
a. sex ratio
b. urban population
c. industrial population
d. rural & urban population
The given graph of spatial point random
sampling is

87.

11 of 15
a. simple
b. nested
c. stratified
d. systematic
Consider the following labels in the given
diagram of a vertical air photograph:

1. Negative plane
2. Nadir point
3. Camera axis
4. Ground plane
Which of the above labels are correct?
a. 1, 2 and 3
b. 2, 3 and 4
c. 1, 3 and 4
d. 1, 2, and 4
88.
The correct sequence of the given parts of
Electromagnetic spectrum in terms of their
wavelength is
a. Ultraviolet, Visible, Gama, Infrared,
Radio
b. Gamma, Ultraviolet, Visible, Infrared,
Radio
c. Gamma, Radio, Infrared, Ultraviolet
Visible
d. visible, Ultraviolet, Gamma, Infrared,
Radio
Directions: The following 12 (twelve) items
consist of two statements, one labelled as
Assertion A and the other labelled as Reason
R. You are to examine these two statements
carefully and decide if the Assertion A and the
Reason R are individually their and if so, whether
the Reason is a correct explanation of the
Assertion, Select your answers to these items
using the codes given below and mark your
answer sheet accordingly.
89.

Assertion (A): Basalt is a fine-grained and


dark-coloured igneous rock which is
formed below the earths surface.
Reason (R): The rate of cooling and
solidification of magma inside the earth is
very slow.
a. Both A and Rare true and R is the
correct explanation of A

90.

91.

92.

93.

b. Both A and R are true but R is NOT


the correct explanation of A
c. A is true but R is false
d. A is false but R is true
Assertion (A): Explosive types of volcanic
eruptions are associated with the
destructive or convergent plate boundaries
in which the heavier plate is sub ducted
beneath the lighter plate.
Reason (R): Materials of the upper mantle
lying below the mid-oceanic ridges are
melted and move upward in the form of
lave during volcanic eruptions.
a. Both A and Rare true and R is the
correct explanation of A
b. Both A and R are true but R is NOT
the correct explanation of A
c. A is true but R is false
d. A is false but R is true
Assertion (A): The structure of the interior
of the earth is deduced with the he of
seismic waves.
Reason (R): The S waves travel in solid,
liquid and gas while the P waves travel
only in solid.
a. Both A and Rare true and R is the
correct explanation of A
b. Both A and R are true but R is NOT
the correct explanation of A
c. A is true but R is false
d. A is false but R is true
Assertion (A): The annual range of
temperature is greater over areas.
Reason (R): Land areas warm and cool
more rapidly than water bodies.
a. Both A and Rare true and R is the
correct explanation of A
b. Both A and R are true but R is NOT
the correct explanation of A
c. A is true but R is false
d. A is false but R is true
Assertion (A): In oligotrophic Lakes, high
concentration of nutrients result in algae
blooms.
Reason (R): A higher concentration of
nutrients result in higher organic
productivity.
a. Both A and Rare true and R is the
correct explanation of A
b. Both A and R are true but R is NOT
the correct explanation of A
c. A is true but R is false

94.

95.

96.

97.

98.

12 of 15
d. A is false but R is true
Assertion (A): The concept of plantation
agriculture has changed in recent years.
Reason (R): Change the size of
landholdings led to the introduction of
small and medium farmers in the market.
a. Both A and Rare true and R is the
correct explanation of A
b. Both A and R are true but R is NOT
the correct explanation of A
c. A is true but R is false
d. A is false but R is true
Assertion (A): About 80 per cent of
Canadian manufacturing is concentrated in
South Ontario and St. Lawrence owland
regions.
Reason (R): There are abundant deposits
of coal in South Ontario and St. Lawrence
lowland regions.
a. Both A and Rare true and R is the
correct explanation of A
b. Both A and R are true but R is NOT
the correct explanation of A
c. A is true but R is false
d. A is false but R is true
Assertion (A): Tropical seas in India are
rich in fish species but there are few
important fishing grounds around the
country.
Reason (R): The quantity of fish of a
particular species is lacking and hence not
commercially exploitable.
a. Both A and Rare true and R is the
correct explanation of A
b. Both A and R are true but R is NOT
the correct explanation of A
c. A is true but R is false
d. A is false but R is true
Assertion (A): Aksai Chin is desolate and
the driest part of the Ladakh region.
Reason (R): The whole area bears a stamp
of excessive aeolian erosion,
a. Both A and Rare true and R is the
correct explanation of A
b. Both A and R are true but R is NOT
the correct explanation of A
c. A is true but R is false
d. A is false but R is true
Assertion (A): The followers of
Quantitative Revolution promoted Human
Geography as Spatial Science.

99.

100.

101.

102.

Reason (R): They assumed people to be


rational and who by to optimise their
benefits.
a. Both A and Rare true and R is the
correct explanation of A
b. Both A and R are true but R is NOT
the correct explanation of A
c. A is true but R is false
d. A is false but R is true
Assertion (A): Mercator Projection is used
by navigators.
Reason (R): It shows the true area of the
countries.
a. Both A and Rare true and R is the
correct explanation of A
b. Both A and R are true but R is NOT
the correct explanation of A
c. A is true but R is false
d. A is false but R is true
Assertion (A): Plano lens is used in pocket
stereoscope
Reason (R): Photographs must overlap
while viewing a three-dimensional image.
a. Both A and Rare true and R is the
correct explanation of A
b. Both A and R are true but R is NOT
the correct explanation of A
c. A is true but R is false
d. A is false but R is true
In support of his hypothesis on the origin
of the Earth, which one of the following
scholars said, Give me matter and I Will
build a world out of it?
a. James Jeans
b. T. C. Chamberlin
c. Immanuel Kant
d. Laplace
Match list I with List II and select the
correct answer
List I (Geological Period)
A. Carboniferous
B. Pre-Cambrian
C. Pleistocene
D. Upper Creataceous
List II (Events associated with the period)
1. Start of Himalayan Orogeny
2. Birds made their appearance
3. Formation of rocks containing coal,
petroleum and natural gas
4. Large areas subjected to glaciation
5. Rock formation having rich metallic
materials s

103.

104.

105.

106.

13 of 15
A
B
C
D
a.
5
3
2
4
b.
3
5
4
1
c.
5
3
4
1
d.
3
5
2
4
Consider the following statements in
relation to Earths topmost layer 1. Transverse earthquake waves move at
a speed of 3.5 km /second in that layer.
2. The specific gravity is 2.7.
3. It is shallowest under Continents and
deepest under Oceans.
4. Abudance of magnesium.
Which of the above statements are correct?
a. l and 2
b. 2 and 3
c. 3 and 4
d. 1, 2 and 4
Consider the following statements
1. A tectonic scarp is a steel slope that
results from differential movement of
the Earths surface.
2. The typical angle of repose for scams
of all origins is between 250 and 400
3. A scarp produced by structurally
controlled erosion at an ancient fault is
known as fault-line scarp.
4. A capable fault is one that shows
movement at or near the ground
surface at least once within the past
35000 years.
Which of the above statements are correct?
a. 1, 2, 3 and 4
b. 2, 3 and 4
c. l and 3
d. 2 and 3
Which of the following factors influence
the type and rate of weathering?
1. Glacier
2. Climate
3. Vegetation cover
4. Rock structure
5. Topography
A
B
C
D
a.
1
2
3
4
b.
2
3
4
5
c.
1
2
3
5
d.
1
2
4
5
In the given diagram, the portion of the
river marked as X is called

110.

107.

108.

109.

a. Braided river
b. Distributary
c. Incisde meander
d. Yezoo
Match List I with List U and select the
correct answer
List I (Cause)
A. Normal Faulting
B. Saltation
C. Ice scouring
D. Infiltration of water
List II (Result)
1. Aquifer
2. Kettle hole
3. Escarpment
4. Transportation of stream-load
A
B
C
D
a.
3
4
1
2
b.
4
3
1
2
c.
3
4
2
1
d.
4
3
2
1
Match List I with List II and select the
correct answer
List I (Characteristic)
A. A plain, largely composed of recent
alluviam
B. A deep sea plain
C. A plain formed by wind action
D. A level surface lightly covered with
thin layer of alluvium
List I (Landform)
1. Pedipla
2. Loess plain
3. Food plain
4. Abyssal plain
A
B
C
D
a.
3
4
2
1
b.
4
3
2
1
c.
3
4
1
2
d.
4
3
1
2
The given block diagram shows:

111.

112.

113.

14 of 15
a. Soil creep
b. Debris flow
c. Rock slide
d. Slump
The percentage of radiant energy reflected
back by a surface is called
a. Albedo
b. Greenhouse effect
c. Insolation
d. Refraction
Match List I with List II and select the
correct answer:
List I (Layer of Atmosphere)
A. Stratosphere
B. Exosphere
C. Troposphere
D. Ionosphere
List II (Characteristics related with the
layers)
1. Temperature decreases with increasing
height
2. Aurora Borealis and Aurora Australis
are produced
3. Contains much of the total atmospheric
Ozone
4. No etilucent clouds are observed over
high latitude in summer
5. Atoms of oxygen, hydrogen and
helium form the tenuous atmosphere
A
B
C
D
a.
5
3
2
4
b.
3
5
1
2
c.
3
5
2
4
d.
5
3
1
2
Tarim Basin of Sinkiang experiences
strong heat wave causing discomfort
during early spring. The wind is popularly
known as
a. Karaburan
b. Katabatic
c. Chinook
d. Fohn
Consider the following conditions for the
formation of temperature inversions:
1. Cloudy sky
2. Strong winds
3. Long winter nights
4. Cold dry air
Which of the above conditions are ideal?
a. 1, 2 and 3

114.

115.

116.

117.

b. 1 and 2
c. 2, 3 and 4
d. 3 and 4
Consider the following statements
1. In the high latitudes, east-coast regions
are wetter than west-coasts.
2. Precipitation is abundant on the
windward slopes but sparse on the
leeward slopes.
3. In the subtropics, west-coasts are
wetter than the east-coasts.
4. Precipitation is abundant in the middle
latitudes.
Which of the above statements is/are
correct?
a. 1, 2 and 3
b. 2 only
c. 2 and 4
d. 3 and 4
Identify the correct sequence of the given
processes regarding rainfall
a. unsaturated air, condensation, dew
point, precipitation
b. dew point, condensation, unsaturated
air, precipitation
c. unsaturated
air,
dew
point,
condensation, precipitation
d. dew point, precipitation, condensation,
unsaturated air
Identify the climatic type which has cool
summer (100C to 150C) and cold winter
(00C to 50C) and rainfall throughout the
year with no dry season
a. Df
b. Cw
c. Cs
d. Cf
The layer of water in the oceans and lakes
that separates the warmer surface layer
from the deeper colder layer is called
a. Epilimnion
b. Hypolimnion

118.

119.

120.

15 of 15
c. Thermocline
d. Hypothermia
The area marked X in the given diagram
represents

a. Kurile Trench
b. Mindanao Trench
c. Marianas Trench
d. Tonga Trench
Consider the following statements
1. Continental shelves are wider where
high mountains are very close and
parallel to the coast.
2. Continental shelves are formed by
prolonged deposition of detritus under
sea water in those areas where ea
conditions are cairn.
3. Extensive continentatal shelves are
formed by marine erosion of the
continental margins during the time of
negative change of the sea level.
4. Continental shelves are formed by the
submergence of continental margins
due to tilting of land towards sea
Which of the above statements are correct?
a. 1, 2 and 3
b. 2, 3 and 4
c. 1, 3 and 4
d. 1, 2 and 4
Which reef type is shown by X; mark on
the diagram given below?
a. Fringing reef
b. Bather reef
c. Patch reef
d. Atoll

C.S.E-Pre 2002

1 of 16

GEOGRAPHY
1.

2.

3.

4.

5.

Precambrian rocks in India are most found


in the
a. Aravallis
b. Western Ghats
c. Gondwanas
d. Siwaliks
Consider the following details:
Avem July temperature: 26C to 32C
Average January temperature: 19C to
28C
Rainfall > 200 cm
These climatic characteristics are typical
of which one of the following climatic
zones?
a. Humid south-eastern India
b. Sub -humid littorals
c. Humid north-eastern India
d. Humid Sthyadri and west coast
Shabar belt is found in the
a. Chota Nagpur plateau
b. Himalayan Piedmont zone
c. Western Ghats
d. Coastal Orissa
Match List I with List II and select the
correct answer:
List I (Trees)
A. Sal
B. Acacia
C. Sundari
D. Juniper
List II (Vegetation Type)
1. Tropical dry deciduous
2. Littoral and swamp
3. Alpine forest
4. Tropical moist deciduous
A
B
C
D
a.
3
2
1
4
b.
4
2
1
3
c.
4
1
2
3
d.
3
1
2
4
Consider the following statements

6.

7.

The belt of coniferous forests in Nepal,


Sikkim and Assam Himalayas is at a
higher altitude than those in Kashmir
Himalayas because Sikkim and Assam
Himalayas
1. are at lower latitudes
2. get lesser snowfall at lower heights
3. have higher temperatures at lower
heights
4. have lower altitudes
5. have more rainfall at lower heights
Which of these are correct?
a. 1, 2 and 3
b. 2, 3 and 4
c. 1, 4 and 5
d. 1, 2, 3 and 5
Which one of the following States is the
largest producer of thermal power in
India?
a. Bihar
b. Tamil Nadu
c. Maharashtra
d. Chattisgarh
Match List I with List II and select the
correct answer
List I (Minerals)
A. Copper
B. Iron ore
C. Zinc
D. Manganese
List II (Regions)
1. Balaghat and Chindwara
2. Badampahar and Kudarmukh
3. Khetri and Manbhandar
4. Zawar
A
B
C
D
a.
3
2
4
1
b.
4
2
3
1
c.
4
1
3
2
d.
3
1
4
2

8.

9.

10.

11.

12.

13.

14.

Which one of the following pairs of forest


resources are animal products?
a. Fibres and flosses
b. Gums and resins
c. Lac and wax
d. Drugs and spices
The process that results in the breakdown
of rocks and minerals in situ is known as
a. attrition
b. erosion
c. weathering
d. corrosion
Which one of the following is most
abudant in Kerala?
a. Thorium
b. Uranium
c. Mica
d. Monazite
Consider the following statements
1. Rice and tobacco are the major crops
of Andhra Pradesh
2. Rice and tea are the major crops of
West Bengal
3. Rice and jowar are the major crops of
Orissa
4. Rice and coffee are the major crops of
Tamil Nadu
Which of these are correct?
a. 1, 2 and 3
b. 1, 2 and 4
c. 1, 3 and 4
d. 2, 3 and 4
Consider the following States
1. Andhra Pradesh
2. Gujarat
3. Maharashtra
4. Tamil Nadu
The correct sequence of these States in
descending order in terms of percentage of
urban population (as per 2001 Census), is
a. 4, 2, 3, 1
b. 4, 3, 2, 1
c. 1, 2, 3, 4
d. 3, 2, 1, 4
As per 2001 Census of India, the number
of million cities is

15.

16.

17.

18.

19.

20.

2 of 16
a. 25
b. 30
c. 35
d. 40
Which one of the following States,
according to the 2001 Census, has the
highest sex ratio in the age group of 0 to
6 years?
a. Nagaland
b. Nagaland
c. Sikkim
d. Tripura
The aboriginal population of the Andaman
Islands be-tongs to which one of the
following racial categories?
a. Negroids
b. Negrito
c. Palaeo-Australoids
d. Negrillos
Which one of the Following pairs is NOT
correctly matched
a. Wildlife Protection Act: 1972
b. Water Preservation Act: 1977
c. Ganga Action Plan: 1981
d. Environment Protection Act:- 1986
Who among the following contributed
most to the development of Astronomy
a. Greeks
b. Romans
c. Indians
d. Arabs
Consider the following statements:
1. Plaska-Dwipa lies to the south of
Kraunca-Dwipa
2. Salmali-Dwipa lies to the south of
Kusa-Dwipa
3. Pushkara-Dwipa lies to the south-east
of Jambu-Dwipa
4. Kusa-DvApa lies to the west of JambuDwipa
Which of these are correct?
a. 1, 2 and 3
b. 1, 2 and 4
c. 2, 3 and 4
d. 1, 3 and 4
Climatic influences are persistent, often
obdurate in their control. Arid regions

21.

22.

23.

permit agriculture only through irrigation.


The economic prosperity of Egypt today
depends on the distribution of the Nile
water as in the days of Pharaohs. This
statements was made by
a. Herodotus
b. Huntington
c. Ratzel
d. Semple
Consider the following cities
1. Canton
2. Peking
3. Singapore
4. Tehran
The coned sequence of these cities, in
ascending order in terms of average annual
temperatures, is
a. 1, 3, 2, 4
b. 2, 4, 1, 3
c. 3, 1, 4, 2
d. 4, 2, 3, 1
The major oilfields of South -West Asia
are located in
e. Shore regions of F Gulf
f. Tigris-Euphrates basin
g. Rub-al-Khati desert
h. Off-shore region
Match List I with List II and select the
correct answer:
List I (Arab Geographers)
A. Ibn - Hawqal
B. AJ-Masudi
C. Al-Biruni
D. Pd Idrisi
List II (Books)
Qanum-alMasudi
1. Mirajal-Dhdhab
2. Book of routes and Realms
3. Amusements for who Desires to Travel
Around the World
A
B
C
D
a.
4
1
2
3
b.
4
2
1
3
c.
3
1
2
4
d.
3
2
1
4
Consider the following statements

24.

25.

26.

27.

28.

29.

3 of 16
1. Herodotus declared Earth as a flat disc
over which the Sun traveled in an arc
from east to west
2. In the opinion of Ptolemy, Earth is
situated in the centre of the universe
3. The T-Q map was oriented towards the
east
4. The Arab geographers oriented their
maps towards the south
Which of these are correct?
a. 1, 2 and 3
b. 2 and 4
c. 3 and 4
d. 1, 2, 3 and 4
Varenius clearly distinguished between
a. Physical and Human geography
b. General and Special geography
c. Quantitative and Qualitative geography
d. Historical
and
Contemporary
geography
The National Highway No. 5 joins
a. Delhi with Mumbai
b. Varanasi with Bangalore
c. Kolkata with Mumbai
d. Chennai with Kolkata
Which one of the following is a third-order
relief?
a. Abyssal plains
b. Cliffs
c. Plateau
d. Trench
In addition to his Anthropogeography,
Ratzel made important contributions to
a. cultural geography
b. regional geography
c. physical geography
d. political geography
The new geography of 1950s was
characterized by emphasis on
a. empirical enquiry
b. geographical aspects of social issues
c. locational theory and use of
quantitative methods and models
d. human ecology
Consider the following statements:
1. Exceptionalism is closely related to
homothetic approach in geography

30.

2. Ididgraphic approach is concerned


mainly with the unique and the
particular
3. Radicalism developed in opposition of
Marxism
4. Humanism gives the central and active
role to human awareness
Which of these are correct?
a. 1 and 2
b. 2 and 3
c. 2 and 4
d. 1 and 4
Consider the following statements of Kant
1. Knowledge can be classified either
logically or physically
2. The logical classification collects all
individual items according to similarity
of morphological features
3. The physical classification collects
individual items which belong to same
space and time
4. History is a narrative and geographic a
description
Which of these are correct?
a. 1, 2 and 3
b. 1, 3 and 4
c. 2 and 4
d. 1, 2, 3 and 4

4 of 16
D. Bab-el-Mandeb
Identify the straits marked as 1, 2, 3 and 4
on the given map diagram and select the
correct answer using the codes given
below

a.
b.
c.
d.

A
4
4
3
3

B
2
1
2
1

C
1
2
1
2

D
3
3
4
4

33.

In the above diagram of marine biological


zones, A represents
a. epipelagic zone
b. abyssopelagic zone
c. meropelagic zone
d. bathypelagic zone

31.
34.

32.

The mean monthly temperature and mean


monthly rainfall plotted in the above
diagram pertain to
a. London
b. New York
c. Rome
d. Vancouver
Consider the following straits
A. Bosporus
B. Hormuz
C. Dardanelles

The circumference of geography given in


the above diagram was propounded by
a. Ackerman, E. A.
b. Anuchin, V. A.
c. Baker, J. N. L
d. Fennerman, N. M.
35.

5 of 16
38.

36.

The dotted areas in the above map of


Attica represents
a. equatorial forests
b. nomadic herding areas
c. pygmy region
d. Tsetse Infested areas
Consider the following cities of
Afghanistan
A. Ghazni
B. Kandabar
C. Kunduz
D. Mazar-i-Sharif
Identify the cities marked as 1, 2, 3 and 4
on the given map of Afghanistan and
select the correct answer using the codes:

a.
b.
c.
d.

A
3
4
4
3

B
4
3
3
4

C
1
1
2
2

D
2
2
1
1

39.

In the above map, the shaded area


represents
a. wheat belt
b. intensive agriculture area
c. grazing lands
d. forested area
Consider the following cities of Japan:
A. Tokyo
B. Osaka
C. Hiroshima
D. Nagoya
Identify the cities marked as 1, 2, 3 and 4
on the given map of Japan and select the
correct answer using the codes given:

a.
b.
c.
d.

A
2
1
2
1

B
3
3
4
4

C
4
4
3
3

D
1
2
1
2

40.

37.

In the above map, the shaded area shows


city of
a. Gibraltar
b. Hong Kong
c. Melbourne
d. Singapore

In the above map of Japan, the broken line


( ) stands for
a. oil pipe-line
b. national highway
c. railway line
d. waterway
41.

44.

42.

In the above map of Australia, the shaded


area indicates
a. wheat cultivation and livestock raising
area
b. sheep raising area
c. cattle raising area
d. cotton cultivation area
Consider the following seas
A. Arafura
B. Banda
C. Coral
D. Solomon
Identify the seas marked as 1, 2, 3 and 4
on the given map and select the correct
answer using the codes given below

a.
b.
c.
d.

A
1
2
2
1

B
4
4
3
3

C
2
1
1
2

D
3
3
4
4

A
B
C
D
a.
3
2
1
4
b.
4
2
1
3
c.
3
1
2
4
d.
4
1
2
3
Directions: The following 23 (Twenty-Three)
items consist of two statements, one labelled as
Assertion A and the other labelled as Reason
R. You are to examine these two statements
careful and decide If the Assertion A and the
Reason R are individually true and it so, whether
the Reason is a correct explanation of the
Assertion. Select your answers to these items
using the codes given below and mark your
answer sheet accordingly.
45.

43.

In the above map of India, the shaded area


records mean July temperature between
a. 20C22.5C
b. 250C-27.50C
c. 27.5 C29 C
d. 29C310C

6 of 16
Consider the following c of Greek and
Roman periods
A. Carthage
B. Byzantium
C. Sardis
D. Load
Identify the cities marked as 1, 2, 3 and 4
the given map and select the correct
answer us the codes given below:

46.

Assertion (A): Circum-Pacific belt


accounts for major percentage share of
worlds total earthquakes of varying
magnitude arid deep focus.
Reason (R): Maximum occurrence of
earthquakes along Circuim-Pacific belt is
caused by divergent motion of tectonic
plates and consequent formation of faults.
a. Both A arid R are true and R is the
coned explanation of A
b. Both A and R are true but R is NOT
the correct explanation of A
c. A is true but R is false
d. A is false but R is true
Assertion (A): Polar areas have high
atmospheric pressure.
Reason (R): These areas receive minimum
sunshine throughout the year.

47.

48.

49.

50.

a. Both A arid R are true and R is the


coned explanation of A
b. Both A and R are true but R is NOT
the correct explanation of A
c. A is true but R is false
d. A is false but R is true
Assertion (A): Westerlies are stronger in
the northern hemisphere than in the
southern hemisphere.
Reason (R): Northern hemisphere has
more land area than the southern
hemisphere.
a. Both A arid R are true and R is the
coned explanation of A
b. Both A and R are true but R is NOT
the correct explanation of A
c. A is true but R is false
d. A is false but R is true
Assertion (A): The temperature in the
southern hemisphere is considerably lower
than that in the northern hemisphere.
Reason (R): The large mass of icecovered Antarctic continent is an
important source of cold in the south.
a. Both A arid R are true and R is the
coned explanation of A
b. Both A and R are true but R is NOT
the correct explanation of A
c. A is true but R is false
d. A is false but R is true
Assertion (A): The Great Barrier Reef is
located near Australia.
Reason (R): The coral polyps grow in
shallow, saline and warm Waters.
a. Both A arid R are true and R is the
coned explanation of A
b. Both A and R are true but R is NOT
the correct explanation of A
c. A is true but R is false
d. A is false but R is true
Assertion (A): Apart from large-scale
climatic aberrations in the Tropics, El
Nino has a disastrous consequence on the
fishing industry of Peru.
Reason (R): El Nino leads to massive
displacements of the rainfall bringing
droughts over large pails of the normal
raining areas and torrential rains to arid
regions of the Tropics.

51.

52.

53.

54.

7 of 16
a. Both A arid R are true and R is the
coned explanation of A
b. Both A and R are true but R is NOT
the correct explanation of A
c. A is true but R is false
d. A is false but R is true
Assertion (-A): The Tundra environment is
characterized by a few important climatic
and environmental elements.
Reasons (R): The Tundra biomes are
found in Arctic or Alpine regions.
a. Both A arid R are true and R is the
coned explanation of A
b. Both A and R are true but R is NOT
the correct explanation of A
c. A is true but R is false
d. A is false but R is true
Assertion (A): Out-migration results in
increase in employment opportunity and
wage rates but a loss in investment.
Reason (R) Generally young people
migrate and out-migration areas have
fewer numbers of young adults relative to
the number of children and aged people.
a. Both A arid R are true and R is the
coned explanation of A
b. Both A and R are true but R is NOT
the correct explanation of A
c. A is true but R is false
d. A is false but R is true
Assertion (A): Denmark supplies dairy
products and bacon to neighbouring
markets in northern Europe.
Reason (R): Denmark produces high
quality grain and grass and its neighbours
are industrialized.
a. Both A arid R are true and R is the
coned explanation of A
b. Both A and R are true but R is NOT
the correct explanation of A
c. A is true but R is false
d. A is false but R is true
Assertion (A): The oldest urban
settlements developed around 8000 BC.
Reason (R): Human settlements developed
with the domestication of plants and
animals.
a. Both A arid R are true and R is the
coned explanation of A

55.

56.

57.

58.

b. Both A and R are true but R is NOT


the correct explanation of A
c. A is true but R is false
d. A is false but R is true
Assertion (A): In developed countries, the
economically
active population in
agriculture sector has declined.
Reason (R): The per capita arable land has
also significantly fallen down.
a. Both A arid R are true and R is the
coned explanation of A
b. Both A and R are true but R is NOT
the correct explanation of A
c. A is true but R is false
d. A is false but R is true
Assertion (A): Butter wool and frozen
meat are the most important exports of
New Zealand.
Reason (R): New Zealand has cool moist
climate and vast grasslands to support
livestock.
a. Both A arid R are true and R is the
coned explanation of A
b. Both A and R are true but R is NOT
the correct explanation of A
c. A is true but R is false
d. A is false but R is true
Assertion (A): The habitat of Date Palms
is warm deserts with little or no rainfall.
Reason (R): Data Palm is a tree of
waterless desert.
a. Both A arid R are true and R is the
coned explanation of A
b. Both A and R are true but R is NOT
the correct explanation of A
c. A is true but R is false
d. A is false but R is true
Assertion (A): Increase in agricultural
production in China has been due to higher
per hectare yields.
Reason (R): China
accomplished
socialist transformation in alt branches of
economy, including agriculture.
a. Both A arid R are true and R is the
coned explanation of A
b. Both A and R are true but R is NOT
the correct explanation of A
c. A is true but R is false
d. A is false but R is true

59.

60.

61.

62.

8 of 16
Assertion (A): Sri Lanka produces more
tea in central and eastern regions.
Reason (R): The seasonally occurring
heavy rainfall is responsible for much
higher production.
a. Both A arid R are true and R is the
coned explanation of A
b. Both A and R are true but R is NOT
the correct explanation of A
c. A is true but R is false
d. A is false but R is true
Assertion (A) : Peninsular India is a
region free from tectonic disturbances.
Reason (R): The configuration and
drainage of F India are influenced by
fracturing and lilting of the massif.
a. Both A arid R are true and R is the
coned explanation of A
b. Both A and R are true but R is NOT
the correct explanation of A
c. A is true but R is false
d. A is false but R is true
Assertion (A): In winters, the northern half
of India is warmer than areas at similar
latitudes outside the country by 3C to
8C.
Reason (R): The presence of the
Himalayas with their east and west extent
is the cause.
a. Both A arid R are true and R is the
coned explanation of A
b. Both A and R are true but R is NOT
the correct explanation of A
c. A is true but R is false
d. A is false but R is true
Assertion (A): Laterite, the typical soil of
tropical region, is rich in oxides of iron
and aluminium compounds.
Reason (R): Heavy rainfall promotes
teaching of lime and silica content of the
soil.
a. Both A arid R are true and R is the
coned explanation of A
b. Both A and R are true but R is NOT
the correct explanation of A
c. A is true but R is false
d. A is false but R is true

63.

64.

65.

66.

Assertion (A): Despite deforestation, in the


existing forest areas of the Chhattisgarh
Region, Sat free is predominant species.
Reason (R): Sat tree is more aggressive,
resistant to burning and has better
longevity.
a. Both A arid R are true and R is the
coned explanation of A
b. Both A and R are true but R is NOT
the correct explanation of A
c. A is true but R is false
d. A is false but R is true
Assertion (A): Sugar industry has a high
concentration in Uttar Pradesh State of
India.
Reason (R): The sugarcane produced in
Uttar Pradesh is of low quality
a. Both A arid R are true and R is the
coned explanation of A
b. Both A and R are true but R is NOT
the correct explanation of A
c. A is true but R is false
d. A is false but R is true
Assertion (A) :
The
concept
of
Lebensraum was misused by Karl
Hansofer.
Reason (R): It was misused for the
furtherance of German expansion under
Nazi.
a. Both A arid R are true and R is the
coned explanation of A
b. Both A and R are true but R is NOT
the correct explanation of A
c. A is true but R is false
d. A is false but R is true
Assertion
(A):
Despite
marked
development and large volume of output,
Indian geography is yet to make a
significant impact on the international
scene.
Reason (R): Indian geographers accept the
themes and teachings of Western
geographers without criticism.
a. Both A arid R are true and R is the
coned explanation of A
b. Both A and R are true but R is NOT
the correct explanation of A
c. A is true but R is false
d. A is false but R is true

67.

68.

69.

70.

9 of 16
Assertion (A): In comparative scales two
plain scales of different units with the
same RE are made one over the other.
Reason (R): The length of both the
scales may not necessarily be the same.
a. Both A arid R are true and R is the
coned explanation of A
b. Both A and R are true but R is NOT
the correct explanation of A
c. A is true but R is false
d. A is false but R is true
Consider the following statements with
respect to Quantitative Revolution
1. It focused on areal differentiation
2. It reduced geography to space
geometry
3. It followed the methodology of spatial
science
4. It helped in the development of
methods of spatial structure
Which of these are correct?
a. 1, 2 and 3
b. 2, 3 and 4
c. 1, 3 and 4
d. 1, 2 and 4
Consider the following statements
associated with representation of scale by
representative fraction (RF)
1. In RE the units of the numerator and
the denominator are taken as the same
2. The units are never mentioned when
the scale is expressed in the form of
RF
3. The denominator is always 100
4. The numerator should always be 1
Which of these are correct?
a. 1, 2 and 4
b. 1 and 4
c. 2 and 3
d. 3 and 4
Consider the following statements
1. Spatial analysis is closely associated
with positivism
2. The behaviouralists concentrated on
the issues of inequality
3. Humanistic geography attempts at
understanding meanings and value of
life events

71.

72.

73.

Which of these are correct?


a. l and 2
b. 1 and 3
c. 2 and 3
d. 1, 2 and 3
In which of the following projections,
scale is correct along 45 latitude?
a. Equal area cylindrical projection
b. Galls projection
c. Mercatprs projection
d. Mollweides projection
Match List I with List II and select the
correct answer:
List I (Projections)
A. Bonns
B. Cylindrical Equal Area
C. Conical with one standard parallel
D. Conical with two standard paralle
List I (Their Suitability)
1. Trans- Siberian Railway
2. USA and Canada boundary
3. Continent of Europe
4. Rubber and spices production in the
world
A
B
C
D
a.
4
3
1
2
b.
4
3
2
1
c.
3
4
1
2
d.
3
4
2
1
Match List I with List II and select the
correct answer:
List I (features)
A. The unit of mapping is areal
B. The linear data are expressed
C. Make use of colours to distinguish
different areas
D. Make use of lines of equal value to
show distribution
List-II (Types of Maps)
1. Flow map
2. Isopleth
3. Choropleth
4. Chromo-chromatic
A
B
C
D
a.
3
1
4
2
b.
2
4
1
3

74.

75.

76.

77.

10 of 16
c.
3
4
1
2
d.
2
1
4
3
Consider the following statements
Colour - composite imagery based on
satellite data is mostly seen in un-natural
colour scheme because
1. blue wavelengths of solar radiation are
strongly scattered
2. red wavelengths of solar radiation are
marl absorbed
3. it contains an invisible wavelength data
which must also be assigned a specific
colour filter
Which of these is/are correct?
a. only 1
b. 1 and 2
c. only 3
d. 1, 2 and 3
Consider the following statements about
Geographical Information Systems (CIS)
1. It handles attribute data on a technical
base
2. It can draw maps and other graphics
with great ease and speed
3. Is a slow and tedious task involving
creation of GIS data base
4. I work only in combination with
remote sensing data.
5. It can analyses data for specified task
and thus help decision making
Which of these are correct?
a. 1, 2 and 4
b. 2, 3 and 4
c. 3, 4 and 5
d. 1, 2, 3 and 5
If LANDSAT data (or an area is received
today then the data for an adjacent area to
its west will be available
a. in the next orbit of the satellite same
day
b. next day at same local time
c. after certain number of days at same
local time
d. after a variable controllable time
A straight road across a series of hills and
valleys appears as a marked zigzag road
near the margin of a vertical aerial
photograph because

11 of 16
D. Trellis
Identify the drainage patterns marked as 1,
2, 3 and 4 in the given below diagram and
select the correct answer using the codes
given below the diagram:

a. here is some tilt in the photograph


b. geometric fidelity is poor in the
margins in the photo graph
c. there is variable movement of images
due to camera (or aircraft) being in
motion during camera exposure
d. there is variable parallax of point in the
flight direction due to variable heights
of points
78.

79.

80.

The diagram given above represents a


a. normal fault
b. reverse fault
c. hinge fault
d. transform fault
Consider the following river basins
A. Chang Jiang
B. Danube
C. Niger
D. Amur
Identify the river basins marked as 1, 2, 3
and 4 in the given diagram and select the
correct answer using the codes given
below:

A
B
C
D
a.
4
1
2
3
b.
3
2
1
4
c.
3
1
2
4
d.
4
2
1
3
Consider the following drainage patterns:
A. Rectangular
B. Radial
C. Annular

a.
b.
c.
d.

A
1
2
1
2

B
3
4
4
3

C
4
3
3
4

D
2
1
2
1

81.

In the above diagram, A represents


a. arete
b. bergschrund
c. cirque
d. roches moutonnees
82.

83.

In the above diagram, wind shown is an


example of
a. anabatic wind
b. adiabatic wind
c. katabatic wind
d. stroph
Which one of the following illustrations
correctly
represents
an
average
temperature
cyclone
in
northern
hemisphere?

a.

87.

b.

c.

84.

85.

86.

d.
Where CF stands for COW FRONT, WF
srtands for WARM FRONT and WS
stands for WARM SECTOR
The arrangement of planets with smallest
on either ends and big planets in the
middle, supports which one of the
following theories of origin of the solar
system
a. Big Bang theory
b. Tidal hypothesis
c. Binary Star theory
d. Cepheid theory
The most recent geological epoch of the
Quaternary Period covering the 10000
years or so from the end of the Pleistocene
epoch to the present day is known as
a. Pilocene
b. Miocene
c. Oligocene
d. Holocene
Match List I with List Ii and select the
correct answer
List I (Planets)
A. Mars
B. Jupiter

88.

89.

12 of 16
C. Saturn
D. Uranus
A
B
C
D
a.
4
1
2
3
b.
3
2
1
4
c.
4
2
1
3
d.
3
1
2
4
Match List I with List II and select the
correct answer
List I (Technical Terms)
A. Magma Chamber
B. Epicentre
C. Vent
D. Seismic Focus
List II (Definitions)
1. A point on earths surface where impact
of the earthquake is felt maximum
2. An opening in the earths crust through
which magma flow out
3. A point below the earths surface
earthquake originates
4. A place below the earths surface lava
is formed and acts as a source of
magma outflow
A
B
C
D
a.
3
1
2
4
b.
3
2
1
4
c.
4
2
1
3
d.
4
1
2
3
Consider the following statements:
1. Weathering encompasses a group of
processes by which surface and
subsurface rocks disintegrate
2. Extreme dryness reduces most
weathering to a minimum
3. Physical weathering dominates in brier
and cooler climates
4. In equatorial rain-forest climate, most
rocks weather slowly
Which of these are correct?
a. 1, 2 and 3
b. 1 and 4
c. 2, 3 and 4
d. 1, 2, 3 and 4
A mountainous coast having presence of
large number of narrow, steep-sided,

90.

91.

92.

93.

94.

95.

elongated and inundated coastal valley is


called
a. Ria coast
b. Fiord coast
c. Half coast
d. Dalmatian coast
Most of the weather phenomena takes
place in troposhare only because
a. temperature remains almost constant in
troposphere
b. it contains electrically charged
particles
c. it absorbs harmful ultraviolet radiation
from the sun
d. it contains almost all the water vapour
and most dust particles
The actual moisture content of a sample of
air as a parentage of that contained in the
same volume of saturated air at the same
temperature is called
a. absolute humidity
b. relative hum
c. specific humidity
d. vapour pressure
Which one of the following regions
receives the rainfall thro the year due to
westerly winds?
a. South-West Australia
b. Iran and Iraq
c. South Africa
d. South Chile
Which one of the following combinations
is responsible for higher salinity in ocean
water
a. High evaporation, high temperature
and high rainfall
b. High evaporation, high temperature
and low rainfall
c. Low evaporation, low temperature and
high rainfall
d. Low evaporation, low temperature and
low rainfall
Which one of the following is NOT a
factor modifying ocean currents?
a. Direction and shape of the coast-line
b. Tidal wave in the ocean
c. Seasonal variations n winds
d. Bottom topography
Consider the following statements

96.

97.

98.

99.

13 of 16
The horizontal distribution of temperature
of ocean water is largely affected by
1. depth of water in the ocean
2. ocean currents
3. prevailing winds
4. latitude
Which of these are correct?
a. 1, 2 and 3
b. 2, 3 and 4
c. 1, 3 and 4
d. 1, 2 and 4
Consider the following statements
1. Hydrogenous deposits are formed very
slowly
2. Phosphoresces are found either in the
form of nodules or in the form of thin
crust
3. Glauconite is a biogenous sediment
Which of these are correct?
a. 1 and 2
b. 2 and 3
c. 1 and 3
d. 1, 2 and 3
Which one of the following is the correct
order of current system in the South
Atlantic Ocean
a. South equatorial current Brazil
currentAntarctic drift Benguela
current
b. Beguela current Brazil current
South equatorial current Antarctic
drift
c. Antarctic drift Brazil current
Benguela current South equatorial
current
d. South equatorial current Benguela
current Brazil cogent Antarctic
drift
Oceanic waves caused by art earthquake
are known as
a. S-waves
b. L-waves
c. P-waves
d. Tsunamis
The tallest trees of the world are found in
the
a. Equatorial biome
b. Tropical-deciduous biome
c. Temperate rain-forest biome

100.

101.

102.

103.

d. Mediterranean biome
Match List I with List II and select the
correct answer
List I (Concepts)
A. Human ecology
B. Neo-determinism
C. Cultural determinism
D. Environmental determinism
List I (Geographers)
1. Griffith Taylor
2. Harlan Borrows
3. Carl flitter
4. Eduard Ullman
A
B
C
D
a.
1
2
4
3
b.
2
1
4
3
c.
2
1
3
4
d.
1
2
3
4
Consider the following
1. The Geographical Pivot of History
2. The Normal Cycle of Erosion
3. Revensteins Laws of Migration
4. RatzeI Anthropogeographic
Their correct chronological sequence is
a. 2, 3, 1, 4
b. 3, 1, 2, 4
c. 4, 2, 3, 1
d. 8, 4, 2, 1
The Semitic people belong to the
a. Caucasoid race
b. Mongoloid race
c. Negroid race
d. Austroloid race
Match List I with List II and select the
correct answer:
List I (Tribes)
A. Babinga
B. Coola
C. Oromo
D. Tswa
List-II (Countries)
1. Canada
2. Angola
3. Ethipia
4. Zaire
A
B
C
D
a.
3
1
4
2
b.
3
2
4
1

104.

105.

106.

107.

14 of 16
c.
4
2
2
1
d.
4
1
3
2
Consider the following statements
1. Nomadism is a high degree of spatial
mobility as a basis for a particular way
of life
2. Nomadism is confined to the semi-and
regions of the
3. Nomadism is presumed to be a
synonymous with movements of herds
and livestock rearing
4. Hunter gatherers who typically have no
domesticated livestock ate also defined
as nomads
Which of these are correct?
a. 1, 2 and 3
b. 2, 3 and 4
c. 1, 3 and 4
d. 1, 2 and 4
Consider the of following activities
1. Fishing and forestry
2. Pastoral hearing
3. Milk and meat diet
Which of these are common among the
Kurds and Kirghiz?
a. 1 and 2
b. 2 and 3
c. 1 and 3
d. 1, 2 and 3
Consider the following relief features
1. Zankar Range
2. Dhauladhar Range
3. Laddakh Range
4. Karakoram Range
The correct northward sequence of these
relief features is
a. 2, 1, 3, 4
b. 2, 4, 3, 1
c. 4, 3, 2, 1
d. 4, 2, 1, 3
Match List I with List II and select the con
answer
List I (Regions)
A. Humid Tropics
B. Hot Desert
C. Monsoon Asia
D. Temperate Grassland
List II (Activities)
1. Nomadic herding

108.

109.

110.

2. Shifting cultivation
3. Commercial livestock fanning
4. Intensive substance agriculture
A
B
C
D
a.
1
2
3
4
b.
2
1
3
4
c.
2
1
4
3
d.
1
2
4
3
Match List I with List II and select the
correct answer:
List I (Terms)
A. Podzole
B. Terra-rossa
C. Chemozem
D. Chestnut
List II (Explanations)
1. Black earth soil in Russia
2. Ash oil of Siberia
3. A brown pedocal soil of Great Plains
of USA
4. Mediterranean reddish clay loam soil
A
B
C
D
a.
1
2
3
4
b.
2
1
3
4
c.
2
1
4
3
d.
1
2
4
3
Match List I with List II and select the
correct answer
List I (Economic Base)
A. Break basket
B. Big game country
C. Mixed farming
D. Maximum biodiversity
List II (Regions)
1. Savanna
2. Steppes
3. Prairies
4. Monsoon lands
A
B
C
D
a.
3
2
1
4
b.
3
1
2
4
c.
4
1
2
3
d.
4
2
1
3
Consider the following statements:
Tropics crops, which are in great demand
in Anglo-America and Europe, are
produced in great variety and volume in
Latin America and Africa because of

111.

112.

113.

15 of 16
1. availability of a cultivable land and
suitability of soil conditions
2. combination
of
climatic
and
topographical conditions
3. lack of mineral resources for trading
purpose
4. non-existence of agro-based industries
Which of these are correct?
a. 1 and 2
b. 2 and 3
c. 1, 3 and 4
d. 2, 3 and 4
Match List I with List I and select the
correct answer
List I (Country)
A. Germany
B. Japan
C. Canada
D. USA
List II (Leading Producer of)
1. Sugarbeet
2. Newsprint
3. Fish
4. Cotton
A
B
C
D
a.
1
3
2
4
b.
2
4
1
3
c.
2
3
1
4
d.
1
4
2
3
Hawaii Islands are known for active
volcanoes because
a. they are locked in the subduction zone
of covering plates
b. faults and fractures are found there
c. they are situated over a hot plume
d. they are situated on a mild oceanic
ridge with rising convective currents
Match List I with List 1I and select the
correct answer:
List I (Places in USA)
A. Butte Hills
B. Chicago
C. Wisconsin and Minnesota
D. Pittsburgh
List II (Known for/as)
1. Biggest railway junction
2. Famous copper mine
3. Coal and steel city

114.

4. Core of dairy-belt
A
B
C
D
a.
2
1
3
4
b.
1
2
4
3
c.
1
2
3
4
d.
2
1
4
3
Match List I with List II and select the
correct answer:
List I (Minerals)
A. Iron ore
B. Coal
C. Petroleum
D. Copper
List II (Major Areas in USA)
1.
2.
3.
4.
5.

115.

116.

117.

Arizona region
Lake Superior region
Gulf Coastal region
Pennsylvanian region
Lake Michigan region
A
B
C
D
a.
2
4
3
1
b.
1
3
5
2
c.
1
4
3
2
d.
2
3
5
1
The American cotton textile industry
moved from New England to the Southern
States because
a. the latter have a more humid climate
b. New
England
diversified
its
manufacturing industries
c. the population growth in the south
provided a major market
d. the south offered lower cost of
production
Consider the following countries
1. Belgium
2. Italy
3. Netherlands
4. United Kingdom
Their coned sequence, in descending order
in terms of population density, is
a. 3, 1, 2, 4
b. 2, 4, 1, 3
c. 3, 1, 4, 2
d. 4, 2, 3, 1
Consider the following statements

118.

119.

120.

16 of 16
The contrast between areas north and
south of the 37th parallel in Japan is
largely on account of differences in
1. climate
2. settlement process
3. topography
4. Industrial activity
Which of these are correct?
a. 1, 2 and 3
b. 2, 3 and 4
c. 1 and 2
d. 3 and 4
Consider the following cities on the
eastern coasts of Australia
1. Brisbane
2. Cairns
3. Melbourne
4. Sydney
The correct sequence of these cities, from
north to south, is
a. 1, 3, 2, 4
b. 2, 1, 4, 3
c. 3, 2, 1, 4
d. 4, 2, 3, 1
Consider the following countries of South
Africa
1. Botswana
2. Lesotho
3. Zambia
4. Zimbabwe
The correct sequence of these countries, in
terms of their proximity to the equator, is
a. 3, 4, 2, 1
b. 4, 2, 1, 3
c. 3, 4, 1, 2
d. 4, 1, 3, 2
Earthquakes along the Himalayas and the
foothills are best explained in terms of
a. plate tectonics
b. young age of the Himalayas
c. fragile and largely unconsolidated
rocks
d. divergence of plates

C.S.E-Pre 2003

1 of 13

GEOGRAPHY
1.

2.

3.

4.

Match list I with List I and select the


correct answer:
List I (Greek Cities)
A. Carthage
B. Sardis
C. Susa
D. Syrene
List II (Label shown in the Map)

A
B
C
D
a.
2
4
3
1
b.
1
3
4
2
c.
2
3
4
1
d.
1
4
3
2
Select the cone sequence in the
chronological order of the following
Greeks who have contributed vastly to the
evaluation of geographical thought during
the ancient period
a. Aristotle - Eratosthense - Anaximander
- Ptolemy
b. Eratosthenes -Anaximander - Ptolemy
- Aristotle
c. Anaximander Anstole-EratosthenesPtolemy
d. Ptolemy -Anaximander AristotleEratoshenes
From amongst the following Arab
scholars, who is best known for his
extensive travels and written account of
geographical importance?
a. Ibn-Haukal
b. Al-Masudi
c. Al-Biruni
d. lbn-Batuta
The principal contribution of Varenius was
to
synthesis
the
approaches
of
geographical studies as
1. Special Geography
2. Physical Geography
3. General Geography
4. Human Geography

5.

6.

7.

8.

Select the correct answer using the codes


given be
a. 1, 2 and 3
b. 1 and 4
c. 3 and 4
d. 1 and 3
Match List I with list I and select the
correct answer
List I (Ancient Period Contributors)
A. Herodotus
B. Ptolemy
C. Al-Biruni
D. Hippanchut
List II (Main Thrust of Geographical
Studies)
1. The art of map making
2. History must be treated geographically
and all geog must be treated
historically
3. Mathematical, physical and regional
geography
4. Suggested use of a grid of latitude and
longitude as a basis for locating places
on Earth
A
B
C
D
a.
2
1
3
4
b.
1
4
2
3
c.
2
1
4
3
d.
1
4
3
2
The idea for digging a canal across
Panama Isthmus was put forward by
a. Ptolemy
b. Strabo
c. Ratzel
d. Numboldt
Environment presents a range of
opportunities and man s free to choose
between them. This basic pi in geography
is known as
a. Determinism
b. Neo-determinism
c. Possibilism
d. Probabilism
Consider the following statements:
1. Ratzel dominated German Geography
in the second half of the 19th Century.
2. Ratzel coined the term Anthropology.

2 of 13

9.

3. Ratzel was highly influenced by


Darwin theory of evolution of species.
Which of these are correctly associated
with Ratzel?
a. 1, 2 and 3
b. 1 and 2
c. 2 and 3
d. 1 and 3
Match list I with list II and select the
correct answer
List I (Projections)
A. Sinusoidal Projection
B. Mollweids projection
C. Zenithal equal area projection
D. Bonnes projection

2.

3.

4.

11.

10.

A
B
C
D
a.
3
4
2
1
b.
2
1
3
4
c.
3
1
2
4
d.
2
4
3
1
Match List I with List II and select the
correct answer
List I (Curves)
A. Clinographic curve
B. Altimetric frequency histogram
C. Hypsometric curve
D. Columnar diagram of slopes
List II (Diagrams)

1.

12.

A
B
C
D
a.
2
1
4
3
b.
4
1
2
3
c.
2
3
4
1
d.
4
3
2
1
Which one of the following statements is
not correct?
a. LANDSFT is a gee-stationary Satellite
b. IRS a sun-synchronous sate
c. INSAT is a geo-stationary satellite
d. SPOT is a sun-synchronous satellite
Match List I with List II and select the
correct answer:
List I (Maps)
A. Cadastral map
B. Topographical map
C. Wall map
D. Atlas map
List II (Scales)
1. 1 cm : 50 km
2. 1 cm : 52 km
3. 1 cm : 100 km
4. 1 cm : 25 m
A
B
C
D
a.
4
1
2
3
b.
3
2
1
4
c.
4
2
1
3
d.
3
1
2
4

3 of 13
13.

14.

15.

16.

17.

Which of the following depict relief?


1. Contour lines
2. Isochrones
3. Hachures
4. Isabars
Select the correct answer using the codes
given below:
a. 1 and 4
b. l and 2
c. 1 and 3
d. 2 and 4
Each 1: Million series (India and adjacent
countries) topographical sheet contains
sixteen 1:250000 sheets and each
1:250000 sheets contains sixteen 1:50,000
sheets. How many 1:25000 sheets are
contained in each 1:50.000 sheet?
a. 2
b. 4
c. 8
d. 16
The radical thinking in geography is
focussed on
a. Attempt to engage the discipline in
socially relevant issues
b. Only surface manifestations of societal
irregularities
c. The premise that progress is
independent of the material conditions
d. The mental images that the decisionmakers have of the environment
Which one of the following is a key
concept in humanistic geography?
a. Development and quality of life
b. Spatial location
c. Spatial concentration
d. Geographical distribution
Which one of the following statements is
not correct?
a. IKONOS satellite image has a 1 m
resolution
b. IRS satellite image has a 5.8 m
resolution
c. INSAT satellite image has a 10 m
resolution
d. SPOT satellite image has a 10 m
resolution

DIRECTIONS: The following 13 (Thirteen)


items consist of two statements one labelled as
Assertion A and the other labelled as Reason
R. You are to examine these two statements
carefully and select the answers to these items
using the codes given below:

18.

19.

20.

21.

Assertion (A): The condition of forest


cover is very deplorable in states like Ha
Punjab, Rajasthan and Gujarat.
Reason (R): There are areas where
intensification of afforestation-programme
is urgently needed
a. Both A and R are individually true and
R is the correct explanation of A
b. Both A and R are individually true but
R is not the correct explanation of A
c. A is true but R is false
d. A is false but R is true
Assertion (A): Great Britain and New
Zealand are remarkably similar in terms of
area. oceanic location and climate yet
Great Britain is densely populated and an
important centre of political and economic
power.
Reason (R): New Zealand unlike Great
Britain suffers from the disadvantage of
the location in the vast expanse of ocean in
the southern hemisphere.
a. Both A and R are individually true and
R is the correct explanation of A
b. Both A and R are individually true but
R is not the correct explanation of A
c. A is true but R is false
d. A is false but R is true
Assertion (A): Major industrial regions in
India developed in the immediate
hinterlands of the ports of Kolkata.
Mumbai and Chennai.
Reason (R): The ports provide access to
the raw materials available in the
hinterlands of these ports as well as to the
world market.
a. Both A and R are individually true and
R is the correct explanation of A
b. Both A and R are individually true but
R is not the correct explanation of A
c. A is true but R is false
d. A is false but R is true
Assertion (A): Agriculture in Monsoon
Asia is characterised by the predominance
of food crop cultivation.
Reason (R): Farming in Monsoon Asia is
mainly meant for subsistence purpose.
a. Both A and R are individually true and
R is the correct explanation of A
b. Both A and R are individually true but
R is not the correct explanation of A
c. A is true but R is false
d. A is false but R is true

22.

23.

24.

25.

26.

Assertion (A): The North Atlantic Ocean


Route is the worlds most important ocean
route.
Reason (R): The North Atlantic Ocean
Route brings together the developing and
developed nations.
a. Both A and R are individually true and
R is the correct explanation of A
b. Both A and R are individually true but
R is not the correct explanation of A
c. A is true but R is false
d. A is false but R is true
Assertion (A): Cyclone is developed in a
region of extremely low pressure of air
with steep gradient of pressure.
Reason (R): It has outward spiralling air at
low elevations and inward air flowing at
the high elevations.
a. Both A and R are individually true and
R is the correct explanation of A
b. Both A and R are individually true but
R is not the correct explanation of A
c. A is true but R is false
d. A is false but R is true
Assertion (A): In the deep ocean, primary
production occurs near hydrothermal
vents.
Reason (R): Over there, microbes produce
food through chemosynthesis which forms
the basis for a food-chain.
a. Both A and R are individually true and
R is the correct explanation of A
b. Both A and R are individually true but
R is not the correct explanation of A
c. A is true but R is false
d. A is false but R is true
Assertion (A): The tarai soils in India are
rich in nitrogen and organic matter but are
deficient in phosphate.
Reason (R): These soils are generally
covered by tall grass and forests.
a. Both A and R are individually true and
R is the correct explanation of A
b. Both A and R are individually true but
R is not the correct explanation of A
c. A is true but R is false
d. A is false but R is true
Assertion (A): Graben is a major relief
feature resulting from the faulting
activities. It is infact a valley or narrow
trough bounded by one or more parallel
faults
Reason (R): Blocks of the Earths crust
may be relatively raised or lowered

27.

28.

29.

30.

4 of 13
between more or less parallel faults. The
lower part known as graben is formed due
to the subsidence of middle part between
two nor faults.
a. Both A and R are individually true and
R is the correct explanation of A
b. Both A and R are individually true but
R is not the correct explanation of A
c. A is true but R is false
d. A is false but R is true
Assertion
(A):
In
contemporary
geography, there is an increasing
awareness of the need for an
interdisciplinary approach to the regional
problems.
Reason (R): Groups of scholars with
diverse backgrounds and skills focus on
specific sets of regional problems more
comprehensively and scientifically.
a. Both A and R are individually true and
R is the correct explanation of A
b. Both A and R are individually true but
R is not the correct explanation of A
c. A is true but R is false
d. A is false but R is true
Assertion (A): The location of the
epicentre of an earthquake may be
estimated using the time-lag between the
arrival of V and S waves.
Reason (R): The L wave cannot start
until the P wave hits the surface.
a. Both A and R are individually true and
R is the correct explanation of A
b. Both A and R are individually true but
R is not the correct explanation of A
c. A is true but R is false
d. A is false but R is true
Assertion (A): Bajadas or Piedmont
alluvial plains extend for several miles
away from a mountain front.
Reason (R): A series of adjacent alluvial
fans sometimes coalesce to form an
extensive piedmont alluvial plain which is
also called Bajadas.
a. Both A and R are individually true and
R is the correct explanation of A
b. Both A and R are individually true but
R is not the correct explanation of A
c. A is true but R is false
d. A is false but R is true
Assertion (A): Vernier scale enables to
estimate the fraction of a division in both
linear and angular measurements, with
greater accuracy.

31.

Reason (R): It consists of a moving scale


which s along a primary scale.
a. Both A and R are individually true and
R is the correct explanation of A
b. Both A and R are individually true but
R is not the correct explanation of A
c. A is true but R is false
d. A is false but R is true
The shaded area in the given diagram
represents

36.

37.

38.

32.

33.

34.

35.

a. Namib desert
b. Inter tropical convergence zone
c. Sahel region
d. Permafrost region
Consider the following Statements:
1. Nearly 30% of the petroleum
production in the world is from the
offshore areas.
2. The exclusive economic zone in the
oceans extends up to 100 nautical
miles from the coast
3. Only less than 1% of marine area is
declared as protected area.
4. The largest marine protected area is the
Great Barrier Reef.
Which of these statements are correct?
a. 1 and 2
b. 1, 3 and 4
c. 1 and 4
d. 2 and 3
Eutrophication of lakes a consequence
effect of the accumulation of
a. Silt load
b. Sewage
c. Nitrates and Phosphates
d. Vegetation
The spread of cultural elements or
complexes from one society to another is
called
a. Diffusion
b. Acculturation
c. Assimilation
d. Transmission
Who said Similar locations lead to similar
mode of life?
a. F.Ratzel

39.

40.

41.

5 of 13
b. Carl Ritter
c. Alexander Von Humboldt
d. E. C. Semple
The Red Indians or the America Indians
belong to the
a. Mongolian race
b. Caucasoid race
c. Australoid race
d. Negrito race
Which one of the following is correctly
matched?
Tribes Native: State
a. Tharu:
Uttaranchal
b. Bhotia:
Uttar Pradesh
c. Munda: Bihar
d. Kol:
Rajasthan
Celite, Germanic and Ural-Altaic language
groups migrated to west because
a. the grasslands of Central Asia dried up
b. the expanding Chinese Empire
disrupted the life of nomadic groups
c. they were pushed westwards when the
easternmost groups moved west
d. these were forced migration which
resulted from the slave trade
In Lees opinion, which of the following
actors are responsible for migration?
1. Factors operating in the area of origin
2. Factors operating at the destination
3. Factors that act as intervening
obstacles
4. Personal factors that are specific to
individuals
Select the correct answer using the codes
given below
a. 1, 2 and 3
b. 1, 2 and 4
c. 3 and 4
d. 1, 2, 3 and 4
Which one of the following states was
formed exclusively by the migrants in the
20th century?
a. Maldives
b. Mauritius
c. Israel
d. Myanmar
The Human Development Index (HDI) has
shown a positive trend all over the world
except
a. Sub-Saharan Africa
b. East Asia and the Pacific
c. Latin America and the Caribbean
d. Central and Eastern Europe and the
CIS countries

42.

43.

44.

45.

46.

47.

48.

Which one of the following does not


represent the prim gathering in the high
latitudes
a. Yukaghirs
b. Yahgans
c. Eskimos
d. Auca Indians
Which one of the following countries is
the largest producer of marine fish?
a. China
b. Russia
c. Chile
d. Japan
Which one of the following regions is
practicing most intensive subsistence
farming?
a. Pampas Region
b. Murray - Dar Basin
c. California Valley
d. Monsoor Asia
Which one of the following stages
indicates the initiation of population
explosion in the demographic transition?
a. Stage I
b. Stage I
c. Stage III
d. Stage IV
Which one of the following is the most
important basis of the Whittleseys
classification of agriculture of the world?
a. Functioning forms of agriculture
b. Distribution of climatic elements
c. Forms of livestock grazing activities
d. Regional distribution of the principal
crops and animals of commercial
significance
The south-western part of California is
known for
a. Subsistence agriculture
b. Forestry
c. Temperate grain farming
d. Mediterranean agriculture
Match List I with List II and select the
correct answer
List I (Minerals)
A. Coal
B. Petroleum
C. Manganese
D. Mica
List II (Chief Producing States)
1. Karnataka, Madhya Pradesh
2. Bihar, Andhra Pradesh
3. Bihar, West Bengal
4. Maharashtra, Karnataka

49.

50.

51.

52.

53.

6 of 13
5. Assam, Gujarat
A
B
C
D
a.
3
5
1
2
b.
2
1
4
3
c.
3
1
4
2
d.
2
5
1
3
The highest hydro-power potential in the
African Continent is in
a. North-western Africa
b. North-eastern Africa
c. Equatorial Africa
d. Southern Africa
Weber introduced a famous locational
theory for localization of manufacturing
industries, known as
a. Hexagon
b. Circle
c. Triangle
d. Rectangle
The shortest route from Los Angeles to
Moscow is over the
a. Atlantic Ocean
b. Pacific Ocean
c. North Po
d. South Pole
Consider the following statements:
1. The shelter of Bedonins is known as
Khaimas.
2. The shelter of Kirghiz is called as
Kraals.
3. The cattle shade of the Bakarwals of
Jammu and Kashmir is known as
Kothas.
4. The Innuits shelters are knows as
igloos.
Which of these statements are correct?
a. 1, 2 and 3
b. 2, 3, and 4
c. 1 and 3
d. 1 and 4
Match List I with list II and select the
correct answer
List I (Settlements/Hamlets)
A. Nucleated settlements
B. T-shaped settlements
C. Fan shaped settlements
D. Dispersed hamlets
List II (Description)
1. Remote and inaccessible are as
2. Rich agricultural plain areas
3. Junction of village road with main
highway
4. Converging at religious centre
A
B
C
D

54.

55.

56.

a.
2
3
4
1
b.
4
1
2
3
c.
2
1
4
3
d.
4
3
2
1
Match List I with list II and select the
correct answer
List I (Minerals)
A. Bauxite
B. Coal
C. Iron Ore
D. Petroleum
List II (Places)
1. Alabama
2. Baku
3. Kuzbas
4. Surinam
A
B
C
D
a.
1
3
4
2
b.
4
3
1
2
c.
1
2
4
3
d.
4
2
1
3
In which one of the following mountain
range of South Africa, are the worlds
greatest goldfields located?
a. Roggeveldberg
b. Groot Swartberg
c. Witwatersrand
d. Drakensberg
Match List I with list II and select the
correct answer
List I (Natural Region)
A. Equatorial
B. Dry summer sub-tropical
C. Interior lowlands (warm)
D. High plateau type in the tropical belt

58.

59.

60.

61.

57.

List II (Location)
1. Around the Mediterranean Sea
2. Amazon and Congo basin
3. Columbia in South America Plateaus
of Ecuador and
4. Plateaus of Tibet and Bolivia
5. From Caspian eastwards towards the
Pamirs
A
B
C
D
a.
2
5
4
3
b.
3
1
5
2
c.
2
1
5
3
d.
3
5
4
2
The fovourable percentage of sea water
salinity from the point of view of fish
production is
a. 2.0 to 2.5
b. 2.5 to 3.0

7 of 13
c. 3.0 lo 3.5
d. 3.5 to 4.0
The sequence of the Great Lakes of U.S.A.
from east to west is
a. Huron - Michigan - Ontario - Superior
- Erie
b. Super - Ontario - Erie - Michigan Huron
c. Ontario - Erie- Huron - Michigan Superior
d. Erie - Huron - Michigan - Ontario Superior
The Canadian Pacific Railway runs
between
a. Edmonton and Halifax
b. Montreal and Vancouver
c. Ottawa and Prince Rupert
d. Halifax and Vancouver
Match List I with List II and select the
correct answer:
List I (Towns)
A. Louisiana
B. Detroit
C. Houston
D. Minneapolis
List II (Industrial/Business Activity)
1. Automobile Industry
2. Flour milling
3. Refining and chemical industry
4. Oil and gas
A
B
C
D
a.
3
2
4
1
b.
4
1
3
2
c.
3
1
4
2
d.
4
2
3
1
Match List I with List II and select the
correct answer:
List I (Characteristic Features)
A. Largest in population size
B. High population density
C. Largest in area
D. Highest urban population
List II (Countries)
1. Belgium
2. France
3. Germany
4. Greece
5. Nether
A
B
C
D
a.
2
1
4
5
b.
3
5
2
1
c.
2
5
4
1
d.
3
1
2
5

62.

63.

64.

65.

Match List I with List II and select the


correct answer:
List I (Towns)
A. Genoa
B. Florence
C. Sicily
D. Naples
List II (Features)
1. Museum town of Italy
2. Leading producer of sulphur
3. Citrus fruit production
4. Leading port of Italy
A
B
C
D
a.
4
3
2
1
b.
2
1
4
3
c.
4
1
2
3
d.
2
3
4
1
Match List I with List II and select the
correct answer:
List I (Russian region)
A. Central Industrial
B. Groznyy
C. Votga
D. Urals
List II (Industrial Activity)
1. Agro-based
2. Cotton textiles
3. Machinery
4. Metallurgy
5. Petroleum refining
A
B
C
D
a.
2
4
1
3
b.
1
3
5
4
c.
2
3
1
4
d.
1
4
5
3
Lake Baikal is the only fresh water inland
sea discharging its outflow into
a. Amur river
b. Lena river
c. Ob river
d. Yenisei river
Consider the following countries of South
and South-east Asia
1. Bangladesh
2. Indonesia
3. Philippines
4. Vietnam
The correct sequence in the decreasing
order of the population site is
a. 2, 1, 4, 3
b. 2, 4, 1, 3
c. 1, 2, 3, 4
d. 4, 2, 3, 1

66.

67.

68.

69.

70.

71.

72.

8 of 13
The lowest rate of population growth
among the following Indian States during
1991-2001 was registered in
a. Madhya Pradesh
b. Sikkimm
c. Arunachal Pradesh
d. Kerala
Which of the following coasts generally
remains dry during the months of July and
August?
a. Coroniandel Coast
b. Konkan Coast
c. Malabar Coast
d. Northern Circars
Which one of the following correctly
states the location of Cold desert of India?
a. To the north-east of Karakoram Range
b. To the w of Pir Panjal Range
c. To the south of Siwalik Range
d. To the west of Aravali Range
Match List I with List II and select the
correct answer:
List I (Name)
A. Bhabar
B. Siachen Glacier
C. Bundelkhand Plateau
D. Rohtang Pass
List II (Location)
1. Karakoram Range
2. Himachal Pradesh
3. Along the foothills of the Siwaliks
4. Between Ganga Plain and Vindhyan
scam land
A
B
C
D
a.
2
4
1
3
b.
3
1
4
2
c.
2
1
4
3
d.
3
4
1
2
The Western Syntaxial bend of the
Himalayas is near
a. Zaskar Range
b. Pir Panjal Range
c. Nanga Parbat
d. Siwalik Hills
The Kathiawar Peninsula is an example of
a. Emerged shorelines
b. Submerged shorelines
c. Ria shorelines
d. Dalmatian shorelines
The Meghalaya Plateau is largely formed
of
a. Cretacecus lava
b. Gondwana rocks
c. Dharwarjarj Quartzites

73.

74.

75.

76.

77.

d. Tertiary sedimentary rocks


Which of the following three rivers of
Peninsular India have the Amarkantak
region as their source?
a. Chambat, Betwa, Luni
b. Godavari, Krishna, Cauvery
c. Son, Mahanadi, Narmada
d. Narmada, Krishna, Wainganga
Which of the following types of soils is
produced by granite and gneiss on
weathering?
a. Red
b. Black
c. Yellow
d. Laterite
Match List I with List II and select the
correct answer:
List I (Type of Vegetation)
A. Mangrove
B. Tropical wet evergreen
C. Tropical wet deciduous
D. Dr Savanna and Steppe
List II (Region )
1. Orissa
2. Western Chats
3. West Bengal
4. Gujarat
A
B
C
D
a.
3
2
1
4
b.
1
4
3
2
c.
3
4
1
2
d.
1
2
3
4
Consider the following regions
1. Guff of Khambhat
2. Gulf of Kachchh
3. Sunderbans
The correct sequence of these regions in
the do order of their tidal energy potential
is
a. 1, 2, 3
b. 3, 2, 1
c. 3, 1, 2
d. 1, 3, 2
Consider the following statements
regarding Kants view on origin of the
Earth:
1. Kant introduced he New aw of
gravitation in his theory.
2. Kant developed his theory accepting
the principle of conservation of angular
momentum.
3. Though Laplace put forward the
nebular hypothesis of origin of the
Earth, Kant is regarded by many as the

78.

79.

9 of 13
real propounder of the nebular
hypothesis.
Which of these statements are correct?
a. 1, 2 and 3
b. 1 and 3
c. 2 and 3
d. 1 and 2
Tertiary period had five epochs. Select the
correct order in which these epochs
appeared in the geological time scale from
ancient to more recent times
a. Palaeocene - Oligocene - Miocene Eocene - Pliocen
b. Palaeocene - Pliocen OligoceneMiocene Eocene
c. Oligocene-Palaeocene MioceneEocene Pliocene
d. PalaeoceneEoceneOligoceneMiocene- Pliocene
Pelagic deposits are mainly found below
the depth of
a. 1000 fathoms
b. 100 fathoms
c. 600 fathoms
d. 200 fathoms

80.

The given fault in the diagram is an


example of
a. Normal fault
b. Overthrust fault
c. Reverse fault
d. Transcurrent fault
81.

82.

83.

84.

85.

86.

87.

In the given diagram, the point marked as


x in B part indicates:
a. Flood plain
b. Natural levee
c. Meander scar
d. Wind gap
Laterite is produced by
a. Hydration and carbonation
b. Hydrolysis and oxidation
c. Leaching and oxidation
d. Hydration and calcification
Which of the following is not a greenhouse
gas?
a. Carbon dioxide
b. Methane
c. Nitrous oxide
d. Argon
The most abundant gas emitted from
volcanoes is
a. Water vapour
b. Helium
c. Sulphur dioxide
d. Carbon dioxide
The blueness of the sky is mainly due to
a. absorption of blue light by the
atmosphere
b. Scattering of sunlight by air molecules
c. Emission of b light by air
d. Presence of water vapour
Warming in the Earths stratosphere is
mainly caused by
a. Release of latent heat energy during
condensation
b. Frictional heating caused by meteorites
c. Chemical reaction between ozone and
chiorofluoro-carbons
d. Absorption of solar radiation by ozone
Match List I with List II and select the
correct answer:
List I (Cloud)
A. Cirrus
B. Startus
C. Nimbus
D. Cumulus
List II (Characteristics)
1. Rain giving
2. Feathery appearance
3. Vertically growing
4. Horizontally spreading
A
B
C
D
a.
2
1
4
3
b.
3
4
1
2
c.
2
4
1
3
d.
3
1
4
2

88.

89.

90.

91.

10 of 13
Match List I with List II and select the
correct answer:
List I (Types of Cloud)
A. Convective cloud
B. Layer cloud
C. Orographic cloud
D. Funnel cloud
List II (Mechanism for Formation)
1. Forced lifting of stable air
2. Adiabatic expansion and cooling due
to rapid local reduction in pressure
3. Local ascent of warm. buoyant air
parcel in a conditionally unstable
environment
4. Forced lifting of air as it passes over
mountains or hills
A
B
C
D
a.
3
2
4
1
b.
4
1
3
2
c.
3
1
4
2
d.
4
2
3
1
A relatively small rotating funnel that
extends down ward from the base of a
towering cloud with thunder, lightning and
strong gusty wind is known as a. Hurricane
b. Tornado
c. Tropical cyclone
d. Extratropical cyclone
In the northern hemisphere, tropical
cyclones and middle latitude cyclone are
similar in that both
a. originate over warm ocean
b. intensify with increasing height above
the ground
c. generally move from east to west
d. have winds that blow counterclockwise around their centres
Match List I with List II and select the
correct answer:
List I (Climatic Conditions)
A. Temperature
B. F- Deserts
C. Equatorial
D. Mediterranean
List II (Tree Species)
1. Oak
2. Acacia
3. Spruce
4. Mahogany
A
B
C
D
a.
4
2
3
1
b.
3
1
4
2
c.
4
1
3
2

92.

93.

94.

95.

96.

d.
3
2
4
1
A submarine mountain rising more than
1000 metres above the ocean floor is
known as
a. Sea mount
b. Abyssal bill
c. Guyots
d. Submarine ridge
The ocean relief is generally
a. flare diverse than hat of the continents
b. more uniform than that of the
continentsd
c. showing major variations only
d. not much of a significance
Consider the following statements
1. Among the major oceans, Indian
Ocean is warmer than the Atlantic and
Pacific
Oceans
in
both
the
hemispheres.
2. The average surface temperature of
Pacific Ocean is higher than that of the
Atlantic Ocean in the southern
hemisphere 3. Among the major oceans, the Atlantic
Ocean is the - coldest in the equatorial
region,
Which of these statements are correct?
a. 1 and 2
b. 2 and 3
c. 1 and 3
d. 1, 2 and 3
Consider the following statements
Salinity of water in the equatorial ocean
decreases because
1. large rivers like Amazon and Congo
enter into the sea.
2. evaporation is less.
3. waters from Arctic and Antarclic mix
with the waters of Equator.
4. of heavy rainfall.
Which of these statements are correct?
a. 1 and 2
b. 3 and 4
c. 1 and 3
d. 1 and 4
Which one of the following sequences in
the
increasing
order
of
salinity
concentration in their waters is correct?
a. Gulf of California - Baltic Sea - Red
Sea - Arctic Sea
b. Baltic Sea Arctic Ocean - Gulf of
California Red Sea
c. Red Sea - Gulf of California - Arctic
Ocean Baltic Sea

97.

98.

99.

100.

101.

102.

103.

11 of 13
d. Arctic Sea - Gulf of California - Baltic
Sea - Red Sea
Oozes are associated with
a. Volcanic deposits
b. Terrigenous deposits
c. Pelagic deposits
d. Coral reefs
Match List I with List II and select the
correct answer:
List I (Deposits)
A. Silt
B. Red day
C. Ooze
D. Gravel
List II (Locations)
1. Continental shelves
2. Oceanic plains
3. Continental slopes
4. Oceanic deeps
A
B
C
D
a.
2
1
3
4
b.
3
4
2
1
c.
2
4
3
1
d.
3
1
2
4
Which of the following is a warm current?
a. South Pacific current
b. Kuroshiwo current
c. Labrador current
d. Rip current
Which of the following conditions are
associated with El Nino phenomena
a. Heavy rains in South America and
droughts in Australia
b. Droughts in South America and heavy
rains in Australia
c. Heavy rains in both South America
and Australia
d. Droughts in both South America and
Australia
Spring tide means
a. Higher high tide, higher low tide
b. Higher high tide, lower low tide
c. Lower high tide, lower low tide
d. Lower high tide, higher low tide
The
important
species
of
the
Mediterranean biome are
a. Mahogany, ebony, rosewood
b. Cork, olive, cirtrus trees
c. Eucalyptus, oak, wattle
d. Pine, cedar, fir
Plants of tropical and sub-tropical regions
are called
a. Megatherms
b. Mesotherms

104.

105.

106.

107.

108.

c. Microtherms
d. Heksotherms
Plants having thorny leaves and
penetrating roots are classified as
a. Xerophytes
b. Hygrophytes
c. Mesophytes
d. Desert plants
Match List I with List II and select the
correct answer:
List I (Regions)
A. Selvas
B. Savannas
C. Monsoon lands
D. Tundras
List II (Characteristics)
1. Tropophytes
2. Mosses and lichens
3. Epiphytes
4. Grasses and trees
A
B
C
D
a.
3
2
1
4
b.
1
4
3
2
c.
3
4
1
2
d.
1
2
3
4
Match List I with List II and select the
correct answer:
List I (Biomes)
A. Coniferous forest
B. Tropical rain forest
C. Tropical deciduous forest
D. Temperate forest
List II (Species)
1. Spruce
2. Rubber
3. Sal
4. Oak
A
B
C
D
a.
3
4
1
2
b.
1
4
3
2
c.
3
2
1
4
d.
1
2
3
4
Select the correct sequence of food chain
in the Arctic waters from the following:
a. Zooplankton - diatoms - fish - polar
bear - seal
b. Diatoms - zooplankton - fish - seal polar bear
c. Fish - diatoms - seal - zooplankton polar bear
d. Diatoms - fish - polar bear - seal
zooplankton
Which of the following is the largest coal
field in India?

109.

110.

111.

112.

113.

114.

12 of 13
a. Jharia
b. Ranigani
c. Bolero
d. Girdih
In which of the following states is the
marine production of fish less than the
inland production
a. Tamil Nadu
b. West Bengal
c. Andhra Pradesh
d. Kerala
In India which of the following crops is
the largest consumer of irrigation waters?
a. Rice
b. Wheat
c. Pulses
d. Sugarcane
Match List I with List II and select the
correct answer:
List I (Industrial Activity)
A. Tannery
B. Rubber
C. Vanaspati
D. Atomic Energy
List II (Place)
1. Narora
2. Kozhikode
3. Tonk
4. Kankroli
A
B
C
D
a.
3
2
4
1
b.
1
4
2
3
c.
3
4
2
1
d.
1
2
4
3
Which one of the following pairs is not
correctly matched?
a. Nasik Pune: N.H. 50
b. Durgapur Kolkata: Express Highway
c. Hyderabad : Headquarters of SouthCentral Railway
d. Haldia- Allahabad :
National
Waterway
The
Palghat
Gap
serves
inland
communication from
a. Panaji to Kochi
b. Kochi to Chennai
c. Mangalore to Mumbai
d. Madurai to Thiruvananthapuram
Consider the following Union Territories
1. Andaman and Nicobar islands
2. Dadra and Nagar Haveli
3. Daman and Diu
4. Lakshadweep

115.

116.

The correct sequence in the descending


order of the percentage change in the
growth of population in respect of these
during 1991 is
a. 2, 4, 1, 3
b. 1, 3, 2, 4
c. 2, 3, 1, 4
d. 1, 4, 2, 3
Match List I with List II and select the
correct answer:
List I (Natural Hazards)
A. Cyclones
B. Droughts
C. Earthquakes
D. Floods
List II (Areas)
1. Mid-eastern India.
2. Plains of Uttar Pradesh and Bihar
3. Jharkhand and Northern Orissa
4. Himalayan foothill zone
A
B
C
D
a.
3
2
4
1
b.
4
1
3
2
c.
3
1
4
2
d.
4
2
3
1
Match List I with List II and select the
correct answer:
List I (Mountain Pass)
A. Mana Pass
B. Niti Pass
C. Shipki La
D. Thaga La
List II (Label shown in the Map)

118.

119.

120.

117.

A
B
C
D
a.
3
4
1
2
b.
1
2
3
4
c.
3
2
1
4
d.
1
4
3
2
Match List I with List II and select the
correct answer:

13 of 13
List I (Cities)
A. Chennai
B. Kolkata
C. Mumbai
D. Delhi
List II (Approximate Height above M.S.I
in meters)
1. 6
2. 11
3. 216
4. 16
A
B
C
D
a.
4
1
2
3
b.
2
1
4
3
c.
4
3
2
1
d.
2
3
4
1
Which of the following
parts
is/are
located on the Arabian Sea coast of India?
1. New Mangalore
2. Kandla
3. Tuticorin
4. Mormugao
Select the correct answer using the codes
given below
a. 1 only
b. 1 and 3
c. 1, 2 , 3 and 4
d. 1, 2 and 4
With which one of the following is the
term Maitri associated?
a. HYV of rice
b. SAARC
c. Indo-Russian Treaty
d. Antarctica Research Centre
Match List I with List II and select the
correct answer:
List I (Biosphere Reserves)
A. Manas
B. Nokrek
C. Gulf of Mannar
D. Nanda Devi
List II (States)
1. Meghalaya
2. Assam
3. Uttranchal
4. Tamil Nadu
A
B
C
D
a.
2
3
4
1
b.
4
1
2
3
c.
2
1
4
3
d.
4
3
2
1

C.S.E-Pre 2004

1 of 12

GEOGRAPHY
1.

2.

3.

4.

Which of the following is/are the correct


characteristic (s) of primary seismic wave?
a. It is a longitudinal and compressional
wave
b. It is analogous to sound waves
c. It travels with faster speed through
solids but slowly through liquids
d. All the above three
Which one of the following is the correct
chronological order of the mountain
building?
a. Alpine - Caledonian - Hercynian
Precambrian
b. Precambnan - Hercynian - Caledonian
Alpine
c. Precambnan - Caledonian - Hercynian
Alpine
d. Caledonian - Precambrian - Alpine Hercynia
Which of the following are formed due to
collision of continental plates?
1. The Alps
2. The Himalayas
3. The Rockies
4. The Caucasus mountains
Select the correct answer using the codes
given below:
a. 1, 2 and 3
b. 2, 3 and 4
c. 1,2 and 4
d. 1, 3 and 4
Match List I (Rock Type) with List II
(Example) and select the correct answer :
List I
A. Clastic
B. Extrusive
C. Metamorphic
D. Plutonic
List II
1. Granite
2. Conglomerate
3. Gneiss
4. Tuff
A
B
C
D
a.
2
4
3
1
b.
1
3
4
2
c.
2
3
4
1
d.
1
4
3
2

5.

6.

7.

8.

The given diagram represents a/an

a. Asymmetrical fold
b. Monoclinal fold
c. Overfold
d. Recumbent fold
Which of the following plate movements
is responsible for the formation of mid ocean ridge?
a. Divergent movement
b. Convergent movement
c. Transform fault movement
d. Parallel movement
Match List I (Landforms) with List II
(Representative Sites) and select the
correct answer :
List I
A. Rift valley lake
B. Basic lava sheet
C. An active volcano
D. Batholith
List II
1. Himalayas
2. Upland of Brittany, France
3. Deccan trap region, India
4. Tanganyika Stramboli
A
B
C
D
a.
4
2
5
3
b.
5
3
1
2
c.
4
3
5
2
d.
5
2
1
3
Consider the following statements :
1. P - wave are the first seismic waves to
reach a place
2. S - waves travel only through liquids
3. Arrival time of P and S - waves is used
to locate the epicentre.
4. Richter scale is a logarithmic scale.
Which of the statements given above are
correct?
a. 1, 2 and 3
b. 1, 3 and 4

9.

10.

11.

c. 1 and 4
d. 3 and 4
A weathering process in which layers of
rocks peel off as expansion alternates with
contraction, is known as
a. Shattering
b. Block separation
c. Granular disintegration
d. Exfoliation
Match List I (Term) with List II (Process
Involved) and select the correct answer :
List I
A. Valley - in - valley profile
B. Collapse sinks
C. Shadow zone
D. Exhumed erosion surface
List II
1. Karst topography
2. Earthquake
3. Planation
4. Rejuvenation
A
B
C
D
a.
3
1
2
4
b.
4
2
1
3
c.
3
2
1
4
d.
4
1
2
3
Residual hill in the desert region is known
as
a. Inlier
b. Inselberg
c. Playa
d. Pediment

12.

13.

In the diagram given above, which of the


1,2,3,4 and 5 represent bluffs?
Select the correct answer using the codes
given below:
a. 1 and 3
b. 3 and 4
c. 1 and 5
d. 2 and 3
The drainage pattern developed on folded
sedimentary rocks is termed as
a. Rectangular
b. Radial
c. Dendritic
d. Trellis

2 of 12
Bowl shaped depression created as a
result of glacial erosion high on a
mountainside is called
a. Glacial trough
b. Hum
c. Hanging valley
d. Cirque
15.
Which one of the following statements is
correct?
a. Jet streams are -best developed in
winters
b. Jet streams often develop oscillations
c. Jet streams greatly influence our
surface weather
d. All the above three are correct in
respect of jet streams
16.
What is the correct sequence of the
following zonal pressure belts?
1. Subpolar low
2. Equatorial low
3. Polar high
4. Subtropical high
Select the correct answer using the codes
given below:
a. 2-3-1-4
b. 1-4-2-3
c. 2-4-1-3
d. 1-3-2-4
17.
Temperature generally decreases towards
the poles because
a. Air movement is generally towards the
equator
b. Cold polar air masses prevent surface
heating of the land
c. Cold surfaces do not absorb solar
energy as readily as warm surfaces
d. Progressively lesser solar energy per
unit area falls on the earth's surface as
we move to polar regions
18.
A temperature inversion is most likely to
occur under which of the following
conditions?
a. Mountain top in late evening
b. Windy but cloudy night
c. Calm, cloudy and humid night
d. Calm, clear and cool winter night
19.
The clouds extending from one side of
horizon to other as a dark grey sheet
having uniform base are called
a. Stratus clouds
b. Cumulus clouds
c. Cirrus clouds
d. Nimbus clouds'
20.
In the Hadley cell, rising air is found in the
14.

a.
b.
c.
d.

21.

22.

23.

24.

25.

Subtropical high pressure zone


Trade winds
Westerlies
Inter - tropical convergence zone
(ITCZ)
Which of the following are source regions
of polar continental air masses?
1. Northern Eurasia
2. Australia
3. North Atlantic,
4. Antarctica
Select the correct answer using the codes
given below:
a. 1 and 2
b. 2 and 3
c. 1 and 4
d. 2 and 4
Which one of the following statements is
correct? Hurricanes
a. form at the equator
b. are not as large as tornadoes
c. develop over warm ocean areas
d. tend to intensify when they move over
land surfaces
Which one of the following regions has a
Mediterranean type of climate?
a. Cape Town" region of South Africa
b. Great Plains of North America
c. North - Eastern Australia
d. Southern Algeria
Consider the following statements
1. Where fold mountains run parallel and
close to the coast, continental shelf is
narrow or absent
2. The average depth for continental shelf
is 100 metres
3. Submarine canyons are mostly found
in continental slope
4. The submarine flat topped mountains
are called guyots
Which of the statements given above are
correct?
a. 1, 2, 3 and 4
b. 1,2 and 4
c. 1. 3 and 4
d. 2 and 3
Match List I (Ridges) with List II
(Oceans) and select the correct answer
List I
A. Mendeleyev
B. Keruguelen
C. San Felix-Juan Fernandez
D. Walvis
List II

3 of 12
North Atlantic Ocean
South Atlantic Ocean
Arctic Ocean
Indian Ocean
South Pacific Ocean
A
B
C
D
a.
5
4
1
2
b.
3
2
5
4
c.
5
2
1
4
d.
3
4
5
2
The coastline formed by the submergence
of mountain ridges running parallel to the
coast is known as
a. Dalmatians coast
b. Ria coast
c. Fiord coast
d. Haff coast
Coral reefs are formed by
a. Volcanic rocks
b. Marine sediments
c. Chlorine material precipitated from sea
water
d. Tiny colonial marine animals which
construct limestone skeleton material
Match List I with List II and select the
correct answer:
List I
A. Gulf Stream
B. West wind drift
C. Peru current
D. West Australian current
List II
1. Pacific Ocean
2. A slow eastward movement of water
over the zone of westerly winds
3. Indian Ocean
4. Warm current
A
B
C
D
a.
4
2
1
3
b.
1
3
4
2
c.
4
3
1
2
d.
1
2
4
3
1.
2.
3.
4.
5.

26.

27.

28.

29.

30.

31.

In the diagram given above, which out of


1, 2, 3 and 4 are cold currents?
a. 1 and 3
b. 2 and 4
c. 2 and 3
d. 1 and 2
The term 'syzygy' is referred to when the
a. Earth is at perihelion and moon at
perigee
b. Earth is at aphelion and moon at
apogee
c. Moon and sun are at right angles with
reference to the earth
d. Moon, sun and earth lie along a
straight line
What happens to tropical soils when the
rainforest is cleared?
a. The soil improves to the extent that
soluble nutrients are no longer depleted
by the trees
b. There is little effect on the soil because
of its depth of richness
c. Clearing interrupts the critical nutrient
cycle and leads to soil leaching
d. The soils stabilize and become
excellent for grain crops

35.

36.

32.

4 of 12
on the scale starting from the largest map
scale :
a. Atlas map - Topographical map Cadastral map
b. Cadastral map - Atlas map Topographical map
c. Cadastral map - Topographical map Atlas map
d. Topographical map - Atlas map Cadastral map
Which one of the following statements
about maps is not correct?
a. A map is a graphic representation of
the milieu
b. A map is a tangible product
c. A map at small scale shows a small
area on a single map sheet
d. A map is mental and manual creation
of man
Match List I (Projections) with List II
(Graticules) and select the correct answer:
List I (Projections)
A. Equatorial orthographic zenithal
B. Sinusoidal projection
C. Homolographic projection
D. Hornolosine projection
List II (Graticules)

1.

2.

33.

34.

In the diagram given above, the areas


marked 'X' and 'Y' respectively, represents
which of the following soil types?
a. Sand and clay
b. Loam and clay
c. Clay and sandy loam
d. Silt and sand
Among the following climatic factors,
which one has the least effect upon a
terrestrial ecosystem?
a. Temperature variation
b. Wind
c. Conditions of sunlight
d. Availability of water
For the following three types of maps,
identify the correct descending order based

3.

4.
a.
b.
c.
d.
37.

A
3
4
3
4

B
1
2
2
1

C
4
3
4
3

D
2
1
1
2

42.

38.

39.

40.

41.

In the diagram shown above, points A and


B are located on the ground surface 7.5
Km apart. These two points appear on a
vertical aerial photograph as a and 'b'
separated by a photographic distance of
100 mm the scale of the photograph is:
a. 1: 75,00,000
b. 1: 7,50,000
c. 1: 75,000
d. 1: 750
The biome with the shortest growing
season is
a. Savanna
b. Taiga
c. Chaparral
d. Tundra
Match List I (Location/Area in Canada)
with List II (Major Mineral) and select the
correct answer :
List I
A. Labrador
B. Athabasca
C. Sudbury
D. British Columbia
List II
1. Zinc
2. Petroleum
3. Iron ore
4. Manganese
A
B
C
D
a.
4
1
3
5
b.
3
2
4
1
c.
4
2
3
1
d.
3
1
4
5
The Snake River plateau in the USA is
formed of
a. Great sheets of lava
b. Enormous quantities of river borne
material
c. Wind blown sand
d. None of the above three
Which one of the following countries has
the highest agricultural production in
Europe?
a. France
b. Switzerland
c. Denmark
d. Germany

43.

44.

45.

46.

5 of 12
Match List I (Main Characteristic
Feature) with List II (Country) and select
the correct answer:
List I
A. Largest Area
B. Highest population density
C. Largest population
D. Largest urban population
List II
1. Iran
2. Israel
3. Lebanon
4. Saudi Arabia
5. United Arab Emirates
A
B
C
D
a.
4
3
1
2
b.
1
2
4
5
c.
4
2
1
5
d.
1
3
4
2
Which one of the following statements
correctly defines the term 'Karroo' of
Southern Africa?
a. Rift valleys formed due to faulting
b. Dome shaped landforms caused by
earth's movements
c. Steps from the coastal lowland on to
the high plateau
d. None of the above three
Structurally, the Great Himalaya is
a. An autochthonous zone
b. A par-autochthonous zone
c. An allochthonous zone
d. A suture zone
Which one of the following statements is
correct?
a. The mountains of the Peninsular India
are about 40 million years old
b. The Eastern Ghats extend as a
continuous range for a distance of
about 1600 km
c. The Western Ghats consist of a series
of discontinuous hills with no
structural continuity
d. The Siwalik Ranges have hogback
appearance
Match List I (Drainage Pattern) with List
II (Rivers) and select the correct answer :
List I
A. Consequent
B. Subsequent
C. Antecedent
D. Superimposed
List II
1. Sindhu, Brahmaputra

47.

48.

2. Swarnrekha, Son
3. Chambai, Betwa
4. Sharavati, Periyar
A
B
C
D
a.
2
1
3
4
b.
4
3
1
2
c.
2
3
1
4
d.
4
1
3
2
In which one of the following countries, is
the ratio of hydroelectric generation to the
total electricity generation the highest?
a. Norway
b. Sweden
c. Italy
d. Denmark
Consider the following statements about
the soils of India:
1. High temperature during most of the
year retards formation of humus.
2. Soils which have humus require
regular application of chemical
fertilizers
to
remove
nitrogen
deficiency.
3. Titanium salts in basalt impart black
colour to the soils of the Indian
plateau.
4. Red soils are confined to the peripheral
areas of the Indian plateau.
Which of the statements given above are
correct?
a. 1 and 2
b. 1, 3 and 4
c. 2 and 3
d. 2 and 4

51.

52.

53.

54.

49.

55.

50.

The location 'X' marked in the map shown


above is that of a hydel power station. This
is
a. Koyna on Krishna river
b. Saravathy on Saravathy river
c. Thungabhadra on Thungabhadra river
d. Nagarjun Sagar on Krishna river
The Himalayan region is poor in mineral
resources because

6 of 12
a. it is made of crystalline rocks
b. the displacement of rock strata has
disturbed the arrangement of rocks and
made it complex
c. the climatic conditions are not suitable
for exploitation of minerals
d. the terrain makes exploitation of
minerals difficult and very costly due
to transport difficulties
Which one of the following agencies is
working for the development of shrimp
culture for the farmers?
a. Fish Farmers' Development Agency
b. Technology
Mission
on
Dairy
Development
c. Fishing Survey of India
d. Brackish
Water
Fish
Farmers'
Development Agency
Canal irrigation is most important in the
Northern Plains of India because
a. the soil is porous
b. the level of underground water is high
c. the sources of canals are perennial
rivers
d. the region is densely populated
The present demographic transition of
India is indicative of
a. High population growth potential but
low actual growth
b. High population growth potential and
high actual growth
c. Partial industrialized economy
d. Typically increasing urbanization
Tar sands containing large molecular
hydrocarbons like crude oil are found in
limited quantities in several parts of the
world but the most important known
occurrence is in
a. USA
b. Canada
c. Australia
d. Russia
Identify the correct descending order of
production of wood pulp of the following
countries :
1. USA
2. Sweden
3. Japan
4. Canada
Select the correct answer using the codes
given below:
a. 1-3-2-4
b. 2-4-1-3
c. 1-4-2-3

56.

57.

d. 2-3-1-4
French Canadians are largely concentrated
in
a. the lowlands along the St. Lawrence
river in Quebec
b. the areas' all along the US border with
Canada
c. the province of Ontario bordering the
Great Lakes
d. the wheat belt in the lower interior
plains
The Appalachian Mountains were formed
in which of the following periods?
a. Cretaceous
b. Mesozoic
c. Palaeozoic
d. Tertiary

62.

63.

64.

58.

59.

60.

61.

The shaded area in the map of the USA


shown above has
a. Mediterranean type of climate
b. Humid subtropical type of climate
c. Steppe type of climate
d. St. Lawrence type of climate
Most of the Japanese industries are located
on the
a. Hokkaido coast
b. Honshu coast
c. Shikoku coast
d. Kyushu coast
Which of the following are associated with
Saudi Arabia?
1. High proportion of urban population
2. Extensive agriculture pattern
3. Considerable extent of land under
pasture lands
Select the correct answer using the codes
given below:
a. 1, 2 and 3
b. 1 and 2
c. 2 and 3
d. 1 and 3
The area between Chhotanagpur plateau
and Shillong plateau is
a. a Graben
b. formed by erosion of Ganga
c. a down faulted region

65.

66.

67.

7 of 12
d. a down warped region in which the
Ganga has deposited sediments
The boundary of the Siwalik Belt with the
Indo-Gangetic through is known as
a. Main boundary thrust
b. Main boundary fault
c. Great boundary fault
d. Scrap fault
The Himalayan front fault is located
between
a. Siwaliks and Lesser Himalaya
b. Great and Trans Himalayas
c. Lesser and Great Himalayas
d. Siwaliks and Piedmont zone
The temperature of Thiruvananthapuram is
lower (ban that of Mumbai in May and
higher than that of Mumbai in January,
because
a. Thiruvananthapuram has cold current
and Mumbai is faced with warm
current
b. Thiruvananthapuram
has
higher
rainfall in summer and it is nearer to
the equator
c. Thiruvananthapuram
is
on
the
windward side and Mumbai is on the
leeward side
d. Thiruvananthapuram
is
thickly
vegetated while Mumbai is not
Xerophytic vegetation is a characteristic
feature of
a. Chhotanagpur plateau
b. Khasi hills
c. Eastern Ghats
d. Kuchch
Match List I (Sources of Energy) with List
II (Places) and select the correct answer :
List I
A. Geothermal
B. Wind
C. Wave
D. Tidal
List II
1. Khambhat
2. Manikaran
3. Bhavnagar
4. Vijinjham
A
B
C
D
a.
3
4
1
2
b.
2
1
4
3
c.
3
1
4
2
d.
2
4
1
3
Match List I (Forest Type) with List II
(Regions) and select the correct answer :

68.

69.

70.

71.

List I
A. Tropical evergreen
B. Monsoon
C. Temperate
D. Mangrove
List II
1. Chhotanagpur plateau, Siwaliks
2. Shillong plateau, Sahyadris
3. West Bengal, Terai
4. Eastern and Western Himalayas,
Nilgiris
A
B
C
D
a.
2
4
1
3
b.
3
1
4
2
c.
2
1
4
3
d.
3
4
1
2
Match List I (Shifting Cultivation) with
List II (State) and select the correct answer
:
List I
A. Jhum
B. Podu
C. Beera
D. Ponam
List II
1. Kerala
2. Madhya Pradesh
3. Andhra Pradesh
4. Assam
A
B
C
D
a.
4
3
2
1
b.
2
1
4
3
c.
4
1
2
3
d.
2
3
4
1
The Arab geographer who authored the
book 'Amusement for Him Desires to
Travel Around the World', also called the
book of Roger and who is famous for his
'Silver Map' was
a. A1 Idrisi
b. Al Biruni
c. Al Maqdisi
d. Al Istakhari
Who amongst the following was the first
to propose a classification of sciences
dealing with reality?
a. Alfred Hettner
b. Immanuel Kant
c. Alexander von Humboldt
d. Richard Hartshorne
Humboldt synthesized his ideas in
'Cosmos' with the specific objective
a. To define the contents and methods of
modern physical geography

72.

73.

74.

75.

76.

8 of 12
b. To lay foundation of the Universal
Science a knowledge of the universe
in its totality
c. To establish geography as the study of
the earth surface of its areal segments
d. To explain the concept of dualism
between man and the physical world
Erdkunde written by Ritter is a/an
a. Earth science concentrating on every
aspect of the earth
b. Regional geography dealing with the
earth surface as mere aggregate of
parts
c. Regional geography dealing with areas
as organic entities
d. Science of earth in relation to nature
and history of mankind
One of the pioneer German geographers of
19th century, who presented the organic
theory of state, was
a. Alexander von Humboldt
b. Carl Ritter
c. Friedrich Ratzel
d. Alfred Hettner
Which of the following were objectives of
the quantitative revolution in geography?
1. To get rid of deterministic thinking
2. To make geography more scientific
3. To make geographical study more
precise and accurate
4. Formulation of models and theories
Select the correct answer using the codes
given below:
a. 1, 2 and 3
b. 1, 2 and 4
c. 2, 3 and 4
d. 1, 3 and 4
Behaviouralism came into existence as
reaction against
a. Environmental determinism
b. Quantification
c. Possibilism
d. Regionalism
Which of the following are three important
approaches to humanistic geography?
1. Idealistic
2. Phenomenal
3. Naturalistic
4. Existential
Select the correct answer using the codes
given below:
a. 1, 2 and 3
b. 1, 2 and 4
c. 1, 3 and 4

77.

78.

79.

80.

81.

82.

d. 2, 3 and 4
The central idea of various cosmogenic
theories of Vedic and post-Vedic period is
the creation of a cosmic nucleus, called
a. Vishwakarma
b. Prajapati
c. Brahrnand
d. Surya
From among the following Greek scholars,
who was the first to conceptualize the
earth as spherical?
a. Pythagoras
b. Plato
c. Eratosthenes
d. Aristotle
Consider the following statements :
Arab geographers contributed to the
development of geography by
1. translating and absorbing of learning
from Greek, Persian and Sanskrit
sources.
2. preserving important writings.
3. expanding geographical knowledge in
Europe and Asia.
4. their
original
contributions
to
Mathematics.
Which of the statements given above are
correct?
a. 1, 3 and 4
b. 1, 2 and 3
c. 2, 3 and 4
d. 1, 2 and 4
Almagest of Ptolemy is a standard
reference of
a. The precise location of known places
b. Tables of latitudes and longitudes
c. Map projections and world maps
d. To movement of celestial bodies
Which of the following methods were
employed by Humboldt in his universal
science-Cosmos?
1. Empirical and inductive
2. Deductive procedure of science
3. Comparative regional geography
4. Graphical representation
Select the correct answer using the codes
given below:
a. 1, 2 and 3
b. 1, 3 and 4
c. 2, 3 and 4
d. 1, 2 and 4
Which one of the following pairs is
correctly matched?

83.

84.

85.

86.

87.

88.

89.

9 of 12
a. Ellen
Semple:Stop-and-GoDeterminism
b. Friedrich Ratzel:- Pulsation Hypothesis
c. Vidal de la Blache :- Genre de vie
d. Griffith Taylor:- Lebensraum
Physical environment offers opportunities
for a range of possible directions of
development and it depends on human
initiatives in particular directions of
progress. This basic premise in geography
is known as
a. Determinism
b. Neo-determinism
c. Possibilism
d. Probabilisrn
Who advocates climate as the main
determinant of history, culture, lifestyle
and occupations?
a. G. Taylor
b. F. Ratzel
c. E. Semple
d. E. Huntington
Who amongst the following founded the
French School of Regional Geography?
a. Paul Vidal de la Blache
b. Philippe Pinchemel
c. Elisee Reclus
d. Lucien Gallois
Which of the following techniques is
useful for pre-dieting the wheat yield?
a. Regression
b. Chi-square
c. Simulation
d. Correlation
Marxist geography found its most detailed
and lucid expression in the writings of
a. Richard Peet
b. M. Quani
c. E. W. Soja
d. David Harvey
Who among the following is regarded as
the founder of humanistic approach in
geography?
a. Richard Peet
b. Wiiiiam Bunge
c. Brian J. L. Berry
d. Yi-Fu-Tuan
Which state amongst the following has the
highest percentage of the Scheduled Caste
population of the State's population?
a. Jharkhand
b. Madhya Pradesh
c. Punjab
d. Uttar Pradesh

90.

91.

92.

93.

94.

95.

Large part of the non-ecumene has


remained sparsely populated because of
a. Its
natural
environment
being
inhospitable
b. Its restriction on large scale inmigration
c. Its limited and unknown resources
d. Its limited development vis--vis
infrastructure
Which one of the following geographers
has developed the idea of the 'mobility
transition'?
a. L. A. Kosinski
b. E. G. Ravenstein
c. E. S. Lee
d. W. Zelinsky
Which one of the following activities
occupies the largest area of the world"?
a. Commercial plantation
b. Pastoral herding
c. Commercial grain farming
d. Subsistence agriculture
In which of the following countries, has
maximum damage to the forests been
caused by acid rain?
a. Spain
b. Netherlands
c. Poland
d. Italy
Which one of the following statements is
not correct?
a. Shifting cultivation is characterized by
field rotation rather than by crop
rotation
b. In intensive subsistence agriculture,
animal farming is more developed
c. Plantation crops require heavy outlay
d. The economy of livestock ranchers is
purely commercial
Match List I (Minerals) with List II (Major
Mining Countries) and select the correct
answer ;
List I
A. Bauxite
B. Manganese
C. Mica
D. Tin
List II
1. India
2. Japan
3. Jamaica
4. Russia
5. Malaysia
A
B
C
D

96.

97.

98.

99.

100.

101.

10 of 12
a.
5
4
1
3
b.
3
1
2
5
c.
5
1
2
3
d.
3
4
1
5
Which one of the following pairs is not
correctly matched?
States of the USA: Geographic Division
a. (a) Iowa: West North Central
b. (b) Texas: West South Central
c. (c) California: Pacific
d. (d) New Jersey: South Atlantic
The longest inland waterways in the world
is
a. Mississippi river system
b. The Great Lakes
c. St. Lawrence
d. River Rhine
Which one of the following natural regions
is best known for the production of
grapes?
a. Hot deserts
b. Monsoon
c. Equatorial
d. Mediterranean
Which one of the following countries
makes maximum use of the geothermal
energy?
a. New Zealand
b. Japan
c. Iceland
d. Russia
Consider the following rural settlement
patterns :
1. Block pattern
2. Circular pattern
3. Fan pattern
Select the correct combination of areas
from amongst the following in which 1, 2
and 3 are found, respectively:
a. Delta, Desert, Lakeside
b. Desert, Delta, Lakeside
c. Desert, Lakeside, Delta
d. Delta, Lakeside, Desert
Consider the following statements :
1. The Taiga biome stretches across large
parts of Canada, Europe and Asia.
2. Taiga is the largest biome in the world.
3. A lot of coniferous trees grow in the
Taiga.
Which of the statements given above are
correct?
a. 1 and 2
b. 2 and 3
c. 1 and 3

102.

103.

104.

105.

106.

107.

d. 1, 2 and 3
Which one of the following countries has
the highest average of road length on per
thousand square kilometer area basis?
a. India
b. Japan
c. USA
d. France
Match List I (Tribes) with List II (Places
of Habitat) and select the correct answer:
List I
A. Inuit
B. Bantu
C. Shompen
D. Chukchi
List II
1. Northern Russia
2. Nicobar Islands
3. Northern Canada
4. Central Africa
A
B
C
D
a.
3
2
4
1
b.
1
4
2
3
c.
3
4
2
1
d.
1
2
4
3
Most of the people in the world live in
lowland areas except those in
a. Central and Western Africa
b. Southern and South-eastern Europe
c. Southern and South-eastern Asia
d. Central America and North Western
South America
Which one of the following continents has
the lowest birth and death rates?
a. Europe
b. Australia
c. North American
d. South America
Which one of the following was
formulated by George K Ziph in 1940 to
explain the observed movement of people?
a. Principle of least effort
b. Intervening opportunities
c. Gravity model
d. Mean information field
Match List I (Factors of Population
Problem) with List II (Corresponding
Examples) and select the correct answer:
List I
A. Ban on migration
B. Extraordinary growth
C. Inadequate use of natural
D. resources
E. D. Unequal distribution

108.

109.

110.

111.

112.

113.

11 of 12
List II
1. Brazil
2. England
3. South Africa
4. South East Asia
5. Tropical Lands
A
B
C
D
a.
1
5
2
3
b.
3
4
5
1
c.
1
4
5
3
d.
3
5
2
1
Which one of the following areas is
dominant in hunting and gathering?
a. Hot desert
b. Cold desert
c. Tropical forest
d. Temperate grassland
Maximum amount of herring fish is
obtained from
a. The North Sea around the Netherlands
b. The coastal areas from Alaska to
California
c. The Sea of Japan
d. The Gulf of Mexico
Which one of the following is the junction
point of the Eastern Ghats and Western
Ghats?
a. Javadi Hills
b. Anaimalai Hills
c. Nilgiri Hills
d. Shevaroy Hills
Consider the following statements :
1. Germany and America are the two
largest producers of wind-power.
2. America has nuclear energy generation
capacity of around 1,00,000 MW
3. America has more nuclear reactors
than any other country in the world.
Which of the statements given above are
correct?
a. 1 and 2
b. 2 and 3
c. 1 and 3
d. 1, 2 and 3
Shaba province in the Democratic
Republic of the Congo is known for which
of the following reserves?
a. Bauxite
b. Iron ore
c. Tungsten
d. Copper
Match List I (Book) with List II (Author)
and select the correct answer :
List I

A. A Prologue to Population Geogra


B. Geography of Population
C. The Geography of Life and Death
D. Population Geography
List II
1. G. T. Trewartha
2. J. I. Clarke
3. W. Zelinsky
4. L. D. Stamp
A
B
C
D
a.
3
1
4
2
b.
4
2
3
1
c.
3
2
4
1
d.
4
1
3
2
Directions: The following 7 (Seven) items consist
of two statements, one labelled as the 'Assertion
(A)' and the other as 'Reason (R)'. You are to
examine these two statements carefully and select
the answers to these items using the codes given
below:
114. Assertion (A): Oceanic salinity is higher
along Tropics of Cancer and Capricorn.
Reason (R): Increased evaporation coupled
with relatively more insulation along these
latitudes causes higher oceanic salinity.
a. Both A and R are individually true
and R is the correct explanation of A
b. Both A and R are individually true but
R is not the correct explanation of A
c. A is true but R is false
d. A is false but R is true
115. Assertion (A): A composite profile
includes only the lowest parts of a series of
parallel profiles.
Reason (R): A composite profile is
constructed to represent the overall relief
of an area as viewed from a distance.
a. Both A and R are individually true
and R is the correct explanation of A
b. Both A and R are individually true but
R is not the correct explanation of A
c. A is true but R is false
d. A is false but R is true
116. Assertion (A): Generally, temperature
decreases from the equator towards poles
which is called temperature gradient. But
the highest temperature is never recorded
at the equator, instead; it is recorded near
both the tropics.
Reason (R): A sizeable portion of the
incoming solar radiation is reflected by
clouds and a large part of heat is spent in
the process of evaporation.
a. Both A and R are individually true
and R is the correct explanation of A

117.

118.

119.

120.

12 of 12
b. Both A and R are individually true but
R is not the correct explanation of A
c. A is true but R is false
d. A is false but R is true
Assertion (A): Until about the end of
eighties of the last century, the Indian
industries showed a clear attitude of
structural shift from consumer goods to
basic and capital goods industries.
Reason (R): The demand for consumer
goods declined during that period.
a. Both A and R are individually true
and R is the correct explanation of A
b. Both A and R are individually true but
R is not the correct explanation of A
c. A is true but R is false
d. A is false but R is true
Assertion (A): French geographers
developed human geography as a complete
unit.
Reason (R): French geographers believed
that environment sets a limit to human
ambitions but man is more powerful than
the environment.
a. Both A and R are individually true and
R is the correct explanation of A
b. Both A and R are individually true but
R is not the correct explanation of A
c. A is true but R is false
d. A is false but R is true
Assertion (A): Radicalism is entirely based
on quantitative revolution.
Reason (R): The consideration of social
values is the core of radicalism.
a. Both A and R are individually true and
R is the correct explanation of A
b. Both A and R are individually true but
R is not the correct explanation of A
c. A is true but R is false
d. A is false but R is true
Assertion (A): The Ransomed Lake in
Rajasthan has dried up for the first time in
its history of 300 years.
Reason (R): The region had deficiency of
rainfall by 69% during 1999-2003 and had
been confronted with acute drought for the
last four years
a. Both A and R are individually true
and R is the correct explanation of A
b. Both A and R are individually true but
R is not the correct explanation of A
c. A is true but R is false
d. A is false but R is true

C.S.E-Pre 2005

1 of 15

GEOGRAPHY
1.

2.

3.

4.

5.

6.

Which one of the following faults has been


shown in the figure given above?
a. Reverse fault
b. Normal fault
c. Strikeslip fault
d. Step fault
Folds in which one limb inclines
moderately with regular slope while the
other inc steeply at right angle and the
slope is almost vertical are known as
a. lsoclinal
b. Monoclinal
c. Recumbent
d. Asymmetrical
Sedimentary rocks are formed when
sediments become compressed and
cemented together in a process known as
a. Crystallization
b. Sedimentation
c. Solidification
d. Lithification
Which one of the following periods is
largely associated with extinction of
Dinosaurs and the increase in flowering
plants and reptiles?
a. Jurassic
b. Triassic
c. Cretaceous
d. Permian
Which one of the following statements is
correct Plateaus are the product of?
a. mature stage of cycle of erosion
b. youth stage of cycle of erosion
c. old stage of cycle of erosion
d. rejuvenation of the old mountain
Match list I (Major Biome) with List II
(Physical Characteristic) and select the

7.

8.

correct answer using the - code given


below the lists:
List I
A. The northern most of the temperate
formations (i.e. a continuous belt
across North America and Northern
Eurasia)
B. Arctic Tundra Vegetation
C. Marine Environment Classification on
D. The terrestrial biomes of the tropics
List-II
1. Floristically poor
2. Boreal Forest
3. Pelagic division
4. Savanna woodland
5. Soviet Steppe and North American
Prairie
A
B
C
D
a.
2
4
3
1
b.
2
1
3
4
c.
4
3
5
2
d.
4
1
3
2
Match List I (Soil Forming Process) with
List II (Related Characteristic) and select
the correct answer using the code given
below the lists:
List I
A. Composition of true soil
B. Origin of inorganic particle
C. Large quantities of soil colloids
D. Formation of thinner soil
List-II
1. Sticky soil formation
2. Steeper slope
3. Mineral and organic particles
4. Flat upland area
5. Weathering of rocks
A
B
C
D
a.
3
5
1
2
b.
4
1
2
5
c.
3
5
4
1
d.
4
2
5
1
Match List I (Ocean current) with List II
(Name of Ocean) and select the correct

9.

10.

11.

answer using the code given below the


Lists:
List I
A. Agulhas current
B. El Nino current
C. Kuroshio current
D. Benguela current
List-II
1. North Atlantic
2. South Atlantic
3. North Pacific
4. South Pacific
5. Indian Ocean
A
B
C
D
a.
5
3
1
5
b.
5
4
3
2
c.
2
4
3
5
d.
5
3
1
2
Which one of the Following sequences
correctly represents the percentage of
given salts in sea water in decreasing
order?
a. Magnesium chloride Sodium
chloride Magnesium sulphate
Calcium sulphate
b. Magnesium sulphate Magnesium
chloride Calcium sulphate
Sodium chloride
c. Sodium chloride Magnesium
chloride Magnesium sulphate
Calcium sulphate
d. Sodium chloride Magnesium
sulphate Magnesium chloride
Calcium sulphate
What are the causes for adiabatic
temperature changes in the atmosphere
a. Convection and advection
b. Latent heat of condensation
c. Expansion on compression of the air
d. Partial absorption of solar radiation by
the atmosphere
Consider the following statements
The Circum Pacific belt is highly prone to
the occurence of earthquakes because it is
a zone of
1. Young folded mountains
2. active volcanoes.
3. divergent plate boundaries.
4. convergent plate boundaries.
Which of the statements given above are
correct?

12.

13.

14.

2 of 15
a. 1, 2 and 3
b. 1, 2 and 4
c. 2, 3 and 4
d. 1, 3 and 4
Match List I (Environmental Degradation)
with list II (Constituent
Causing
Degradation) and select the given below
the lists:
List I
A. Acid rain
B. Automobile smoke
C. Ozone Depletion
D. Global warming
List II
1. Nitrogen
2. Carbon dioxide
3. Nitrogen oxide
4. Oxide of sulphur
5. Chlorofluorocarbon
A
B
C
D
a.
4
2
5
3
b.
5
3
1
2
c.
4
3
5
2
d.
5
2
1
3
Match List I (Ocean Deposit of Organic
Remain) with List II (Origin of Organic
Remain) and select the correct answers
using the code given below the lists:
List I
A. Neritic remains
B. Pelagic remains
C. Pteropod ooze
D. Diatom ooze
List II
1. Shells of Plank tonic molluscs
2. Microscopic plant shells
3. Dead skeletons of marine animals and
plants
4. A type of algae
5. Shells of various foraminifera
A
B
C
D
a.
1
4
5
2
b.
3
2
1
4
c.
1
2
5
4
d.
3
4
1
2
Which one of the following sequences
correctly indicates the increasing order of
the distance from the coast?
a. Abyssal plain continental shelf
continental slope continental rise

15.

16.

17.

18.

b. Continental shelf Continental slope


abyssal plain continental rise
c. Continental shelf continental slope
continental rise abyssal plain
d. Continental slope continental shelf
abyssal plain continental rise
Which one of the following represents
wall like formation of Solidified magma
and are mostly perpendicular to the beds of
the sedimentary rocks?
a. Dyke
b. Sills
c. Lapilli
d. Breccia
Consider the following statements:
1. New Zealand has very mild winters
owing to its oceanic situation and only
the extreme north has very cold
winters
2. In New Zealand, the situation of the
mountain chain produces much sharper
climate contrasts between cast and
west than in a north-south direction.
Which of the statements given above is/are
correct?
a. 1 only
b. 2 only
c. Both 1 and 2
d. Neither 1 nor 2
Consider the following statements
1. The network of Indian railways is
spread over 80,000 Route Kilometers
(RKm).
2. More than 50% of RKm, network of
the Indian Railways is electrified.
Which of the statements given above is/are
correct?
a. 1 only
b. 2 only
c. Both 1 and 2
d. Neither 1 nor 2
Which one of the following statements is
not correct?
a. Europeans emigrated m the densely
populated tropical and subtropical
coastlands.
b. The great exodus from Ireland in mid
1900 was mainly due to failure of
potato crops
c. Chinese immigrates have contributed
greatly to economic development of
Malaysia

3 of 15
d. The cities of the north-eastern United
States are now experiencing migration
losses
19.
Which one of the following pairs is not
correctly matched?
Theory Relating to Urban Structure:
Proponent
a. Concentric Theory:- Sinclair
b. Sector Theory:- Homer Hoyt
c. Central Place Theory:- Christaller
d. Multiple Nuclei Theory:- Harris and
Ullman
20.
Which one of the following pairs is not
correctly matched?
Concept:
Scholar
a. Rank-size rule:- G.K. Zipf
b. Law bf retail gravitation:- W.J. Reilly
c. Concept of conurbation:- Patrick
Geddes
d. Concept of megalopolis:- E.W.
Burgess
21.
Traveling from the equator towards pole,
what would he the correct sequence of
vegetation type you would encounter?
a. Tropical rain forest -tundra - savanna
b. Tropical rain forest -desert scrub tundra
c. Desert scrub - tropical rain foresttundra
d. Mediterranean scrub -desert scrubtundra
22.
Which one of the following pairs is not
correctly matched?
Type of Agriculture Country
a. Viticulture:- France
b. Sericulture:- Poland
c. Horticulture:- Netherlands
d. Pisciculture:- Japan
Directions: The following 8 (Eight) items consists
of two statements, one labelled as the Assertion
(A) and the other as Reason (R). You are to
examine these two statements carefully and select
b answers to these items using the codes given
below:
23.
Assertion (A): The amount of salinity in
the tropical zone is recorded higher in the
eastern than the western coasts of
continents.
Reason (R): The trade wind causes
upwelling of water in the western coasts
and the water is piled up in the eastern
coasts of con tine nets.

24.

25.

26.

27.

a. Both A and R are individually true and


R is the correct explanation of A.
b. Both A and R are individually true hut
R is not the correct explanation of A
c. A is true hut R is false
d. A is false hut R is true
Assertion
(A):
Intensification
and
diversification of agriculture in the Central
Valley has made California the leading
American state in agricultural output.
Reason (R): California has experienced
massive & progressive extension of
irrigation.
a. Both A and R are individually true and
R is the correct explanation of A.
b. Both A and R are individually true hut
R is not the correct explanation of A
c. A is true hut R is false
d. A is false hut R is true
Assertion (A): A river system can continue
to cut down its valleys at approximately
the same rate as uplift and so maintains its
general pattern and direction.
Reason (R): In case of an antecedent
drainage system, the river is said to have
originated before a period of uplift and
folding of land as a result of cart
movement.
a. Both A and R are individually true and
R is the correct explanation of A.
b. Both A and R are individually true hut
R is not the correct explanation of A
c. A is true hut R is false
d. A is false hut R is true
Assertion (A): A strip of land near the
south-western shore of San Francisco Bay
is referred to as Silicon Valley.
Reason (R): In the early part of California
history the south-western shore of San
Francisco Bay was a major area of silica
mining.
a. Both A and R are individually true and
R is the correct explanation of A.
b. Both A and R are individually true hut
R is not the correct explanation of A
c. A is true hut R is false
d. A is false hut R is true
Assertion (A): On small scale conformal
projections, large areas may be imperfectly
represented with respect to shape.

28.

29.

30.

31.

4 of 15
Reason (R): There is no angular
deformation at any point on a conformal
projection.
a. Both A and R are individually true and
R is the correct explanation of A.
b. Both A and R are individually true hut
R is not the correct explanation of A
c. A is true hut R is false
d. A is false hut R is true
Assertion (A): Both Remote Sensing and
GIS are involved in the analysis of
phenomena of spatial significance.
Reason (R): Remote Sensing serves as an
important data source for GIS and vice
versa.
a. Both A and R are individually true and
R is the correct explanation of A.
b. Both A and R are individually true hut
R is not the correct explanation of A
c. A is true hut R is false
d. A is false hut R is true
Assertion (A): The outer core of the
earths interior is in molten state while the
inner core is in solid state.
Reason (R): The P-seismic waves
disappear in outer core while S-waves
penetrate up to the inner core.
a. Both A and R are individually true and
R is the correct explanation of A.
b. Both A and R are individually true hut
R is not the correct explanation of A
c. A is true hut R is false
d. A is false hut R is true
Assertion (A): In a resequent fault line
scarp, the down throw side will form
Lower ground.
Reason (R): The down throw side
possesses the softer rock at the level of
present erosion surface.
a. Both A and R are individually true and
R is the correct explanation of A.
b. Both A and R are individually true hut
R is not the correct explanation of A
c. A is true hut R is false
d. A is false hut R is true
Consider the following statements
regarding peninsular India:
1. It is largely con, posed of Archaean
crystalline rocks.
2. Most of the mountains here are of
relict type.

32.

33.

34.

35.

3. The interior parts have post-cambrian


marine deposists.
4. Deccan basalts are formed at the end of
cretaceous.
Which of the statements given above are
correct?
a. 1, 2, 3 and 4
b. 1, 2 and 4
c. 3 and 4
d. 1 and 2
Which one of the Following is the correct
statement?
The Alpine vegetation in the eastern
Himalayas is found up to greater altitudes
than the Western Himalayas because.
a. the eastern Himalayas are higher than
the western Himalayas
b. the eastern Himalayas are nearer to the
equal or and sea coast than the western
Himalayas
c. the eastern Himalayas receive more
rainfall than the western Himalayas
d. the eastern Himalayan soils are more
fertile than those of the western
Himalayas
The irregularity in the amount of rainfall in
different parts of the north Indian plains is
mainly due to
a. Irregular intensity of low pressure in
the north-western parts of India
b. Variation in the location of the axis of
low pressure trough
c. Difference in frequency of cyclones
d. Variation in the amount of moisture
carried by winds every year
Consider the following statements
1. Canyon is an extreme type of Vshaped valley with very steep sides and
no valley floor.
2. Grand Canyon is associated with the
Colorado River.
Which of the statements given above is/are
correct?
a. 1 only
b. 2 only
c. Both 1 and 2
d. Neither 1 nor 2
Which one of the following is correct
sequence of the stratigraphy in India from
oldest to the youngest
a. Deccan Lava - Gondwana sediment Dharwarian sediment - Tertiary folding

36.

37.

38.

39.

40.

5 of 15
b. Gondwana sediment - Deccan Lava Tertiary folding- Dharwarian sediment
c. Dharwarian sediment-Tertiary folding
- Deccan Lava -Gondwana sediment
d. Dharwarian sediment Gondwana
sediment- Deccan Lava Tertiary
Folding
Consider the following statements
1. The coastal area between the Eastern
Ghats and the Bay of Bengal is
narrower than the coastal area between
the Western Ghats and the Arabian
Sea.
2. The Indus rises near Munsarover in
Tibet and finally falls in the Arabian
Sea near Karachi.
Which the statement given above is/are
correct?
a. 1 only
b. 2 only
c. Both 1 and 2
d. Neither 1 nor 2
Which one of the following regions is not
characterized by the Mediterranean type of
climate?
a. South Africa
b. South Western Australia
c. South Western United States of
America
d. North Western Europe
Consider the following countries
1. Brazil
2. Indonesia
3. Iran
4. Pakistan
Which one of the following is the correct
sequence of the above countries in the
descending order of their population size?
a. 1 3 2 4
b. 2 4 3 1
c. 2 1 4 3
d. 3 2 1 4
Which city has the maximum average
annual rainfall among following European
cities?
a. Warsaw
b. Bucharest
c. Athens
d. Belgrade
Which country among the following has
the highest proportion of its electricity
generation by nuclear plants?

41.

42.

43.

44.

a. United States of America


b. United Kingdom
c. France
d. Germany
Consider the following statements in
respect of the United Arab Emirates:
1. United Arab Emirates are surrounded
in the northwest by Oman.
2. United Arab Emirates have highest
percentage of the foreigners in its
population n the world.
3. Dubai is the most populous of the
Emirates.
Which of the statements given above is/are
correct?
a. 1 and 3
b. 2 and 3
c. 2 only
d. None
Which of the following is/are not correctly
matched?
Geographic Division of USA State of the
USA
1. West North Central Iowa
2. Middle Atlantic Maryland
3. South Atlantic New Jersey
4. West South Central Texas
Select the correct answer using the code
given below:
a. 1 and 4
b. 2 and 3
c. 2 only
d. 3 only
Which one of the following countries is
the largest producer-of electricity by wind
power in the world?
a. Germany
b. Australia
c. Norway
d. Switzerland
Consider the following statements
1. As per Census 2001, the sex ratio
(Females per 1000 males) decreased
during 1991-2001 in comparison to
that during 1981-1991 in India.
2. The population of India as recorded at
each decennial census from 1901 has
grown steadily except for a decease
during 1951-61.
Which of the statements given above is/are
correct?

45.

46.

47.

48.

6 of 15
a. 1 only
b. 2 only
c. Both 1 and 2
d. Neither 1 nor 2
As per census 2001, which of the
following pairs are not correctly matched?
Stoic/Union Territory Population
Characteristics
1. Danian and Diu Lowest % of urban
population
2. Pondicherry Sex ratio in excess of
1000
3. Himachal Pradesh Lowest sex ratio
(Female per 1000 males)
4. Arunachal
PradeshLowest
population density
Select the correct answer using the codes
given below:
a. 2 and 4
b. 1 and 3
c. 1 and 4
d. 2 and 3
Which one of the following African
countries is not land-locked?
a. Mali
b. Zambia
c. Chad
d. Libya
Consider tile following statements
1. The population density of Japan is
dose to that of India
2. Crude birth-rate of China is much
lower than that of India.
3. Surface area of India is about five
times that of United Kingdom.
Which of the statements given above are
correct?
a. 1 and 2
b. 2 and 3
c. l and 3
d. l, 2 and 3
Which one of the following statements is
correct?
When contours are placed very close to
each other in a river valley, they represent
a
a. waterfall
b. rapid
c. convex lens
d. concave slope

49.

50.

51.

52.

Who among the following scholars


followed the teleological approach in the
study of the earth?
a. Alexander Von Humboldt
b. Carl Ritter
c. Immanuel Kant
d. Bernard Varenius
Select the correct sequence of the SQI
topographic map Index Numbers which
would lie to the North-East-South and
West of the sheet bearing number 48 I/2.
a. 48 I/1, 48 E/14, 48 I/16, 48 I/3
b. 48 I/6, 48 I/1, 48 E/14, 48 I/3
c. 48 I/1, 48 I/6, 48 I/3, 48 E/14
d. 48 E/14, 48 I/3, 48 I/1, 48 I/6
Match List I (Diagram with List II (Data)
and select the correct answer using the
codes given below
List I
A. Climography
B. Star diagram
C. Dispersion diagram
D. Columnar diagram
List II
1. Anuual rainfall for 40 years
2. Mean rainfall at a station or 12 months
3. Velocity and frequency of wind from
different direction
4. Mean rainfall and mean temperature
for 12 months
A
B
C
D
a.
1
2
4
3
b.
4
3
1
2
c.
4
1
3
2
d.
1
4
2
3
Consider the following axioms regarding
Tidal Hypothesis on origin of the earth:
1. The solar system was formed from the
primitive sun and another intruding
star
2. The intruding star was smaller in size
than the primitive sun
3. The primitive sun was rotating on its
axis
4. The tidal force of intruding star was
greater than the primitive sun
Which of the axioms given above were
presented by Sir James Jeans?
a. 1, 2 and 3
b. 2, 3 and 4
c. 1, 2 and 4

53.

54.

55.

7 of 15
d. 1, 3 and 4
Match List I (Geological Period) with list
TI (Distinctive Life / Incidence) and select
the correct answer using the code given
below the lists:
List I
A. Late Precambrian
B. Carboniferous
C. Devonain
D. Upper Cretaceous
List II
1. Himalayan Orogeny
2. Sponges
3. Man
4. Coal
5. Fish
A
B
C
D
a.
5
4
3
1
b.
2
1
5
4
c.
5
1
3
7
d.
2
4
5
1
Which of the following characteristics are
associated with great masses of
metamorphic rocks generally resulting
from regional metamorphism?
1. Such rocks become wholly ctystaliine.
2. Such
rocks
become
partially
ctystaliinc.
3. A streaking or parallel arrangement of
their constituent crystals which lie with
their longer axes pointing in the same
direction.
Select the correct answer using the code
given below:
a. 1, 2 and 3
b. 2 and 3
c. 1 and 3
d. 1 and 3
Consider the following statements
Vulcanicity refers to
1. all those processes in which molten
rock material or magma rises into the
crust.
2. the greater bulk of the volcanic rocks
of the earths surface were erupted
from volcanoes.
3. the process of solidification of rock
into crystalline or semi crystalline from
molten rock material after being
poured out on the surface.

56.

57.

58.

Which of the statements given above are


correct?
a. 1, 2 and 3
b. 1 and 2
c. 2 and 3
d. 1 and 3
Match List I (Characteristic of Weathering
with List (I (Relationship of Various
Aspects of Weathering) and select the
correct answer using the code given
below:
List I
A. Disintegration of rock without
chemical alteration
B. Rotting and decomposition of
rocks
chemical weathering
C. Process of oxidation
D. Effect of oxidation upon iron
compounds
List II
1. A universal feature in weathering
2. Red, yellow or brown tints on
3. Mechanical weathering
4. Chemical weathering
5. Dominant weathering process of the
humid temperature regions
A
B
C
D
a.
1
3
4
5
b.
3
4
1
2
c.
1
5
2
4
d.
3
4
2
1
Consider the following statements:
An aerial photograph will he considered to
he truely vertical if
1. the sky does not appear in the scene.
2. the camera axis is held perpendicular
to horizon
3. the area covered in the image is
relatively small.
Which of the statements given above arc
correct?
a. 1 and 2
b. 1 and 3
c. only 2
d. only 3
Consider
the
following
landform
characteristics:
1. Stream divided are sharp and ridge-like
resulting in a minimum of inter stream
uplands.

59.

60.

61.

62.

8 of 15
2. Extensive areas are at or near the base
level of erosion
3. There are a few consequent trunk
streams hut few large tributaries.
Numerous short tributaries and gullies
arc extending themselves by head ward
erosion and developing valley systems.
Which one of the following is the orderly
sequence of landform characteristics given
above in an idealized Fluvial Cycle?
a. 1 - 3 - 2
b. 2 - 3 - 1
c. 3 1 - 2
d. 2 1 - 3
Which one of the following provides the
force needed to drive the atmospheric
circulation ?
a. Higher biotic content of the tropical
latitudes and lower biotic content of
the polar latitudes
b. The energy contrasts between high
insolation tropical latitudes and the low
insolation polar latitudes
c. Higher rotational speed of lower
latitudes and lower rotational speed of
higher latitudes
d. Equatorial radius of the earth is longer
than the polar radius
Which one of the following1 statements is
correct? Collision-coalescence
process
of precipitation: is applicable to
a. clouds which extend beyond freezing
level
b. all types of clouds
c. cumulo-nimbus clouds
d. those clouds which do not extend
beyond the freezing level
Which one of the following statements is
correct? The innermost region
of a
hurricane is known
as hurricane-eye
where
a. temperature is abnormally low and
pressure is the lowest
b. sky is clear and temperature is the
lowest
c. temperature is abnormally high and
pressure is the lowest
d. sky is under dense cloud cover and
pressure is the lowest
Consider the following statements :
1. Contour ploughing promotes soil
erosion and the formation of gullies.

9 of 15

63.

64.

65.

2. Gully erosions the removal of top soil


and creation of many steep-sided
cuttings in a hillside.
Which of the statements given above is/are
correct?
a. 1 only
b. 2 only
c. Both 1 and 2
d. Neither 1 nor 2
Match List I (Region of Dispersed Rural)
with List II (Associated Factor) and select
the answer using the code given below the
lists :
List I
A. Alps and Himalayas
B. Delta of Ganga
C. Farm lands of the U.S.A.
D. Clayey soil areas
List II
1. Large areas to be reached
2. Marshy conditions hinder movement
3. Racial tradition of settling
4. Rugged topography divides the land
5. Too many streams divide the land
A
B
C
D
a.
1
2
5
3
b.
4
1
5
2
c.
1
3
4
5
d.
4
5
1
2
Which one of the following is not correct
about the North-eastern United States of
America?
a. The region experienced expansion of
manufacturing industry for which the
population provided all graces of
labour, entrepreneurial experience, vast
amounts of money to invest and very
large market close at hand
b. Clothing manufacture is carried on in
many locations, but is specially
concentrated in the city of Cleveland,
Ohio
c. Photographic equipment manufacture
is dominated by the world's largest
photographic company with its
headquarters a( Rochester, New York
d. Manufacture of aircraft engines and
helicopters is heavily concentrated in
Hartford, Connecticut

66.

67.

68.

Match List I (Major Cities of Brazil) with


List II (Mark on the Map) and select the
correct answer using the code given below
the Lists :
List I
A. Sao Paulo
B. Racife
C. Brasilia
D. Rio de Janeiro
List II
1. X1
2. X2
3. X3
4. X4
A
B
C
D
a.
2
4
3
1
b.
4
1
2
3
c.
2
3
1
4
d.
4
1
3
2
Following four cities are located almost
on 760E longitude:
1. Indore
2. Solapur
3. Ludhiana
4. Jaipur
Which one of the following is the correct
sequential order of the above cities from
North to South if one travels almost
parallel along those longitudes?
a. 4 1 3 2
b. 3 4 1 2
c. 2 3 1 4
d. 3 4 2 1
Mohorovicic discontinuity separates
a. Upper mantle and lower crust
b. Mantle and asthenosphere
c. Inner solid core and outer liquid core
d. Core and Mantle
Kuroshio is a warm ocean current which
runs from
a. Philippines to Japan
b. Indonesia to Philippines
c. Japan to China

10 of 15
69.

70.

71.

72.

d. Sri Lanka to Indonesia


Which of the following properties are
associated with the Conical Map
Projection?
1. The distance between the meridians
decreases towards the pole
2. The standard parallel is correct to the
scale
3. The distance between the parallels
decreases towards the pole
4. The central meridian is a straight line
Select the correct answer using the code
given below:
a. 1 and 4
b. 2 and 3
c. 1, 2 and 3
d. 1, 2 and 4
What is Gomia in Jharkhand known for?
a. Coalfields
b. Manganese mining
c. Fertilizer plant
d. Explosive factory
Which one of the following terms is
credited to Humboldt?
a. Uranography
b. Biogeography
c. Geomorphology
d. Climatology
Which one of the following pairs is not
correctly matched?
a. Hecataeus
:
Ges-periodos
b. Anaximander
:
Gnomon
c. Eratosthenes
:
Pangaea
d. Hipparchus
:
Astrolabe

75.

76.

73.
77.

What is represented by the shaded portion


in the map given above?
a. Black soil
b. Desert soil
c. Grey and brown soil
d. Red and yellow soil
74.

78.

In the map given above, four regions of


natural vegetation are marked as 1, 2, 3
and 4. Which of these are tropical rain
forests?
a. 1, 2 and 3
b. 1 and 3
c. 2, 3 and 4
d. 2 and 4
Which one of the following statements is
not correct?
a. Makeup of the soil accor4ing lo
proportions of sand, silt and clay is
known as soil texture
b. The way in which sand, silt, clay and
humus bond together to form peds is
known as soil structure
c. The soil moisture budget is the balance
of water in the soil
d. A group of soils formed from different
parent rocks and having different
horizones and soil profiles is known as
soil series
What is the approximate average annual
water availability of India?
a. 1150 billion cubic metre (BCM)
b. 1250 billion cubic metre (BCM)
c. 1650billion cubic metre (BCM)
d. 1850 billion cubic metre (BCM)
Which of the following has/have salt
marshes as an ideal place for its/their
growth
1. Halophytes
2. Mesophytes
3. Xerophytes
Select the correct answer using the code
given below
a. 1 only
b. 1 and 2
c. 2 only
d. 1 and 3

79.

80.

81.

82.

Which type of fold is formed at A in the


figure given above?
a. Isoclinal fold
b. Plunge fold
c. Open fold
d. Recumbent fold
What is the Representative Fraction (R.F.),
if the given scale is 211:1 mile?
a. 1 : 31680
b. 1 : 62360
c. 1 : 126720
d. 1 : 63360
Which one of the following cartographic
techniques is suitable for measuring spatial
association among different attributes of
the regional economy?
a. Dot method
b. Pie diagram
c. Bar diagram
d. Chloroplast method
Match List I (Projection) with List II
(Property) and select the correct answer
using the code given below the Lists :
List I
A. Bonne's Projection
B. Gall's Stereographic Projection
C. Cylindrical Equal Area Projection
D. Mercator's Projection
List II
1. Length of the equator is 2R cost 45
2. Pole is a straight line
3. All parallels are standard parallels
4. Length of any parallel is 2R cos
5. The scale along the parallel &
meridian is same at a point
A
B
C
D
a.
3
2
4
1
b.
1
5
2
4
c.
3
1
2
5
d.
1
3
5
2
Consider the following statements :
1. National Highway No. 7 is the longest
among India's National Highways
2. Nagpur, Hyderabad, Vijayawada,
Chennai and Kanyakumari fall on the
National Highway No. 7

11 of 15
3. Gwalior falls on the National Highway
No. 2 and 3
Which of the statements given above is/are
correct?
a. 1 only
b. 1 and 2
c. 2 and 3
d. 1, 2 and 3
83.

84.

85.

86.

87.

88.

X in the figure given above is a river. In


digital database terms, it is an/a
a. Node and denotes a linear feature
b. Arc and denotes a linear feature
c. Polygon and denotes an area! feature
d. Arc and denotes point feature
Which one of the following is not a GIS
software?
a. ARCVIEW
b. OSIRIS
c. ILWIS
d. IMAGE ANALYST
The terms longitude and latitude were first
used by
a. Eratosthenes
b. Anaximander
c. Ptolemy
d. Homer
Which one of the following scholars of
pre-modern period clearly established the
relationship between regional geography
and systematic geography?
a. Cluverius
b. Bushing
c. Buache
d. Varenius
Which one of the following concepts is
associated with Vidal de la Blache?
a. Landscape chorology
b. Mathematical Geography
c. Terrestrial whole
d. Lands haft
Which one of the following is not a
tributary of the Barak River?
a. Jiri

89.

90.

91.

92.

b. Rukni
c. Dhaleswari
d. Pagladiya
What is shading, in the form of lines
showing a hill or slope on a map, termed
as?
a. Spot height
b. Contour
c. Hachure
d. Farm lines
Match List I (Classification of Valleys)
with List II (Characteristic of Valleys) and
select the correct answer using the code
given below the Lists :
List I
A. Consequent Valley
B. Subsequent Valley
C. Insequent Valley
D. Obsequent Valley
List II
1. Draining in a direction opposite to that
of the original consequent valleys
2. Showing no apparent adjustment to
structural control
3. Course shifted from the original
consequent ones to more readily
erosible rocks
4. Course determined by the initial slope
of the land
5. Valleys draining in the direction
opposite to rock dip
A
B
C
D
a.
4
3
2
1
b.
2
1
4
3
c.
4
3
1
2
d.
2
4
5
3
Towards which direction does the
magnetic north pole show a declination
from the geographic North Pole?
a. East
b. West
c. North
d. South
Match List I (Numerical Scale) with List II
(Equivalent Map Distance for J km
Ground Distance) and select the correct
answer using the code given below the
lists :
List I
A. 1: 1000000
B. 1:50000

93.

94.

95.

12 of 15
C. 1:250000
D. 1:25000
List II
1. 40 mm
2. 04 mm
3. 20 mm
4. 01 mm
A
B
C
D
a.
4
3
1
2
b.
3
1
2
4
c.
4
3
2
1
d.
3
1
4
2
Which of the following are associated with
the origin of dispersed settlements in the
U.K. and the U.S.A.?
1. Desire of family privacy
2. Farmer's easy access to fields
3. Law and order in countryside
4. Uneven topography of farm lands
Select the correct answer using the code
given below:
a. 1 and 3
b. 2 and 4
c. 1 and 2
.
d. 3 and 4
Match List I (Climatic Type) with List II
(Region) and select the correct answer
using the code given below :
List I
A. Amw
B. Aw
C. Bshw
D. Cwg
List II
1. Most parts of peninsular plateau
2. Interior parts of peninsula, western
slopes of Aravallis and parts of
Haryana
3. North Indian plains
4. Western coast of India in the south of
Goa
A
B
C
D
a.
4
3
2
1
b.
2
1
4
3
c.
2
3
4
1
d.
4
1
2
3
Match List I (Coal Mining Area) with List
II (Country) and select the correct answer
using the code given below the Lists :
List I
A. Karaganda Basin

96.

97.

98.

B. Pechora Basin
C. Upper Silesia
List II
1. United Kingdom
2. Russia
3. Poland
4. Kazakhastan
A
B
C
a.
4
3
1
b.
4
2
3
c.
1
2
3
d.
1
2
4
Match List I (type of agriculture) with List
II (area Associated with the particular
types of agriculture) and select the correct
answer using the code given below the
lists:
List I
A. shifting
B. Subsistence
C. Commercial
D. Plantation
List II
1. American Prairies
2. Ganga Delta
3. Gardens of Assam
4. Naga Hills
5. Northern Canada
A
B
C
D
a.
4
5
2
3
b.
3
1
4
5
c.
4
2
1
3
d.
3
4
5
2
Tribes belonging to which one of the
following groups earn their livelihood
through food-gathering and hunting?
a. Bushman, Pigmy and Eskimo
b. Massi, Kirghiz and Boro
c. Pigmy, Eskimo and Kirghiz
d. Boro, Bushman and Masai
Consider the following statements:
1. Natives of Andaman Nicobar islands
belong to Negrito races.
2. Triba living in Central India belong to
Mongoloid races.
3. Tribals living in the NorthEast India
belongs to the Negroid races.
4. Indo Aryans belong to the Nordic
races.
Which of the statements given above are
correct?

99.

100.

101.

102.

13 of 15
a. 1 and 2
b. 2 and 3
c. 1 and 4
d. 3 and 4
Match List I (Concept) with List II
(Proponent) and select the correct answer
using the code given below the Lists
List I
A. Determinism
B. Possibilism
C. Neo-Deternhinism
D. Probabi1isrn
List II
1. O.H. K. Spate
2. F. Ratzel
3. Brunhes
4. G Taylor
A
B
C
D
a.
2
3
4
1
b.
4
1
2
3
c.
2
1
4
3
d.
4
3
2
1
Consider the following statements
1. The
environmental
determinism
dominated the writings of early Arab
geographers.
2. Vidal tie La Blache supplemented
classical geographical determinism
With Social Darwinism
Which of the statements given above is/are
correct?
a. 1 only
b. 2 only
c. Both 1 and 2
d. Neither 1 and 2
Which one of the following is the correct
sequence in the evolution of geographical
ideologies
a. DeterminismQuantitative
RevolutionPossibilism Radicalism
b. DeterminismPossibilism

Quantitative Revolution Radicalism


c. PossibilismDetermthism

Radicalism Quantitative Revolution


d. RadicalismPossibilism
DeterminismQuantitative
Revolution
Match List I (Book) with List II (Author)
and select the correct answer using the
code given below the Lists
List I

103.

104.

105.

106.

A. Cosmos
B. Erdkunde
C. Anthropogeography
D. Principles de Geographies Humaine
List II
1. Cad Ritter
2. Alexander Von Humboldt
3. Vidal de Ia Blache
4. Friedrich Ratzel
A
B
C
D
a.
1
2
3
4
b.
2
1
4
3
c.
1
3
4
2
d.
2
1
3
4
The Mangrove vegetation, in India, is most
extensively found in
a. Malabar coast
b. The Sunderbans
c. The Rann of Kachchh
d. Coastal Orissa
Which one of the following regions of
India has the highest percentage of its area
under forest cover?
a. Andaman and Nicobar Islands
b. Arunachal Pradesh
c. Mizoram
d. Jammu and Kashmir
Which one of the following states in India
has the highest percentage of tank
irrigation?
a. Tamil Nadu
b. West Bengal
c. Kerala
d. Karnataka
Consider the following statements
Larger bodies of intrusive rocks called
batholiths are
1. very commonly formed of granites
though not invariably.
2. lens shaped masses of rock occupying
the saddles of anticlines or the keels of
synclines of mountains.
3. large dome shaped masses the sides
whose plunge steeply to unknown
depths.
Which of the statements given above are
correct?
a. 1, 2 and3
b. 1 and 2
c. 2 and 3
d. 2 and 3

107.

108.

109.

14 of 15
Which one of the following statements is
correct?
Monocline folds are normally associated
with
a. faulting
b. distorted beds
c. inverted beds
d. inclined beds
Which of the following pairs are correctly
matched?
Symbol /form
Term used
1.

Delta

2.

Plateau

3.

Spring Line

4.
:
Waterfall
Select the correct answer using the code
given below
a. 1 and3
b. 2, 3 and 4
c. 1 and 4
d. l, 2, 3 and 4
Which of the following pairs are correctly
matched?
Multipurpose Project : State

110.

111.

112.

1. Ukai Project
: Gularat
2. Kukadi Project : Maharashtra
3. Salal Project
: Jammu & Kashmir
Select the correct answer using the codes g
below
a. 1, 2 and 3
b. 1 and 2
c. 2 and 3
d. 1 and 3
Which one of the following is a land of
geysers, hot springs, lava flows and snow
fields?
a. Alaska
b. Iceland
c. Ireland
d. Finland
Tropic of Capricorn passes through which
one of the following countries
a. Botswana
b. Swaziland
c. Zimbabwe
d. Zambia
Amphibious wooden houses, floating on
the canals, are found in

113.

114.

115.

116.

a. Bangkok
b. Phnon, Penh
c. Vientiane
d. Yangoon
Which of the following pairs is not
correctly matched?
a. Ribbon Falls
: California
b. Tugela Falls
:
Natal,
South
Africa
c. Powers court Falls : Ireland
d. Sutherland Falls : Norway
In Central Place Theory, k = 7 settlement
denotes
a. Market town
b. Market-cum-administrative town
c. Administrative town
d. Transport town
Consider the following statements
1. The present Infant Mortality Rate in
India is about 80 per 1000 live births.
2. The Tenth Plan envisages a reduction
in Infant Mortality Rate to 45 per 1000
live births.
3. The Tenth Plan envisages a reduction
in decadel growth rate of the
population between 20012011 to 10
per cent.
Which of the statements given above is/are
correct?
a. 1 and 2
b. 2 only
c. l and 3
d. l, 2 and 3
Match List I (Proponent) with List II
(Criterion for Functional Classification)
and select the correct answer using the
code given below the lists
List II
A. Harris
B. Aurousseau
C. Nelson
D. McKenzie
List II
1. Dominant functions
2. Statistical criterion
3. Ecological consideration
4. Multiplicity and specialization of
activities
A
B
C
D
a.
2
1
3
4
b.
4
2
1
3

117.

118.

119.

120.

15 of 15
c.
2
3
4
1
d.
4
1
2
3
Match List I (Process of Urbanization)
with List II (Stage of Urbanization) and
select the correct answer using the code
given below the Lists:
List I
A. Beginning of urban settlement
B. Town formation
C. Urban maturity
List II
1. Megalopolis
2. Lopolis
3. Polls
A
B
C
4.
2
3
1
5.
1
3
2
6.
3
2
1
7.
1
2
3
The most important agriculture is feature
of Mediterranean agriculture is
a. Rainfed agriculture
b. Mixed cropping
c. Viticulture
d. thy field farming
Match List I (Settlement Pattern) with List
II (Region) and select the correct answer
using the code given below the Lists
List I
A. Circular or semi circular
B. Star like
C. Triangular
D. Block like
List II
1. At the confluence of rivers.
2. In the vicinity of crater lakes or on the
levees of oxbow lake
3. In deserts and semideserts
4. At the convergence or roads
A
B
C
D
a.
1
2
4
3
b.
2
4
1
3
c.
1
2
3
4
d.
2
3
1
4
The dominant rural settlement pattern
along the coastal region of Kerala is
known as
a. Clustered
b. Compact
c. Nucleated
d. Dispersed

C.S.E-Pre 2006

1 of 13

GEOGRAPHY
1.

2.

3.

4.

5.

Consider the following statements:


1. The granite and gneisses give rise to
red soil through weathering.
2. The laterite soil contains the hydrated
oxides of iron and Aluminium.
3. Both the red and laterite soils are poor
in nitrogen but rich in potash and
phosphorus.
Which of the statements given above is
correct?
a. 1 and 2, only
b. 2 only
c. 1 and 3, only
d. 1, 2 and 3
An area which has not been glaciated may
show:
a. Hanging valleys
b. Corries & pyramidal peaks
c. Drumlins
d. V-shaped valleys
Consider the following statements
1. In an overturned fold, the axial plane is
inclined and both limbs dip in the same
direct
2. A recumbent fold is one in which the
axial plane is essentially horizontal
Which of the statements given above is/are
correct?
a. 1 only
b. 2 only
c. Both 1 and 2
d. Neither 1 nor 2
The tropical cyclones do not occur close to
the equator because of :
a. Weak coriolis force
b. Light & variable winds
c. Excessive humidity
d. Convective activity
Among the
following
types
of
ecosystems, the annual average rate of
net plant production is highest in :
a. Temperate grassland
b. Savannah
c. Coniferous forest
d. Temperate forest

6.

7.

8.

9.

10.

Which one of the following rivers has a


cuspate delta?
a. Mississippi
b. Niger
c. Nile
d. Tiber
Which one of the following statements is
not correct?
a. The largest known flood basalt region
in the world is the Deccan trap area of
peninsular India.
b. The largest flood basalt region in the
United States of America is the
Columbia Plateau.
c. The two shield volcanoes Mauna Loa
and Mauna Kea have summit
elevations of more than 6500 m above
sea level.
d. The island of Hawaii includes at least
seven coalesced volcanic domes.
The progressive wave theory regarding the
origin of tides was put forth by :
a. R.A. Harris
b. Isaac Newton
c. William Whewell
d. G.B. Airy
With reference to earthquakes which one
of the following statements is not correct?
a. L waves travel along the surface of the
earth's crust
b. As the P waves propagate, they move
material in a path parallel to the
direction of movement
c. S waves move objects at right angles to
their direction of motion
d. S waves propagate through even liquid
material
Consider the following statements :
1. Youthful topography is characterized
by comparatively a few streams but
usually with low gradients.
2. In mature topography, the main
streams have their valleys cut to base
level and are at grade.
Which of the statements given above is/are
correct?
a. 1 only

11.

12.

13.

14.

b. 2 only
c. Both 1 and 2
d. Neither 1 nor 2
Spits develop due to :
a. Circular movement of water currents
along the bays
b. Formation of long shore drifts towards
the coast
c. Formation of large scale deposits of
fine grained dust
d. Oblique movement of water currents
along the shores
Match List - I with List- II and select the
correct answer using the code given below
the- lists :
List-I (Landforms)
A. Natural Levees
B. Eskar
C. Polje
D. Barchan
List II (Processes)
1. Aeolin
2. Karst
3. Fluvial
4. Glacial/Glaciofluvial
A
B
C
D
a.
1
3
2
4
b.
3
2
4
1
c.
1
2
3
4
d.
3
4
2
1
Consider the following statements
1. General characteristics of westerlies
are largely modified by cyclones and
anticyclones.
2. Westerlies are the most dominant
wind systems in middle latitudes,
3. Westerlies become more vigorous in
the southern hemisphere.
Which of the statements given above is/are
correct?
a. 1 and 2, only
b. 2 only
c. 1 and 3, only
d. 1, 2 and 3
Consider the following statements:
1. Chernogem soils originate under
tall grass prairie vegetation.
2. Laterites form under hot and humid
climatic conditions.
Which of the statements given above is/are
correct?
a. 1 only

15.

16.

17.

18.

19.

2 of 13
b. 2 only
c. Both 1 and 2
d. Neither 1 nor 2
What are Geosynclines?
a. Arched unfolds in the strata of the
earth's crust.
b. Major structural down folds in the
earth's crust.
c. Large scale rises in the earth's crust.
d. Sliding of plate margins inside the
arth's crust.
Pelagic deposits consist of:
a. Terrigenous materials
b. Cosmic materials
c. Inorganic materials
d. Organic ooze materials
Consider the following statements :
1. Chaparral biomes develop under
conditions of high moisture with
less pronounced differences in the
temperature between summer and
winter.
2. Moist temperate coniferous forest
biomes occur along the west coast of
North America from central California
to Alaska.
Which of the statements given above is/are
correct?
a. 1 only
b. 2 only
c. Both 1 and 2
d. Neither 1 nor 2
Consider the following statements :
1. The decrease of temperature with
increasing latitude is more pronounced
in North Atlantic Ocean than the South
Atlantic Ocean.
2. The vertical distribution of temperature
in enclosed seas of higher latitudes
registers inversion of temperature.
Which of the statements given above is/are
correct?
a. 1 only
b. 2 only
c. Both 1 and 2
d. Neither 1 nor 2
Consider the following statements :
1. Montreal Protocol in the year 1987
called for the complete cut in the use
of Chlorofluorocarbons (CFCs) by the
year 2000.

20.

21.

22.

23.

2. India was a signatory to the Montreal


Protocol in the year 1987.
Which of the statements given above is/are
correct?
a. 1 only
b. 2 only
c. Both 1 and 2
d. Neither 1 nor 2
Consider the following statements:
1. The boundary of a National Park is
circumscribed by legislation whereas
that of a Biosphere Reserve is not
circumscribed by legislation.
2. In a National Park, particular attention
is paid to the conservation of gene
pools whereas it is not so in a Wildlife
Sanctuary.
Which of the statements given above is/are
correct?
a. 1 only
b. 2 only
c. Both 1 and 2
d. Neither 1 nor 2
With reference to Koppen's classification
of climatic types, consider the following
statements :
1. The minor/sub type indicated through
small letter f means rains all the year
round.
2. The minor/sub type indicated through
small letters means dry summer.
Which of the statements given above is/are
correct?
a. 1 only
b. 2 only
c. Both 1 and 2
d. Neither 1 nor 2
Consider the following statements :
1. Florida current passes through the
Yucatan channel before entering Gulf
of Mexico.
2. Florida
current
possesses
the
characteristics of the equatorial water
mass.
Which of the statements given above is/are
correct?
a. 1 only
b. 2 only
c. Both 1 and 2
d. Neither 1 nor 2
Collision-coalescence
process
of
precipitation is applicable to:

24.

25.

26.

27.

28.

29.

3 of 13
a. Orrus
b. Clouds that extend beyond freezing
level
c. Clouds that do not extend beyond
freezing level
d. All types of clouds
What is a seismic zone extending at an
angle of about 45 from the base of an
ocean trench, down through lithosphere to
the asthenosphere known as?
a. Appleton layer
b. Benioff zone
c. Conard discontinuity
d. Convergence zone
Which one of the following primitive
people use 'Krall' for their shelter?
a. Pygmy
b. Papuan
c. Bedouins
d. Masai
Consider the following tribes:
1. Masai
2. Kirghiz
3. Boro
Which of the above are
nomadic
herders?
a. 1 and 2, only
b. 2 and 3, only
c. 1 and 3, only
d. 1, 2 and 3
Which one of the following ethnic groups
does not belong to Mongoloids?
a. The Yakuts
b. The Samoyeds
c. The Red Indians of North America
d. The Bantus
Regarding the Zone Strata Theory of
Taylor, which one of the following
statements is not correct?
a. Central Asia has been the cradle land
of human races
b. Oldest human races are farthest away
from the cradle land
c. Recent human races are nearest to the
cradle land
d. Caucasords are the latest human races
Consider the following statements:
1. Khasi society is patrilinear.
2. Pastoralism is the main occupation of
the Tharus.
Which of the statements given above is/are
correct?

a.
b.
c.
d.

1 only
2 only
Both 1 and 2
Neither 1 nor 2

30.

36.

37.

31.

32.

33.

34.

35.

In the given map, which one of the


following types of soil is indicated by the
marked area?
a. Black soil
b. Grey and Brown soil
c. Laterite soil
d. Red and Yellow soil
Consider the following tribes :
1. Abujhmaria
2. Bhotia
3. Gaddi
For which of the above tribes is the
economic activity
mainly
pastoral
herding?
a. 1 and 2, only
b. 2 and 3, only
c. 1 and 3, only
d. 1, 2 and 3
What does Caribou provide?
a. Meat and skin
b. Meat, milk and non- shrinkage wool
c. Meat and fur
d. Meat and blubber for fuel
Where is the Dogger Bank which is a
major fishing area, located?
a. North Pacific Ocean
b. South Pacific Ocean
c. North Sea
d. South Atlantic Ocean
Which one of the following is the speech
area of "Austric Linguistic Family"?
a. Assam, Nagaland and Manipur
b. Bihar, Orissa and West Bengal
c. Tamil Nadu, Karnataka and Kerala
d. Punjab, Himachal Pradesh and
Rajasthan
'Syntaxial Bend' is a feature of which one
of the following?

4 of 13
a. Himalayas
b. Aravallis
c. Satpuras
d. Vindhyas
Where does the ethnic group called Dayak
live?
a. (a) Bali
b. (b) Borneo
c. (c) Tenggara Sunda Islands
d. (d) West Timor
Ratzel's
second
volume
of
Anthropogeography emphasized on :
a. Influence of physical environment on
man
b. Lebensraum
c. Human migration
d. Influence of man on natural
environment

38.

In the given map, four republics of


Russian Federation are marked as 1, 2, 3
and 4. Which one of them is Chechnia?
a. l
b. 2
c. 3
d. 4
39.

In the given map, four areas of sea/gulf


bordering Australia viz.,
A. Arafura Sea,
B. Coral sea,
C. Gulf of Carpentaria and
D. Timor sea

are marked with numbers. Match them and


select the correct answer using the code
given below:
A
B
C
D
a.
2
3
4
1
b.
2
4
3
1
c.
1
3
4
2
d.
1
4
3
2

43.

44.

40.

45.

41.

42.

In the map given here, four Emirates/Cities


viz.,
A. Sharjah,
B. Dubai,
C. Abu Dhabi, and
D. Doha
are marked with numbers. Using the code
given below, match them correctly and
select the correct answer:
A
B
C
D
a.
4
2
3
1
b.
1
3
2
4
c.
1
2
3
4
d.
4
3
2
1
Dead Sea is situated in which one of the
following
a. A Rift Valley
b. An Intermundane Plateau
c. Intermundane Plains
d. Canyons
Consider the following:
1. Strong winds
2. Long winter
3. Cloudy sky
4. Cold dry air
Which
of the above are the ideal
conditions
for the formation of
temperature inversions?
a. 1, 2 and 3
b. 3 and 4
c. 1, 2 and 4
d. 2 and 4, only

46.

47.

5 of 13
Who propounded the concentric circle
theory of city morphology?
a. Burgess
b. Hoyt
c. Ulman
d. Mumford
Which of the following explains the 'Late
Expanding Phase' Of Demographic
Theory?
a. High Birth Rate and High Death Rate
b. High Birth Rate and Declining Death
Rate
c. Declining Birth and Declining Death
Rate
d. Low Birth Rate and Low Death Rate
Consider the following statements:
Immigrants wherever settled
1. are culturally different from their
receivers.
2. are visible minorities.
3. largely fulfill certain functions.
4. are granted full legal rights only in
stages.
Which of the statements given above are
correct?
a. 1 and 2, only
b. 1, 2 and 4, only
c. 3 and 4, only
d. 1, 2, 3 and 4
Among the following States, which one is
the largest producer of rubber?
a. Assam
b. Meghalaya
c. Tripura
d. Sikkim
Match List - I with List - II and select the
correct answer using the code given below
the lists :
List - I (Tribes)
A. Hmar
B. Dimasa
C. Konyak
D. Nishi
List - II (States)
1. Arunachal Pradesh
2. Nagaland
3. Mizoram
4. Assam
5. Sikkim
A
B
C
D
a.
1
2
4
5

48.

49.

50.

51.

b.
3
4
2
1
c.
1
4
2
5
d.
3
2
4
1
Match List - I with List - II and select the
correct answer using the code given below
the lists :
List I (State)
A. Minnesota
B. North Carolina
C. Kentucky
D. West North Central
List - II (Geographic Division of the USA)
1. South Atlantic
2. East South Central
3. West South Central
4. West North Central
A
B
C
D
a.
4
3
2
1
b.
2
1
4
3
c.
4
1
2
3
d.
2
3
4
1
Consider the following places:
1. San Francisco
2. Adelaide
3. Cape Town
4. Sydney
Which of the above have Mediterranean
climate?
a. 1 and 3, only
b. 2 and 4
c. 1, 3 and 4
d. 1, 2 and 3
Which one of the following is the correct
statement?
Kalahari is a desert because it
a. lies in the tropics
b. is on the leeward side of a mountain
c. underlies a high pressure cell
d. receives only land winds
Consider the following :
1. Broad evergreen leaves
2. Shining, waxy leaf surface
3. Epiphytes
Which of the above is/are the salient
characteristics of natural vegetation of
Selves?
a. 1 and 2, only
b. 2 only
c. 1 and 3, only
d. 1, 2 and 3

52.

53.

54.

55.

56.

57.

58.

6 of 13
Which one of the following pairs is not
correctly matched?
a. Chicago : Lake Michigan
b. Cleveland : Lake Erie
c. Detroit
: Lake Superior
d. Toronto : Lake Ontario
Which one of the following is not a part
of the Polynesia region?
a. New Zealand
b. Timor
c. Tonga Islands
d. Tuvalu
In the USA, where is the Silicon Valley
located?
a. Northern end of Salton Sea
b. Northern end of San Francisco Bay
c. Southern end of the Strait of Georgia
d. Southern end of San Francisco Bay
In which one among the following
countries is the per capita arable land
highest?
a. Germany
b. Norway
c. Sweden
d. United Kingdom
Consider the following pairs:
1. St. Petersburg : Gulf of Finland
2. Stockholm
: Baltic Sea
: Norwegian Sea
3. Osic
Which of the pairs
given above is/are
correctly matched?
a. 1 only
b. 1 and 2, only
c. 2 and 3, only
d. 1, 2 and 3
In which one among the following
countries is the urban population as per
cent of its total population highest?
a. Australia
b. Japan
c. New Zealand
d. USA
Match List - I (Area) with List - II
(Mineral) and select the correct answer
using the code given below the Lists :
List - I (Area)
A. Caucasus
B. Sakhalin
C. Magnitogorsk
D. Karaganda
List II (Mineral)

1.
2.
3.
4.

59.

60.

61.

62.

Mineral Oil
Iron ore
Coal
Copper
A
B
C
D
a.
3
2
1
4
b.
3
1
2
4
c.
4
2
1
3
d.
4
1
2
3
Match List - I with List - II and select the
correct answer using the code given below
the lists :
List - I (Islands)
A. Azores
B. Bermuda
C. Canary
D. Reunion
List - II (Terristories of)
1. U.K.
2. France
3. Portugal
4. Spain
A
B
C
D
a.
3
4
1
2
b.
2
1
4
3
c.
2
4
1
3
d.
3
1
4
2
Consider the following statements :
1. Australia has the world's largest
artesian basin.
2. Australia produces nearly one-third
of the wool production of the world.
3. Broken Hill has good reserves of
Diamond deposits.
Which of the statements given above are
correct?
a. 1 and 2, only
b. 2 and 3, only
c. 1 and 3, only
d. 1, 2 and 3
Which one of the following pairs is not
correctly matched?
a. Bandung ; Jawa Island
b. Bangkok : Mekong River
c. Denpasa : Bali Island
d. Mandalay : Irrawady River
Among the following Asian countries,
whose share in the world exports is the
highest?
a. India
b. Malaysia

63.

64.

65.

66.

67.

7 of 13
c. Singapore
d. Thailand
Consider the following statements :
1. China spans five time zones but runs
on a single national time using the
standard meridian of Beijing.
2. Russia uses eight standard time zones.
Which of the statements given above is/are
correct?
a. 1 only
b. 2 only
c. Both 1 and 2
d. Neither 1 nor 2
Where are Mulberry trees grown mainly
for silk production in China?
a. Hwang Ho basin
b. Si Kiang valley
c. Manchurian plain
d. Yangtze delta
Consider the following statements :
1. Most of the coal reserves in China
occur in the southern parts bordering
Vietnam, Laos and Myanmar.
2. The largest share of coal in United
States of America is mined in the
western half of the country.
Which of the statements given above is/are
correct?
a. 1 only
b. 2 only
c. Both 1 and 2
d. Neither 1 nor 2
Consider the following
1. Andes mountains
2. New Zealand
3. Philippines
4. Taiwan
Which of the above is/are part(s) of the
Ring of fire?
a. 1 only
b. 1 and 2, only
c. 2, 3 and 4, only
d. 1, 2, 3 and 4
As per the World Development Indicators
of World Bank, in which one among the
following South Asian countries is the
average annual growth rate of population
during 2005-2015 expected to be highest?
a. Bangladesh
b. India
c. Nepal
d. Pakistan

68.

69.

70.

71.

72.

73.

74.

When does escarpment develop?


a. When a block steps down along a fault
plane
b. When a block moves down
vertically along a fault plane
c. When a block moves horizontally
along a fault plane
d. When a block moves upward along a
fault plane
Which one of the following terms best
defines South Asia as a region?
a. Physiographic
b. Economic
c. Geographic
d. Political
Among the following States of India,
which one has the largest forested area?
a. Andhra Pradesh
b. Assam
c. Gujarat
d. Tamil Nadu
Which one of the following pairs is not
correctly matched?
Types of waterfallNames of waterfalls
a. Caprock fall :
Gersoppa falls
b. Scarp fall :
Dasam falls
c. Knickpoint fall : Dhunwadhar falls
d. Hanging Valley fall : Johan falls
Which one of the following statements is
not correctly matched?
a. Vindhyan system mostly consists
of sandstones, limestones and shale.
b. Gneisses and schist of archaeans are
the oldest rocks in India.
c. Rocks of Gondwana system are of
fluvial or lacustrine origin.
d. Deccan trap formations were due to
volcanic eruption during Oligocene.
Which one of the following pairs is not
correctly matched?
Hydel Power Plant
State
a. Kundah :
Karnataka
b. Rihand
:
Uttar Pradesh
c. Sileru
:
Andhra Pradesh
d. Rangeet :
Sikkim
Consider the following statements:
1. The light sandy red and yellow soils in
Balaghat, Durg and Raipur districts are
of alluvial origin.
2. In Tamil Nadu, the red soils occupy
over two-thirds of its total area.

75.

76.

77.

78.

8 of 13
3. The red soils of Burdwan, Birbhum
and Bankura are derived and
transported from Chotanagpur plateau.
Which of the statements given above is/are
correct?
a. 1 and 2, only
b. 2 only
c. 1 and 3, only
d. 1, 2 and 3
The
National
River
Conservation
Directorate functions under which Union
Ministry?
a. Agriculture Ministry
b. Environment and Forests Ministry
c. Science and Technology Ministry
d. Water Resources Ministry
When a person traverses from Mangalore
(West) to Chennai (East), which one of the
following sequences (from West to East)
of forest types does he observe?
a. Tropical evergreen - Tropical wet
deciduous - Tropical semi-evergreen Tropical dry
b. Tropical wet deciduous - Tropical
evergreen Tropical semi-evergreen Tropical dry
c. Tropical semi-evergreen - Tropical
evergreen Tropical wet deciduous Tropical dry
d. Tropical evergreen - Tropical wet
deciduous Tropical dry - Tropical
semi-evergreen
Consider the following pairs:
River
Merges with
1. Parvati
:
Beas
2. Chandra :
Sutlej
3. Pindar
:
Alaknanda
4. Tons
:
Bhagirathi
Which of the above pairs is/are correctly
matched?
a. 1 and 2, only
b. 3 only
c. 1 and 3, only
d. 1, 2, 3 and 4
If one traverses from West to East in India,
which one of the following is the correct
sequence of coal field deposits (from West
to East) does he observe?
a. Karanpura Singrauli Bokaro
Raniganj
b. Singrauli Karanpura Bokaro
Raniganj

79.

80.

81.

82.

c. Karanpura Singrauli Raniganj


Bokaro
d. Singrauli Karanpura Raniganj
Bokaro
Consider the following statements:
1. In India, Tertiary deposits of coal
occur in Assam, Arunachal Pradesh,
Meghalaya and Nagaland.
2. In India, lignite does not occur
anywhere except in Tamil Nadu and
Rajasthan.
3. In India, metallurgical coal is restricted
to Jharkhand and Andhra Pradesh only
Which of the statements given above is/are
correct?
a. 1 only
b. 2 and 3, only
c. 1 and 3, only
d. 1, 2 and 3
Temperate cyclones are characterized by
which of the following?
a. Fronts
b. Diverging winds
c. Dry winds
d. Hot winds
The following have been proposed as the
climatic regions of India by Koppen:
1. Aw
2. Cwg
3. As
4. E
Which one of the following is the correct
sequence- of the climatic regions given
above as they occur from south to north
India?
a. 3 - 1 - 2 4
b. 2 - 4 - 3 - 1
c. 3 - 4 - 2 - 1
d. 2 - 1 - 3 - 4
Match List - I with List - II and select the
correct answer using the code given below
the lists :
List - I (Institute)
A. Rain Forest Research Institute
B. Tropical Forestry Research Institute
C. Institute of Forest Genetics and Tree
Breeding
D. Institute for Forest Productivity
List - II (Location)
1. Coimbatore
2. Dehradun
3. Jabalpur

83.

84.

85.

86.

9 of 13
4. Jorhat
5. Ranchi
A
B
C
D
a.
4
3
2
1
b.
4
3
1
5
c.
3
4
1
5
d.
3
4
2
1
Which among the following is/are the
major factor/factors responsible for the
monsoon type of climate in India?
1. Location
2. Thermal contrast
3. Upper air circulation
4. Inter-tropical .convergence zone
Select the correct answer using the code
given below:
a. 1 only
b. 2 and 3, only
c. 2, 3 and 4, only
d. 1, 2, 3 and 4
Match List - I with List - II and select the
correct answer using the code given below
the lists:
List - I (Primitive people)
A. Semang
B. Maasai
C. Bushmen
D. Kirghiz
List - II (Native Place)
1. Temperate glassland
2. Savannah grassland
3. Equatorial forests
4. Hot deserts
A
B
C
D
1
4
2
3
a.
b.
3
2
4
1
c.
1
2
4
3
d.
3
4
2
1
Which one of the following patterns of
settlements is most likely to develop along
the convergence of several transport
routes?
a. Circular
b. Linear
c. Rectangular
d. Star-shaped
Other than Kerala, which one of the
following States has already achieved the
replacement level of fertility?
a. Assam
b. Gujarat

87.

88.

89.

c. Karnataka
d. Tamil Nadu
Match List - I with List - II and select the
correct answer using the code given
below:
List-I (Industry)
A. Silk
B. Rail wagon
C. Paper
D. Leather
List-II (Important Producing)
1. Kolar
2. Sirpur
3. Titagarh
4. Chennai
A
B
C
D
a.
1
2
3
4
b.
4
3
2
1
c.
3
2
3
1
d.
1
3
2
4
Match List - I with List - II and select the
correct answer using the code given below
the lists:
List - I (National Highway)
A. NH3
B. NH6
C. NH7
D. NH12
List - II (Passes through)
1. Bhopal
2. Indore
3. Jabafpur
4. Raipur
A
B
C
D
a.
1
3
4
2
b.
2
4
3
1
c.
1
4
3
2
d.
2
3
4
1
Consider the following statements about
the Quick Bird satellite :
1. It operates at an altitude of 950 km.
2. Its sensors collect both multispectral
and panchromatic images concurrently.
Which of the statements given above is/are
correct?
a. 1 only
b. 2 only
c. Both 1 and 2
d. Neither 1 nor 2

90.

91.

92.

93.

94.

10 of 13
Match List - I with List - II and select the
correct answer using the code given below
the lists :
List - I (Mineral Resources)
A. Coal
B. Oil and gas
C. Non-fuel minerals
D. Uranium
List - II (Australian Region)
1. New South Wales
2. Northern territory
3. South Australia
4. Victoria
5. Western Australia
A
B
C
D
a.
2
1
3
5
b.
4
5
2
1
c.
2
5
3
1
d.
4
1
2
5
On an aerial photograph, symbol '+'
denotes which one of the following?
a. Collimating marks
b. Principal points
c. Plumb point
d. Iso-centre
Among the following environmental
pollutants, which one leads to the
occurrence of acid rain?
a. Carbon dioxide
b. Hydrogen peroxide
c. Carbon monoxide
d. Sulphur dioxide
With reference to contours used for
mapping the terrain, consider the
following statements :
1. Contours are horizontal to the dip of
the land and perpendicular to the
direction in which the surface water
runs.
2. Steeper the slope, farther the contour.
3. Contours bend up-stream while
crossing a river.
4. Contours do not show up minor relief
features.
Which of the statements given above are
correct?
a. 1, 2 and 3
b. 1, 3 and 4
c. 2, 3 and 4
d. 1, 2 and 4
While travelling from north to south along
the east coast of China, which one of the

95.

following is the correct sequence of the


given Chinese cities ?
a. Shanghai - Guangzhou - Harbin Beijing
b. Harbin - Beijing - Shanghai Guangzhou
c. Shanghai - Beijing - Harbin Guangzhou
d. Harbin - Guangzhou - Shanghai Beijing
For the purpose of identification of
backward areas in India, which one of the
following is not a measure of
identification?
a. Percentage of Scheduled Caste and
Scheduled Tribe to the total population
of the area
b. Ratio of population to the cultivable
waste land
c. Ratio of urban to rural population
d. Percentage of working force engaged
in agriculture

98.

99.

96.
100.

101.

97.

In the given map, four cities viz., A.


Bordeaux, B. Lyons, C. Marseilles and D.
Paris are marked with numbers. Match
them and select the correct answer using
the code given below:
A
B
C
D
a.
1
2
3
4
b.
1
3
2
4
c.
4
2
3
1
d.
4
3
2
1
Consider the following statements in
respect of formation of landforms in India:
1. Structurally, the Meghalaya Plateau is
an extended part of the Deccan
Plateau.
2. The Valley of Kashmir was formed in
a synclinorium.
Which of the statements given above is/are
correct?
a. 1 only

102.

11 of 13
b. 2 only
c. Both 1 and 2
d. Neither 1 nor 2
Who of the following was the first
scientific geographer who ascertained the
length of equator on sound principles?
a. Herodotus
b. Hecataeus
c. Anaximander
d. Eratosthenes
Consider the following statements :
1. Columbus estimated that Asia must lie
very close to Europe on the west on the
basis of the work of Ptolemy.
2. Strabo gave a correct explanation of
the floods of the Nile, attributing them
to the heavy rains in Ethioipia.
Which of the statements given above is/are
correct?
a. 1 only
b. 2 only
c. Both 1 and 2
d. Neither 1 nor 2
Who among the following was the first to
distinguish between general geography
and special geography and to show the
mutual dependence of these approaches?
a. Varenius
b. Kant
c. Humboldt
d. Strabo
Who among the following rejected the
teleological Idea of final causes and
insisted that explanations must be sought
in what is chronologically antecedent and
in this view, opposed Linnaeus and
Leibniz but supported and amplified the
ideas of Hume and Goethe?
a. Varenius
b. Kant
c. Humboldt
d. Ratzel
Consider the following statements :
1. Humboldt gave the first scientific
description of the relation of altitude,
air temperature, vegetation and
agriculture in tropical mountains.
2. Ritter developed the concept that
continental climates are colder in
winter and warmer in summer than the
climates at places near the oceans at
the same latitude.

103.

104.

105.

106.

Which of the statements given above is/are


correct?
a. 1 only
b. 2 only
c. Both 1 and 2
d. Neither 1 nor 2
Consider the following statements :
1. Ritter saw in all his geographical
studies the evidence of God's plan.
2. All the volumes of the Die Erdkunde
completed by Ritter covered only
Africa and a part of Asia.
3. Like Humboldt, Ritters great work
was largely put together on the basis of
his own observations.
Which of the statements given above are
correct?
a. 1 and 2, only
b. 1 and 3, only
c. 2 and 3, only
d. 1, 2 and 3
Consider the following statements :
1. Ellen Simple accepted the Ritzel's
ideas about the state as an organism
and introduced them to America.
2. Ellen Simple published her first
professional article dealing with the
role of the Appalachians as a barrier in
American history.
Which of the statements given above is/are
correct?
a. 1 only
b. 2 only
c. Both 1 and 2
d. Neither 1 nor 2
Consider the following statements:
Behaviorism refers to
1. spatial behaviour of people.
2. spatial perception.
3. spatial experience and imagination.
4. mental maps.
Which of the statements given above are
correct?
a. 1 and 2
b. 1 and 3
c. 2 and 3
d. 3 and 4
Consider the following statements :
1. Humanistic geography depended on
the growing need of social relevance of
the subject.

107.

108.

109.

110.

111.

112.

12 of 13
2. Humanistic geography developed due
to reaction to excessive quantification
and model building in geography.
3. Humanistic geography drew much
from the radical trend in geography.
Which of the statements given above are
correct?
a. 1 and 2, only
b. 1 and 3, only
c. 2 and 3, only
d. 1, 2 and 3
Consider the following statements :
1. Radicalism in geography is critical of
geography as a spatial science.
2. Radicalism aimed at a general
restructuring of capitalist society.
3. Radicalism in geography relied heavily
on empiricist methods.
Which of the statements given above is/are
correct?
a. 1 only
b. 1 and 2, only
c. 2 and 3, only
d. 1, 2 and 3
The number of a topographic map is
47A/16/NW. Its scale must be :
a. 1 : 250,000
b. 1 : 100,000
c. 1 : 50,000
d. 1 : 25,000
Which one of the following is a special
case of Bonne's projection?
a. Zenithal Polar Equidistant Projection
b. Cylindrical Equidistant Projection
c. Conical Equidistant Projection
d. Sinusoidal Projection
Which one of the following projections is
suitable for large maps of small area fitting
on all sides?
a. Sinusoidal Projection
b. Modified Polyconic Projection
c. Conical Projection with one Standard
Parallel
d. Lambert's Projection
Which one of the following is more useful
to a sailor?
a. Mercator's Projection
b. Gnomonic Projection
c. Sinusoidal Projection
d. Mollweide's Projection
If the focal length (f) of a camera lens is
20 cm and the height of the plane (h) is

113.

114.

115.

1000 metres above ground, what will be


the scale of the air-photographs ?
a. 1 : 500
b. 1 : 5000
c. 1 : 1000
d. 1 : 50000
Which one of the following pairs is not
correctly matched?
Software Organization
a. Arc Info : ESRI
b. GRASS : DST
c. MGE
: Intergraph Corporation
d. IDRISI
: Clark University
Under which of the following conditions
would you not consider a given pair of
aerial photographs as a stereopair?
1. No overlap
2. 25% overlap
3. 60% overlap
4. 100% overlap
Select the correct answer using the code
given 'below:
a. 1 and 2
b. 1 and 3
c. 1 and 4
d. 2 and 4
Geographical Information Systems (GIS)
are designed to manipulate spatial objects.
Which one of the following is not a spatial
object?
a. An attribute data cell
b. A polygon
c. A point
d. A line

Directions: The following FIVE (5) items consist


of two statements, one labelled as the "Assertion
(A)' and the other as 'Reason (R). You are to
examine these two statements carefully and select
the answers to these items using the code given
below:
116.

Assertion (A): The Himalayan Rivers are


older than the mountain ranges they cross.
Reason (R): The Himalayan Rivers cut
their channels deeply and incisively.
a. Both A and R are individually true and
R is the correct explanation of A.
b. Both A and R are individually true but
R is not the correct explanation of A.
c. A is true but R is false
A is false but R is true

117.

118.

119.

120.

13 of 13
Assertion (A): In India, the net sown area
increased by about 19% from the year
1950-51 to 1999-2000.
Reason (R): During the same period, there
had been a concurrent decrease in the area
under forest.
a. Both A and R are individually true and
R is the correct explanation of A.
b. Both A and R are individually true but
R is not the correct explanation of A.
c. A is true but R is false
A is false but R is true
Assertion (A): Cyclones over Bay of
Bengal are characterized by .clockwise
circulation.
Reason (R): Their circulation is governed
by pressure gradient and carioles force.
a. Both A and R are individually true and
R is the correct explanation of A.
b. Both A and R are individually true but
R is not the correct explanation of A.
c. A is true but R is false
A is false but R is true
Assertion (A): Cirrus clouds primarily
contain ice particles.
Reason (R): Cirrus clouds occur at very
light altitudes.
a. Both A and R are individually true and
R is the correct explanation of A.
b. Both A and R are individually true but
R is not the correct explanation of A.
c. A is true but R is false
A is false but R is true
Assertion (A): The per cent of population
whose income is less than one US
dollar/day is less in Pakistan than that in
India.
Reason (R): The adult literacy rate
(percentage of age 15 years and above
with literacy) is more than in Pakistan than
in India.
a. Both A and R are individually true and
R is the correct explanation of A.
b. Both A and R are individually true but
R is not the correct explanation of A.
c. A is true but R is false
d. A is false but R is true

C.S.E-Pre 2007

1 of 16

GEOGRAPHY
1.

2.

3.

4.

Consider the following statements:


1. The Benguela current had its influence
in the formation of Kalahari Desert.
2. The Agulhas current had its influence
in the formation of Thar Desert.
3. The currents of Northern Indian Ocean
change their course of flow twice a
year.
Which of the statements given above is/are
correct?
a. 1 only
b. 2 and 3 only
c. 1 and 3 only
d. 1, 2 and 3
Consider the following statements:
As per Census 2001
1. except in some dstricts of Kerla, there
is no district in other States where
females outnumbered males.
2. the sex ratio in Orissa and Manipur in
less than that of the national average.
Which of the statements given above is/are
correct?
a. 1 only
b. 2 only
c. Both 1 and 2
d. Neither 1 nor 2
If a truck has to carry goods from Delhi to
Bhopal via. Gwalior, by National
highways (NH) following the shortest
route, which one of the following groups
of national Highways it has to go by?
a. NH2, NH11, NH12
b. NH2, NH3, NH12
c. NH8, NH3, NH11
d. NH2, NH25, NH11
In which one of the following projections
is the equator twice the length of the
central meridian?
a. Gnomonic projection
b. Mercator projection
c. Mollweide projection
d. Zenithal orthographic projection

5.

6.

7.

8.

9.

Consider the following statements:


1. The Rourkela plant was commissioned
in the year 1959.
2. The Rourkela plant was constructed
with the collaboration of UK.
3. The Rourkela plant gets iron ore from
Sundargarh.
Which of the statements given above are
correct?
a. 1 and 2 only
b. 1 and 3 only
c. 2 and 3 only
d. 1, 2, and 3
Consider the following statements:
1. In India, groundnut crop is raised
mostly during the Kharif season under
rain fed conditions.
2. In India, tobacco is cultivated only in
black cotton soils.
Which of the statements given above is/are
correct?
a. 1 only
b. 2 only
c. Both 1 and 2
d. Neither 1 nor 2
Which one among the following pairs of
States is the leading producer of tabocco in
India?
a. Andhra Pradesh and Gujarat
b. Karnataka and Madhya Pradesh
c. Maharashtra and Chhattisgarh
d. Uttar Pradesh and Rajasthan
The uranium mines being developed by
UCIL at which of the following States?
a. Jharkhand and Arunachal Pradesh
b. West Bengal and Assam
c. Andhra Pradesh and Meghalaya
d. Maharashtra and Sikkim
Consider the following Statements:
1. Screw pines (Pandanus tectorius) are
commonly found in montane wet
temperate forests.

2 of 16

10.

11.

12.

13.

2. Sal (Shorea robusta) in one of the tree


found in the sub-Himalayan tract from
Kangra district in Himachal Pradesh to
the Darrang & Nowgong districts of
Assam.
Which of the statements given above is/are
correct?
a. 1 only
b. 2 only
c. Both 1 and 2
d. Neither 1 nor 2
Match List I with List II and select the
correct answer using the code given below
the lists:
List I (Major Irrigation Project)
A. Malaparabha
B. Lower Bhavani
C. Salandi
D. Vamsadhara
List II (State)
1. Andhra Pradesh
2. Karnataka
3. Orissa
4. Tamil Nadu
A
B
C
D
a.
2
3
4
1
b.
2
4
3
1
c.
1
4
3
2
d.
1
3
4
2
Kurnool-Cuddapah canals are taken off
from which one of the following rivers?
a. Cauvery
b. Penneru
c. Palar
d. Tungabhadra
Which one of the following is not a
landform which results due to rejuvenation
of a river?
a. Incised meander
b. Knick point
c. Oxbow lake
d. River terrace
Match List-I with List-II and select the
correct answer by using the code given
below the lists:
List-I (Mineral)
A. Mica

B. Diamond
C. Manganese
D. Copper
List-II (Place)
1. Chindwara
2. Bellary
3. Panna
4. Hazaribag
Code:
a. A-1, B-2, C-3, D-4
b. A-1, B-3, C-2, D-4
c. A-4, B-3, C-2, D-1
d. A-4, B-2, C-3, D-1
14.

Consider the following statements :


1. In the wet temperate forests at above
15(30 m on the Nilgiris and
Anaimalais, the Magnolias and
Rhododendrons are commonly found.
2. Tropical dry evergreen forests are
mostly found in Andhra Pradesh,
Chhattisgarh and Orissa.
Which of the statements given above is/are
correct?
a. 1 only
b. 2 only
c. Both 1 and 2
d. Neither 1 nor 2

15.

Match List-I with List-II and select the


correct answer by using the code given
below the lists :
List-I (Soil)
A. Red and loamy soils
B. Calcareous sierozemic soil
C. Grey and brown soils
D. Laterite soil
List-II (Region)
1. Nagaland, Arunachal Pradesh and the
Sahyadri
2. Gujarat and adjacent areas
3. Punjab, Haryana and Malwa
4. Western parts of the western Ghats
Code:

3 of 16
a. A cylindrical projection with two
standard parallels is particularly useful
for representing the low-latitude zone
straddling the equator between the pair
of standard parallels

a. A-1, B-3, C-2, D-4


b. A-1, B-2, C-3, D-4
c. A-3, B-1, C-4, D-2
d. A-3, B-4, C-1, D-2
16.

Taking into account the amount of rainfall


occurring from June to September, which
one of the following is the correct order of
cities in terms of rainfall?

b. In a Mercator projection, the spacing


of parallels decreases towards the poles
c. A straight line on planar projection is
the shortest route between two points
on earth's surface

a. Kolkata > Ahmedabad > Allahabad


b. Kolkata > Allahabad > Ahmedabad

d. On a conic projection, meridians are


shown as straight lines that converge
towards the poles

c. Allahabad > Kolkata > Ahmedabad


d. Ahmedabad > Kolkata > Allahabad
17.

Which type of climate (s) prevail(s) in the


long corridor (Leeward side) of the
Western Ghats and the Nilgiri Hills?

a. 0.63-0.69 micrometer
b. 0.76-0.90 micrometre

b. Tropical wet and dry with winter rain

c. 2.08-2.35 micrometre

c. Tropical semi-arid steppe

d. 10.40-12.50 micrometre
23.

1. The sea water temperature of Red Sea


is higher than that of Persian Gulf.

a. Wall maps

2. The Baltic Sea water is more saline


than that of Black Sea.

c. Small-scale maps

Which of the statements given above is/are


correct?

d. Medium-scale maps
A star diagram is used to represent which
of the following?

a. 1 only

a. Duration of winds only

c. Both 1 and 2
d. Neither 1 nor 2
Assertion (A) : For the onset of wet
summer monsoons of India, the south-east
Trade Winds over Indian Ocean cross the
equator and recurve into a south westerly
flow.
Reason (R): The North-Easterly surface
winds of Siberian High blowing towards
south Asia cause the deflection of these
winds.
Assertion (A): Vertisols are most
extensive in Australia, India and Sudan.
Reason (R): The clay-producing materials
are available in mesothermal or tropical
climates with periodic dry and wet
seasons.

b. Flows between central places and


dependent places

b. 2 only

24.

c. Wind direction and duration


d. Direction of winds only
20.

Construction of a comfort diagram


requires which of the following data?
a. Monthly temperature and rainfall
b. Monthly temperature
humidity

and

relative
25.

c. Monthly relative humidity and rainfall


d. Monthly temperature,
relative humidity
21.

Consider the following statements :

Diagonal scales are generally used for


which of the following?
b. Cadastral maps

19.

What is the thermal band region of Land


sat Thematic Mapper Sensor?

a. Tropical wet and dry climate

d. Sub-tropical monsoon rainforest


18.

22.

rainfall and

Which one of the following statements is


not correct?

Assertion (A): Hurricanes do not originate


in South Atlantic Ocean and South-Eastern
Pacific Ocean.
Reason (R): In South Atlantic and SouthEastern Pacific Oceans, the Intertropical
Convergence Zone seldom occurs.
Codes:
a. Both A and R are individually true and
R is the correct explanation of A
b. Both A and R are individually true but
R is not the correct explanation of A

4 of 16
Reason (R): The Gondwana coal is
laminated bituminous coal, characterized
by high volatile and ash contents.
a. Both A and R are individually true and
R is die correct explanation of A
b. Both A and R are individually true but
R is not the correct explanation of A
c. A is true but R is false
d. A is false but R is true
Assertion (A): Black cotton soils are ideal
for dry farming.
Reason (R): Black cotton soils are fine
grained.
a. Both A and R are individually true and
R is die correct explanation of A
b. Both A and R are individually true but
R is not the correct explanation of A
c. A is true but R is false
d. A is false but R is true
Assertion (A): Greenland and Antarctica
regions are permanent centres of thermal
anti-cyclones.
Reason (R): The high pressure is the
common feature of Greenland and
Antarctica because of the higher latitudes
and free radiation
Codes:
a. Both A and R are individually true and
R is the correct explanation of A
b. Both A and R are individually true but
R is not the correct explanation of A

c. A is true but R is false

c. A is true but R is false

d. A is false but R is true

d. A is false but R is true

a. Both A and R are individually true and


R is the correct explanation of A
b. Both A and R are individually true but
R is not the correct explanation of A
c. A is true but R is false
d. A is false but R is true
26.

Assertion (A) : After carbon, nitrogen is


the most abundant element in the
biosphere.
Reason (R) : Nitrogen is essential for
synthesizing the living matter in nature.
Codes :

30.

a. Both A and R are individually true and

R is the correct explanation of A


b. Both A and R are individually true but

R is not the correct explanation of A


c. A is true but R is false
d. A is false but R is true

27.

28.

29.

Assertion (A): Laterite soils are red in


colour and coarse in texture.
Reason (R): Abundant phosphoric acid
and silica give these soils, die particular
colour and texture.
Codes:
a. Both A and R are individually true and
R is die correct explanation of A
b. Both A and R are individually true but
R is not the correct explanation of A
c. A is true but R is false
d. A is false but R is true
Assertion (A): In Gondwana coal, the
carbon percentage is rarely above 30.

31.

32.

Which one of the following deserts is


famous for its deposits of nitrates?
a. Atacama
b. Kalahari
c. Gobi
d. Sahara

33.

Who, among the following, propounded


the law of Primate City?
a. Carl Sauer
b. Isaiah Bowman
c. Mark Jefferson
d. Walter Christaller

5 of 16
34.

Which one of die following pairs is not


correcdy matched?

a. QuebecMontrealOttawa
Toronto

a. Ol Doinyo Lengai : Northern Tanzania

b. MontrealQuebecOttawa
Toronto

b. Mount Egmont

: Hawaii

c. Volcan Poas

: Costa Rica

c. QuebecMontrealToronto
Ottawa

d. Mount Klyuchevskaya : Kamchatka


Peninsula
35.

36.

Which one of the following types of


volcanic eruptions is not usually
explosive?

d. MontrealQuebecToronto
Ottawa
39.

a. Pelean

a. Gulf of Aqaba

: Jordan

b. Hawaiian

b. Gulf of Aden

: Yemen

c. Strombolian

c. Ligurian Sea

: Greece

d. Vulcanian

d. Cardigan Bay

: United Kingdom

Niagara Falls is located between which of


the following pairs of lakes?

40.

b. Lake Huron and Lake Erie


c. Lake Erie and Lake Ontario

a. convergence

d. Lake Ontario and Lake Huron

b. divergence

Match List-I with List-II and select the


correct answer by using the code given
below the lists:
List-I (Region/City)

Which one of the following is the correct


statement?
The boundary zone between the North
American plate and the Eurasian plate
exhibits conditions of

a. Lake Superior and Lake Huron

37.

Which one of the following pairs is not


correctly matched?

c. sinking
d. sliding
41.

Consider the following :

A. West Virginia

1. Nebka

B. Colorado

2. Lunette

C. Milwaukee

3. Draa

D. New Orleans

4. Seif

List-II (Corresponding Region)

Which of the above are sand-dune forms?

1. Appalachian Mountains

a. 1 and 2 only

2. Lake Michigan

b. 2 and 3 only

3. Gulf of Mexico

c. 1, 3 and 4 only

4. Rocky Mountains

d. 1, 2, 3 and 4

Code:

42.

a. A-1, B-4, C-2, D-3


b. A-1, B-2, C-4, D-3
c. A-3, B-4, C-2, D-1
d. A-3, B-2, C-4, D-1
38.

From east to west, which one of the


following sequences of cities of Canada is
correct ?

Consider the map of India given above.


The shaded area in the above outline map

of India represents which one of the


following climatic types as proposed by
C.W. Thornthwaite?
a. CA'w

6 of 16
d. Alpine
orogenyHercynian
orogenyCaledonian orogeny
45.

c. EA'd

Which one of the following colours used


to show the cultivated area in Survey of
India's topographical maps of 1:50000
scale?

d. CB'w

a. Light green

b. DB'd

b. Brown

43.

c. Yellow
d. Dark green
46.

List-I (Lake)

In the map given above, four cities of Java


are marked with numbers (1), ((2), (3), and
(4). Consider the above map and match
List-I with List-II, and select the correct
answer using the code given below the
lists :

A. Great Bear Lake


B. Lake Winnipeg
C. Reindeer Lake
D. Lake Melville
List-II (Province/Territory)

List-I (Number in the Map)

1. Manitoba

A. (1)

2. Saskatchewan

B. (2)

3. New foundland and Labrador

C. (3)

4. North-west Territories

D. (4)

Code:

List-II (Name of City)

a. A-2, B-1, C-4, D-3

1. Bandung

b. A-4, B-3, C-2, D-1

2. Semara

c. A-2, B-3, C-4, D-1

3. Surabaya
4. Yogyakarta
Code:

d. A-4, B-1, C-2, D-3


47.

2. Vosges mountains

b. A-3, B-2, C-4, D-1

3. Eastern highlands of Madagascar

c. A-1, B-2, C-4, D-3

Which of the above are Block Mountains?

d. A-3, B-4, C-2, D-1

a. 1 and 2 only

Which one of the following is the correct


order of the geological events on the earth
from ancient to more recent times ?
a. Hercynian
orogeny
Caledonian
orogeny Alpine orogeny
b. Caledonian
orogenyHercynian
orogenyAlpine orogeny
c. Hercynian
orogenyAlpine
orogenyCaledonian orogeny

Consider the following :


1. Harz mountains

a. A-1, B-4, C-2, D-3

44.

Match List-I with List-II and select the


correct answer by using the code given
below the lists:

b. 2 and 3 only
c. 1 and 3 only
d. 1, 2 and 3
48.

Consider the following statements :


1. Intrusive igneous rocks tend to have
larger mineral crystals than extrusive
igneous rocks.

7 of 16
c. Both 1 and 2

2. Vast majority of sedimentary rocks are


neoclassic.
3. Shale is the finest grained clastic
sedimentary rock.

d. Neither 1 nor 2
52.

1. Mauna Loa is an active shield volcano.

Which of the statements given above is/


are correct?

2. Cinder Cones are common in the areas


of East Africa.

a. 1 only
c. 1 and 3 only

3. The Ngorongoro Crater inTanzania is a


caldera. Which of the statements given
above are correct?

d. 1, 2 and 3

a. 1 and 2 only

Consider the following statements :

b. 2 and 3 only

1. Equatorial counter current is more


strongly developed in Pacific Ocean
than in Atlantic Ocean.

c. 1 and 3 only

b. 2 and 3 only

49.

2. Equatorial counter current


in westward direction.

d. 1, 2 and 3
53.

flows

a. Aleutian Range
c. Cascade Range

a. 1 only

d. Mohave Desert

b. Alaska Range

54.

c. Both 1 and 2
d. Neither 1 nor 2

51.

Which one of the following has the


volcanic Mount St. Helens?

Which of the statements given above is/are


correct?
b. 2 only

50.

Consider the following statements :

Match List-I with List-H and select the


correct answer by using the code given
below the lists:

San Andreas fault passes through which of


the following?

List-I (Weathering)

a. Beaufort Sea and Rocky Mountains

B. Maximum mechanical weathering

b. Pacific Ocean, north of San Francisco


and Gulf of California

C. Maximum chemical weathering

A. Maximum frost action

c. Bering Strait and Alaska Range

D. Seasonal
weathering

d. Mackenzie Mountains and Western


Cordillera

List-II (Affected Morphogenetic Region)

Consider the following statements :

2. Tropical wet-dry (Savanna)

1. The
temperature
and
salinity
differences, that trigger thermohaline
circulation of deep-sea, are generated
at the ocean surface in the low-latitude
wind belts.

3. Arid

a. 1 only
b. 2 only

chemical

1. Humid tropics

4. Periglacial
Code:
a. A-4, B-3, C-1, D-2
b. A-1, B-2, C-4, D-3

2. The deep-sea currents of Indian Ocean


are generated in the water girdling
Antarctica.
Which of the statements given above is/are
correct?

maximum

c. A-4, B-2, C-1, D-3


d. A-1, B-3, C-4, D-2
55.

When river is overloaded with fine


sediments and river water is lighter than
the seawater, which type of delta is
formed?

8 of 16

56.

a. Arcuate delta

a. 1 only

b. Bird-foot delta

b. 1 and 2 only

c. Truncated delta

c. 2 and 3 only

d. Estuarine delta

d. 1, 2 and 3

Consider the following statements :

60.

1. The acid lava usually melts at a


relatively higher temperature as
compared to basic (basaltic) lava.

1. Ptolemy indicated on his map that the


Indian Ocean was enclosed by land in
the south.

2. The acid lava solidifies into glass-like


sheets and the basaltic lava solidifies
into rough surfaces.

2. Al-Masudi described the evaporation


of moisture from water surfaces and
the condensation of the moisture in the
form of clouds.

Which of the statements given above is/are


correct?

57.

a. 1 only

Which of the statements given above is/are


correct?

b. 2 only

a. 1 only

c. Both 1 and 2

b. 2 only

d. Neither 1 nor 2

c. Both 1 and 2

Which one of the following is the correct


sequence of the Union Territories in terms
of population density from the lowest to
the highest value?

d. Neither 1 nor 2
61.

b. LakshadweepPondicherry
(Puducherry) Chandig arhDelhi

b. Al-Masudi
c. Al-Idrisi
d. Al-Biruni
62.

c. Pondicherry
(Puducherry)
LakshadweepChandi garh Delhi

Consider the following statements :


1. Al-Baruni first presented the idea that
the Southern Hemisphere was mostly
an open ocean and that most of the
world's land area was in the Northern
Hemisphere.

d. LakshadweepPondicherr
(Puducherry) Delhi Chandigarh
In remote sensing, the strong 'chlorophyll
absorption band' lies in the wavelength
bands of which one of the following
ranges?

2. Ibn-Batuta quoted in his writings that


Hindus believed that the tides were
caused by the moon.

a. 0.45-0.67 urn

59.

Which medieval geographer made


extensive corrections on Ptolemy's works?
a. da Vinci

a. ChandigarhLakshadweepDelhi
Pondicherry (Puducherry)

58.

Consider the following statements :

b. 0.30-0.38 urn

Which of the statements given above is/are


correct?

c. 0.12-0.28 jxm

a. 1 only

d. 0.72-1.30 urn

b. 2 only

Consider the following :

c. Both 1 and 2

1. The Dead Sea

d. Neither 1 nor 2

2. The Red Sea


3. Midland Plain of Scotland
Which of the above is/are Rift Valley/Rift
Valleys?

63.

Which one of the following is the correct


statement?
Varenius's 'special geography was
a. systematic geography

9 of 16

64.

65.

b. regional geography

b. E. C. Semple

c. an empirical idea proved by experience

c. W.M. Davis

d. a teleological concept

d. F. Ratzel

Who, among the following geographers


adopted the 'deductive method' of study in
geography?

68.

a. Varenius

a. Canary Island

b. Cluverius

b. Alexandria

c. Kant

c. Rome

d. Humboldt

d. Syene (Aswan)

Consider the following statements :

69.

1. Ritter made use of the regional


approach to geography rather than the
systematic study of individual features.
2. De LA Blache presented an effective
refutation of the idea of environmental
determinism.

b. satellite images
c. block diagrams
d. projections
70.

Which of the statements given above is/are


correct?

Which one of the following terms


describes not only the physical space
occupied by an organism, but also its
functional role in the community of
organisms?
a. Ecotone

a. 1 and 2 only

b. Ecological niche

b. 2 and 3 only

c. Habitat

c. 1 only

d. Home range

d. 2 only

71.

Consider the following statements :


1. Simple was very careful to make the
point that the environment controls
human action.

Under normal conditions, which one of the


following is the correct sequential
development of features made by fluvial
action?
a. WaterfallsMeander
terracesOxbow lakes

2. Huntington posited climate, 'the,


quality of people' and culture as a
triadic causation of human progress.

barsRiver

b. Meander
barsRiver
WaterfallsOxbow lakes

terraces

Which of the statements given above is/are


correct?

c. WaterfallsRiver terracesMeander
barsOxbow lakes

a. 1 only

d. River
terracesMeander
WaterfallsOxbow lakes

b. 2 only
c. Both 1 and 2
d. Neither 1 nor 2
67.

The concept of Vanishing Point' is used in


the context of
a. aerial photographs

3. From Ratzel's second volume of


Anthropogeographie,
Huntington
formulated the concept of possibilism.

66.

For preparing the world map, Eratosthenes


used prime meridians through which one
of the following?

Who related the rise of civilization in the


mid-latitudes and lack of development in
the tropics to climatic conditions?
a. E. Huntington

72.

bars

Consider the following statements :


1. The acid lava usually melts at a
relatively higher temperature as
compared to basic (basaltic) lava.

Which of the statements given above is/are


correct?

10 of 16
Hemisphere, whereas in Southern
Hemisphere, the outgoing radiation
exceeds the incoming radiation.
Which of the statements given above is/are
correct?

a. 1 only

a. 1 only

b. 2 only

b. 2 only

c. Both 1 and 2

c. Both 1 and 2

d. Neither 1 nor 2

d. Neither 1 nor 2

2. The acid lava solidifies into glass-like


sheets and the basaltic lava solidifies
into rough surfaces.

73.

74.

1. Outwash deposits

Match List-I with List-II and select the


correct answer by using the code given
below the lists:

2. Yardang

List-I (Climate)

3. Striation

A. Savanna climate

4. Sandbank

B. Steppe climate

Select the correct answer using the code


given below:

C. Humid sub-tropical climate

Code:
a. 2 and 3 only

List-II
(Code
Classification)

b. 1, 3 and 4 only

1. Aw

c. 1 and 3 only

2. Bs

d. 2 and 4 only

3. Cs

Consider the following statements :

4. Cfa

1. Most rainfall and snowfall in the midlatitudes are formed by the ice-crystal
process.

Code:

2. The base of cirrus clouds invariably


occurs at an altitude below 5000 m.

c. A-3, B-2, C-4, D-1

Which of the following distinct features


are found over the glaciated topography?

Which of the statements given above is/are


correct?
a. 1 only
b. 2 only
c. Both 1 and 2
d. Neither 1 nor 2
75.

Consider the following statements :


1. The variation in average annual range
of temperature at 15 latitude of
Southern Hemisphere is much less than
that at 15 latitude of Northern
Hemisphere.
2. At latitudes poleward of 35, the
incoming radiation exceeds the
outgoing radiation in Northern

76.

D. Dry sub-tropical climate


in

Koppen's

a. A-1, B-2, C-4, D-3


b. A-1, B-4, C-2, D-3
d. A-3, B-4, C-2, D-1
77.

Consider the following statements :


1. Pearl millet (Pennisetum typhoides)
can under hot-humid conditions only.
2. Pigeon pea (Cajanus cajari) can be
cultivated both under hot humid and
hot dry conditions.
Which of the statements given above is/are
correct?
a. 1 only
b. 2 only
c. Both 1 and 2
d. Neither 1 nor 2

11 of 16
78.

79.

Where is Mt. Waialeale, one of the


heaviest rainfed places in the world,
located (11990 mm in a year)?

B. 540

a. Brazil

List-II (City)

b. Hawaii

1. Durban (South Africa)

c. Philippines

2. Marseille (France)

d. Sulawesi

3. Punta Arenas (Chile)

Consider the following statements :

4. Tokyo (Japan)

1. In the tropical rain forest of Brazil, the


average diurnal range of temperature is
much greater than the average annual
range of temperature.

Code:

2. In the humid sub-tropical climate of


Kyushu island of Japan, the rainfall for
the year almost totals that of the
rainfall in tropical rain forest.

C. 1040
D. 1560

a. A-4, B-2, C-1, D-3


b. A-4, B-1, C-2, D-3
c. A-3, B-2, C-1, D-4
d. A-3, B-1, C-2, D-4
83.

Which of the statements given above is/are


correct?

80.

1. The ocean currents

a. 1 only

2. The temperature and salinity

b. 2 only

3. The depth of ocean waters

c. Both 1 and 2

4. Length of day and night

d. Neither 1 nor 2

Select the correct answer by using the code


given below:

What is the proportion of landmass of the


ocean floor plotted against the given
datum line called?

a. 1, 2 and 3 only
b. 1 and 3 only

a. Altimetric curve

c. 2 and 4 only

b. Hypsographic curve

d. 1, 2, 3 and 4
84.

c. Hygrometric curve
d. Hydrometric curve
81.

The term 'truncated spur' is associated with


which one of the following?

Which one of the following is the correct


statement?

a. Aeolian process

Granite of plutonic
essentially of

c. Glacial process

origin

b. Fluvial process

consists

d. Weathering process

a. shale, conglomerate and sandstone

82.

Which of the following determine the


amount of phytoplanktons in the ocean
waters?

b. quartz, schist and phyllite

Which one of the following pairs neither is


nor correctly matched?

c. biotite, hornblende and augite

Tributary

d. quartz, feldspar and mica

a. Kinnarasani

Match List-I with List-II and select the


correct answer by using the code given
below the lists:

b. Amaravati

List-I (Approximate
precipitation in mm)
A. 400

mean

annual

85.

c. Malaprabha
d. Pranhita
86.

: River
: Godavari
: Krishna
: Krishna
: Godavari

Match List-I with List-II and select the


correct answer by using the code given
below the lists:

12 of 16
a. Flood plain

List-I (Name of Confluence)


A. Vishnu Prayag

b. Peneplain

B. Kama Prayag

c. Pediment

C. Rudra Prayag

d. Pediplain
89.

D. Deva Prayag
List-II
(River
Alaknanda)

merging

with

a. Gondwana

1. Bhagirathi

b. Dharwar

2. Mandakini

c. Vindhya

3. Pindar

d. Tertiary

4. Dhauli Ganga

90.

Code:
a. A-1, B-2, C-3, D-4
b. A-1, B-3, C-2, D-4
c. A-4, B-2, C-3, D-1
d. A-4, B-3, C-2, D-1
87.

Which system of rocks in India produces


manganese?

Match List-I with List-II and select the


correct answer by using the code given
below the Lists :

a. Andosols
b. Histosols
c. Mollisols

List-I (Valley)
A. Dzukou Valley

Which one of the following soils is


characterized by a very dark surface
horizon of great thickness (25 to 100 cm),
with a high proportion of calcium among
the exchangeable cations and forms mainly
under grasslands in climates with a marked
seasonal moisture deficiency?

d. Ultisols

B. Yumthang Valley

91.

The Damodar-Subarnarekha system is an


example of which one of the following?

C. Neora Valley

a. Superimposed drainage pattern

D. Sangla Valley

b. Dendritic drainage pattern

List-II (State)

c. Radial drainage pattern

1. Himachal Pradesh
2. Sikkim

d. Rectangular drainage pattern


92.

Code:

Which one of the following pairs is


associated with the shortest distance
between mainlands of India and Sri
Lanka?

a. A-1, B-2, C-4, D-3

a. Jaffna

b. A-1, B-4, C-2, D-3

b. Talaimannar : Dhanushkodi

c. A-3, B-2, C-4, D-1

c. Colombo

d. A-3, B-4, C-2, D-1

d. Talaimannar : Tuticorin

3. Nagaland
4. West Bengal

93.

88.

: Vedaranniyam
: Kanyakumari

Northern part of Australia is typified by


which one of the following?
a. Equatorial climate
b. Mediterranean climate
c. Temperate climate
d. Monsoon climate

What is the name of landform marked by


Y in the above diagram?

13 of 16
94.

Match List-I with List-II and select the


correct answer by using the code given
below the lists:

c. Germany
d. Zambia
97.

List-I (Term)
A. Hanging wall

Match List-I with List-II and select the


correct answer using the code given below
the lists:
List-I (City in USA/Canada)

B. Stone polygon
C. Slump

A. Phoenix

D. Wind gap

B. Detroit
C. Montreal

List-II (Associated with)

D. Vancouver

1. Per glacial region

List-II (Prominent Industry)

2. River capture
4. Fault

1. Chemical, iron and steel, and car


manufacturing industries

Code:

2. Electronics and high-tech industry

a. A-2, B-3, C-1, D-4

3. Food processing and publishing, shipbuilding and timber processing

3. Mass movement

b. A-4, B-1, C-3, D-2

4. Brewing,
chemicals,
processing

c. A-2, B-1, C-3, D-4


d. A-4, B-3, C-1, D-2
95.

car
finance

manufacturing,
and
timber

Code:
a. A-2, B-4, C-1, D-3
b. A-2, B-1, C-4, D-3
c. A-3, B-4, C-1, D-2
d. A-3, B-1, C-4, D-2
98.

In the map given above, four cities, viz.,


A. Birmingham, B. Bristol, C. Liverpool
and D. Manchester are marked with
numbers. Using the code given below,
match them correctly and select the correct
answer:
Code:
a. A-3, B-4, C-1, D-2
b. A-4, B-3, C-2, D-1
c. A-3, B-4, C-2, D-1
d. A-4, B-3, C-1, D-2
96.

Among the following, which one has the


largest
number
of
land-border
neighbouring countries?
a. Brazil
b. Russia

In the map given above, four cities of USA


are marked with numbers (1), (2), (3), and
(4). Consider the above map and match
List-I with List-II, and select the correct
answer by using the code given below the
lists :
List-I (Number in the Map)
A. (1)
B. (2)

14 of 16
c. The ability of organisms in a food
chain to convert energy received into
living matter

C. (3)
D. (4)
List-II (City)

d. Population explosion of species in an


ecosystem due to the removal of their
predators

1. Boston
2. New York
3. Philadelphia
4. Washington DC

Boulder clay is a feature associated with


which one of the following?

Code:

a. Mechanical weathering

a. A-1, B-2, C-3, D-4

b. Mass movement

b. A-1, B-3, C-2, D-4

c. River deposition

c. A-4, B-2, C-3, D-1

d. Glacial deposition

d. A-4, B-3, C-2, D-1


99.

100.

101.

103.

Which one of the following countries has


Red River delta?

What is the maximum potential number of


inhabitants which can be supported in a
given ecosystem known as?

a. Japan

a. Biotic potential

b. Indonesia

b. Carrying capacity

c. Malaysia

c. Reproductive capacity

d. Vietnam

d. Tolerance limit

The Hkakabo Razi (5885 m), the highest


point in mainland South-East Asia, is at
the border between which of the following
countries?

104.

105.

List-1 (Tribe)

a. China and Myanmar

A. Aka

b. China and Vietnam

B. Bondo Poraja

c. Cambodia and Laos

C. Riang

d. Cambodia and Thailand

D. Gaddi

Among the following types of biomess,


which one has the highest species
diversity?

List-II (State)
1. Arunachal Pradesh
2. Himachal Pradesh

a. Temperate forests

3. Orissa

b. Tropical rainforests

4. Tripura

c. Temperate deciduous forests

Code:

d. Temperate grasslands
102.

Match List-I and List-H and select the


correct answer by using the code given
below the lists:

a. A-1, B-3, C-4, D-2

What is eutrophication?
a. Process
by
which
organisms
concentrate
certain
chemical
substances to levels above those found
in their natural environment
b. Excessive growth of algae in nutrientrich water leading to reduced oxygen
levels and the death of many
organisms

b. A-2, B-4, C-3, D-1


c. A-1, B-4, C-3, D-2
d. A-2, B-3, C-4, D-1
106.

Match List-I with List-II and select the


correct answer by using the code given
below the lists:
List-I (Natural Region)
A. Tundra Region

15 of 16
B. Equatorial Forest Region

a. North America

C. Steppe Region

b. South America

D. Savanna Region

c. Australia
d. Europe

List-II (Tribe)
1. Kirghiz

111.

1. A hurricane acquires its spin from the


Coriolis effect,

2. Samoyed
3. Semang

2. The diameter of the hurricane


decreases as it moves away from low
latitudes.

4. Bedouin
5. Masai
Code:

3. The diameter of a hurricane is never


below 150 km.

a. A-2, B-1, C-4, D-5


b. A-5, B-3, C-1, D-2

107.

c. A-2, B-3, C-1, D-5

Which of the statements given above is/are


correct?

d. A-5, B-1, C-4, D-2

a. 1 only

The Minamata incident, an example for


environmental degradation, is due to
which one of the following?

b. 2 and 3 only

a. Air pollution
b. Nuclear disaster

c. 1 and 3 only
d. 1, 2 and 3
112.

c. Water pollution
d. Thermal pollution
108.

109.

Match List-I with List-II and select the


correct answer by using the code given
below the lists:
List-I (District)

During Pleistocene, epoch, four glacial


phases were identified. Which one of the
following is the correct chronological
order in which they appeared during this
epoch from early times?

A. Almora

a. GunzMindelWurmRiss

1. Gypsum

b. GunzMindelRissWurm

2. Kyanite

c. WurmRissMindelGunz

3. Magnesite

d. MindelGunzRissWurm

Code:

Which one of the following climates is


found in coastal areas of California in
North America, Central Chile in South
America and South-West tip of West
Australia?

a. A-3, B-1, C-2

a. Tropical savanna
b. Mediterranean
c. Humid continental
d. Low-latitude steppe
110.

Consider the following statements :

Among the following continents, which


one has the lowest growth rate of
population?

B. Bikaner
C. Singhbhum
List-II (Rich source of)

b. A-3, B-2, C-1


c. A-2, B-1, C-3
d. A-1, B-3, C-2
113.

Match List-I with List-II and select the


correct answer by using the code given
below the lists:
List-I (Important
Plantation)
A. Mauritius
B. Formosa
C. South-Eastern Brazil

Regions

of

16 of 16
List-I (Mineral)

D. Coastal Plains of Caribbean Islands

A. Iron ore

List-II (Names of Plantation)


1. Coffee

B. Copper

2. Rubber

C. Aluminium

3. Sugarcane

D. Nickel
List-II (Producing Area)

4. Tea

114.

Code:

1. Arkansas

a. A-2, B-4, C-1, D-3

2. Cuba

b. A-2, B-1, C-4, D-3

3. Kazakhastan

c. A-3, B-4, C-1, D-2

4. Krivoy Rog

d. A-3, B-1, C-4, D-2

Code:

Among the following countries, which one


has the largest concentration of Railways?

a. A-1, B-2, C-4, D-3

a. South Africa

c. A-4, B-2, C-1, D-3

b. Egypt

d. A-4, B-3, C-1, D-2

c. Algeria

b. A-1, B-3, C-4, D-2

118.

d. Sudan
115.

Among the following, which country is


reliant on a single export of oil/petroleum?

a. Silurian

a. South Africa

c. Triassic

b. Zimbabwe

d. Ordovician

c. Namibia

b. Devonian

119.

d. Nigeria
116.

Consider the following statements :

2. The migration of Equatorial Low


(ITCZ) into die Southern Hemisphere
in January is
far less pronounced
than its migration into the Northern
Hemisphere in July.
Which of the statements given above is/are
correct
a. 1 only
b. 2 only
c. Both 1 and 2
d. Neither 1 nor 2
Match List-I with List-II and select the
correct answer by using the code given
below the lists:

Consider the following statements :


1. Carajas region of Brazil is an area of
iron ore.
2. Pilbara region of Australia is an area of
copper ore.

1. The sub-tropical jet stream is evident


throughout the year but the polar front
jet stream is strong only during the
half-year centred on winter.

117.

In which geological period did the shallow


sea reptile Ichthyosaurus first appear?

Which of the statements given above is/are


correct?
a. 1 only
b. 2 only
c. Both 1 and 2
d. Neither 1 nor 2
120.

Which one of the following countries


produces the largest amount of crude steel
of the world?
a. Japan
b. USA
c. South Korea
d. China

C.S.E. Pre. 2008

1 of 13

GEOGRAPHY
1.

2.

3.

4.

5.

Match List-I with List-II and select the


correct answer using the code given below
the lists:
List-I
(Description)
A. Each parallel is projected as standard
parallel
B. Sine curves are used as meridians
C. 900 N or S latitude is not shown
D. North and South poles are lines
List-II
(Projection)
1. Polyconic projection
2. Sinusoidal projection
3. Mercator projection
4. Galls
stereographic
cylindrical
projection
Code:
a. A4, B3, C2, D1
b. A1, B2, C3, D4
c. A4, B2, C3, D1
d. A1, B3, C2, D4
If the tilt angle than three degrees, the air
photograph is known as
a. Oblique photograph
b. Vertical photograph
c. Ortho photograph
d. Trimetrogon photograph
Who of the following devoted himself to
the study of the modes of life of Germans
living outside Germany ?
a. Ratzel
b. Mackinder
c. Jafferson
d. Sauer
What is a line drawn through a series of
points which have been equally elevated
(or depressed) by relative movements of
land and sea, termed as ?
a. Isobase
b. Isorhythm
c. Isogram
d. Isonoetic line
Consider the following statements The
central themes of Vidal de La Blache are,
that

6.

7.

8.

9.

1. nature sets limits and offers


possibilities for development.
2. mans adjustment to nature is the
function of traditions and mental
structuring.
Which of the statements given above is/are
correct?
a. 1 only
b. 2 only
c. Both 1 and 2
d. Neither 1 nor 2
Regarding von Humboldt, consider the
following statements:
1. He assembled materials to show how
environments varied, with differences in agricultural practices, reflecting
interaction among altitude, temperature
and vegetation.
2. He developed synthetic reasoning
leading to inductive generalization.
Which of the statements given above is/are
correct?
a. 1 only
b. 2 only
c. Both 1 and 2
d. Neither 1 nor
Who of the following authored the book
Influences of Geographic Environment?
a. Huntington
b. La Blache
c. Ratzel
d. Semple
Consider the following statements:
1. Ritters scientific geography was based
on the concept of unity in diversity.
2. Ritters concepts regarding the
meaning of the observed geographical
patterns on earth were teleological.
Which of the statements given above is/are
correct?
a. 1 only
b. 2 only
c. Both 1 and 2
d. Neither 1 nor 2
Which one of the following statements is
correct?

10.

11.

12.

13.

14.

a. Digita1 Raster Graphics (DRG) s a


scanned image of a USGS topographic
map
b. DRG is a hard copy of a resource map
of India
c. DRG is a survey map of NATMO
d. DRG is a cadastral map
Who of the following named the human
dominance theory of man-environment
re1atjonshipas Possibilism
a. Lucien Febvre
b. Lucien Gallois
c. Vidal de La Blache
d. Jean Brunhes
Who of the following is the author of
Almagest and Guide to Geography,
two great books of Roman period?
a. Strabo
b. Plini
c. Ptolemy
d. Pomponious
Who of the following geographers stated
that the Earth is a it inseparable organic
whole?
a. von Humboldt
b. Ritter
c. Richthofen
d. Ratzel
Match List-I with List-II and select the
correct answer using the code given below
the lists
List-I
(Landform)
A. Nickpoint
B. Sink hole
C. Point bar
D. Accordant summit
List -II
(Process)
1. Karst
2. Fluvial
3. Plantation
4. Rejuvenation
Code:
a. A3, B2, C1, D4
b. A4, B1, C2, D3
c. A3, B1, C2, D4
d. A4, B2, C1, D3
Assertion (A): Laterite soils are not very
productive for crop plants.

15.

16.

17.

18.

2 of 13
Reason (R): Laterite soils contain
high concentrations of silica and calcium
carbonate.
a. Both A and R are individually true and
R is the correct explanation of A
b. Both A and R are individually true but
R is not the correct explanation of A
c. A is true but R is false
d. A is false but R is true
Assertion (A): Large deserts are located at
low latitudes.
Reason (R): Continental drift has moved
them to lower latitudes from higher
latitudes.
a. Both A and R are individually true and
R is the correct explanation of A
b. Both A and R are individually true but
R is not the correct explanation of A
c. A is true but R is false
d. A is false but R is true
Assertion (A): In the Mediterranean
climate, a pronounced winter precipitation
is observed.
Reason (R): The Mediterranean regime is
controlled by sub-tropical anticyclone in
winter.
a. Both A and R are individually true and
R is the correct explanation of A
b. Both A and R are individually true but
R is not the correct explanation of A
c. A is true but R is false
d. A is false but R is true
Assertion (A): The westerlies of the
southern hemisphere are stronger and more
constant in direction than those of the
northern hemisphere.
Reason (R) : The broad expanses of ocean
in southern hemisphere rule out the
development of, stationary pressure
systems.
a. Both A and R are individually true and
R is the correct explanation of A
b. Both A and R are individually true but
R is not the correct explanation of A
c. A is true but R is false
d. A is false but R is true
Assertion (A): In aerial and satellite
remote sensing and photography, the
spectral bands of green and near infra-red
radiation are used.
Reason (R): The vegetation reflects green
and near infra-red radiation which appear

19.

20.

21.

22.

on the air photographs and imagery as


green and red tones.
a. Both A and R are individually true and
R is the correct explanation of A
b. Both A and R are individually true but
R is not the correct explanation of A
c. A is true but R is false
d. A is false but R is true
Assertion (A): Species diversity is quite
low in tropical Savannah in comparison to
the adjacent tropical rain forests.
Reason (R): The fire factor plays a role in
controlling the species diversity in
Savannah.
a. Both A and R are individually true and
R is the correct explanation of A
b. Both A and R are individually true but
R is not the correct explanation of A
c. A is true but R is false
d. A is false but R is true
Assertion (A): The circumpolar current
encircles Antarctica from west to east.
Reason (R): The westerly winds in
southern hemisphere blow constantly from
west to east and do not
encounter any major landmasses.
a. Both A and R are individually true and
R is the correct explanation of A
b. Both A and R are individually true but
R is not the correct explanation of A
c. A is true but R is false
d. A is false but R is true
In spherical diagram, the radii of spheres
are proportionate to which one of the
following ?
a. Square root of the quantities
b. Cube root of the quantities
c. Percentage of the quantities
d. Actual quantities
With reference to Molkweide projection,
consider the following statements
1. Mollweide projection is elliptical in
shape.
2. The parallels are straight lines.
Which of the statements given above is/are
correct?
a. 1 only
b. 2 only
c. Both 1 and 2
d. Neither 1 nor 2

23.

24.

25.

26.

3 of 13
With reference to the satellites
launched by India, which one of the
following is the correct chronological
order ?
a. Bhaskara II, IRS-lA, Cartosat, INSAT1B
b. Bhaskara II, INSAT-1B, IRS1A,
Cartosat
c. IRS- 1A, IN SAT-1 B, Bhaskara II,
Cartosat
d. INSAT-1B, Bhaskara II, Cartosat, IRSIA
Consider the following statements:
1. An Albers equal-area projection,
distortion zones are arranged parallel
to the standard lines.
2. Albers equal-area projection is good
for a middle-latitude area of greater
east-west than north-south extent.
Which of the statements given above is/are
correct?
a. 1 only
b. 2 only
c. Both 1 and 2
d. Neither 1 nor 2
Consider the following statements:
1. In Mercator projection, all the rhumbs
appear as straight lines.
2. Mercators projection enlarges areas at
a rapidly decreasing rate towards the
higher latitudes.
Which of the statements given above is/are
correct?
a. 1 only
b. 2 only
c. Both 1 and 2
d. Neither 1 nor 2
Consider the following statements:
1. In India natural rubber is produced in
Southern India and Andaman and
Nicobar Islands only.
2. Among the coffee growing States of
India, the lowest average yield per
hectare of plucked coffee is in Kerala.
Which of the statements given above is/are
correct?
a. 1 only
b. 2 only
c. 1 and 2
d. Neither 1 nor 2

27.

28.

29.

30.

31.

32.

33.

How is the rectilinear drainage pattern


formed where two sets of structural
controls occur at right angles, termed ?
a. Rectangular
b. Radial
c. Dendritic
d. Trellis
In Thornthwaites scheme of classification
of climate, what does E indicate in the
context of India ?
a. Semi-arid climate
b. Humid climate
c. Arid climate
d. Dry sub-humid climate
Which one of the following pairs of cities
have the largest annual average rainfall
difference as compared to the other three
pairs ?
a. Jabalpur and Nagpur
b. Mumbai and Pune
c. Kolkata and Bhubaneshwar
d. Guwahati and Shillong
As per census 2001, what is the
approximate percentage of population of
the five notified minority communities
together in the total population of India?
a. 16.2%
b. 17.3%
c. 18.4%
d. 19.5%
Consider the following statements:
1. Nokrek Biosphere Reserve contains
tropical wet evergreen forest type of
vegetation.
2. The river Pindar flows through
Nandadevi Biosphere Reserve.
The river Pindar flows through Nandadevi
Biosphere Reserve.
a. 1 only
b. 2 only
c. Both 1 and 2
d. Neither 1 nor 2
Who of the following advocated the idea
that the state is an organism attached to
the land?
a. Ratzel
b. Ritter
c. Kant
d. Humboldt
In the context of India, in the 1: 50,000
scale toposheet, how many meters are
represented by 1 cm ?

34.

4 of 13
a. 1000 m
b. 500m
c. 250 m
d. 100 m
Among the following States, which one
has the largest net irrigated area ?
a. Andhra Pradesh
b. Maharashtra
c. Gujarat
d. Tamil Nadu

35.

36.

37.

38.

In the map given above, four cities


namely, A. Adelaide, B. Brisbane, C.
Melbourne and D. Sydney are marked with
numbers. Match the city with the correct
number and select the correct answer using
the code given below
Code:
a. A1, B2, C4, D3
b. A1, B4, C2, D3
c. A3, B2, C4, D1
d. A3, B4, C2, D1
Which one of the following pairs is
correctly matched ?
a. Gulf of Martaban : Yangon
b. Gulf of Tongking : Shanghai
c. Gulf of Mannar : Trincomalee
d. Gulf of Aqaba : Cairo
Among the following South Asian
countries, in which one is the Maternal
Mortality Ratio (per 100,000 live births)
lowest ?
a. Bangladesh
b. India
c. Nepal
d. Pakistan
In Brazil, iron ore is found chiefly in
which one of the following Federal Units ?
a. Amazonas
b. Maranhao
c. Minas Gerais
d. Rio Grande do Norte

39.

40.

41.

42.

43.

Consider the following rivers:


1. Don
2. Syr Darya
3. Volga
4. Ural
Which of the above rivers flow into the
landlocked Caspian Sea?
a. 1 and 2
b. 2 and 3
c. 3 and 4
d. 1 and 4
In which one of the following countries is
Barail Range located?
a. Afghanistan
b. Sri Lanka
c. Pakistan
d. India
As per Koppens classification of climate,
in which of the following lo the regions of
Central Mahashtra and Telangana fall
a. Tropical Wet and Dry climate
(Monsoon Savannah)
b. Tropical Wet and Dry climate with
Winter Rain (Monsoon Savannah with
Winter Rain)
c. Tropical Semi-Arid Steppe
d. Humid Sub-Tropical with Dry Winter
Which one of the following groups of
States and major producers of Sunflower?
a. Tamil Nadu, Haryana and Rajasthan
b. Kerala, Orissa and Uttar Pradesh
c. Madhya
Pradesh,
Punjab
and
Chhattisgarh
d. Karnataka, Maharashtra and Andhra
Pradesh
With reference to north-east monsoon
(winter monsoon) in India, consider the
following statements
1. Unlike the summer monsoon, there is
no easterly jet stream over the
peninsula during the winter monsoon:
2. More than two-thirds of annual rainfall
in Coimbatore, Salem and Nilgiri
districts occurs from October to
December.
Which of the statements given above is/are
correct?
a. 1 only
b. 2 only
c. Both 1 and 2
d. Neither 1 nor 2

44.

45.

46.

47.

5 of 13
With reference to Indian forests,
consider the following pairs:
1. Tropical Moist Deciduous Forests :
Sandalwood (Santalum album)
2. Tropical Dry Deciduous Forests : Sal
(Shorea robusta)
3. Tropical Thom Forests : Shisham
(Dalbeigia sissoo)
Which of the pairs given above is/are
correctly matched?
a. 1 only
b. 1 and 2 only
c. 2 and 3 only
d. 1, 2 and 3
Consider the following areas of forests:
1. Andaman and Nicobar Islands forests
2. Nilambur tract of Kerala
3. Low outcrop of Garo and Khasi hills
4. Mikir hills upto 1000 meters
Which of the above areas have tropical wet
evergreen forests?
a. 1 and 2 only
b. 2 and 3 only
c. 1, 3 and 4 only
d. 1, 2, 3 and 4
Consider the following statements:
1. As per Census 2001, the population
growth rate of Tamil Nadu during
1991-2001 was the lowest among the
States in India.
2. As per Census 2001, the population
growth rate of Nagaland during 19912001 was the highest among the States
in India.
Which of the statements given above is/are
correct?
a. 1 only
b. 2 only
c. Both 1 and 2
d. Neither 1 nor 2

48.

49.

50.

51.

52.

In the map given above, four cities of


Germany namely, A. Berlin, B. Cologne,
C. Frankfurt and D. Munich are marked
with numbers. Match the city with the
correct number and select the correct
answer using the code given below
Code:
a. A1, B2, C3, D4
b. A1, B3, C2, D4
c. A4, B2, C3, D1
d. A4, B3, C2, D1
On which one of the following rivers are
Tikarpara and Naraj dams constructed
under a multipurpose project.
a. Damodar
b. Kosi
c. Mahanadi
d. Sutlej
Which one of the following pairs is not
correctly matched ?
Irrigation project / District
a. Bargi project : Jabalpur
b. Kadana project : Panchmahals
c. Pochampad project Guntur
d. Malaprabha project Belgaum
Consider the following statements:
1. Cherrapunji is located on the south
side of Garo hills.
2. Cherrapunji is about 1700 m above
mean sea level.
Which of the statements given above is/are
correct?
a. 1 only
b. 2 only
c. Both 1 and 2
d. Neither 1 nor 2
Which one among the following rivers has
the largest catchment area ?
a. Cauvery
b. Krishna
c. Mahanadi
d. Narmada
Consider the following:
1. Jammu hills
2. Mikir hills
3. Zaskar range
On which of the above is annual rainfall
generally above 100 cm?
a. 1 and 2 only
b. 2 only
c. 1 and 3 only
d. 1, 2 and 3

53.

54.

55.

56.

57.

6 of 13
Among the following elements,
which one is in the highest amount in the
composition of basalt rock?
a. Aluminium
b. Calcium
c. Iron
d. Silicon
Consider the following pairs
Mineral / Composition
1. Amphibole : Calcium-magnesium-iron
silicate
2. Mica : Magnesium-iron-aluminium
silicate
3. Quartz: Silicon dioxide
Which of the above pair/pairs is/are
correctly matched?
a. 1 and 2 only
b. 1 and 3 only
c. 3 only
d. 1, 2 and 3
Consider the following statements:
1. The Mid-Atlantic Ridge surfaces
above the sea level in Iceland.
2. The San Andreas fault is a transform
fault.
Which of the statements given above is/are
correct?
a. 1 only
b. 2 only
c. Both 1 and 2
d. Neither 1 nor 2
Consider the following statements:
1. When feldspar undergoes chemical
weathering, kaolinite is produced.
2. Water is essential for chemical
weathering of feldspar.
Which of the statements given above is/are
correct?
a. 1 only
b. 2 only
c. Both 1 and 2
d. Neither 1 nor 2
With reference to seismic waves, consider
the following statements:
1. P-waves travel through a solid rock
faster than sound waves travelling
through air.
2. S-waves travel at about half the speed
of P-waves.
Which of the statements given above is/are
correct?
a. 1 only

58.

59.

60.

61.

62.

63.

b. 2 only
c. Both 1 and 2
d. Neither 1 nor 2
Tonle Sap Lake, one of the worlds largest
fresh water fisheries, is replenished by
which one of the following rivers ?
a. Yangtze
b. Irrawaddy
c. Mekong
d. Saiween
Consider the following statements:
1. Glacial valley floors are flat and their
walls steep, in contrast to the V-shaped
valleys of many mountain rivers.
2. Unlike rivers; valley glaciers at
coastlines may erode their valley floors
far deeper than sea level.
Which of the statements given above is/are
correct?
a. 1 only
b. 2 only
c. Both 1 and 2
d. Neither 1 nor 2
Consider the following statements
1. Nearly 99 percent of the total mass of
the atmosphere lies within 30 km of
the Earths surface.
2. The proportion of ozone in the
atmosphere increases to a maximum at
about 60 km from Earths surface.
Which of the statements given above is/are
correct?
a. 1 only
b. 2 only
c. Both 1 and 2
d. Neither I nor 2
The albedc of which one among the
following is the highest?
a. Cirrus cloud
b. Cumulonimbus cloud
c. Melting snow
d. Sand
An observer in the northern hemisphere
who stand s with his hack to the wind will
have lower pressure to his left and higher
pressure to his right. This is known by
which one of the following ?
a. Bergeron-Findeisen theory
b. Buys Ballots law
c. Walker circulation
d. Stokes law
Consider the following statements :

64.

65.

66.

7 of 13
1. The Jet Streams are located near
the altitude of the tropopause.
2. The Polar Front Jet Stream is evident
throughout the year, but the subtropical
Jet Stream is strong only during the
half-year centered on winter.
Which of the statements given above is/are
correct?
a. 1 only
b. 2 only
c. Both 1 and 2
d. Neither 1 nor 2
Consider the following phenomena :
1. Shifting of the sub-tropical high
pressure belt towards the equator.
2. Lower sea-surface temperatures in the
eastern North Atlantic.
3. Expansion of the circumpolar westerly
vortex,
Which of the above is/are attributed to the
prolonged drought in the Sahel region of
Africa?
a. 1 only
b. 2 and 3 only
c. 1 and 3 only
d. l, 2 and 3
Consider the following statements:
1. Convective type of precipitation is
associated with towering cumulus and
cumulonimbus clouds.
2. In tropical. cyclones, cumulonimbus
cells become organised around the
centre in spiralling bands.
Which of the statements given above is/are
correct?
a. 1 only
b. 2 only
c. Both 1 and 2
d. Neither 1 nor 2
Consider the following statements:
1. Tropical cyclones occur only in
oceanic areas where the sea
temperature exceeds 3 3C.
2. In order to achieve the rotational
component, the tropical cyclone must
be spawned at least 5N or 5S of the
equator.
Which of the statements given above is/are
correct?
a. 1 only
b. 2 only
c. Both 1 and 2

67.

68.

d. Neither 1 nor 2
In which one of the following regions is
the maximum amount of worlds
phosphate produced ?
a. Northern Africa
b. Northern Europe
c. North America
d. Northern Australia
What is the formula for calculating the
cephalic index ?

73.

a.
b.
c.

69.

70.

71.

72.

d.
Consider the following areas
1. Central Africa
2. Borneo and Papua-New Guinea
3. Amazon basin
In which of the above areas is the
primitive agriculture such s. shifting
cultivation or rotational bush-fallow type
of cultivation found?
a. 1 only
b. 1 and 2 only
c. 2 and 3 only
d. 1, 2 and 3
Among the following countries, which one
is the most densely populated?
a. Belgium
b. France
c. Germany
d. The Netherlands
Consider the following statements:
1. Costa Rica is a leading exporter of
cocoa.
2. Malaysia is a leading exporter of
rubber.
Which of the statements given above is/are
correct?
a. 1 only
b. 2 only
c. Both 1 and 2
d. Neither 1 nor 2
Consider the following pairs:
Nomadic tribes / Region
1. San : Kalahari desert

74.

75.

8 of 13
2. Samoyed : Siberia
3. Wodaabe : North-Eastern Asia
4. Yakut : Lena basin
Which of the pairs given above are
correctly matched ?
a. 1 and 2 only
b. 2 and 4 only
c. 1, 2 and 4
d. 1, 3 and 4
Match List-I with List-II and select the
correct answer using the code given below
the lists
List-I
(Tribe)
A. Chaimal
B. Dafla
C. Dharua
D. Garasia
List-II
(State)
1. Arunachal Pradesh
2. Orissa
3. Rajasthan
4. Tripura
Code:
a. A3, B1, C2, D4
b. A3, B2, C1, D4
c. A4, B2, C1, D3
d. A4, B1, C2, D3
In which one of the following countries do
the ethnic communities called Karen,
Kachin and Chin live?
a. Australia
b. Indonesia
c. Myanmar
d. Sri Lanka
Match List-I with List-II and select the
correct answer using the code given below
the lists:
List-I
(Tribe)
A. Semang
B. Beduin
C. Masai
D. Kirghiz
List-II
(Habitat)
1. Hot desert
2. Equatorial belt
3. Temperate grassland
4. Savannah grassland

76.

77.

78.

Code:
a. A1, B2, C3, D4
b. A1, B2, C4, D3
c. A2, B1, C4, D3
d. A2, B1, C3, D4
Among the following countries, which one
is a major producer of Wolfram (tungsten
ore) ?
a. Australia
b. China
c. Indonesia
d. Russia
Ravenstein and Zelinsky are well known
for their work in the context of which one
of the following model /theories ?
a. Locational theories
b. Nearest neighbour analysis
c. Simulation models
d. Migration
Among the following countries, which one
has the highest total fertility rate?
a. India
b. Pakistan
c. Nepal
d. Bangladesh

81.

82.

79.

83.

80.

In the map given above, which one of the


following types of climates based on
Koppens system is indicated by the
shaded portion ?
a. Aw
b. Caf/Cfa
c. Csa
d. Daf
Consider the following climate data:
Mean
Precipitation
Temp.
(mm)
(C)
D
10
104
J
9
102
F
10
88
M
12
68
A
13
33
M
15
12

84.

9 of 13
June
16
3
July
17
1
A
17
I
S
18
5
0
16
19
N
13
40
The above climate data resembles that of
which one of the following cities?
a. Buenos Aires
b. Tokyo
c. Melbourne
d. San Francisco
Which one of the following States in India
has the broadest continental shelf?
a. Andhra Pradesh
b. Gujarat
c. Karnataka
d. Tamil Nadu
Which one of the following sequences of
ocean trenches is correct in terms of the
ascending order of their depths?
a. Aleutian TrenchMindanao Trench
New Herbides Trench
b. New Herbides TrenchMindanao
TrenchAleutian Trench
c. Aleutian
TrenchNew
Herbides
TrenchMindanao Trench
d. New
Herbides
Trench--Aleutian
TrenchMindanao Trench
Consider the following statements:
1. The Walker circulation weakens
during El Nino events.
2. El Nino events often result in, fooding
in California.
3. During El Nino years, there fewer
hurricanes in the Atlantic.
Which of the statements given above is/are
correct?
a. 1 only
b. 2 and 3 only
c. 1 and 3 only
d. l, 2 and 3
Consider the following statements:
1. Neap tides occur when the Moon,
Earth and Sun are collinear.
2. When an intense storm passes near the
shore during a spring tide, the waves at
high tide may cause tidal waves.
Which of the statements given above is/are
correct?
a. 1 only
b. 2 only

85.

86.

87.

88.

89.

90.

c. Both 1 and 2
d. Neither 1 nor 2
Consider the following areas:
1. Orinoco basin in South America
2. Zaire basin in Africa
3. New Guinea
4. Isthumus of Central America
Which of the above said areas have
tropical rain forests?
a. 1 and 3 only
b. l, 2 and 4 only
c. 2, 3 and 4 only
d. 1, 2, 3 and 4
Which of the following kinds of organisms
are the diatoms ?
a. Unicellular algae
b. Protozoans
c. Free floating bryophytes
d. Detritivores
Which one of the following statements is
not correct ?
a. Most reef-forming corals prefer sea
temperature between 17 C and 33 C.
b. Cora1 growth is usually restricted to
the upper 25 or 30 m of the sea.
c. Coral animals belong to the organisms
of Phylum Porifera.
d. Coral animals live in a mutualistic
relationship with algae.
Of the solar energy coming into the
Biosphere, what percentage of it is actually
used in the photosynthesis?
a. Less than 1%
b. Between 1% and 15%
c. Between 15% and 2%
d. More than 2%
Among the following greenhouse gases,
the emission of which one of the following
is entirely anthropogen?
a. Chlorofluorocarbons
b. Methane
c. Nitrous oxide
d. Hydrogen sulphide
With reference to a freshwater, lake,
consider the following characteristics
1. Richness in nutrients
2. Rapid turnover of phytoplankton
3. Depletion of dissolved oxygen
Which of the above is/are observed in a
lake after its eutrophication?
a. 1 only
b. 1 and 2 only

91.

92.

93.

94.

95.

96.

97.

10 of 13
c. 3 only
d. l, 2 and 3
With reference to Central Place Theory,
consider the following statements
1. Christaller dealt only with retailing
functions in his theory.
2. Lsch attempted to incorporate
manifesting into his model.
Which of the statements given above is/are
correct?
a. 1 only
b. 2 only
c. Both 1 and 2
d. Neither 1 nor 2
Which one of the following types of spurs
is typically associated with val1ey
glaciers?
a. Interlocking
b. Juxtaposed
c. Overlapping
d. Truncated
How is the amount of horizontal
displacement in a normal fault called ?
a. Throw
b. Tear
c. Heave
d. Strike
Between which one of the following pairs
of islands is Duan Passage located ?
a. South Andaman and Little Andaman
b. Little Andaman and Car Nicobar
Island
c. Car Nicobar Island and Little Nicobar
Island
d. Little Nicobar Island and Great
Nicobar Island
Among the following States of India,
which one has the oldest rock formations
in the country?
a. Assam
b. Bihar
c. Karnataka
d. Uttar Pradesh
Among the following States, which one
has the largest forest area ?
a. Gujarat
b. Karnataka
c. Orissa
d. Tamil Nadu
In which one of the following National
Parks has a climate, that varies from

98.

99.

100.

101.

102.

tropical td sub-tropical temperate and


arctic ?
a. Kangchendzonga National Park
b. Nandadevi National Park
c. Neora National Park
d. Namdhapa National Park
Consider the following statements:
1. Gondwana coal is a laminated,
bituminous coal.
2. Gondwana coal is completely free
from
moisture,
sulphur
and
phosphorus.
Which of the statements given above is/are
correct?
a. 1 only
b. 2 only
c. Both 1 and 2
d. Neither 1 nor 2
Consider the following pairs:
1. Amarkantak area : Bauxite deposits
2. Barabil-Koira
valley:
Haematite
deposits
3. Salem district : Magnesite deposits
Which of the pair/pairs given above is/ are
correctly matched ?
a. 1 only
b. 1 and 2 only
c. 3 only
d. 1, 2 and 3
Which one of the following statements is
not correct ?
a. The Karanphuli river originates in
Meghalaya and flows into Bang1adesh.
b. The Sabarmati river originates in
Rajasthan and flows into the Gulf of
Khambhat
c. The Ghaghara river originates in Tibet
and enters India through Nepal.
d. The Jhelum river originates in Kashmir
and enters Pakistan though Pir Panjal
range
In which one among the following districts
is the highest point of Eastern Ghats
located ?
a. Ganjam
b. Visakhapatnam
c. Kumool
d. North Arcot
Consider the following statements:
1. Calcareous alluvial soils are found in
north-eastern districts of Uttar Pradesh.

103.

104.

105.

11 of 13
2. Calcareous sierozemic soils
are found in coastal districts of south
Orissa and north Andhra Pradesh.
Which of the statements given above is/are
correct?
a. 1 only
b. 2 only
c. Both 1 and 2
d. Neither 1 nor 2
Match List-I with List-II and select the
correct answer using the code given below
the lists
List-I
(Power project)
A. Dehar (hydro)
B. Gandhar (thermal)
C. Kalinadi (hydro)
D. Uran (thermal)
List-II
(State)
1. Gujarat
2. Himachal Pradesh
3. Karnataka
4. Maharashtra
Code:
a. A4, B3, C1, D2
b. A2, B1, C3, D4
c. A4, B1, C3, D2
d. A2, B3, C1, D4
Consider the following statements:
1. Most of the iron ores found in India are
of the limonite type.
2. The magnetite type of iron ore occurs
in the Dharwad and Cuddapah rock
systems of the peninsular India.
3. In India, sphalerite and galena occur in
the pre-cambrian rocks.
Which of the statements given above is/are
correct?
a. 1 and 3 only
b. 2 only
c. 2 and 3only
d. 1, 2 and 3
Consider the following pairs:
Crop Important producing district
1. Turmeric : Kottayam, Kollam and
Kojhikode
2. Rubber: Kottayam, Kollam and
Kojhikode
3. CoffeeEast Godavari, Visakhapatnam
and Srikakulam

12 of 13

106.

107.

108.

109.

110.

111.

112.

Which of the pairs given above is/are


correctly matched?
a. 1 only
b. 1 and 2 only
c. 2 and 3 only
d. 1, 2 and 3
In which one of the following is the
Himalayan pass Shipki La located ?
a. Chandra valley
b. Hunza valley
c. Nubra valley
d. Sutlej valley
Consider the following statements:
1. Bajra (pearl millet) is essentially a rabi
crop in India.
2. Though maize is cultivated as a rabi
crop in some areas, it is also grown as
kharif crop.
Which of the statements given above is/are
correct?
a. 1 only
b. 2 only
c. Both 1 and 2
d. Neither I nor 2
Among the following South Asian
countries, which one has the highest level
of urbanization?
a. Sri Lanka
b. Bangladesh
c. Pakistan
d. Nepal
Which one of the following countries has
the largest forest area in the world?
a. Brazil
b. Canada
c. Russian Federation
d. United States of America
Which one of the following is not required
for the formation of photochemical smog?
a. Oxygen
b. Oxide of nitrogen
c. Carbon monoxide
d. Sun light
The important fishing areas called Grand
Banks and Georges Bank located in the
Waters of which one of the following?
a. North Pacific
b. South Pacific
c. South-west Atlantic
d. North-west Atlantic

In the map given above, four important


towns of South Africa namely, A: Durban,
B : Cape Town, C : East London and D :
Port Elizabeth are marked with numbers.
Match the city with the correct number and
select the correct answer using the code
given below:
Code:
a. A1, B2, C4, D3
b. A1, B4, C2, D3
c. A3, B2, C4, D1
d. A3, B4, C2, D1
113.

114.

In the map given above, four lakes namely,


A. Lake Turkana (Lake Rudolf), B. Lake
Nyasa, C. Lake Tanganyika and D. Lake
Victoria are marked with numbers. Match
the lake with the correct number and select
the correct answer using the code given
below:
Code:
a. A1, B2, C4, D3
b. A1, B4, C2, D3
c. A3, B2, C4, D1
d. A3, B4, C2, D1
Match List-I with List-II and select the
correct answer using the code given below
the lists
List-I

115.

116.

A. Paper mill at Tuli


B. Karbi-Langpi hydroelectric project
C. Loktak Hydroelectric Project
D. Naini Saini Airstrip
List-II
1. Assam
2. Manipur
3. Nagaland
4. Uttarakhand
Code:
a. A3, B4, C1, D2
b. A3, B1, C4, D2
c. A2, B4, C1, D3
d. A2, B1, C4, D2
Match List-I with List-II and select the
correct answer using the code given below
the lists:
List-I
(Geographical features)
A. Death valley
B. Edwards plateau
C. Sonoran desert
D. The Everglades
List -II
(American State)
1. Arizona
2. California
3. Florida
4. Texas
Code:
a. A3, B4, C1, D2
b. A3, B1, C4, D2
c. A2, B4, C1, D3
d. A2, B1, C4, D3
In the context of manual methods of image
interpretation, consider the following
statements:
1. In truly vertical photograph, the nadir
coincides with the photos principal
point.
2. Straight lines connecting opposite
fiducial marks intersect at the principal
point.

117.

118.

119.

120.

13 of 13
Which of the statements given
above is/are correct?
a. 1 only
b. 2 only
c. Both 1 and 2
d. Neither 1 nor 2
Consider the following cities:
1. Antwerp
2. Maastricht
3. Rotterdam
Which of the above cities has/have port(s)
connected to North Sea?
a. 1 only
b. 2 and 3 only
c. 1 and 3 only
d. 1, 2 and 3
Consider the following pairs:
1. Edinburgh : North Sea
2. Liverpool : Irish Sea
3. Plymouth : Sea of the Herbides
4. Cardiff : English Channel
Which of the pairs given above are
correctly matched?
a. 1 and 2 only
b. 3 and 4 only
c. 1, 2 and 4 only
d. 1, 2, 3 and 4
In which one of the following basins is
Taklamakan desert located ?
a. Tarim basin
b. Lopnor basin
c. Kerulin basin
d. Red basin
Consider the following pairs
1. Biwa Ko : Largest lake in Japan
covering about 670 km2
2. Sichuan Pendi : One of Chinas rice
producing areas
Which of the pairs given above is/are
correctly matched ?
a. 1 only
b. 2 only
c. Both 1 and 2
d. Neither 1 nor 2

CSE Pre-2009

1 of 15

GEOGRAPHY
1.

2.

3.

4.

Consider the following cities :


1. Agra
2. Indore
3. Nashika
4. Vadodara
Which of the above are million-plus cities
as per Census 2001?
a. 1 and 4 only
b. 1, 2 and 3 only
c. 2, 3 and 4 only
d. 1, 2, 3 and 4
Consider the following statements:
1. The sex ratio in India as per Census
2001 is lower as compared to that of
Census 1951.
2. From 1991 to 2001, the sex ratio in
India has increased.
Which of the statements given above is/are
correct?
a. 1 only
b. 2 only
c. Both 1 and 2
d. Neither 1 nor 2
Consider the following pairs :
1. Centre for Environmental Management
of Degraded Ecosystems : New Delhi
2. Centre of Social Forestry and EcoRehabilitation : Allahabad
3. National Coral-Reef Research Centre:
Thiruvananthapuram
Which of the above pairs is/are correctly
matched?
a. 1 only
b. 1 and 2 only
c. 2 and 3 only
d. 1, 2 and 3
Match List I with List II and select the
correct answer using the code given below
the lists :
List I
(Polluted Area)
A. Greater Kochi
B. Nagda-Ratlam

5.

6.

C. Pali
D. Parwanoo
List II
(Type of Industry)
1. Cotton textiles and dyeing
2. Food processing and electroplating
3. Oil
refineries,
chemical
and
metallurgical industries
4. Viscose rayon, caustic, dyes and
distillery
Code:
A
B
C
d
a.
4
3
2
1
b.
3
4
2
1
c.
4
3
1
2
d.
3
4
1
2
Match List I with List II and select the
correct answer using the code given below
the lists :
List I
(Country)
A. Canada
B. India
C. Russia
D. Zaire
List II
(Tribe)
1. Onga
2. Chukchi
3. Haida
4. Lese
Code:
A
B
C
D
a.
2
4
3
1
b.
3
1
4
2
c.
2
3
4
1
d.
3
1
2
4
The draught animal 'Llamas' are found in
a. Tibet
b. Arctic
c. Alps

2 of 15

7.

8.

9.

10.

11.

d. Andes
Which one of the following regions is the
world's best cod-fishing ground ?
a. North-east Atlantic
b. South-east Pacific
c. West Central Pacific
d. Indian Ocean
The Dogger Bank, the main fishing area of
Europe, lies in
a. Baltic Sea
b. English Channel
c. North Sea
d. Norwegian Sea
Match List I with List II and select the
correct answer using the code given below
the lists :
List I
(Country)
A. Philippines
B. Thailand.
C. Sri Lanka
List II
(Names of shifting cultivation the country
is associated with)
1. Chena
2. Caingin
3. Milpa
4. Tamrai
Code:
A
B
C
a.
3
4
1
b.
3
1
4
c.
2
4
1
d.
2
1
4
Factory farming is the characteristic of
a. North-western North America
b. North-western Europe
c. Eastern Europe
d. South-east Asia
In which one of the following countries is
the iron mining centre called Scunthorpe
located ?
a. France
b. Germany
c. Sweden

12.

13.

14.

15.

d. United Kingdom
Which of the three countries were
members of European Free Trade
Association (EFTA) when it was founded
in 1960?
a. Italy, Belgium, Poland
b. Hungary, Finland, Spain
c. Austria, Denmark, Norway
d. The Netherlands, France, Romania
The coal basin region called Donbas lies to
the
a. North of Black Sea and the Sea of
Azov
b. North of Caspian Sea
c. East of Gulf of Finland
d. South of White Sea and south-west of ;
Kanin Peninsula
Kampung is a form of rural settlement
found in
a. China
b. Malaysia
c. Philippines
d. Cambodia
Match List I with List II and select the
correct answer using the code given below
the lists :
List I
(Patterns of rural settlements)
A. Arrow
B. Checkerboard
C. Circular
D. Fan
List II
(Locations associated with)
1. Delta
2. Lake
3. Mountain slope
4. Road crossing at right angle
5. Road meeting a river bend
Code:
A
B
C
D
a.
5
4
2
1
b.
3
4
2
1
c.
5
2
4
3
d.
3
2
4
1

3 of 15
16.

17.

18.

In vertical distribution of ocean deposits,


what is the correct sequence (from oceanic
surface towards bottom) of the following
deposits?
1. Calcareous oozes
2. Siliceous oozes
3. Terrigenous deposits
4. Red clay
Select the correct answer using the code
given below :
a. 1, 2, 3, 4
b. 3, 1, 2, 4
c. 3, 2, 1, 4
d. 4, 3, 2, 1
Match List I with List II and select the
correct answer using the code given below
the lists :
List I
(Genetic Phases)
A. The Sun and the Moon are at right
angles with the Earth
B. The Sun, the Moon and the Earth are
in a straight line
C. The Moon is nearest to the Earth
D. The Moon is farthest from the Earth
List II
(Resultant Tides)
1. Spring tide
2. Neap tide
3. Apogean tide
4. Perigean tide
Code:
A
B
C
D
a.
2
1
4
3
b.
2
4
1
3
c.
3
4
1
2
d.
3
1
4
2
Consider the following pairs :
Area : Well-known for
1. Kinta Valley : Tin fields
2. Sudbury region : Nickel production
3. Chihuahua : Silver production
Which of the pairs given above is/are
correctly matched ?
a. 1 only
b. 2 and 3 only

19.

20.

21.

c. 1 and 3 only
d. 1, 2 and 3
Match List I with List II and select the
correct answer using the code given below
the lists :
List I
(Oil-field)
A. Bahregan
B. Dhahran
C. Kirkuk
D. Palembanng
List II
(Country)
1. Iraq
2. Iran
3. Indonesia
4. Saudi Arabia
Code:
A
B
C
D
a.
2
4
1
3
b.
2
1
4
3
c.
3
4
1
2
d.
3
1
4
2
With reference to the availability of
minerals in India, consider the following
statements :
1. The
highest
concentration
of
manganese is found in the Dharwar
system of rocks.
2. The chromite mainly occurs in ultrabasic igneous rocks.
Which of the statements given above is/are
correct?
a. 1 only
b. 2 only
c. Both 1 and 2
d. Neither 1 nor 2
With reference to the occurrence of
minerals in India, consider the following
statements :
1. Silver is a mineral of primary origin
occurring in acid igneous rocks.
2. Bauxite is a secondary mineral mainly
associated with lateritic rocks formed
during tertiary period.

4 of 15

22.

23.

24.

25.

Which of the statements given above is/are


correct ?
a. 1 only
b. 2 only
c. Both 1 and 2
d. Neither 1 nor 2
Among the following states, in which one
is the percentage of forest area to total
geographical area the highest ?
a. Arunachal Pradesh
b. Himachal Pradesh
c. Sikkim
d. Uttarakhand
Consider the following pairs :
Tree : Location of its natural habitat
1. Rosewood : Slopes of Western Ghats
of Karnataka and Kerala
2. Spruce : Western Himalayas at
elevations from 2100 m to 3600 m
3. Walnut : Lower elevations of Eastern
Himalayas
Which of the pairs given above is/are
correctly matched?
a. 1 only
b. 1 and 2 only
c. 2 and 3 only
d. 1, 2 and 3
Consider the following statements :
1. The Buckingham Canal runs through
the coast from Guntur district to South
Arcot district.
2. For Indira Gandhi Canal, additional
waters of Beas and Yamuna are
diverted and the canal terminates in
Barmer district.
Which of the statements given above is/are
correct?
a. 1 only
b. 2 only
c. Both 1 and 2
d. Neither 1 nor 2
Consider the following statements :
1. The genre de vie of Vidal de la Blache
refers to the inherited traits that
members of a human group learn, what
we may call culture.

26.

27.

28.

2. The genre de vie is a basic factor in


determining which are the various
possibilities offered by nature a
particular human group will select.
Which of the statements given above is/are
correct ?
a. 1 only
b. 2 only
c. Both 1 and 2
d. Neither 1 nor 2
Consider the following statements :
1. The Kangra Valley is a strike valley
extending from the foot of the Dhaula
Dhar Range to the South of the Beas
River.
2. The Kulu Valley is a transverse valley
in the upper course of Ravi.
Which of the statements given above is/are
correct ?
a. 1 only
b. 2 only
c. Both 1 and 2
d. Neither 1 nor 2
Match List I with List II and select the
correct answer using the code given below
the lists :
List I
(Wetland)
A. Barilla
B. Dihaila
C. Pariyej
D. Pong
List II
(State)
1. Bihar
2. Gujarat
3. Himachal Pradesh
4. Madhya Pradesh
Code:
A
B
C
D
a.
1
4
2
3
b.
1
2
4
3
c.
3
4
2
1
d.
3
2
4
1
Consider the following rivers :
1. Banas

5 of 15

29.

30.

31.

2. Ghaggar
3. Machhu
4. Rupen
Which of the above neither flow into the
sea nor merge with any major river that
flows into the sea, but are lost in the
desert?
a. 1 and 2 only
b. 2, 3 and 4 only
c. 1, 3 and 4 only
d. 1, 2, 3 and 4
Match List I with List II and select the
correct answer using the code given below
the lists :
List I
(Mangrove Area)
A. Coringa
B. Coondapur
C. Ramnad
D. Vikhroli
List II
(State)
1. Andhra Pradesh
2. Karnataka
3. Maharashtra
4. Tamil Nadu
Code:
A
B
C
D
a.
1
2
3
4
b.
1
2
4
3
c.
2
1
3
4
d.
2
1
4
3
In which of the following places are
deposits of Wolfram found ?
a. Chandigiri and Amarkantak
b. Chendipathar and Degana
c. Bellary and Radhanagari
d. Guntur and Hazaribagh
Consider the following statements:
1. The Cochin Shipyard was constructed
with the Japanese assistance.
2. The Integral Coach Factory was set up
with German collaboration.
Which of the statements given above is/are
correct ?
a. 1 only

32.

33.

34.

35.

36.

b. 2 only
c. Both 1 and 2
d. Neither 1 nor 2
Match List I with List II and select the
correct answer using the code given below
the lists :
List I
(National Highway)
A. NH 9
B. NH 12
C. NH 15
D. NH 17
List II
(Route)
1. Pune-Machilipatnam
2. Pathankot-Samakhiali
3. Panvel Edapally
4. Jabalpur Jaipur
Code:
A
B
C
D
a.
1
2
4
3
b.
1
4
2
3
c.
3
2
4
1
d.
3
4
2
1
Among the following, the narrowest
portion of English Channel lies between ,
a. Dover and Calais
b. Portsmouth and Le Havre
c. Hull and Rotterdam
d. Plymouth and Brest
Alps Mountains are spread over
a. France, Switzerland, Italy, Romania
b. France, Switzerland, Italy, Austria
c. France, Switzerland, Italy, Spain
d. France, Switzerland, Italy, Belgium
The high altitude tropical forest which has
dwarfed trees is known as
a. Alpine forest
b. Elfin forest
c. Montane forest
d. Sclerophyll forest
Consider the following pairs :
1. Coal : Donbas
2. Iron ore : Magnitogorsk
3. Mica : Mama

6 of 15

37.

38.

39.

Which of the pairs given above is/are


correctly matched ?
a. 1 only
b. 1 and 2 only
c. 2 and 3 only
d. 1, 2 and 3
Match List I with List II and select the
correct answer using the code given below
the lists :
List I
(Island)
A. Hokkaida
B. Honshu
C. Shikoku
D. Kyushu
List II
(City)
1. Takamatsu
2. Nagasaki
3. Yokohama
4. Hakodate
Code:
A
B
C
D
a.
2
1
3
4
b.
2
3
1
4
c.
4
1
3
2
d.
4
3
1
2
Consider the following pairs :
1. Kumamoto : Ship-building
2. Fukuoka : Chemicals
3. Hamamatsu : Musical instruments
Which of the pairs given above is/are
correctly matched?
a. 1 only
b. 1 and 2 only
c. 2 and 3 only
d. 1, 2 and 3
The Hanshin region of Japan is an
industrial conurbation formed by which
one of the following groups of three cities?
a. Tokyo, Kawasaki, Yokohama
b. Yawata, Kokura, Mauji
c. Osaka, Kobe, Kyoto
d. Osaka, Tokyo, Kyoto

40.

41.

42.

43.

44.

45.

46.

47.

Summer rains in Australia broadly


decrease from
a. East to west
b. West to east
c. North to south
d. South to north
Among the following, which one has the
largest number of neighbouring countries
sharing the land border ?
a. Sudan
b. Brazil
c. China
d. Zaire
Which one of the following rivers
demarcates the partial boundary between
South Africa and Botswana?
a. Limpopo
b. Zambezi
c. Cubango
d. Kwando
Vaal is a major tributary of which one of
the following rivers?
a. Limpopo
b. Nile
c. Orange
d. Zambezi
The large coffee plantation in Brazil is
called
a. Estate
b. Farm
c. Fazendas
d. Kolkhozes
Arakan and Pegu Yomas are located in
a. Myanmar
b. Nepal
c. Bhutan
d. China
On the banks of which river is the city of
Kathmandu located?
a. Arun
b. Saptakosi
c. Bagmati
d. Sunkosi
Of the following, who enunciated that "all
history should be studied geographically
and all geography historically"?

7 of 15

48.

49.

50.

a. Herodotus
b. Strabo
c. Hecataeus
d. Posidonius
Consider the following statements :
1. Ptolemy presumed that the prime
meridian passes through Bermuda
Island.
2. Ptolemy believed that Ceylon marks
the southern limit of the world.
Which of the statements given above is/are
correct ?
a. 1 only
b. 2 only
c. Both 1 and 2
d. Neither 1 nor 2
Caledonian orogenic movements are
related to the geological history of
a. Late Jurassic and early Cretaceous
period
b. Late Silurian and early Devonian
period
c. Middle Triassic period
d. Late
Devonian
and
early
Carboniferous period
Match List I with List II and select the
correct answer using the code given below
the lists :
List I
(Discontinuities)
A. Weichert Gutenberg discontinuity
B. Mohorovicic (Moho) discontinuity
C. Lehman discontinuity
D. Conrad discontinuity
List II
(Layers of Earth's interior associated with
discontinuity)
1. Outer core and inner core
2. Lower mantle and outer core
3. Upper mantle and lower mantle
4. Lower crust and upper mantle
5. Division of Sial and Sima within the
crust
Code:
A
B
C
D
a.
5
1
3
2

51.

52.

53.

54.

55.

b.
2
1
3
5
c.
2
4
1
5
d.
5
4
1
2
Consider the following minerals :
1. Corundum
2. Quartz
3. Topaz
What is the correct sequence of the above
in terms of descending order of their
hardness?
a. 1-2-3
b. 2-3-1
c. 1-3-2
d. 3-1-2
A lens-shaped igneous intrusion situated
beneath an anticlinal fold or in the base of
a syncline is called
a. Lapolith
b. Lacolith
c. Phacolith
d. Batholith
Open folds are those in which the angle
between the two limbs of the fold is
a. more than 90 but less than 180
b. more than 75 but less than 150
c. more than 45 but less than 90
d. more than 120 but less than 210
Consider the following statements :
1. Kaolinite is the weathered silicate clay.
2. Kaolinization process occurs due to
hydrothermal alteration of feldspar.
Which of the statements given above is/are
correct ?
a. 1 only
b. 2 only
c. Both 1 and 2
d. Neither 1 nor 2
Consider the following statements:
1. Pyramidal peak is a gentle-sided
mountain summit.
2. Tarn is formed often in the cirques
3. The long axis of drumlins are parallel
to the direction of glacial movement
Which of the statements given above are
correct?
a. 1 and 2 only

8 of 15

56.

57.

58.

59.

b. 2 and 3 only
c. 1 and 3 only
d. 1, 2 and 3
A landform developed by the process of
deposition occurring towards the centre of
intermontane basins in arid or semi-arid
regions is known as
a. Pediment
b. Bajada
c. Playa
d. Peneplain
Eskers are of glacio-fluvial origin. Which
one of the following statements
characterize them?
a. They are long, narrow and sinuous
ridges of sands.
b. They are small, irregular mounds of
bedded sands.
c. They are small alluvial cones.
d. They are flat-topped terraces.
Consider the following statements
regarding the distribution and pattern of
isotherms :
1. In the northern hemisphere, isotherms
bend sharply equator-ward over the
continents during winter.
2. Temperature gradients are greater in
summer than in winter in the northern
hemisphere.
Which of the statements given above is/are
correct ?
a. 1 only
b. 2 only
c. Both 1 and 2
d. Neither 1 nor 2
Match List I with List II and select the
correct answer using the code given below
the lists :
List I
(Local Wind)
A. Harmattan
B. Haboob
C. Bora
D. Zonda
List II
(Country /Region)
1. Argentina

60.

61.

62.

63.

2. Adriatic coast
3. West Africa
4. Sudan
Code:
A
B
C
D
a.
1
2
4
3
b.
1
4
2
3
c.
3
4
2
1
d.
3
2
4
1
With reference to convectional type of
rainfall, consider the following statements:
1. It is a warm weather phenomenon
2. This rainfall is associated with cumulonimbus clouds.
Which of the statements given above is/are
correct ?
a. 1 only
b. 2 only
c. Both 1 and 2
d. Neither 1 nor 2
With reference to the sub-tropical high
pressure belt, consider the following
statements :
1. It is affected by Earth's rotation and
descent of winds from higher altitudes.
2. It is dynamically induced and
characterised
with
anti-cyclonic
conditions.
Which of the statements given above is/are
correct ?
a. 1 only
b. 2 only
c. Both 1 and 2
d. Neither 1 nor 2
In Koppen's classification of world
climate. Aw climates refer to
a. Humid tropical wet monsoon climate
with a short dry season
b. Tropical wet and dry climate with dry
season in winter
c. Tropical desert climate
d. Humid meso-thermal climates
With reference to equatorial climate,
consider the following statements :
1. Thermally induced low pressure belt
develops around the equator but the
pressure gradient is very low.

9 of 15

64.

65.

66.

67.

68.

2. Temporal variability in the amount of


rainfall is not found.
Which of the statements given above is/are
correct ?
a. 1 only
b. 2 only
c. Both 1 and 2
d. Neither 1 nor 2
Who introduced the term anticyclone for
the first time ?
a. Fitzroy
b. J. Bjerknes
c. F. Galton
d. V. Bjerknes
For obtaining greater accuracy in the
fraction of a division in both linear and
angular measurements, which scale is
used?
a. Linear Scale
b. Vernier Scale
c. Plane Scale
d. Graphic Scale
Which one of the following RF represents
the 1 cm map distance to 5 km of ground
distance?
a. 1 : 500
b. 1 : 5000
c. 1 : 50000
d. 1 : 500000
A form of diagram in which values are
plotted as radii from a point of origin, is
known as
a. Star diagram
b. Columnar diagram
c. Block pile diagram
d. Dispersion diagram
With reference to Dot method, consider
the following statements :
1. Dot method is the most convenient
method for illustrating the distribution
of absolute figures.
2. The size of the dots depends upon the
frequency of the element to be
represented.
3. In it each dot has an identical value.

69.

70.

71.

4. It is used very effectively when


plotting values in detail on a large
scale.
Which of the statements given above are
correct?
a. 1, 3 and 4
b. 2, 3 and 4
c. 1 and 2 only
d. 1, 2 and 3
Match List I with List II and select the
correct answer using the code given below
the lists :
List I
(Map type)
A. Choropleth
B. Isopleth
C. Chorochromatic
D. Choroschematic
List II
(Information Content)
1. Quantitative map showing imaginary
lines joining places of equal value for
any distribution
2. Qualitative map showing areal
distribution by symbols, index letters
etc.
3. Qualitative map showing areal
distribution by shades and tints
4. Quantitative map showing regional
variation of distribution using shades
Code:
A
B
C
D
a.
2
1
3
4
b.
2
3
1
4
c.
4
3
1
2
d.
4
1
3
2
A graph showing wavelength and
reflectance amount is known as
a. Spectral Reflectance Curve
b. Spectral Signature
c. Spectral Band
d. Spectral Regions
For representing the data relating to cost
control, quality control and assessment of
progress, which of the following is/are the
correct use of the diagram ?
a. Cycle diagram

10 of 15

72.

73.

74.

75.

b. Chorochromatic maps
c. Chorographic compass maps
d. Both
Chorochromatic
and
chorographic compass maps
A vector-based data structure is made for
storing terrain information in digital terrain
modelling in which each sample point has
an x, y co-ordinate and a height, or z
value. All the points are connected by
edges to form a network of nonoverlapping triangles that collectively
represent the terrain surface. This network
is known as
a. TIN
b. DTM
c. TQM
d. TIGER
In the Government of India, under which
Ministry
is
the
National
River
Conservation Directorate?
a. Ministry of Agriculture :
b. Ministry of Earth Sciences
c. Ministry of Environment and Forests
d. Ministry of Water Resources
Consider the following statements :
1. The Rhine rift valley stretches for
more than 800 km.
2. The floor of Dead Sea is more than
1100 m below sea level.
Which of the statements given above is/are
correct ?
a. 1 only
b. 2 only
c. Both 1 and 2
d. Neither 1 nor 2
Consider the following statements :
1. Mistral is a cold wind blowing in
North America.
2. Sirocco is a hot dry wind blowing
across the North African coast.
Which of the statements given above is/are
correct ?
a. 1 only
b. 2 only
c. Both 1 and 2
d. Neither 1 nor 2

76.

77.

78.

79.

In the context of the inflow of South


Westerly monsoon into India, consider the
following statements :
1. The Arabian Sea current merges with
the Bay of Bengal current over the
Punjab and adjoining Himalayas.
2. "Breaks" or spells of dry weather are
not the characteristic features of southwest monsoon.
Which of the statements given above is/are
correct ?
a. 1 only
b. 2 only
c. Both 1 and 2
d. Neither 1 nor 2
Consider the following statements
regarding
general
characteristics
associated with tropical cyclones :
1. Tropical cyclones, move with very low
velocity over the oceans but become
strong while moving over the land
areas.
2. They are confined to a particular
period of the year (mainly summer)
and normally move from east to west.
Which of the statements given above is/are
correct ?
a. 1 only
b. 2 only
c. Both 1 and 2
d. Neither 1 nor 2
There is a submarine ridge in the Bay of
Bengal. What is it called ?
a. Ninety East ridge
b. Mozambique ridge
c. Chagos-Laccadive ridge
d. Carlsberg ridge
Consider the following statements :
1. The
continental
shelves
of
Lakshadweep originated due to coral
reefs.
2. The continental shelves of western
coast are due to faulting and
submergence.
Which of the statements given above is/are
correct?
a. 1 only
b. 2 only

11 of 15

80.

81.

82.

83.

c. Both 1 and 2
d. Neither 1 nor 2
Consider the following statements :
1. Temperature of surface water is
comparatively higher in the northern
hemisphere than in the southern
hemisphere.
2. The isotherms in the southern
hemisphere are not regular and do not
follow the latitudes while they
(isotherms) are regular and follow the
latitudes in the northern hemisphere.
Which of the statements given above is/are
correct?
a. 1 only
b. 2 only
c. Both 1 and 2
d. Neither 1 nor 2
Consider the following :
1. Gulf of California
2. Mediterranean Sea
3. Baltic Sea
In terms of decreasing salinity, which one
of the following is the correct sequence?
a. 1-2-3
b. 2-1-3
c. 3-1-2
d. 2-3-1
Consider the following statements:
1. Globigerina ooze is a calcareous
pelagic deposit.
2. Globigerina ooze is found mostly in
the tropical and temperate zones of the
Atlantic Ocean.
Which of the statements given above is/are
correct ?
a. 1 only
b. 2 only
c. Both 1 and 2
d. Neither 1 nor 2
Regular oscillations in the water surface of
large water bodies are formed because of
a. Gravitational effect of Moon
b. Difference in temperature of sea water
and the air above
c. Rotation of Earth
d. Friction of wind on the water surface

84.

85.

86.

87.

88.

89.

Consider the following statements :


1. Laterite consists essentially of oxides
of iron and aluminium.
2. Ferrallitic soils occur widely in the
humid tropics where chemical
weathering and leaching are intense.
Which of the statements given above is/are
correct ?
a. 1 only
b. 2 only
c. Both 1 and 2
d. Neither 1 nor 2
Consider the following pairs :
1. Podzolization : Eluviation of clays,
organic matter, calcium, sodium, iron,
and aluminium out of the A horizon
2. Gleization : Accumulation of organic
matter in a swampy environment
Which of the pairs given above is/are
correctly matched?
a. 1 only
b. 2 only
c. Both 1 and 2
d. Neither 1 nor 2
Chernozems are
a. Silica rich temperate grassland soils
b. Iron rich humid upland soils
c. Humus rich dark brown colour soils in
semi-arid region
d. Humus rich tropical grassland soils
In which one of the following regions are
the hurricanes apparently absent?
a. Coast of Texas
b. Gulf of Mexico
c. South-eastern coast of USA
d. South-eastern coast of Pacific
The shifting cultivation that is leading to
soil erosion in Brazil is known as
a. Milpa
b. Conuco
c. Tavy
d. Roca
Two main cold currents in the North
Pacific Ocean are known as
a. Oyashio and California currents
b. Alaska and Kuroshio currents

12 of 15

90.

91.

92.

c. North
Equatorial
and
Counter
Equatorial currents
d. North Pacific and Kuroshio currents
Match List I with List II and select the
correct answer using the code given below
the lists :
List I
(Author)
A. Davis
B. Hack
C. Hutton
D. King L.C
List II
(Concept)
1. Pediplanation
2. Peneplanation
3. Dynamic Equilibrium
4. Uniformitarianism
Code:
A
B
C
D
a.
1
4
3
2
b.
1
3
4
2
c.
2
4
3
1
d.
2
3
4
1
With reference to migration, who of the
following formulated the 'Gravity Model'
which demonstrates the relationship
between population size, distance and
migration?
a. Zipf
b. Miller
c. Malthus
d. Zelinsky
Consider the following statements :
1. The location of objects in geographic
space and the associated attributes are
handled by GIS.
2. Spatial objects are represented by
graphical elements of points, lines and
polygons.
Which of the statements given above is/are
correct?
a. 1 only
b. 2 only
c. Both 1 and 2
d. Neither 1 nor 2

93.

94.

95.

96.

Match List I with List II and select the


correct answer using the code given below
the lists:
List I
(Country)
A. Thailand
B. Singapore
C. Indonesia
D. Vietnam
List II
(Feature)
1. Entrepot
2. Mekong delta
3. Pattaya beach
4. Banda Aceh
Code:
A
B
C
D
a.
2
4
1
3
b.
2
1
4
3
c.
3
4
1
2
d.
3
1
4
2
Geography was first divided into 'General'
and 'Special' by
a. Varenius
b. Kant
c. Humboldt
d. Ritter
Consider the following statements :
1. Ritter identified races by skin, colour
2. Ritter's concepts regarding the Earth's
geographical
patterns
were
teleological.
Which of the statements given above is/are
correct?
a. 1 only
b. 2 only
c. Both 1 and 2
d. Neither 1 nor 2
Ratzel
coined
the
term
'Anthropogeography' which he used for the
a. Geography of man in terms of
individuals and races
b. Geography of man and his works in
relation to the Earth's surface
c. Organic theory of society and state

13 of 15

97.

98.

99.

100.

101.

d. Study of harmonious reciprocal


relation of man and nature
Which one of the following geographers
wrote the book 'Influences of Geographic
Environment'?
a. Humboldt
b. Ratzel
c. Ritter
d. Semple
Diorite is an example of
a. Chemically formed sedimentary rock
b. Extrusive igneous rock
c. Plutonic igneous rock
d. Metamorphic rock
Among the following regions of India,
which one receives comparatively least
amount of annual rainfall ?
a. An 80 km wide coastal belt from
Nellore to Point Calimere
b. The middle and lower Assam valley
c. North-eastern Rajasthan
d. The coastal plains of Gujarat south of
Narmada
As per Koppen's classification of climate,
which one of the following is the suitable
description for north-east India including
north Bengal ?
a. Tropical monsoon rainforest (Amw)
b. Sub-tropical monsoon rainforest (Am)
c. Humid subtropical with dry winter
(Cwg)
d. Tropical wet and dry climate
(Monsoon savannah, Aw)
Consider the following statements :
1. In India, groundnut is principally
grown as rain-fed crop.
2. In India, sesamum is grown as kharif
crop in northern states but in south it is
generally grown during the rabi
season.
Which of the statements given above is/are
correct ?
a. 1 only
b. 2 only
c. Both 1 and 2
d. Neither 1 nor 2

102.

103.

104.

105.

Match List I with List II and select the


correct answer using the code given below
the lists :
List
(Crop)
A. Ginger
B. Jute
C. Mustard
D. Turmeric
List II
(Major cultivating districts)
1. Cuddapah and Guntur
2. Idukki and Wayanad
3. Goalpara and Kamrup
4. Bharatpur and Ganganagar
Code:
A
B
C
D
a.
1
4
3
2
b.
2
3
4
1
c.
1
3
2
4
d.
2
4
3
1
Consider the following pairs :
Mineral : District with large/significant /
commercially important deposits
1. Apatite : Visakhapatnam
2. Chromite : Cuttack
3. Gold : Nagaur
4. Gypsum : Anantapur
Which of the pairs given above are
correctly matched?
a. 1 and 2 only
b. 1, 2 and 3
c. 1, 2 and 4
d. 2, 3 and 4
The Bhakra Nangal Multipurpose Project
is a joint venture of
a. Punjab and Haryana only
b. Punjab, Haryana and Rajasthan only
c. Punjab, Haryana, Rajasthan and
Himachal Pradesh
d. Punjab and Himachal Pradesh only
Consider the following statements:
1. Maharashtra has more net irrigated
area as compared to Madhya Pradesh.

14 of 15

106.

107.

108.

109.

2. Punjab has more area under canal


irrigation as compared to Haryana.
Which of the statements given above is/are
correct ?
a. 1 only
b. 2 only
c. Both 1 and 2
d. Neither 1 nor 2
Match List I with List II and select the
correct answer using the code given below
the lists:
List I
(Coalfields)
A. Makum
B. Kamptee
C. Karanpura
D. Singrauli
(Waidhian)
List II
(Districts)
1. Dibrugarh
2. Palarnau and Hazaribagh
3. Nagpur
4. Sidhi and Sahdol
Code:
A
B
C
D
a.
1
2
3
4
b.
1
3
2
4
c.
4
2
3
1
d.
4
3
2
1
The plain formed due to coalescence of
series of alluvial fans in the piedmont zone
is known as
a. Pediment
b. Bajada
c. Pediplain
d. Hamada
Aretes are sharp ridges which develop
between the adjacent
a. Cirques
b. Roche moutonne
c. Medial moraines
d. U-shaped valleys
Calcium
carbonate
deposited
by
precipitation from carbonate-saturated
waters around a hot spring is called

110.

111.

112.

113.

114.

a. Stalactite
b. Stalagmite
c. Travertine
d. Lapies
When a descending air contracts and its
volume decreases, what happens to its
temperature?
a. Its temperature decreases
b. Its temperature increases
c. Its temperature remains constant
d. Its temperature first increases, then
decreases
What are Chinooks?
a. Cold local winds which blow from
north-west to south-east direction in
Spain
b. Extremely dry winds blowing from
north-east and east to west in eastern
parts of Sahara desert
c. Warm and dry winds which blow
down the eastern slopes of Rocky
Mountains of North America
d. Warm and dry winds along the
northern slopes of Alps Mountains
Condensation is a process which occurs
when
a. Temperature falls below 00C
b. Absolute humidity becomes equal to
relative humidity
c. Temperature in the air parcel remains
static
d. Relative humidity becomes 100% and
more
Which of the following sequences of
clouds in ascending order would one meet
500 metres and above the mean sea level?
a. Stratus, Nimbus, Cirrostratus, Cirrus
b. Nimbus, Cirrostratus, Cirrocumulus,
Stratus
c. Cirrostratus, Stratus, Nimbus, Cumulonimbus
d. Stratus, Cirrostratus, Cumulo-nimbus,
Cirrocumulus
A ridge, 64,000 km long and 1000 to 4000
km wide, runs down the middle of the
North and the South Atlantic Ocean
basins, into the Indian Ocean basins, then
passes between Australia and Antarctica to

15 of 15

115.

116.

117.

118.

enter the South Pacific basin. What is this


ridge?
a. Socotra-Lakshadweep-Chagos ridge
b. Pacific-Antarctic ridge
c. Dolphin-Challenger ridge
d. Mid-oceanic ridge
Match List I with List II and select the
correct answer using the code given below
the lists:
List I
(Sediment Types)
A. Terrigenous
B. Biogenous
C. Hydrogenous
D. Comogenous
List II
(Examples)
1. Calcareous oozes
2. Quartz and clay
3. Tektite spheres
4. Manganese nodules
Code:
A
B
C
D
a.
2
4
1
3
b.
2
1
4
3
c.
3
1
4
2
d.
3
4
1
2
Hard pan and clay pan in soil are found in
a. A-horizon
b. B-horizon
c. C-horizon
d. A-C horizon
Very thick A-horizon is often found in
a. Chernozem soil
b. Chestnut soil
c. Brown forest soil
d. Podzolic soil
Match List I with List II and select the
correct answer using the code given below
the lists:
List I
(Soil group)
A. Chernozem
B. Terra Rossa
C. Podsol

119.

120.

D. Sierozem
List II
(Climatic Regimes)
1. Temperate grasslands
2. Mediterranean
3. Cool temperate
4. Hot deserts
Code:
A
B
C
D
a.
4
3
2
1
b.
1
2
3
4
c.
4
2
3
1
d.
1
3
2
4
Match List I with List II and select the
correct answer using the code given below
the lists:
List I
(Highest Peak)
A. Mount Logan
B. Mount McKinley1
C. Mount Kosciusko
List II
(Country)
1. Australia
2. U.S.A.
3. Canada
Code:
A
B
C
a.
1
3
2
b.
1
2
3
c.
3
2
1
d.
2
3
1
In U.S.A. which one of the following
regions receives heavy rainfall throughout
the year under the influence of Westerlies?
a. North Western
b. North Eastern
c. South Western
d. South Eastern

You might also like